You are on page 1of 293

Problem Solving in

Rheumatology
KEVIN PILE MB ChB, MD, FRACP
Conjoint Professor of Medicine, University of Western Sydney, New South Wales, Australia

LEE KENNEDY BSc, MB ChB, MD, PhD, FRCP, FRCPE, FRACP


Professor of Medicine, School of Medicine, Department of Medicine, James Cook University, Queensland, Australia

CLINICAL PUBLISHING OXFORD

Contents
Abbreviations vii

SECTION 1 1. 2. 3. 4. 5. 6. 7.

General Rheumatology and Soft Tissue Rheumatism

New Onset Painful Joints 1 An Acutely Swollen/Hot Joint 6 Painful Shoulders Rotator Cuff and Frozen Shoulder 11 Tennis Elbow and Golfers Elbow 18 Carpal Tunnel Syndrome and Other Entrapment Neuropathies 21 Fibromyalgia Syndrome 27 Plantar Fasciitis 33 SECTION 2

Osteoarthritis

8. 9. 10. 11. 12. 13.

Causes and Prevention 39 Non-Pharmacological Treatment 45 Drug Treatment 50 NSAIDs Gastric Side Effects and Protection 54 NSAIDs Cardiac Complications 60 Joint Replacement Surgery 65 SECTION 3

Rheumatoid Arthritis

14. 15. 16. 17. 18. 19. 20.

Causes 71 Laboratory and Imaging Investigations 77 Managing Rheumatoid Arthritis at Onset 82 Evaluating the Response to Treatment 87 Pregnancy and Rheumatic Diseases 92 Diet and Arthritis 97 Polyarthritis in the Elderly 103 SECTION 4

Systemic Lupus Erythematosus, Sjgrens Syndrome and Scleroderma

21. Antinuclear Factor 109 22. SLE Risk Factors and Diagnosis 116 23. Monitoring and Managing SLE 122

vi

Contents

24. 25. 26. 27.

Sjgrens Syndrome 129 Raynauds Phenomenon 134 Assessing and Treating Scleroderma 139 Immunosuppressive Drugs 147 SECTION 5

Vasculitic Syndromes

28. Vasculitic Disease 153 29. Giant Cell Arteritis and Polymyalgia Rheumatica 159 30. Behets Syndrome 165 SECTION 6 31. 32. 33. 34. 35. 36. 37. 38. 39.

Back and Specic Joint Problems

Acute Back Pain 169 Chronic Back Pain 175 Psoriatic Arthritis 178 Asymptomatic Hyperuricaemia 184 Gout Acute Attack and Beyond 189 Pseudogout Investigation and Management 195 Joint and Bone Infections 199 Viral Arthritis 205 Rheumatological Complications of Diabetes 211 SECTION 7

Bone Diseases

40. 41. 42. 43. 44. 45. 46.

Osteoporosis Prevention and Lifestyle Management 217 Bisphosphonates for Osteoporosis Which Agent and When? 222 Osteoporosis Drugs Other Than Bisphosphonates 227 Male Osteoporosis 233 Glucocorticoid-Induced Osteoporosis 237 Pagets Disease of Bone 241 Bone Complications of Renal Disease 246 SECTION 8

Muscle Diseases

47. Steroid myopathy 253 48. Inammatory Myopathies 260 49. Muscle Complications of Statin Therapy 265 General index 271

Abbreviations
ABD ACE ACR ADAMTS ADFR adynamic bone disease angiotensin-converting enzyme American College of Rheumatology a disintegrin and metalloproteinase with thrombospondin motif Activate, Decrease osteoclast activity, Free of treatment and Repeat adenosine diphosphate adverse drug reaction adenosine monophosphate antinuclear antibody anti-neutrophil cytoplasmic antibodies antinuclear factor alkaline phosphatase activator protein-1 Adenomatous Polyp Prevention on Vioxx study antiphospholipid syndrome ankylosing spondylitis apoptosis-associated speck-like protein adenosine triphosphate parvovirus B19 British Ankylosing Spondylitis Metrology Index bone mineral density body mass index blood pressure bisphosphonates fth cervical segment cytoplasmic anti-neutrophil cytoplasmic antibody calcium channel blocker cytosine-cytosine-thymine-guanine monocyte chemoattractant protein1 (see also MCP-1) cyclic citrullinated peptide corneodesmin circulating endothelial precursor cyclic guanosine monophosphate congenital heart block condence interval CIM CK CKD CKD-MBD CLASS Clc-l CMC CNS CORE COX COX-1 COX-2 CPEO CPPD CREST critical iIlness myopathy creatine kinase chronic kidney disease CKD-mineral and bone disorder Celecoxib Long-term Arthritis Safety Study chloride channel carpometacarpophalangeal central nervous system Continuing Outcomes Relevant to Evista cyclooxygenase cyclooxygenase-1 cyclooxygenase-2 Chronic Progressive External Ophthalmoplegia calcium pyrophosphate dihydrate Calcinosis; Raynauds phenomenon; Esophageal dysmotility; Sclerodactyly, Telangiectasia C-reactive protein ChurgStrauss syndrome computed tomography cytosine-thymine-guanine connective tissue growth factor carpal tunnel syndrome cytotoxic lymphocyte-associated antigen linked to immunoglobulin cardiovascular disease chest X-ray 1,25-dihydroxy-vitamin D3 dendritic cell Dupuytrens disease dual-energy X-ray absorptiometry docosahexaenoic acid dehydroepiandrosterone drug-induced lupus distal interphalangeal diffuse idiopathic skeletal hyperostosis diffusing capacity for carbon monoxide dermatomyositis myotonic dystrophy type 1

ADP ADR AMP ANA ANCA ANF AP AP-1 APPROVe APS AS ASC ATP B19 BASMI BMD BMI BP BPs C5 c-ANCA CCB CCTG CCL2 CCP CDSN CEP cGMP CHB CI

CRP CSS CT CTG CTGF CTS CTLA4-Ig CVD CXR D3 DC DD DEXA DHA DHEA DIL DIP DISH DLCO DM DM1

viii
DM2 DMARD DMOAD DMPK dsDNA EBV EDTA EEG EGF eGFR ELISA

Abbreviations
myotonic dystrophy type 2 disease-modifying antirheumatic drug disease-modifying osteoarthritis drug myotonic dystrophy protein kinase double-stranded DNA EpsteinBarr virus ethylenediaminetetraacetic acid electroencephalogram epidermal growth factor estimated glomerular ltration rate enzyme-linked immunosorbent assay electromyography extractable nuclear antigen endothelial nitric oxide synthase eicosapentaenoic acid erythrocyte sedimentation rate endothelin fatty acid full blood count (18)-F-uorodeoxyglucose-positron emission tomography broblast growth factor 12 kDa FK506-binding protein bromyalgia syndrome forced vital capacity follicle-stimulating hormone Glucosamine/chondroitin Arthritis Intervention Trial giant cell arteritis gestational diabetes glomerular ltration rate gastrointestinal guanosine monophosphate glycogen storage disease guanosine triphosphate graft-versus-host disease histamine H2 receptor antagonist glycosylated haemoglobin hyperbaric oxygen high-density lipoprotein Haemolytic anaemia, Elevated Liver enzymes, Low Platelets human immunodeciency virus human leukocyte antigen (genetic designation for human major histocompatibility complex) hereditary neuropathy with liability to pressure palsies hnRNP HPRT HRCT HRT HSP HTLV-1 IBD IBM IFN Ig IGF-1 I IL IL-1ra IMPDH IMT INR IP IU JSN LBP LDL LFA-1 LFT LIFE LJM LORA LRP-5 LUMINA LH MCP MCP-1 MCTD MELAS MERRF MI MMF MMP MORE MPA heterogeneous nuclear ribonucleoprotein hypoxanthine phosphoribosyltransferase high-resolution computed tomography hormone replacement therapy Henoch-Schnlein purpura human T-lymphotropic virus type 1 inammatory bowel disease inclusion body myositis interferon immunoglobulin insulin-like growth factor-1 inhibitor of kappa-beta interleukin interleukin-1 receptor antagonist inosine monophosphate dehydrogenase intima-media thickness International Normalized Ratio inammatory polyarthritis International Units joint space narrowing low back pain low-density lipoprotein lymphocyte function-associated antigen-1 liver function test Losartan Intervention for Endpoint reduction limited joint mobility late-onset RA LDL receptor-related protein-5 Lupus in minorities: nature versus nurture luteinizing hormone metacarpophalangeal monocyte chemoattractant protein1 (see also CCL2) mixed connective tissue disease Myopathy, Encephalopathy, Lactic Acidosis and Stroke Myoclonic Epilepsy with Ragged Red Fibres myocardial infarction mycophenolate mofetil matrix metalloproteinase Multiple Outcome of Raloxifene Evaluation microscopic polyangiitis

EMG ENA eNOS EPA ESR ET FA FBC FDG-PET FGF FKBP-12 FMS FVC FSH GAIT GCA GDM GFR GI GMP GSD GTP GVHD H2RA HBA1C HBO2 HDL HELLP HIV HLA

HNPP

Abbreviations
MRI MRSA MSA MTOR MTP MUA NALP magnetic resonance imaging methicillin-resistant Staphylococcus aureus myositis-specic antibodies mammalian target of rapamycin metatarsophalangeal manipulation under anaesthesia pyrin domain-containing proteins sharing structural homology with NODs nerve conduction studies nuclear factor of activated T lymphocytes nuclear factor--beta National Health and Nutrition Examination Survey National Institutes of Health nitric oxide nucleotide-binding and oligomerization domain proteins nitric oxide synthase inducible nitric oxide synthase endothelial nitric oxide synthase (eNOS) non-steroidal anti-inammatory drug osteoarthritis oral contraceptive pill 25-hydroxy-vitamin D osteoprotegerin odds ratio partial androgen deciency in aging men peptidylarginine deaminase pulmonary artery hypertension polyarteritis nodosa perinuclear anti-neutrophil cytoplasmic antibody polymerase chain reaction plasma procalcitonin platelet-derived growth factor positron emission tomography prostaglandin prostacyclin proximal interphalangeal polymyositis polymyositis/dermatomyositis polymyalgia rheumatica pyrophosphate peroxisomal proliferator-activated receptor PPI PPRP PRIMO PsA PTH PTNP22 PUFAs QALY RA RANK RANKL RCT REM RF RISC RNA RNP ROD ROS RR RS3PE RUTH SAPHO SE SELENA SERM SHBG SI sIL-6R SJC SLC22A4 SLE Sm SOBOE SOTI SPARC SPECT SRP SRRR SS SSc ssDNA

ix
proton pump inhibitor 5phosphoribosyl 1-pyrophosphate Prediction of Muscular Risk in Observational conditions psoriatic arthritis parathyroid hormone protein tyrosine phosphate nonreceptor type 22 polyunsaturated fatty acids quality-adjusted life year rheumatoid arthritis receptor activator of NF-B receptor activator of NF-B ligand randomized controlled trial rapid eye movement rheumatoid factor RNA-induced silencing complex ribonucleic acid ribonucleoprotein renal osteodystrophy reactive oxygen species relative risk remitting seronegative symmetric synovitis with pitting oedema Raloxifene Use for The Heart Synovitis, Acne, Pustulosis, Hyperostosis and Osteitis shared epitope Safety of Estrogens in Lupus Erythematosus National Assessment selective oestrogen receptor modulator sex hormone binding globulin sacroiliac soluble receptor for IL-6 swollen joint count solute carrier family 22 A4 systemic lupus erythematosus Smith antigen shortness of breath on exertion Spinal Osteoporosis Therapeutic Intervention secreted protein acidic and rich in cysteine single photon emission computed tomography signal recognition particle sibling recurrence risk ratio Sjgrens syndrome systemic sclerosis single-stranded DNA

NCS NFAT NF-B NHANES NIH NO NOD NOS NOS-2 NOS-3 NSAID OA OCP 25(OH)D OPG OR PADAM PADI PAH PAN p-ANCA PCR PCT PDGF PET PG PGI2 PIP PM PM/DM PMR PP PPAR

x
STAT1 sTNFR SSRI TB TBF TGF- Th1 Th2 TIMP TJC TLR TKA TMV TNF TNFR2 TRAP

Abbreviations
signal transducer and activator of transcription-1 soluble receptor for TNF selective serotonin reuptake inhibitor tuberculosis thermal biofeedback transforming growth factor- T helper 1 cells T helper 2 cells tissue inhibitor of metalloproteinase tender joint count Toll-like receptor total knee arthroplasty turnover, mineralization and volume tumour necrosis factor TNF- receptor type 2 tartrate-resistant acid phosphatase TROPOS TSH TxA2 U1RNP UA U/E UDP UK US UV VDR VEGF VIGOR WBC WHO WOMAC XO Treatment Of Peripheral Osteoporosis Study thyroid-stimulating hormone thromboxane A2 uracil-rich 1 ribonucleoprotein uric acid urea and electrolytes uridine diphosphate United Kingdom United States ultraviolet light vitamin D receptor vascular endothelial growth factor Vioxx Gastrointestinal Outcomes Research study white blood cell World Health Organization Western Ontario and McMaster Universities xanthine oxidase

S E C T I O N

O N E

01

General Rheumatology and Soft Tissue Rheumatism


01 02 03 04 05 06 07 New onset painful joints An acutely swollen/hot joint Painful shoulders rotator cuff and frozen shoulder Tennis elbow and golfers elbow Carpal tunnel syndrome and other entrapment neuropathies Fibromyalgia syndrome Plantar fasciitis

P R O B L E M

01 New Onset Painful Joints

Case History
June is a 32-year-old tour guide with an eight-week history of painful stiff hands and difculty walking in the mornings. The symptoms usually last for 90 minutes. For the last six weeks she has been using diclofenac 50 mg bd with moderate benet. Her mother has rheumatoid arthritis treated with methotrexate. What additional history will help to determine a diagnosis? What is the relevance of her family history? What aspects of the examination will be particularly relevant? Which investigations should be performed?

Atlas Medical Publishing Ltd

01 General Rheumatology and Soft Tissue Rheumatism

Background
History
Obtaining a clear history of Junes symptoms will assist greatly in narrowing your initial differential diagnosis as a prelude to examination and investigations. Open questions that encourage the person to start with their initial symptoms provide chronology and the pattern of progression. Gentle prompting can, towards the end of consultation, be supplemented with specic questions. As you listen to the story, you will be assessing the impact of the symptoms on the individuals life and its components of family, work and leisure. Specically: b b b b b Are symptoms related to a musculoskeletal problem? Was there an identied trigger or precipitant? What has been the pattern or progression of symptoms? Are there features of systemic illness or inammatory disease? Has anything helped the problem?

Pain and loss of function are primary presenting symptoms, but do not always coexist. Individuals differ in their descriptors of pain, its intensity and its impact. You will be told when the problem began and where. Is the pain in a joint; in a related joint structure such as tendon, ligament or bursa; or in a bone? What is the nature of the pain; when does it occur; and what is the effect of movement? Malignant pain is usually a dull, deep ache within a bone, occurring at night or when resting. Similar symptoms may occur with Pagets disease or with a fracture. Differentiators of inammatory from non-inammatory/mechanical joint pain are summarized in Table 1.1.
Table 1.1 Differentiators of joint pain
Inammatory pain Non-inammatory/mechanical pain

Pain and stiffness predominant in morning and at end of day Stiffness greater than 30 minutes Symptoms lessen with activity Pain does not improve with rest Localized erythema, swelling, tenderness Systemic features fatigue, weight loss

Short-lived joint stiffness Pain worsens with activity Pain improves with rest

Localization of pain requires clarication as to whether symptoms are recreated by contact or movement in the area, or whether the pain is referred from another site. Referred pain occurs when sensory perception externalizes nociceptive input from the sclerotome or myotome of an affected structure to the relevant dermatome. Table 1.2 shows common referred pain patterns. Onset of symptoms following trauma supports mechanical disruption of a joint, disruption of a joints surrounding capsule and ligaments, or fracture. Less obvious triggers to explore are infections (Table 1.3), vaccinations (Rubella) and recent travel. A tactful approach is required when soliciting information on genitourinary symptoms or a

01 New onset painful joints

Table 1.2 Common presentations of referred pain


Area pain experienced
Shoulder Biceps and lateral upper arm Groin, inner knee Lateral thigh, buttock

Origin of pain
Cervical spine Shoulder and rotator cuff Hip Trochanteric bursa

Table 1.3 Common infections associated with arthritis


Viral
Hepatitis B, C Rubella Parvovirus Arbovirus *

Gastrointestinal
Salmonella typhimurium Shigella exneri Yersinia enterocolitica Campylobacter jejuni

Genitourinary
Chlamydia trachomatis

* Serology should be tested according to exposure.

history of a new sexual partner, as it is not obvious to a patient with arthritis as to why you would be asking such questions. A comprehensive family history is a key part of every clinical history. A familial pattern of a specic diagnosis such as rheumatoid arthritis (RA), ankylosing spondylitis or systemic lupus erythematosus (SLE) highlights that diagnosis, and may also raise related diagnoses that are particularly relevant for seronegative spondyloarthritides such as psoriasis or inammatory bowel disease.

Examination
Examination identies the pattern and number of joints involved and extra-articular features (Table 1.4). Features of inammation are sought: temperature, pulse and blood pressure are measured, and an assessment is made of localized erythema and warmth, tenderness, inammation obscuring the joint margins, and reduced function. You should distinguish monoarthritis from oligoarthritis (4 joints) and polyarthritis (>4 joints), whether these joints are large or small, and whether there is spinal (particularly sacroiliac) involvement. Distal to the wrist and ankle there are at least 56 joints, so that as the number of joints increases, the greater the probability is of involvement of both hands and feet and of the pattern becoming increasingly symmetrical. Fingernails are assessed for pitting or onycholysis suggestive of psoriasis. The scalp, umbilicus, natal cleft and extensor surfaces of knee and elbow should be inspected. The presence of a malar rash or photosensitive rash in a young woman suggests SLE.

Investigations
Investigations serve to: b Conrm or refute a diagnostic possibility

01 General Rheumatology and Soft Tissue Rheumatism

Table 1.4 Patterns of arthritis


Pattern Monoarthritis Inammatory spinal disease Sacroiliitis Ankylosing spondylitis Psoriatic arthritis IBD Asymmetrical large joint arthritis Psoriatic arthritis Reactive arthritis IBD Symmetrical small joint arthritis (MCP, PIP, MTP) RA SLE Psoriatic arthritis DIP hands

Differential diagnosis

Trauma Haemophilia Septic Gout Pseudogout

Inammatory OA (if involves PIP and 1st CMC) Psoriatic arthritis

Further X-ray investigations Aspirate for crystals and culture

Review personal and family history HLA-B27

X-ray lumbar spine and SI joints

Review personal and family history Examine for conjunctivitis and urethritis, and scalp and buttocks for psoriasis Infection screen

Examine X-ray hands rheumatoid nodules Skin rashes, serositis or mucositis Urinalysis RF, CCP antibodies, ANA X-ray hands and feet

ANA, antinuclear antibodies; CCP, cyclic citrullinated peptides; CMC, carpometacarpophalangeal; DIP, distal interphalangeal; IBD, inammatory bowel disease; MCP, metacarpophalangeal; MTP, metatarsophalangeal; OA, osteoarthritis; PIP, proximal interphalangeal; RA, rheumatoid arthritis; RF, rheumatoid factor; SI, sacroiliac; SLE, systemic lupus erythematosus.

b Monitor for known complications of the disease process or proposed treatment b Document a parameter that changes with disease activity or treatment The latter includes the inammatory markers erythrocyte sedimentation rate (ESR) and C-reactive protein (CRP), which are non-specic markers. Whenever the possibility of a septic joint is considered, obtaining aspirate and culture from the joint is mandatory. Aspirated uid is collected into a sterile container and an ethylenediaminetetraacetic acid (EDTA)-containing tube to enable a cell count, and is sent with a request for Gram staining, polarized light microscopy, culture and sensitivity, and cell count and differential cell count. If there will be a signicant delay in the sample reaching the laboratory, uid can be inoculated into a blood culture system. The early signs and symptoms of RA are not always typical. RA is characterized as autoimmune partly on the basis of the presence of rheumatoid factor (RF), an autoantibody (usually immunoglobulin M [IgM]) targeting the Fc portion of IgG. Its sensitivity is low, ranging from 60%80%, and specicity is lower, the antibody being frequently present in other connective tissue diseases, which limits the diagnostic utility.

Recent Developments
1 RF is present in 70% of RA cases but is not specic, occurring in 5% of healthy individuals, and globally is more associated with chronic infection than rheumatic diseases. Non-RF antibodies were rst described in the 1960s, with the target

01 New onset painful joints

epitopes now identied as citrulline residues, which are arginine residues modied by peptidylarginine deaminase (PADI). Assays are now available for the detection of antibodies to cyclic citrullinated peptides (anti-CCP antibodies), which are highly sensitive and specic for RA and are a poor prognostic marker of joint erosion, vasculitis and rheumatoid nodules.1 The specicity of anti-CCP in RA is >90% with sensitivity of 33%87%. When combined with IgM-RF, antiCCP has positive predictive value of >90% for RA.2 A study of undifferentiated polyarthritis found that 93% of subjects positive for anti-CCP at rst clinic visit progressed to RA compared to 25% who were anti-CCP negative.3 2 Smoking increases the risk of RA 24 fold and also inuences the manifestations of the disease with increased RF positivity and erosive disease, nodularity and vasculitis similar to the ndings noted with anti-CCP antibodies. Smoking may break immune tolerance by creating neo-epitopes on IgG and thus leading to RF development. Recent work has shown that smoking is associated with increased citrullination. The subsequent citrullinated antigens bind with more afnity to the HLA-DR4 shared epitope subtypes, leading to increased risk of RA.4

Conclusion
Persistent arthropathy in a younger patient necessitates both accurate diagnosis and effective management. A working knowledge of local infectious triggers is required, with supplemental knowledge of the likely pathologies based on age and gender. History and examination need to include potential exposure to infectious triggers, along with personal and family history. Examination will conrm or exclude signicant joint inammation, and provide information on its pattern and severity (number of joints and functional impact). Targeted investigations will narrow the diagnosis, with the urgent investigation being exclusion of septic arthritis if there is clinical suspicion.

Further Reading
1 2 3 Mimori T. Clinical signicance of anti-CCP antibodies in rheumatoid arthritis. Intern Med 2005; 44: 11226. Schellekens GA, Visser H, De Jong BAW et al. The diagnostic properties of rheumatoid arthritis antibodies recognizing a cyclic citrullinated peptide. Arthritis Rheum 2000; 43: 15563. van Gaalen FA, Linn-Rasker SP, van Venrooij WJ et al. Autoantibodies to cyclic citrullinated peptides predict progression to rheumatoid arthritis in patients with undifferentiated arthritis: a prospective cohort study. Arthritis Rheum 2004; 50: 70915. Gorman JD. Smoking and rheumatoid arthritis: another reason just to say no. Arthritis Rheum 2006; 54: 1013.

01 General Rheumatology and Soft Tissue Rheumatism

P R O B L E M

02 An Acutely Swollen/Hot Joint

Case History
You have been asked to see a 28-year-old man who presents with a 36-hour history of a red and very swollen right knee, upon which he is unable to weight bear. He has been previously well and has no relevant family history. The clinic nurse has recorded his temperature as 37.9C and a random blood glucose is 7.3 mmol/l. What is your preliminary differential diagnosis? What additional history and examination is relevant? What are the key investigations? How should he be managed?

Background
Differential diagnosis
The knee is one of the most common joints affected by monoarthritis, which is fortunate since it is so easy to aspirate. The differential diagnosis of monoarthritis is listed in Table 2.1.
Table 2.1 Differential diagnosis of monoarthritis
Trauma Meniscal or ligamentous tears haemarthrosis Sepsis Gonococcal arthritis, Staphylococcus aureus, penetrating injury, foreign body Reactive arthritis Following gastrointestinal or genitourinary infection Haemophilia Crystal arthritis Gout, pseudogout Inammatory e.g. Rheumatoid, psoriatic

Trauma conjures images of motor vehicle accidents or dramatic tackles in rugby; however, much more mundane twisting injuries or valgus/varus strains when under load are common. A rapidly developing joint swelling within minutes of the injury is suspicious of an anterior cruciate ligament tear with involvement of the blood vessel running along its surface. If internal mechanical derangement is considered possible, then imaging or Atlas Medical Publishing Ltd

02 An acutely swollen/hot joint

Table 2.2 Common errors in diagnosing acute monarthritis


Error
The problem is the joint because the patient has joint pain The presence of intra-articular crystals excludes infection Fever distinguishes infectious causes from other causes A normal serum urate makes gout unlikely, and a high level conrms gout Gram staining and culture of synovial uid are sufcient to exclude infection

Reality
Surrounding soft tissues, including bursitis, may be the source of pain Crystals can be present in a septic joint Fever may be absent in septic monoarthritis, and in the immunocompromised patient. Acute crystal arthritis may cause fever Serum urate is normal for 30% of acute gout attacks, and only 5% of those with hyperuricaemia develop gout each year Fastidious, slow-growing organisms, or fragile organisms, may not be identied in early infection. Liaison with the laboratory is required for specialist media and prolonged incubation

orthopaedic review is warranted. Table 2.2 highlights some common errors in diagnosing acute monoarthritis. The presence of fever suggests infection, and the patient should be questioned and examined to determine the likely source. Septic arthritis is usually exquisitely tender with resistance to joint movement. Staphylococci are the most common cause of musculoskeletal sepsis, with the prevalence of both streptococcal and mycobacterial infection increasing. For infections with staphylococci, streptococci, Gram-negative bacteria and anaerobes, only one joint is usually involved. Polyarticular involvement is more likely in the elderly or immunosuppressed, with infection by Haemophilus inuenza, meningococci and Neisseria gonorrhoeae. Lyme disease can present with knee involvement, although the diagnosis can be quickly excluded if there has been no exposure to the tick vector of Borrelia burgdorferi. In young patients, gonococcal arthritis is the most common non-traumatic acute monoarthritis, and questioning regarding sexual partners and genitourinary symptoms is necessary. In addition to arthritis (often polyarticular), tenosynovitis and a pustular rash of the extremities should be sought. Gonococcal arthritis is 34 times more common in women, who often develop arthritis in the perimenstrual period. Men often experience a urethritis as dysuria, and may notice a morning discharge, whereas women may be asymptomatic. Reactive arthritis is a sterile arthritis, occurring distant in both time and place from an inciting infection (usually gastrointestinal or genitourinary). Lower limb asymmetric oligoarthritis is most common, with associated enthesitis such as Achilles tendinitis, and mucocutaneous features of conjunctivitis, pustular rash on the hands and feet and sterile urethritis. Common triggers are genitourinary infection with Chlamydia trachomatis and gastrointestinal infection with Salmonella typhimurium, Shigella exneri, Campylobacter jejuni and Yersinia enterocolitica. Stool culture and collection of early morning urine for detection of chlamydia DNA by polymerase chain reaction (PCR) should be considered. Crystal arthritis is both dramatic and rapid in onset, with the most commonly implicated crystals being uric acid, calcium pyrophosphate and hydroxyapatite. Gout is

01 General Rheumatology and Soft Tissue Rheumatism unusual in the young and is usually preceded by more distal joint involvement, classically the rst metatarsophalangeal joint (podagra). Pseudogout or calcium pyrophosphate dihydrate (CPPD) deposition disease is uncommon below the age of 50 years and the knee is most often involved, followed by wrist and shoulder. Basic calcium phosphate (hydroxyapatite) results in a calcic periarthritis, which most commonly affects the shoulder.

Aspirating a knee joint


Every medical graduate should feel condent to undertake knee aspiration (Figure 2.1). The knee is exposed with the patient lying so that you can obtain access to either the medial or lateral aspect. The knee is generously cleaned with antiseptic and allowed to dry whilst you are preparing the aspiration syringes. The patella is pinched between thumb and index nger at its midpoint, which allows you to detect tension in the quadriceps muscles and also allows you to distract the patella upwards to increase the infrapatellar space. Local anaesthetic (510 ml) is inltrated via a 21G or 23G needle at a point proximal and inferior to where you are holding the patella, noting that the pain-sensitive structures are the dermis and the thickened synovium as you enter the joint. When the anaesthetic has been given time to work, the joint is aspirated along the same needle track with a fresh 1020 ml syringe and 18G needle. Afterwards, a dressing is applied rmly for several minutes to ensure haemostasis and to prevent synovial uid leakage.

Figure 2.1 Arthrocentesis of the left knee medial approach.

Only 12 ml of uid is sufcient to complete all investigations; however, the joint should be aspirated of as much uid as possible without increasing the trauma of the procedure. Substantial pain relief is achieved by aspirating a tense effusion, and while reaccumulation will occur, it buys some time while the preliminary investigation results are received. As the aspirate is removed, you should note its colour, viscosity and turbidity. Normal synovial uid is similar to egg white (syn = resembling, ovium = egg) and is both viscous and acellular. As the degree of inammation increases from the negligible amount found in osteoarthritis to the mid-range of rheumatoid arthritis and the extreme of septic arthritis the viscosity decreases and the cellularity and turbidity increase.

02 An acutely swollen/hot joint

Table 2.3 Synovial uid characteristics


Normal
Colour Clarity Viscosity WBC ( 106/l) Neutrophils
WBC, white blood cell.

Non-inammatory
Straw yellow Transparent High 2002000 <25%

Inammatory
Yellow Hazy opaque Low 200075 000 25%50%

Septic
Variable Opaque LowThick >75 000 >75%

Clear Transparent High 0200 <25%

Blood-coloured effusions suggest either trauma or CPPD deposition disease. Synovial uid characteristics are shown in Table 2.3. It is suggested that approximately 2 ml of uid is collected into a container plus anticoagulant, and the remaining uid collected in a large-volume sterile container. Tests requested should include an urgent Gram stain, cell count and differential count, crystal examination using polarized light microscopy and culture. If gonococcal or fungal infections are suspected, this needs to be highlighted as it inuences the culture medium and length of culture required. Analgesics, antipyretics and rest should be employed in the rst instance, with the aspiration itself often affording a considerable pain relief. If septic arthritis is suspected, then intravenous antibiotics covering Staphylococcus aureus and N. gonorrhoeae should be commenced after the synovial uid aspiration. The presence of bacteria on Gram staining or subsequent bacterial growth requires specialist medical and orthopaedic review to combine antibiotic therapy with joint lavage. Gout is conrmed by the presence of intracellular, negatively birefringent urate crystals, with intracellular pyrophosphate crystals conrming pseudogout. Both of these conditions are self-limited and spontaneously improve over a few days. Adequate hydration combined with analgesia and the introduction of a non-steroidal anti-inammatory drug will generally sufce. Colchicine at a dose sufcient to impact on acute gout invariably causes diarrhoea. If you have conrmed the joint is sterile, then intra-articular corticosteroid injection provides excellent resolution.

Recent Developments
1 A prospective study of children presenting for investigation of possible septic arthritis of the hip concluded that oral temperature >38.5C was the best predictor, followed by an elevated serum C-reactive protein (CRP), an elevated erythrocyte sedimentation rate, refusal to weight bear and an elevated white cell count.2 CRP >20 mg/l was a strong independent risk factor and a valuable tool for assessing and diagnosing septic arthritis of the hip. As the number of risk factors increases so does the predicted probability of septic arthritis, such that three to ve factors present is associated with 83%98% predictive probability.

10

01 General Rheumatology and Soft Tissue Rheumatism 2 Increased plasma procalcitonin (PCT) may be a useful marker for osteomyelitis but not septic arthritis. Procalcitonin is cleaved in neuroendocrine tissues such as thyroid C cells, lung and pancreatic tissue to calcitonin. During infection, large amounts of PCT are released. The source is probably monocytes stimulated by endotoxin, and hepatocytes stimulated by tumour necrosis factor or interleukin-6. The role of PCT measurement with a rapid immunoassay was investigated in children admitted with suspected osteomyelitis or septic arthritis.3 The authors reported specicity of 100% and sensitivity of 58% for osteomyelitis and the same specicity, but lower 27% sensitivity, in septic arthritis. 3 High-resolution magnetic resonance imaging (MRI) of soft tissues and joints is increasingly used prior to interventions such as arthroscopy. In a cohort of children, Luhmann and colleagues4 compared radiological interpretation of knee MRI with that of the surgeon who integrated the history, clinical examination, plain radiographs, MRI scans and radiologist report. The pre-operative diagnosis by the surgeon was better (P <0.05) than the formal radiology interpretation with respect to anterior cruciate ligament tear, lateral meniscal tear, osteochondritis dissecans and discoid lateral meniscus.

Conclusion
An acutely hot, swollen joint is an urgent presentation. Exclusion of sepsis is mandatory, particularly in children and immunocompromised patients. Joint aspiration remains the investigation of choice. Subsequently, treatment will often include antibiotics, pending laboratory results, combined with judicious use of analgesia and anti-inammatory medications. Analysis of synovial uid is valuable in establishing the diagnosis of gout, particularly in joints other than the classical podagra of the great toe. Patients often interpret the doctors it could be gout comment about their sore joint as either a denitive diagnosis or as a slur on an indulgent lifestyle, when neither may be intended.

Further Reading
1 2 Siva C, Velazquez C, Mody A, Brasington R. Diagnosing acute monoarthritis in adults: a practical approach for the family physician. Am Fam Physician 2003; 68: 8390. Caird MS, Flynn JM, Leung YL, Millman JE, DItalia JG, Dormans JP. Factors distinguishing septic arthritis from transient synovitis of the hip in children. A prospective study. J Bone Joint Surg Am 2006; 88: 12517. Butbul-Aviel Y, Koren A, Halevy R, Sakran W. Procalcitonin as a diagnostic aid in osteomyelitis and septic arthritis. Pediatr Emerg Care 2005; 21: 82832. Luhmann SJ, Schootman M, Gordon JE, Wright RW. Magnetic resonance imaging of the knee in children and adolescents. Its role in clinical decision-making. J Bone Joint Surg Am 2005; 87: 497502.

3 4

03 Painful shoulders rotator cuff and frozen shoulder P R O B L E M

11

03 Painful Shoulders Rotator Cuff and Frozen Shoulder

Case History
Mr Lawrence, a 76-year-old retired driver, is having difculty living independently after returning home following a recent myocardial infarction. On the day of discharge he fell heavily, landing on his left upper arm. His concern is a very painful left shoulder, especially at night and when he tries to move his left arm during the day. How would you determine whether he has adhesive capsulitis (frozen shoulder)? Is there a role for medical imaging, and if so, what modality? What treatment should be initiated?

Background
Shoulder pain is an almost unavoidable life experience; in one study, 7% of an adult population aged 2575 years reported at least one months shoulder pain in the previous year. The peak annual incidence of shoulder disorders is in the fourth and fth decades, at a rate of 0.25%. A Dutch study found that 25% of all 85-year-olds in Leiden suffered from chronic shoulder pain and restriction. Community-based surveys concur with this high incidence of soft tissue lesions about the shoulder, with roughly equal sex incidence. Up to 20% of patients with chronic symptoms and 65% of all diagnoses relate to lesions of the rotator cuff. Rotator cuff disease is the most common cause of shoulder pain found in these studies. An ultrasound study found rotator cuff tears in 13% of 5059-year-olds, 20% of 6069-year-olds, 31% of 7079-year-olds and 51% of subjects aged over 80 years, even when they were asymptomatic. Table 3.1 summarizes causes of shoulder pain. The pain-sensitive structures of the shoulder are mainly innervated by the fth cervical segment (C5), so that pain from these structures is referred to the C5 dermatome creating the sensation of pain over the anterior arm, especially the deltoid insertion. The acromioclavicular joint is innervated by the C4 segment pain arising here is felt at the joint itself and radiates over the top of the shoulder into the trapezius muscle and to the side of the neck.

Clinical assessment
A history of trauma, marked night pain and weakness on resisted abduction strongly suggests a rotator cuff tear. The sleeping position that induces night pain is an important clue: Atlas Medical Publishing Ltd

12

01 General Rheumatology and Soft Tissue Rheumatism

Table 3.1 Causes and clinical characteristics of shoulder pain


Category
Extracapsular lesions

Cause

Clinical features

Rotator cuff and subacromial bursa (e.g. impingement Painful arc of abduction syndromes, calcic tendinitis, cuff tears, bursitis) Pain on resisted cuff muscle movements, with intact passive movement (allowing for pain and guarding) Pain on impingement manoeuvres as the inamed rotator cuff tendons impinge on the inferior surface of the acromion and coracoacromial arch Glenohumeral joint (inammatory arthritis RA, spondyloarthritis, pseudogout) Joint capsule (adhesive capsulitis) Bone disease (Pagets disease, metastases) Cervical spine (facet joint root impingement, discitis) Brachial plexus (brachial amyotrophy) Thorax (Pancoasts tumour) Thoracic outlet syndrome Suprascapular nerve entrapment Subdiaphragmatic (abscess, blood, hepatic lesions) Loss of both active and passive movement Reduced glenohumeral range Night pain Muscle strength, allowing for pain, is intact Arm and hand pain with paraesthesia Marked muscle weakness and wasting Neck pain and stiffness Herpes zoster rash Systemic features with weight loss

Intracapsular lesions

Referred pain

shoulder pain that results in awakening when not lying on that shoulder is found in adhesive capsulitis and inammatory arthritis; pain when lying on the affected shoulder is seen in acromioclavicular joint disease and rotator cuff disease. Prior shoulder problems suggest rotator cuff disease with chronic impingement, or calcic tendinitis. A history of marked shoulder joint swelling suggests inammatory arthropathy with the presence of an anterior bulge in the shoulder usually secondary to a subacromial bursa effusion. Glenohumeral osteoarthritis (OA) is characterized by morning stiffness, pain with use and chronicity of symptoms. OA, however, is less common than rotator cuff dysfunction. Examination of the shoulder is best undertaken with the patient wearing the minimum of upper body clothing. The contours of the shoulder are examined for wasting, asymmetry and muscle fasciculation. Palpation should proceed from the sternoclavicular joint along the clavicle to the acromioclavicular joint, to the tip of the acromion and the humeral head beneath the acromion. The shoulder range of movement should be examined both actively and passively, with muscle strength and pain on resistance assessed. There are essentially three movements to test in the shoulder: abduction due to supraspinatus contraction; external rotation as a result of infraspinatus and teres minor movement; and internal rotation due to subscapularis movement (Box 3.1). Box 3.1 Practice Point Three positive clinical tests (supraspinatus weakness, weakness of external rotation and impingement) or two positive results for a patient older than 60 years are highly predictive of a rotator cuff tear.1 Complete rotator cuff tears will show no active abduction but near full-range movement when passively moved. During examination ask about a painful arc during abduction (Figure 3.1). When examining active and passive abduction you should stand behind the patient and place one hand over the shoulder and scapula. The scapula should not begin to

03 Painful shoulders rotator cuff and frozen shoulder

13

Painful arc of abduction acromioclavicular joint

180

Painful arc of abduction in rotator cuff 120

70

Figure 3.1 Painful arc: the patient slowly abducts the arm as high as possible, describing symptoms as the arm rises.

elevate or rotate until at least 90 degrees of abduction has been reached. Early scapulothoracic movement localizes the abnormality to the glenohumeral joint or capsule, as seen in frozen shoulder. You should examine external rotation at 0 degrees abduction, with the elbow beside the chest, and if external rotation is absent then a frozen shoulder is likely. Next re-examine both internal and external rotation at 90 degrees abduction; if both are restricted, a frozen shoulder is again likely. Bicipital tendinitis is examined by testing resisted exion at 30 degrees external rotation, and feeling for tenderness in the bicipital groove. Shoulder impingement can be reproduced by internally rotating the arm held exed at 90 degrees and bringing the inamed rotator cuff against the anterior acromion. The empty can test is suggestive of a rotator cuff tear: it shows pain on resisted elevation of the inverted arm held extended at 90 degrees, as if emptying a can of drink.

Rotator cuff disease


The glenohumeral joint is, by virtue of its anatomical shape, inherently unstable, relying on the joint capsule as well as the rotator cuff muscles (supraspinatus, infraspinatus and subscapularis) for additional stability. Impingement of the rotator cuff between the proximal humerus and the acromioclavicular arch may occur from anomalies of the arch (structural impingement) and from instability due to joint hyperlaxity or weak rotator cuff muscles (functional impingement). Coracoacromial arch anomalies may be congenital, dependent on acromial shape. Three shapes have been described at, curved and

14

01 General Rheumatology and Soft Tissue Rheumatism hooked although there is poor inter-observer agreement on identifying the shape. Acquired impingement occurs secondary to osteophytes growing from the acromioclavicular joint or calcication of the acromioclavicular ligament. Impingement occurs when the cuff becomes compressed in the subacromial space as the arm is elevated. As the humeral head rotates, the rotator cuff tendons are compressed between the greater tuberosity of the humerus and the anterior edge of the acromion, the coracoacromial ligament, the under-surface of the acromioclavicular joint and with the reactive inammatory subacromial bursa. In addition to the impingement theory, a vascular theory has been proposed. With the arm at the side, it has been suggested that the supraspinatus tendon has a relative avascular area 1 cm proximal to its insertion at the greater tuberosity, directly beneath the impingement zone. This may be affected by the position of the shoulder and increases with age. However, the infraspinatus tendon has a similar vascular watershed area, suggesting that factors other than vascularity are important. Chronic irritation in the avascular region produces tendinitis, leading to local inammation and further compression. Other causes of tendinitis include trauma, instability and possibly infarction of the cuff in patients with vascular disease. The vascular and impingement theories are not mutually exclusive and it is possible that the high incidence of supraspinatus pathology is the result of impingement in and around a critical zone of vascular supply. With time, wearing and attrition of the cuff leads to impaired action or rupture of the short rotators stabilizing the humeral head into the glenoid fossa, so that the deltoid pulls the humerus against the under-surface of the acromion and a vicious impingement cycle is established. Impingement-caused tears are usually incomplete in the supraspinatus and infraspinatus tendons and complete in the long head of biceps. Complications of impingement include a frozen shoulder, rupture of the rotator cuff tendons or long head of biceps and, in elderly patients with a long-standing tear, a feared end-stage lesion, recurrent haemorrhagic shoulder of the elderly. Treatment depends on the mechanism of impingement. Patients with functional impingement are treated with a resting sling for 2436 hours, pendular exercises and fulldose non-steroidal anti-inammatory drug (NSAID). Structural impingement is treated similarly but the surgical options of arthroscopic surgery to remove osteophytes or trim the acromion are available. Corticosteroid injection to the subacromial space can be combined with an initial 47 days of pendulum exercises and avoidance of abduction prior to a programme of shoulder-strengthening exercises. Infraspinatus strengthening may be important to provide stabilization of the humeral head to prevent superior subluxation on abduction. Studies of eccentric loading exercises have shown promising results, particularly in lesion of the Achilles tendon. Eccentric loading exercises involve a load being applied to a muscle in its contracted position and the muscle is lengthened under the load. In the shoulder, the supraspinatus would be contracted with the arm abducted and under load the arm would slowly return to the side. Exercise programmes require highly motivated people and there is concern that exercises can increase symptoms initially.

Frozen shoulder/adhesive capsulitis


Initially described in 1872, this condition remains as difcult to treat and difcult to explain from the point of pathology as Codman observed in 1934. This common disor-

03 Painful shoulders rotator cuff and frozen shoulder

15

der (2% cumulative risk in an at-risk population annually) is frequently misdiagnosed and is characterized by painful restriction of all shoulder movements, both active and passive, with characteristic restriction in the glenohumeral range. There is marked reduction or absence of shoulder external rotation at 0 degrees abduction, reduction of both internal and external rotation at 90 degrees abduction, as well as prominent restriction of placing the hand behind the back on internal rotation. Frozen shoulder is characterized pathologically by brosis and retraction affecting predominantly the anterior and inferior structures of the glenohumeral joint capsule. Patients usually present in the sixth decade and onset before the age of 40 years is uncommon. Table 3.2 lists the diseases associated with frozen shoulder, diabetes being the most signicant. Diabetes, particularly long-standing insulin-dependent diabetes, is associated with glycosylation of subcutaneous collagen and the development of soft tissue contraction so called diabetic cheiroarthropathy.
Table 3.2 Common disorders associated with frozen shoulder

Acute shoulder trauma and shoulder immobilization Diabetes mellitus Thyroid disease (both hyper- and hypothyroidism) Cardiac disease, particularly after cardiac surgery Neurological disease with loss of consciousness or hemiplegia Pulmonary disease tuberculosis and carcinoma Rotator cuff disease, especially cuff tear Acute glenohumeral joint inammation

Three phases of frozen shoulder are recognized: 1 Painful inammatory phase. Beginning insidiously, with often only a minor injury being recalled, nocturnal awakening pain develops. The pain may be constant and prevents the patient lying on the shoulder. Physiotherapy often aggravates symptoms at this stage and corticosteroid injections are of limited benet. This phase lasts 29 months. 2 Frozen shoulder. With time, the night and rest pain eases, but the shoulder remains frozen. Mean duration is 412 months. 3 Recovery phase. After a mean delay of 526 months, shoulder limitation slowly recovers in the majority of patients towards normal range (usually a 10%30% loss of motion, which is often undetected by the patient). The total duration of symptoms lasts 1242 months, with mean disease duration of 30 months. In 10%20% of patients a contralateral frozen shoulder develops, usually milder than the rst, while the original shoulder is thawing. It is important to educate patients that the condition will spontaneously resolve and the stiffness will greatly reduce. NSAIDs and analgesics are used but there are no randomized controlled trials studying efcacy. A prospective study in frozen shoulder compared exercise within the limits of pain with intensive physiotherapy. Those who performed exercises within the limits of pain had better results, recorded as near-normal painless shoulder movement (64% of patients at 12 months, 89% at 24 months), compared to intensive physiotherapy (63% of patients at

16

01 General Rheumatology and Soft Tissue Rheumatism 24 months).2 An early meta-analysis by Hazleman on the use of intra-articular steroids reported that the outcome depended on the duration of symptoms and hence possible stage of disease. Patients who receive the injection earlier in the course of the disease recover more quickly.3 An extensive meta-analysis by Buchbinder et al. found a benet for glenohumeral intra-articular corticosteroid injection for frozen shoulder compared with placebo.4 For those unable to tolerate the pain and disability of a frozen shoulder, manipulation under anaesthesia (MUA) is a reliable way to improve the range of movement. It is particularly indicated when disability persists after six months of non-operative therapy. More recently, arthroscopic release of the capsule has been advocated as a more controlled release of the capsule than MUA. Arthroscopic release also avoids the complications of MUA such as fracture of the humerus and iatrogenic intra-articular shoulder lesions.5

Imaging
Imaging is undertaken primarily when considering referred pain or a malignant process. In the assessment of rotator cuff disease, no imaging may be required initially, and may only be undertaken subsequently if the clinical progression is not as expected. A plain Xray should then be the initial imaging modality, because if there is marked superior migration of the humeral head, there must be complete rotator cuff disruption. Either magnetic resonance imaging or ultrasound can conrm a possible full-thickness rotator cuff tear, although ultrasound is signicantly cheaper and is preferred by patients. Suspected partial-thickness tears are best veried with an ultrasound scan.1

Recent Developments
1 Oral steroids may be useful in frozen shoulder, particularly during the early inammatory phase. Buchbinder et al.6 undertook a randomized controlled trial on a series of 50 patients and found that oral steroid therapy initially improved the frozen shoulder but the effect did not last beyond six weeks. Their subsequent analysis of ve small trials, in which all subjects received physiotherapy or an exercise programme, conrmed that oral prednisolone or cortisone when given for 34 weeks had a modest benet on pain and disability and ability to move the shoulder.7 2 Recently, a neural aetiology for tendinopathy has been considered.8 Tendinopathy was proposed as an appropriate term for a symptomatic primary tendon disorder, as it made no assumption as to the underlying pathological process. Underlying the neural theory are four basic observations: tendons are innervated; substance P has been found in rotator cuff tendinopathy and is a pro-inammatory mediator; the neurotransmitter glutamate is also present in tendinopathy; and tendon nerve cell endings are closely associated with mast cells. It has been tentatively postulated that neural stimuli secondary to overuse or mechanical irritation lead to mast cell degranulation and release of mediators that begin an inammatory cascade.

03 Painful shoulders rotator cuff and frozen shoulder

17

Conclusion
Frozen shoulder is a common and painful condition that impacts adversely on an individuals activities of daily living. Despite being self-limited, recovery is protracted and a high proportion of patients do not regain full function. As a condition, it is largely managed in the community by primary physicians, physiotherapists and occupational therapists. Treatments that aim to mechanically stretch or disrupt the joint capsule (MUA, arthroscopic release or hydrodilation of the capsule) are reserved for those with severe symptoms who have failed to progress with conservative therapy.

Further Reading
1 2 Diehr S, Ison D, Jamieson B, Oh R. Clinical inquiries. What is the best way to diagnose a suspected rotator cuff tear? J Fam Pract 2006; 55: 6214. Diercks RL, Stevens M. Gentle thawing of the frozen shoulder: a prospective study of supervised neglect versus intensive physical therapy in seventy-seven patients with frozen shoulder syndrome followed up for two years. J Shoulder Elbow Surg 2004; 13: 499502. Hazleman BD. The painful stiff shoulder. Rheumatol Phys Med 1972: 11: 41321. Buchbinder R, Green S, Youd JM. Corticosteroid injections for shoulder pain. Cochrane Database Syst Rev 2003; CD004016. Dias R, Cutts S, Massoud S. Frozen shoulder. BMJ 2005; 331: 14536. Buchbinder R, Hoving JL, Green S, Hall S, Forbes A, Nash P. Short course prednisolone for adhesive capsulitis (frozen shoulder or stiff painful shoulder): a randomised, double blind, placebo controlled trial. Ann Rheum Dis 2004; 63: 14609. Buchbinder R, Green S, Youd JM, Johnston RV. Oral steroids for adhesive capsulitis. Cochrane Database Syst Rev 2006; CD006189. Rees JD, Wilson AM, Wolman RL. Current concepts in the management of tendon disorders. Rheumatology 2006; 45: 50821.

3 4 5 6

7 8

18

01 General Rheumatology and Soft Tissue Rheumatism

P R O B L E M

04 Tennis Elbow and Golfers Elbow

Case History
Simon is a 48-year-old labourer. His work has required a large amount of manual screwdriver use, and he presents with a three-month history of an increasingly painful elbow. He now has trouble grasping objects such as a cup. What is the difference between tennis elbow and golfers elbow? What are the characteristics of each condition, and what treatment is indicated?

Background
Tennis elbow
Tennis elbow or lateral epicondylitis is an overload injury, which occurs after minor or unrecognized microtrauma to the proximal insertion of the extensor muscles of the forearm particularly extensor carpi radialis brevis. Tennis elbow is the most frequently diagnosed elbow condition (Box 4.1); it occurs commonly in middle life (age 3555 years) and has an incidence in general practice of 47 cases per 1000. Despite its common name, most cases occur in nontennis players and it is frequently a work-related enthesopathy affecting up to 15% of workers in at-risk industries. Operative specimens reveal tendon glycosaminoglycan inltration and microtears, as well as new bone formation at the attachment site. Both traction injury and ischaemia play a role in its development. Flexion deformity is unusual, occurs late and is minimal. Loss of 20 degrees of extension cannot be attributed to tennis elbow and warrants investigation for arthritis, impingement at the olecranon fossa or a soft tissue mass in the posterior aspect of the elbow. Tennis elbow is usually self-limiting, having an average duration of six months to two years, with 90% of subjects recovering within one year. Various conservative interventions exist including pain-relieving medications, corticosteroid injections, physiotherapy, elbow supports, acupuncture, surgery and shockwave therapy (Box 4.2). Box 4.1 Diagnosis of tennis elbow b Lateral elbow pain with tenderness on palpation just distal to the lateral epicondyle b Worsening pain localizing to the lateral epicondyle on resisted wrist dorsiexion b X-rays excluding calcic tendinitis, exostoses and osteoarthritis of the radio-ulnar joint Atlas Medical Publishing Ltd

04 Tennis elbow and golfers elbow

19

Box 4.2 Treatment of tennis elbow b Structured physiotherapy consisting of elbow manipulation and exercise, supplemented with home exercises and self-manipulation b Practical advice booklet on self-management and ergonomics b Recommend avoidance of corticosteroid injections, as short-term benet is offset by a poorer longer-term outcome

Most important in treatment is activities modication both frequency and method of performance. In tennis players, common errors are inadequate conditioning, incorrect grip size, faulty backhand style and problems with the racquet and its stringing. In the work setting, a review by an occupational therapist is recommended, particularly focusing on pronation/supination movements and grip size. A physiotherapy programme that includes strengthening exercises for the entire upper limb and a graded resistive programme for wrist dorsiexors is recommended. In the setting of localized tenderness it is tempting to inject the lesion. As noted in Box 4.2, the short-term gain may be offset by a poorer long-term outcome. The injection technique is a small volume of corticosteroid and local anaesthetic injected into the tendinous insertion of extensor carpi radialis brevis into the lateral epicondyle. As the injection is not into a potential space but into an already tender, dense area, the injection is against resistance and is both uncomfortable and has the risk of steroid tracking supercially to the subcutaneous tissues, leading to depigmentation and atrophy. In contrast to other painful overuse syndromes in which total tendon ruptures have been reported (Achilles, biceps, patella), the tendon of the extensor carpi radialis brevis is strongly connected and supported by other extensors of the wrist. A small number of patients have recalcitrant lateral epicondylitis and are considered for operative intervention open, arthroscopic and percutaneous. Operative interventions followed for a minimum of two years demonstrate an improvement compared to pre-operative status, with no difference in outcome according to procedure technique.1

Golfers elbow
Golfers elbow or medial epicondylitis is the mirror image of tennis elbow, and is thought also to relate to repetitive traction stress and microtears at the insertion of the forearm exors (exor carpi radialis) and pronator teres into the medial epicondyle. It occurs in both professional and amateur sports players, as well as manual workers such as bricklayers. It is much less common than tennis elbow, with approximately one-twentieth the incidence. Similar to tennis elbow, the diagnosis is clinical, with localized tenderness that worsens on resisted wrist exion and forearm pronation (Box 4.3). Box 4.3 Golfers elbow b Elbow pain at the medial epicondyle b Increasing symptoms on resisted wrist exion and resisted forearm pronation b Treatment includes modication of activities, upper limb exercises and analgesics

20

01 General Rheumatology and Soft Tissue Rheumatism

Recent Developments
A randomized controlled trial compared the effectiveness of physiotherapy, corticosteroid injections and a wait and see approach in 198 patients with tennis elbow who were randomized to the three treatment arms.2 Physiotherapy was eight sessions of mobilization with movement and exercises plus home exercises and self-manipulation. Injection therapy with triaminolone acetonide (10 mg) and 1% lidocaine was the second study arm. The wait and see approach consisted of ergonomic instruction and use of analgesics, heat, cold and braces if needed. At six weeks the main outcome measures (global improvement, pain-free grip strength, assessors rating of complaints, severity of elbow pain and elbow disability) were signicantly better in the corticosteroid-treated group than in the other groups. However, all groups were improving and the benet of the steroid injection was short-lived, such that a crossover occurred around twelve weeks, with the one-year results showing physiotherapy superior to corticosteroid injections for all outcome measures. Importantly, at one year, the injection-treated group was signicantly worse on all outcomes compared with the physiotherapy group, and on two out of three measures compared with the wait and see group. The corticosteroid injection group also had the most reported recurrences. A similar study with only seven weeks of follow-up conrmed the benets of steroid injections in the short term.3

Conclusion
Tennis elbow is a common problem in general practice and is best treated with the knowledge that it is a self-limiting condition, with the majority of patients improving in the medium term. Whilst corticosteroid injections offer short-term benet, there is the potential for both short-term adverse effects and the possibility of a worse outcome at one year. Physiotherapy provides benet that is slower in onset but is more sustained and allows patients to become self-reliant in their own management.

Further Reading
1 Szabo SJ, Savoie FH, Field LD, Ramsey JR, Hosemann CD. Tendinosis of the extensor carpi radialis brevis: an evaluation of three methods of operative treatment. J Shoulder Elbow Surg 2006; 15: 7217. Bisset L, Beller E, Jull G, Brooks P, Darnell R, Vicenzino B. Mobilisation with movement and exercise, corticosteroid injection, or wait and see for tennis elbow: randomised trial. BMJ 2007; 333; 93945. Tonks JH, Pai SK, Murali SR. Steroid injection therapy is the best conservative treatment for lateral epicondylitis: a prospective randomised controlled trial. Int J Clin Pract 2007; 61: 2406.

05 Carpal tunnel syndrome and other entrapment neuropathies P R O B L E M

21

05 Carpal Tunnel Syndrome and Other Entrapment Neuropathies

Case History
Beatrix is a 33-year-old production-line worker. For the last four weeks she has been awakening with painful pins and needles in her left hand and a dull pain that radiates from her wrist to her elbow. Shaking the arm improves the symptoms and she sometimes sleeps with her arm hanging out of the bed. What are the clinical features of the carpal tunnel syndrome? What is the role for imaging and nerve conduction studies? What investigations are appropriate to determine the cause? How would you manage this problem?

Background
Entrapment neuropathies are disorders where peripheral nerves are damaged by compression as they pass through a bony or brous canal. The disorders may be precipitated by repetitive motion or strain, and carpal tunnel syndrome (CTS) is by far the commonest entrapment neuropathy and the most common focal peripheral neuropathy. The median nerve, along with the exor tendons, passes through the carpal tunnel, which is bridged by the transverse carpal ligament (Figure 5.1). CTS affects 3% of the population although there is an imperfect correlation between reported symptoms and electrophysiological ndings. Women are three times more likely than men to be affected with CTS, and a number of predisposing conditions are recognized (Box 5.1). CTS causes pain, numbness and tingling in the distribution of the median nerve: i.e. anteriorly, in the lateral half of the ring nger to the median half of the thumb; and posteriorly, in the distal halves of the ring and middle ngers. If severe, the symptoms may radiate up the arm and they can often occur at night, thus disturbing sleep. In severe cases there is a loss of small muscle function, which impairs manual dexterity and can lead to wasting of muscles of the thenar eminence. The symptoms of CTS are common and clinical signs (Box 5.2) are not always present. Accurate diagnosis is one of the major determinants of successful treatment. The diagnosis should be conrmed wherever possible by nerve conduction studies. Atlas Medical Publishing Ltd

22

01 General Rheumatology and Soft Tissue Rheumatism

Median nerve Tendon sheath

Carpal ligament

Bundle of tendons

Figure 5.1 Anatomy of the carpal tunnel.

Other common nerve entrapment syndromes Thoracic outlet syndromes


These are due to compression of the brachial plexus and brachial vessels in the neck. Costoclavicular syndrome, due to a narrowing of the space between the clavicle and rst rib, may arise from congenital abnormality or because of poor posture. Cervical rib syndrome is due either to an extra rib or to a brous band between the seventh cervical vertebra and the sternum. Compression of nerves and vessels occurs as they pass over the additional structures. Adsons test may be positive: the patient looks to the affected side and takes a deep breath while the examiner lifts the arm to 90 degrees. If compression is present, the radial pulse may disappear.

Suprascapular neuritis
The suprascapular nerve (cervical segments C5/C6) supplies sensation to the shoulder joint and motor supply to the infraspinatus and supraspinatus muscles. It can become compressed as it passes through the suprascapular notch and under the transverse ligament.

Ulnar neuritis
Compression of the ulnar nerve usually occurs in the canal, where it is covered by the arcuate ligament. It may also occur between the two heads of exor carpi ulnaris just distal to the elbow joint. The syndrome may occur as a result of direct trauma or fracture, repetitive

05 Carpal tunnel syndrome and other entrapment neuropathies

23

Box 5.1 Causes of CTS Overuse Repetitive exion or extension of the wrist, particularly while gripping objects rmly Use of walking stick in patients with mobility disorders Occupational use of power tools, assembly-line work Colles fracture Subluxation of the lunate bone Rheumatoid tendon sheath inammation Osteoarthritis Gout or pseudogout Outpouching of the wrist joint capsule Pregnancy Obesity Congestive cardiac failure Lipoma Septic arthritis Lyme disease Tuberculosis Diabetes Hypothyroidism Acromegaly Amyloidosis

Injury Arthritis

Wrist ganglion Increased canal volume

Infections

Metabolic

Box 5.2 Clinical signs of CTS Tinels sign Tapping over the median nerve elicits symptoms in the distribution of the nerve Phalens sign Place both hands together palm to palm, with the wrists extended to 90 degrees, and forearms horizontal and close to the chest. The affected hand will begin to tingle within 12 minutes Reverse Phalens As above, but with the hands placed back to back movements or rheumatoid arthritis affecting the elbow joint. It causes pain and tingling down the inside of the forearm to the little nger and medial aspect of the ring nger. The nerve gives rise to a sensory supply to the skin of the hypothenar eminence and a motor supply to muscles of the hypothenar eminence and other small muscles in the hand.

Median neuritis
This is a much less common syndrome and is usually due to entrapment of the nerve at the elbow. Symptoms are similar to those of the carpal tunnel syndrome and may be exacerbated by pronation of the forearm.

Radial neuritis
Again, this is relatively uncommon. Compression usually occurs at the elbow and causes sensory symptoms in the forearm bone to the base of the thumb.

24

01 General Rheumatology and Soft Tissue Rheumatism

Meralgia paraesthetica
The lateral cutaneous nerve of the thigh passes through the femoral canal, where it is sharply angulated and liable to compression. The syndrome leads to sensory symptoms in the middle and lower part of the lateral aspect of the thigh. It is caused by obesity, direct trauma or by repetitive exion of the thigh.

Anterior compartment syndrome


This part of the lower leg contains the tibialis anterior and extensor digitorum muscles and the deep peroneal nerve (supplies skin between the rst and second toes). The nerve may be injured by unaccustomed running, as a result of tibial or bular fractures or through direct trauma.

Medial compartment syndrome


This is the most common lower-leg nerve entrapment syndrome. The symptoms include pain and tenderness on the medial aspect of the shin (shin splints). It is often precipitated by unaccustomed running on a hard surface.

Posterior compartment syndrome


This compartment contains the soleus and gastrocnemius muscles, which join together to form the Achilles tendon and are responsible for plantar exion. The syndrome is associated with calf pain precipitated by exercise and with altered sensation on the sole of the foot. The management of all of these nerve entrapment syndromes is somewhat similar: the patient should rest wherever possible and avoid movements or actions that exacerbate the symptoms; local injection with anaesthetic or steroid is indicated in some cases, and a minority of patients require surgical decompression of the affected nerve.

Management of CTS1,2
b General measures include trying to relax the grip, using grip-adapted implements such as large pens, taking frequent breaks, keeping the hands warm and considering posture and position (e.g. if using a keyboard, this should be at elbow height). Conservative management with ultrasound has been advocated but there are limited trial data to support this therapy. b Splinting the wrist in neutral position may alleviate symptoms related to soft tissue swelling and is most effective when used soon after the onset of symptoms. Nighttime splinting is often sufcient. b Non-steroidal anti-inammatory drugs are effective in some cases, although improvement may be short-lived. Oral corticosteroids are more effective (e.g. prednisolone 20 mg/day for 23 weeks, followed by reducing doses). Diuretics are widely used but are often disappointing in their effect. b The use of local injection of anaesthetic and steroid into the proximal carpal tunnel is supported by trial data. The outcome is probably comparable to that of systemic steroids, but the patient is not exposed to the risk of side effects associated with highdose steroid therapy. The injection may be directly into the carpal tunnel or proximal to the carpal tunnel. Benet from local injection may last for up to three months and is increased by concurrent splinting. b For patients who have either severe symptoms or do not respond to conservative measures, surgery is required. This has traditionally been carried out by an open

05 Carpal tunnel syndrome and other entrapment neuropathies

25

Confirm symptoms are in median nerve distribution

History and examination to search for underlying causes Enquire about occupation and repetitive strain

Mild symptoms

Moderate symptoms with signs

Severe symptoms

Rest Remove precipitating cause Trial of splinting

NCS imaging

No further action

Confirmed diagnosis

Conservative measures

Local injection Systemic steroids

Repeat treatment at 3 months

Consider surgery Open Endoscopic

Figure 5.2 Investigation and management of CTS. Imaging is with high-resolution ultrasound or with MRI. NCS, nerve conduction studies.

procedure, which can be performed without admission to hospital. More recently, endoscopic carpal tunnel release through two small incisions has been used by many surgeons. This has the advantage of causing less scarring.

Recent Developments
1 Not all patients have ready access to nerve conduction studies. Several studies have shown that high-resolution ultrasound and magnetic resonance imaging (MRI) may be very accurate in diagnosing CTS.3,4 These methods can demonstrate altered

26

01 General Rheumatology and Soft Tissue Rheumatism anatomy and decreased volume of the carpal tunnel, and in patients with CTS show the median nerve is swollen distal to the compression. 2 Some familial cases of nerve entrapment are due to inherited anatomical abnormalities. Recently, the condition of hereditary neuropathy with liability to the pressure palsies (HNPP) has been described.5,6 This condition is inherited in an autosomal dominant manner and is due to a deletion at locus 17p11.2. HNPP is a slowly progressive condition, punctuated by episodes of acute peripheral neuropathy at sites that are liable to nerve entrapment. 3 Endoscopic surgery has revolutionized treatment of CTS. The two-portal endoscopic approach to managing CTS has been adopted in many centres. Although this approach is attractive, recent trials7,8 suggest that it has very little to offer over traditional open surgery. In general, surgical treatment is more successful than medical or conservative treatment in patients with proven CTS.9

Conclusion
CTS is the most common form of entrapment neuropathy. Denitive diagnosis is by nerve conduction studies, but ultrasound and MRI are increasingly being used to conrm the diagnosis. It is worth routinely excluding hypothyroidism and diabetes as predisposing causes but there is not usually a treatable or identiable underlying cause. A limited trial of conservative or medical measures is justied in mild cases but surgery is generally required for severe, progressive or unresponsive cases.

Further Reading
1 2 3 4 5 Viera AJ. Management of carpal tunnel syndrome. Am Family Physician 2003; 68: 26572. Ashworth N. Carpal tunnel syndrome. Clin Evid 2006; 15: 118. Wiesler ER, Chloros GD, Cartwright MS, Smith BP, Rushing J, Walker FO. Use of diagnostic ultrasound in carpal tunnel syndrome. J Hand Surg 2006; 31: 72632. de Noordhout AM. Diagnosing entrapment neuropathies: probes and magnets instead of electrodes and needles? Clin Neurophysiol 2006; 117: 4845. Sander MD, Abbasi D, Ferguson AL, Steyers CM, Wang K, Morcuende JA. The prevalence of hereditary neuropathy with liability to pressure palsies in patients with multiple surgically treated entrapment neuropathies. J Hand Surg 2005; 30: 123641. Koc F, Guzel R, Benlidayi IC, Yerdelen D, Guzel I, Sarca Y. A rare genetic disorder in the differential diagnosis of the entrapment neuropathies: hereditary neuropathy with liability to pressure palsies. J Clin Rheumatol 2006; 12: 7882. Rab M, Grunbeck M, Beck H et al. Intra-individual comparison between open and 2-portal endoscopic release in clinically matched bilateral carpal tunnel syndrome. J Plast Reconstr Aesthet Surg 2006; 59: 7306. Atroshi I, Larsson G-U, Ornstein E, Hofer M, Johnsson R, Ranstam J. Outcomes of endoscopic surgery compared with open surgery for carpal tunnel syndrome among employed patients: randomised controlled trial. BMJ 2006; 332: 14736. Hui ACF, Wong S, Leung CH et al. A randomized controlled trial of surgery vs steroid injection for carpal tunnel syndrome. Neurology 2005; 64: 20748.

06 Fibromyalgia syndrome P R O B L E M

27

06 Fibromyalgia Syndrome
Case History
Sandra is in her early 40s and is seeing you because she hurts from her scalp to her toes. This has been present for at least eight years and is ruining her life. She tires easily and aches with any activity. Her sleeping is restless, she awakes tired and she has an irritable bowel. There are no abnormalities on physical examination. What is bromyalgia and how would you support this diagnosis? Are there any investigations that might help? What treatment, if any, would you suggest to Sandra?

Background
Fibromyalgia syndrome (FMS) is a soft tissue musculoskeletal condition with many features in common with chronic fatigue syndrome, the major difference being the predominance of musculoskeletal features in FMS. Diagnosis of FMS is based on the American College of Rheumatology (ACR) criteria (1990): b b b b Pain on both sides of the body Pain above and below the waist Pain in an axial distribution Local tenderness in at least 11 out of 18 dened trigger points (Figure 6.1)

The pain is often dened as aching or burning and varies in intensity and location from day to day. Other features of FMS are shown in Table 6.1.
Table 6.1 Frequency of FMS symptoms
Symptom
Muscular pain Fatigue Insomnia Joint pains Headaches Restless legs

%
100 96 86 72 60 56

Symptom
Paraesthesiae Memory impairment Leg cramps Poor concentration Anxiety Major depression

%
52 46 42 41 32 20

Atlas Medical Publishing Ltd

28

01 General Rheumatology and Soft Tissue Rheumatism

Figure 6.1 Trigger points for the diagnosis of FMS. There are 18 points in total (nine identical locations on each side). Anterior: anterior aspects of C5, C6 and C7; second rib; lateral epicondyle; knee (medial fat pad). Posterior: suboccipital muscle insertions; supraspinatus muscle origin; trapezius (midpoint upper border); gluteal (upper outer quadrants); greater trochanter. Adapted with permission from Borg-Stein 2006.1

Musculoskeletal pain is the most consistent feature of FMS. Fatigue can be almost as debilitating. Disordered sleep is also a very frequent feature and contributes to fatigue and to the mood disturbances. Sleep abnormalities are strongly correlated with the alpha-electroencephalogram (EEG) abnormality and movement disorders including the periodic jerking of arms and legs, teeth grinding (bruxism) and restless legs. Gastrooesophageal reux disease occurs with high frequency, as does irritable bowel syndrome. Headaches may be of the migraine or tension type. Facial pain is also relatively common, including discomfort related to temporomandibular joint dysfunction. Psychological and psychiatric morbidity are increased. There is high prevalence of anxiety disorders including obsessive-compulsive disorder and post-traumatic stress disorder.2

06 Fibromyalgia syndrome

29

Epidemiology and aetiology


A number of recent studies24 have examined incidence and prevalence of FMS. The estimated prevalence is between 1% and 4%. FMS is between two and six times more likely to occur in women. Incidence in the female population has been estimated at 11.3 per 1000 person-years. It can occur at any age but becomes more common with advancing years. FMS has been associated with other rheumatic disorders including rheumatoid arthritis (RA) and systemic lupus erythematosus (SLE). There is clearly strong interplay between physical and psychological factors in FMS. The onset of illness may be triggered by physical illness (including viral diseases) or by trauma (including surgery). There is some suggestion that heredity may play a part, with components of the serotonergic and dopaminergic systems being potential candidates for involvement. Some of the symptomatology around the trigger points may be due to increased acetylcholine at the motor endplate causing contraction and shortening of the sarcomere. This may lead to increased energy consumption and increased local blood supply, with resulting local tenderness. A number of local and systemic mediators have been implicated. These include bradykinin, calcitonin gene-related peptide, substance P, tumour necrosis factor-a, interleukin-1, noradrenaline and serotonin.

Investigations
Routine investigations including full blood count and biochemistry, plus erythrocyte sedimentation rate (ESR), C-reactive protein (CRP) and other inammatory markers are within the normal range. Because thyroid disease is common, it is useful to include thyroid function tests. There are no specic endocrine abnormalities. X-ray, computed tomography (CT) and magnetic resonance imaging (MRI) scans are generally normal. There are no specic abnormalities on muscle biopsy, electromyography or nerve conduction studies. EEG or more formal sleep studies may be requested in patients who have marked sleep disturbance. This may reveal abnormalities including periodic limb movement disorder, rapid eye movement (REM) sleep disorder or sleep apnoea. The diagnosis of FMS is one of exclusion and is made clinically.

Prognosis, differential diagnosis and treatment


The outlook for FMS is variable and the condition tends to become chronic. However, more widespread understanding and clearer denition, along with a more highly developed treatment ow, are beginning to streamline management and improve the outlook. There is a danger that over-enthusiastic investigation might contribute to making the condition more chronic. However, this should not deter the clinician from making a full investigation and the clinical picture warrants it. The major differential diagnosis is other connective tissue disorders, including rheumatoid disease, SLE and scleroderma. Major differential diagnoses of FMS and investigation of the condition are summarized in Figure 6.2. There is no specic treatment for FMS. Therapeutic measures include the following: b General: investigation and clear diagnosis; educating the patient as to the nature of the diagnosis and reassuring them; attention to psychological and social factors, and encouraging the patient to have a normal sleep pattern as well as to engage in physical activity consistent with their state of health and preferences.

30

01 General Rheumatology and Soft Tissue Rheumatism

Musculoskeletal pain

Affecting the axial skeleton Both sides of the body Above and below the waist At least 11/18 trigger points tender

Associated symptoms

Joint swelling or deformity

? Referred pain Visceral Skeletal

Limited Ix FBC ESR CRP FT4 ve TSH ve Classic FMS (clinical diagnosis)

ve

Further Ix RF ANF Viral serology ve

Imaging X-rays CT scan MRI scan

ve

Consider: RA Scleroderma SLE Lyme disease

Figure 6.2 Investigation of suspected FMS. - ve, negative; + ve, positive; ANF, antinuclear factor ; CRP, C-reactive protein; CT, computed tomography; ESR, erythrocyte sedimentation rate; FBC, full blood count; FMS, bromyalgia syndrome; FT4, thyroxine; Ix, investigation; MRI, magnetic resonance imaging; RA, rheumatoid arthritis; RF, rheumatoid factor; SLE, systemic lupus erythematosus; TSH, thyroid stimulating hormone. Adapted from Schneider et al. 2006.5

b Pain relief. This may range from simple analgesics such as paracetamol to more powerful agents. Inappropriate use of powerful opioid analgesics should be avoided as this may lead to dependence and seldom alleviates the symptoms in the long term. Non-steroidal anti-inammatory drugs have marginal benets over simple analgesics. Tramadol is a weak opioid with some action to inhibit the uptake of serotonin. b Antidepressants. Either tricyclic antidepressants or selective serotonin reuptake inhibitors are of benet in many cases, even when there is not major evidence of depression. Dual inhibitors of both noradrenaline and serotonin uptake, such as duloxetine, may be of particular benet. b a2-adrenergic agonists, such as clonidine or tizanidine, are of benet to some patients and may act by preventing the action of neurotransmitters such as glutamate or substance P within the central nervous system. b Anticonvulsants may contribute to pain relief. There is increasing experience with newer agents such as pregabalin or gabapentin, which have proved to be of use in other painful neurological conditions.

06 Fibromyalgia syndrome

31

b Trigger point injection may be undertaken with local anaesthetic, steroid or botulinum toxin. The latter may be more effective than steroid injection; it may act by diminishing acetylcholine release, thus decreasing muscle activity and local ischaemia. b Postural training, exercise and ergonomic adjustments (particularly in the workplace) may help a patient to adapt to disability associated with FMS. b Stress reduction may be achieved by a variety of means including cognitive behavioural therapy, relaxation techniques and biofeedback methods. b Physical therapies including acupuncture, massage, transcutaneous electrical nerve stimulation and ultrasound may be helpful. These should always be performed by a suitably qualied and reputable practitioner.

Recent Developments
1 There is some discordance between the clinical diagnosis of FMS and the diagnosis using ACR criteria.6 In a survey of 206 patients, FMS was diagnosed in 49.0% while only 29.1% met ACR criteria. The authors of the report proposed a survey method for diagnosis of FMS that did not require clinical examination. 2 Improved understanding of the pathophysiological basis for the syndrome is not only lending credence to the diagnosis, but is also giving rise to more logical management.7 Patients are often mislabelled as having a purely psychological diagnosis, leading to the undertreatment or mismanagement of the physical symptoms. Increased activity of the hypothalamicpituitaryadrenal axis and the sympathetic nervous system has been proposed to be responsible for some of the clinical features.8 3 More than 90% of patients have tried alternative or complementary treatments. Acupuncture has been widely used but not intensively studied. A recent randomized controlled trial of acupuncture in FMS9 reported greater improvements in the treatment group compared with the control group. Symptoms of fatigue and anxiety were particularly improved and the treatment is very well tolerated. These patients also experienced better pain relief. 4 Being overweight or obese is associated with a variety of physical and psychological symptoms, including many of the symptoms that form part of the FMS complex. Results of the recently published behavioural weight loss programme10 conrm that modest weight loss is associated with an improvement in FMS symptoms.

Conclusion
Attempts should be made to establish the diagnosis in the above patient. This should include the exclusion of thyroid disease and connective tissue disorders. Thorough medical assessment should be undertaken, with care not to increase the patients anxiety. Once the diagnosis is established, this should be carefully explained to the patient and a realistic management plan should be agreed. This should include trying to manage pain and sleep disturbance with the minimum of pharmacological intervention. Physical activity should be encouraged within the limits imposed by her condition. Weight loss is

32

01 General Rheumatology and Soft Tissue Rheumatism important in the overweight patient for future health and may also help to alleviate the symptoms of FMS.

Further Reading
1 Borg-Stein J. Treatment of bromyalgia, myofascial pain, and related disorders. Phys Med Rehabil Clin N Am 2006; 17: 491510. 2 Raphael KG, Janal MN, Nayak S, Schwartz JE, Gallagher RM. Psychiatric comorbidities in a community sample of women with bromyalgia. Pain 2006; 124: 11725. 3 Weir PT, Harlan GA, Nkoy FL et al. The incidence of bromyalgia and its associated comorbidities: a population-based retrospective cohort study based on International Classication of Diseases 9th Revision codes. J Clin Rheumatol 2006; 12: 1248. 4 McNally JD, Matheson DA, Bakowsky VS. The epidemiology of self-reported bromyalgia in Canada. Chronic Dis Can 2006; 27: 916. 5 Schneider MJ, Brady DM, Perle SM. Commentary: differential diagnosis of bromyalgia syndrome: proposal of a model and algorithm for patients presenting with the primary symptom of chronic widespread pain. J Manipulative Physiol Ther 2006; 29: 493501. 6 Katz RS, Wolfe F, Michaud K. Fibromyalgia diagnosis: a comparison of clinical, survey, and American College of Rheumatology criteria. Arthritis Rheum 2006; 54: 16976. 7 Dadabhoy D, Clauw DJ. Fibromyalgia: progress in diagnosis and treatment. Curr Pain Headache Rep 2005; 9: 399404. 8 Sarzi-Puttini P, Atzeni F, Diana A, Doria A, Furlan R. Increased neural sympathetic activation in bromyalgia syndrome. Ann NY Acad Sci 2006; 1069: 10917. 9 Martin DP, Sletten CD, Williams BA, Berger IH. Improvement in bromyalgia symptoms with acupuncture: results of a randomized controlled trial. Mayo Clin Proc 2006; 81: 74957. 10 Shapiro JR, Anderson DA, Danoff-Burg S. A pilot study of the effects of behavioural weight loss treatment on bromyalgia symptoms. J Psychsom Res 2005; 59: 27582.

07 Plantar fasciitis P R O B L E M

33

07 Plantar Fasciitis

Case History
Shirley is aged 43 years and presents with exquisite pain beneath her left heel when walking. She is moderately obese (body mass index 32 kg/m2) and nds it hard to walk rst thing in the morning. She prefers to wear open sandals. Should Shirley have X-rays to determine whether she has a plantar spur? What effect, if any, does Shirleys weight have on her condition? What are the current supported therapies?

Background
Plantar fasciitis commonly causes inferior heel pain and occurs in up to 10% of the United States population. It affects both active and sedentary adults of all ages, but is more likely to occur in persons who are obese, who spend most of the day on their feet or who have limited ankle dorsiexion. Plantar fasciitis is a musculoskeletal disorder primarily affecting the fascial enthesis. Although poorly understood, development of plantar fasciitis is thought to have a mechanical origin. In particular, pes planus foot types and lower-limb biomechanics that result in a lowered medial longitudinal arch create excessive tensile strain within the fascia, producing microscopic tears. The roles of both chronic inammation and arch mechanics in the aetiology of plantar fasciitis are controversial.1 Diagnosis is based on the history and physical examination (Table 7.1); the differential diagnosis is listed in Table 7.2. Unaccustomed walking in the sedentary or prolonged running in the athlete may induce fatigue tears of the plantar fascia and avulsion fracture may cause pain at the medial calcaneal tuberosity. The same area is involved in spondyloarthropathy (ankylosing spondylitis, psoriatic arthritis and reactive arthritis) as plantar

Table 7.1 Diagnosis of plantar fasciitis2

Chronic inferior heel pain on weight bearing: throbbing, searing, piercing in character Pain worst with the rst steps in the morning or after rest Pain reduces after mobilization, to recur with continued activity Walking barefoot, on toes or up stairs exacerbates pain Tenderness around medial calcaneal tuberosity at the plantar aponeurosis Bilateral plantar fasciitis is highly suggestive of spondyloarthropathy

Atlas Medical Publishing Ltd

34

01 General Rheumatology and Soft Tissue Rheumatism

Table 7.2 Common causes of heel pain2,3

Posterior calcaneal pain Supercial bursitis Achilles tendonitis Retrocalcaneal bursitis Calcaneal epiphysitis (Severs disease) in adolescents Plantar calcaneal pain Central: fat pad atrophy (primary, or secondary to corticosteroid inltration), obesity Medial: compression neuropathy of nerve to abductor digiti minimi and medial calcaneal branch of the posterior tibial nerve; posterior tibialis tendonitis Medial central: plantar fasciitis, avulsion fractures, fascial microtears Lateral: calcaneal stress fracture, calcaneal cysts

fascia enthesitis. Insertional plantar fascia pain is typically increased by passive dorsiexion of the big toe and is located centromedially at the medial calcaneal tuberosity. Heel pain may be caused by an entrapment neuropathy of the calcaneal branches of the posterior tibial nerve or the rst branch of the lateral plantar nerve. Since encroachment occurs between the abductor hallucis fascia and the quadratus plantae muscle, maximal tenderness is in the medial border of the heel. Diagnostic imaging does not contribute unless another diagnosis is strongly suspected, such as calcaneal stress fracture. The latter may be diagnosed by plain X-ray or by isotope bone scanning for conrmation. Radiography may show calcications in the soft tissues around the heel, or osteophytes on the anterior calcaneus (i.e. heel spurs). Fifty per cent of patients with plantar fasciitis and 20% of persons without plantar fasciitis have heel spurs. Plantar fasciitis is ve times more likely to occur in obese individuals. Although the link between obesity and plantar heel pain is poorly understood, research to date has focused on the impact of adiposity of the subcalcaneal fat pad and the function of the medial longitudinal arch.4 The plantar fascia is the primary structure stabilizing the medial longitudinal arch of the foot. Both abnormal arch structure and movement have been implicated in the development of plantar fasciitis. In particular, pes planus or
Table 7.3 Therapeutic recommendations2
Clinical recommendation
Off-the-shelf (non-magnetic) insoles Custom-made insoles are not more effective than fabricated insoles Plantar fascia stretching more effective than calf stretching and should be recommended to all patients Corticosteroid iontophoresis should be considered for short-term relief if initial therapy fails Custom-made night splints Extracorporeal shock therapy is not effective Walking casts in those who have failed conservative therapy Open or endoscopic surgery for those who have failed all conservative measures
B, inconsistent or limited quality evidence; C, consensus, usual practice or expert opinion.

Evidence rating
B B B B B B C B

07 Plantar fasciitis

35

biomechanics that result in a lower longitudinal arch (i.e. foot pronation) increase tension within the plantar fascia and thereby increase the risk of fascial injury. The plantar fat pad is a specially organized and richly innervated adipose tissue that provides cushioning to the underlying foot structures of the heel, dampening impulses associated with heel strike. The heel pad is particularly receptive to detecting vibration, suggesting a role for detection of gait-induced shock waves. Consequently, changes leading to increased stiffness and reduced compressibility of the subcalcaneal fat pad have been linked to the development of plantar fasciitis, presumably by lowering the attenuation of impulse arising from heel strike. Most patients with plantar fasciitis eventually improve, albeit slowly; 80% of patients treated conservatively are in complete remission at four years. A variety of therapies are used in the treatment of plantar fasciitis and key recommendations, including exercises (Figure 7.1), are summarized in Table 7.3. Limited evidence supports the use of corticosteroid injections; the benet is short-lived and may be associated with adverse effects such as fascial rupture. Corticosteroid-induced atrophy of the subcalcaneal fat pad may also exacerbate the underlying biomechanical predisposition.

Recent Developments
1 Foot orthoses are commonly used for plantar fasciitis but, if custom-made, require a period of weeks between initial consultation and use. Off-the-shelf orthoses are quicker and cheaper, but cost and availability may still be a barrier to use. Short-term treatments, such as supportive taping, are frequently used to alleviate symptoms initially. Low-Dye taping is one of the most frequently applied methods, and improves symptoms by reducing strain in the plantar fascia during standing and walking. Utilizing taping around both the midfoot and the level of the Achilles insertion into the calcaneum, comparison was made between individuals who received taping plus sham ultrasound and individuals receiving sham ultrasound only.5 The primary outcome was rst-step pain one week after tape application, and there was a statistically signicant though clinically small reduction in pain with taping. The 12.3 mm reduction in a visual analogue scale for pain was greater than the 910 mm difference thought to be clinically meaningful. 2 The sensory nerve supplying the skin of the heel and the medial side of the sole is the medial calcaneal branch of the posterior tibial nerve; researchers have suggested that entrapment of this nerve might present as an important factor causing heel pain. Nerve conduction tests have conrmed latency in the medial calcaneal nerve in subjects with plantar fasciitis, but these subjects have normal conduction in their sural, medial and lateral plantar nerves and posterior tibial nerves.6

Conclusion
Plantar fasciitis, if not treated, tends to follow a chronic course. Being overweight, the presence of bilateral symptoms and duration greater than six months are associated with poorer outcome. Treatment should be directed along biomechanical lines weight

36

01 General Rheumatology and Soft Tissue Rheumatism

Figure 7.1 Recommended exercises for plantar fasciitis. (1) Before stepping down, especially after sleeping or resting, stretch the arch of the foot by stretching your legs out in front of you (do not bend the knee). Place a towel around the ball of the foot. Slowly pull on the ends of the towel, pulling the toes and ball of the foot back as far as is comfortable. Hold the foot in this position for ten seconds. Repeat at least ten times. You should feel a pull on the bottom of the foot, especially in the arch. This stretches the plantar fascia and reduces its pull on the heel. (2) Stand about two to three feet from a wall. Lean forward with your hands against the wall. With the painful foot behind, place the other foot forward. Press against the wall, shifting weight over the front foot, while straightening the back leg. Keep the heel of the back foot on the oor and feel the stretch in the heel, Achilles tendon and calf. Hold this position for ten seconds. Repeat at least ten times, and try to do this three times a day.

07 Plantar fasciitis

37

reduction, orthoses and plantar stretching. Since there is limited evidence regarding the value of treatments, a reasonable approach is to start with patient-directed, low-risk, minimal-cost interventions, such as regularly stretching the calf muscles and the plantar fascia, avoiding at shoes and walking barefoot, using over-the-counter arch supports and heel cushions, and limiting extended physical activities. A trial of non-steroidal antiinammatory drugs is reasonable. More costly treatments, such as custom-made orthotics, night splints and immobilization with casts, may be options when the condition does not improve, although the value of these treatments is uncertain.

Further Reading
1 2 3 4 5 Wearing SC, Smeathers JE, Urry SR, Hennig EM, Hills AP. The pathomechanics of plantar fasciitis. Sports Med 2006; 36: 585611. Cole C, Seto C, Gazewood J. Plantar fasciitis: evidence-based review of diagnosis and therapy. Am Fam Physician 2005; 72: 223742. Canoso JJ. Foot pain. In: Rheumatology in Primary Care. WB Saunders, Philadelphia, 1997; Chapter 29. Wearing SC, Hennig EM, Byrne NM, Steele JR, Hills AP. Musculoskeletal disorders associated with obesity: a biomechanical perspective. Obes Rev 2006; 7: 23950. Radford JA, Landorf KB, Buchbinder R, Cook C. Effectiveness of low-Dye taping for the short-term treatment of plantar heel pain: a randomised trial. BMC Musculoskelet Disord 2006; 7: 64. Chang CW, Wang YC, Hou WH, Lee XX, Chang KF. Medial calcaneal neuropathy is associated with plantar fasciitis. Clin Neurophysiol 2007; 118: 11923.

S E C T I O N

T W O

02

Osteoarthritis
08 09 10 11 12 13 Causes and prevention Non-pharmacological treatment Drug treatment NSAIDs gastric side effects and protection NSAIDs cardiac complications Joint replacement surgery

P R O B L E M

08 Causes and Prevention


Case History
John is a 48-year-old accountant who is overweight (body mass index 32 kg/m2). His general health has been good, but he has become concerned recently about twinges of pain in his knees. He was a keen road runner in his teens and 20s, but no longer takes regular exercise. He smokes and takes no regular medications. Both of his parents suffered from osteoarthritis, his father requiring knee replacements in his early 60s. What is the current thinking about the inheritance of osteoarthritis? Which environmental factors play a part in its progression? Can the disease be prevented in a predisposed individual?

Background
Osteoarthritis (OA) is an increasing public health problem because of the rising prevalence of obesity and because of the increasing number of older people in the population.1 Currently, in the United States, 350 000 hip and knee replacements are performed each year. OA is markedly age related, affecting around 12% of those aged 55 years, 50% of Atlas Medical Publishing Ltd

40

02 Osteoarthritis those aged 65 years and as many as 80% of those aged 75 years and over. Prevalence estimates have largely come from population-based X-ray surveys. However, there is a variable relationship between radiological features and symptoms. In clinical practice, OA is only diagnosed when clinical symptoms (pain and stiffness) coexist with evidence of structural damage to the joint. With age, the higher prevalence of OA in females becomes even more exaggerated. The hips, knees and hands are the most commonly affected joints, although almost any joint can be affected. Joint failure arises from a combination of systemic and mechanical factors, including weakness of supporting muscles and ligaments, growth of new bone, erosion of articular cartilage and loss of joint space. A simple grading system is presented in Box 8.1. OA may be primary or secondary; secondary causes are summarized in Box 8.2.

Box 8.1 Radiological severity scoring of OA Grade 0 1 2 3 4 None Possible Minimal Moderate Severe No features Minimal osteophytes Osteophytes, normal joint space Diminution of joint space Decreased joint space, bony sclerosis

Box 8.2 Secondary causes of OA Anatomical Leg length inequality, joint malalignment Slipped femoral epiphyses, epiphyseal dysplasias Perthe disease Congenital dislocation of the hip Hypermobility syndromes Traumatic Occupational arthropathies Fracture through or near a joint Joint surgery Major external trauma Inammatory Inammatory arthropathies Septic arthritis Metabolic Crystal arthropathies Haemochromatosis Acromegaly Ochronosis, chondrodysplasias

08 Causes and prevention

41

The traditional view is that OA is a simple mechanical condition, in which repeated or sustained stresses on the joint lead to articular cartilage damage and loss. The notion that the cartilage is the tissue primarily affected in patients prone to OA has been challenged. In fact, cartilage is a relatively inert tissue with no direct blood supply, nerve input or capacity to become inamed. Severe OA can occur in rare genetic conditions ochronosis and chondrodysplasias but generally in OA, predisposed individuals do not appear to have an alteration in their cartilage structure or composition. In the early stages, changes in modelling of adjacent bone and in the collateral ligaments of the joint may precede the development of changes in cartilage and symptoms. Aging and postmenopausal changes in bone turnover are almost certainly important in the disease. From post-mortem and other studies, it seems clear that changes in the joint capsule and collateral ligaments can also precede the classic structural ndings of OA. These changes may affect joint stability and alignment leading to increased mechanical forces on bone and cartilage. Furthermore, weakening of periarticular muscles further compromises joint stability and predisposes to malalignment. This is a particular factor in knee OA where quadriceps wasting and weakness leads to decreased joint stability. Loss of coordination, vision and proprioception with age contribute to the individual responding less well to unexpected stresses on the joint and thus increasing the impact of minor, potentially traumatic events. A genetic predisposition to OA was rst suspected in the 1940s when prevalence of Heberdens nodes was noted to be three times higher in rst-degree relatives of OA patients. Several segments of the human genome have been identied as susceptibility loci, and at least twelve candidate genes have been studied. Genetic factors account for around 50% of the variance in susceptibility to OA. Twin studies suggest a concordance rate of around 0.65 for monozygotic twins and 0.4 for dizygotic twins.2 In a United Kingdom study,2 rst-degree relatives of patients who had total hip replacement carried a relative risk of 1.9 for also requiring the procedure. The comparable gure for total knee replacement was 4.8. From genome-wide scans, susceptibility loci have been hypothesized at chromosomes 2q, 4q, 6p, 11q and 16p. The only candidate genes identied so far in these regions of the genome are the collagen genes COL29A1 and COL11A2. Other genes are COL2A1, which codes for collagen II (the major protein of cartilage), CRTL1 (coding for a cartilage link protein) and CRTM (coding for a cartilage matrix protein). Polymorphisms of the genes for the vitamin D receptor (VDR) and the oestrogen-a receptor have also been related to risk both are major determinants of bone density. The COL2A1 and VDR genes are located closely together on chromosome 12q. Cartilageremodelling protein gene expression may also play a role; expression of the gene for matrix metalloproteinase-3 (MMP-3), which degrades collagen, and the gene for a disintegrin and metalloproteinase with thrombospondin motif-5 (ADAMTS-5) both increase cartilage breakdown in vitro. Levels of expression of these enzymes may be both genetically determined and increase in response to mechanical stress of cartilage. Obesity is by far the best-studied environmental factor, particularly with respect to increasing risk of knee arthritis. Several factors may underlie the association between obesity and OA. Mechanical stresses predispose to joint damage, although obesity also increases risk of OA in non-weight-bearing joints, and alterations in gait and predisposition to injury contribute to joint alignment. Hormonal changes are factors, including low local levels of androgens because of increased aromatase activity. Hyperglycaemia appears to be a risk factor partly because of its association with obesity, but also because of other

42

02 Osteoarthritis metabolic factors including the inhibitory effect of advanced glycation end products on proteoglycan synthesis. The low-grade inammation found in obese subjects and other pro-inammatory states also predispose. Glucosamine blocks the effect of cytokines on nitric oxide synthesis and also increases glycosaminoglycan production. Other nutritional factors have been identied: low vitamin C intake and vitamin E as an antioxidant and determinant of collagen turnover have been implicated, as has low vitamin D status. Numerous studies have conrmed the increased risk of OA with obesity, particularly when obesity develops at a younger age.35 In the study by Dawson et al.,3 women who smoked heavily were also at increased risk. Even a modest increase in weight within the normal healthy range increases risk of developing OA.4 The latter observation was also conrmed in a national Swedish study,5 although a decreased risk of OA of the hip was reported in smokers. The effect of smoking on OA is therefore uncertain but appears unlikely to be marked. Occupational risk factors are well documented,6 with those who undertake manual labour being at increased risk. Repeated stress on the knee, particularly with the knee
Genetic

Strong family history Joint replacement in first-degree relative Premature onset

Secondary OA

Anatomical

Traumatic Primary OA

Metabolic

Inflammatory

Overweight or obese

Lose weight

Mechanical loading ( occupational)

Avoid repeated strain Support knee

Female gender

Age, post-menopausal

Others Low vitamin C, D or E; neuromuscular weakness; low BMD

Figure 8.1 Risk factors for OA.

08 Causes and prevention

43

bent, increases mechanical stress on the patello-femoral compartment of the joint. Moderate sporting activity does not seem to pose a risk. Elite athletes are at increased risk. Having high bone mineral density (BMD) may predispose to development of osteophytes, but high bone turnover as in post-menopausal osteoporosis leads to microarchitectural deterioration around the joint. The bone in this area consequently becomes stiff and less able to absorb shock, placing the patient at higher risk of OA. Hormone replacement therapy (HRT) has been said to be protective. This may relate to its protective effect on BMD, but it should be noted that use of HRT is associated with a variety of other lifestyle factors. Risk factors for OA are summarized in Figure 8.1. The risk factors for progression are similar and appear to operate more markedly on the knee. These include obesity, injury, crystal arthropathy, neuromuscular dysfunction and poor physical alignment of the joint.

Recent Developments
1 Levels of adiponectin, a hormone product of fat cells, are decreased in obesity. Low levels increase the risk of developing diabetes and vascular disease. High levels of adiponectin have recently been reported in synovial uid from patients with OA.7 The hormone is thought to have a protective role by upregulating the tissue inhibitor of metalloproteinase-2 (TIMP-2) and by inhibiting interleukin(IL)-1induced increases in matrix metalloproteinase-13 (MMP-13). 2 The rate of development of OA in women increases markedly after the menopause, suggesting that oestrogen may have a protective effect. In a large study,8 low plasma oestrogen and low urinary levels of its metabolites were associated with increased risk of OA. Measures to preserve exposure to oestrogen may protect the joints. 3 Biomarkers of disease activity could prove to be a considerable asset.9 Candidates include the urinary C-terminal peptide of collagen type II, serum hyaluronan and cartilage oligomeric matrix protein. Measurement of these markers could be combined with other clinical measures (e.g. C-reactive protein), symptom scores and genetic/molecular markers of risk to help focus programmes of treatment on patients who are either at high risk or have rapidly progressive disease.

Conclusion
OA is a disorder with a strong genetic component. Patients with a rst-degree relative who has a joint replacement, Heberdens nodes or who has developed premature OA should be aware that they are at risk of developing OA. The major, modiable risk factor is being overweight or obese. There is evidence that weight loss in those who are overweight improves OA symptoms and retards progression. It is important to identify factors that place increased strain on the joints, including occupational risks, repetitive injuries and joint malalignment. Simple measures may decrease the risk from physical factors. OA is one of the major degenerative diseases of aging. While we can delay the impact of the disorder on individual patients, we are not at a stage where OA can be routinely prevented.

44

02 Osteoarthritis

Further Reading
1 2 3 Arden N, Nevitt MC. Osteoarthritis: epidemiology. Best Pract Res Clin Rheumatol 2006; 20: 325. Loughlin J. Genetic epidemiology of primary osteoarthritis. Curr Opin Rheumatol 2001; 13: 11116. Dawson J, Juszczak E, Thorogood M, Marks SA, Dodd C, Fitzpatrick R. An investigation of risk factors for symptomatic osteoarthritis of the knee in women using a life course approach. J Epidemiol Community Health 2003; 57: 82330. Holmberg S, Thelin A, Thelin N. Knee osteoarthritis and body mass index: a population-based case-control study. Scand J Rheumatol 2005; 34: 5964. Jrvholm B, Lewold S, Malchau H, Vingrd E. Age, bodyweight, smoking habits and the risk of severe osteoarthritis in the hip and knee in men. Eur J Epidemiol 2005; 20: 53742. Rossignol M. Primary osteoarthritis and occupation in the Quebec national health and social survey. Occup Environ Med 2004; 61: 72935. Chen TH, Chen L, Hsieh MS, Chang CP, Chou DT, Tsai SH. Evidence for a protective role for adiponectin in osteoarthritis. Biochim Biophys Acta 2006; 1762: 71118. Sowers MR, McConnell D, Jannausch M, Buyuktur AG, Hochberg M, Jamadar DA. Estradiol and its metabolites and their association with knee osteoarthritis. Arthritis Rheum 2006; 54: 24817. Kraus VB. Do biochemical markers have a role in osteoarthritis diagnosis and treatment? Best Pract Res Clin Rheumatol 2006; 20: 6980.

4 5 6 7 8

09 Non-pharmacological treatment P R O B L E M

45

09 Non-Pharmacological Treatment

Case History
AP is a 58-year-old woman whose symptoms of osteoarthritis (OA) are beginning to limit her activity. She complains that she feels stiff. She takes paracetamol regularly but does not want to take other drugs. One of her friends recently suffered gastrointestinal haemorrhage while taking a non-steroidal anti-inammatory drug (NSAID). What non-pharmacological interventions are available? Are natural treatments effective at decreasing symptoms and protecting joints? Is cartilage protection a valid treatment goal?

Background
Effective analgesia and use of anti-inammatory drugs can transform the lives of patients with OA. However, pharmaceuticals are not the sole mode of treatment for OA. Nonpharmacological approaches are often underused. Also many patients resort to natural products. Health practitioners need to understand these products and whether they work. Drug treatment should only be considered when lifestyle and non-pharmacological management has failed.

Body weight
Being overweight or obese increases the risk of developing OA and accelerates progression of the disease.1 In the Framingham study, an increase in body mass index (BMI) of only 2 kg/m2 was associated with a 50% increase in the risk of developing OA in women.2 Studies documenting the benet of weight loss have generally been short term, lasting no more than two years. Maintenance of a normal or near-normal body weight is also associated with other health benets including prevention of diabetes and cardiovascular disease. While the cornerstone of weight management is diet and exercise, the role of drugs (including orlistat, sibutramine and rimonabant) should be considered. Weight reduction reduces mechanical stresses on the joints of the axial skeleton. Being overweight also increases the risk of joint malalignment. Some of the benets of a weightmanagement programme may relate directly to the effects of exercise and to improved metabolic factors and lower levels of chronic inammation. It is important that treatment goals are realistic and that the patient receives adequate support. The practitioner should always be alert to the possibility of psychological problems, including eating disorders. Atlas Medical Publishing Ltd

46

02 Osteoarthritis

Exercise
Physical exercise contributes to weight loss or at least to weight maintenance. Independently of this, it increases overall function and well-being, and decreases risk of falling. As well as protection from OA,1 exercise also lowers risk of osteoporosis, diabetes and cardiovascular disease. Exercise is of particular benet in knee protection. Isokinetic exercises that utilize either exion against resistance or extension against resistance are effective. Both aerobic exercise (low impact) and specic strengthening exercises for periarticular muscles have been found to be effective. Whichever form of exercise is used, there is benet from the input of a qualied therapist; undue pain or discomfort should be avoided and the approach should be based on the patients ability and motivation, with only gradual increase in the effort required.

Glucosamine and chondroitin


Glucosamine, available as sulphate or hydrochloride, is a hexosamine that is a natural constituent of keratin sulphate, the glycosaminoglycan of hyaline cartilage. It is frequently administered with chondroitin sulphate, another naturally occurring constituent of the cartilage matrix (being a constituent of aggrecan, which is the major proteoglycan of cartilage). Although studies have reported effects comparable with NSAIDs, results have been extremely variable. This may be partly because of differences between glucosamine hydrochloride and glucosamine sulphate. Studies using the former preparation have tended to show negative results whereas use of the latter compound tends to be more effective. This has been attributed by some to the fact that it is a source of sulphate, rather than because of the glucosamine per se. Glucosamine sulphate is given as a single, daily oral dose of 1500 mg and is well absorbed. Some of its effect is mediated through inhibition of cytokine-induced nitric oxide production and through inhibition of proteolytic enzymes. A Cochrane review in 2000 included 16 studies of patients (total 2029) who used glucosamine sulphate 1500 mg each day for six weeks.3 Sixty per cent of patients reported improved pain and 33% reported improved function. A more recent Cochrane review3 included 20 trials with 2570 patients and, overall, did not conrm improved pain and function with glucosamine. However, ten trials using one preparation (Rotta Pharmaceuticals) did show improvements. A recent study4 randomized 1583 patients with knee arthritis to glucosamine hydrochloride 1500 mg/day, chondroitin sulphate 1200 mg/day, glucosamine and chondroitin together, 200 mg celecoxib or placebo. Up to 4 g/day of paracetamol was allowed as rescue analgesia. Neither glucosamine, chondroitin, nor the combination was successful overall. The combination of the two was somewhat effective in patients with moderate to severe knee pain. It is noteworthy that glucosamine hydrochloride was used in this study.

Other nutritional components and supplements


There is evidence that low vitamin C levels or status is related to the risk of developing OA and to the progression of the disease. The two major essential fatty acids are linoleic acid (18:2 n-6) and a-linolenic acid (18:3 n-3). These two fatty acids form eicosanoids through the action of cyclooxygenase-1 (COX-1), cyclooxygenase-2 (COX-2) and 5-lipoxygenase. When cell membranes comprise predominantly n-6 fatty acids (leading to prostaglandin E2 and leukotriene B4 production) a pro-inammatory state exists,

09 Non-pharmacological treatment

47

while the incorporation of predominantly n-3 fatty acids results in an anti-inammatory state. There is considerable evidence for the benet of n-3 fatty acid supplementation in management of OA. Patients with OA frequently use complementary and alternative medicines up to 40% of patients were using such products in a recent survey.5 Fish oils, garlic extracts and celery extract are among the most commonly used products. This high level of usage reects, to an extent, the fact that patients get incomplete relief from medical approaches, and also that many patients do not want to be reliant on pharmaceuticals.

Viscosupplementation
Hyaluronic acid is a high-molecular-weight polysaccharide that contributes to the physical properties of synovial uid and cartilage. Synovial uid supplementation, or viscosupplementation, was developed in the 1980s as a treatment proposed to relieve pain in the short term and to restore the viscoelastic properties associated with hyaluronan in synovial joints. The products used are derivatives of hyaluronan and hylan and are given by intra-articular injection sometimes weekly for the rst three weeks, but then at intervals of at least four weeks for up to six months. A recent Cochrane review6 considered 76 controlled trials comparing viscosupplementation products, or examining their use with intra-articular steroids or NSAIDs. The conclusion was that viscosupplementation treatment was effective, especially at the 513-week period. Firm conclusions about the relative value of different products could not be made. Two recent studies have conrmed the potential benets of viscosupplementation. Sun et al.7 administered hyaluronic acid to 75 patients with ankle OA. Patients reported increased pain relief and improved function. The effects were apparent both during treatment and for up to six months afterwards. In another study,8 intra-articular hyaluronic acid (three injections) was compared with an exercise regimen for patients with knee OA. Both treatments were effective and there was no statistical difference between the two.

Other measures
Use of a walking stick (cane) may help. The stick should be used on the opposite side to the worst affected joints, with the hand holding the stick at the level of the greater trochanter, and the foot of the affected leg and the stick striking the ground more or less simultaneously. The importance of the joint being as well aligned as possible cannot be overemphasized. Shoe inserts, braces or taping of the affected joint may minimize the effect of malalignment including valgus (knees closer together than ankles) and varus (ankles closer together than knees) deformities. Local application of capsaicin cream (0.025%0.075%), which acts as a counter-irritant, can be useful. Footwear should be chosen according to the following advice: b Thick, soft soles with maximum shock absorption b Minimum heel raise b Broad forefoot, allowing the toes to splay (thus improving balance and decreasing the likelihood of falls and injuries) b Soft uppers b Adequate depth of the shoe

48

02 Osteoarthritis

Recent Developments
1 Glucosamine has recently been studied in chondral and synovial cultures in vitro.9 In this study, glucosamine suppressed the secretion of matrix metalloproteinases (MMP-2 and MMP-9), key enzymes involved in cartilage degradation. Urokinase plasminogen activator has also been implicated in the development of OA, particularly where there are joint effusions. Again, levels of this enzyme were suppressed by glucosamine. 2 The hexosamine biosynthetic pathway may be a major cellular nutrient sensor.10 The rst step in this pathway is the formation of glucosamine-6-phosphate from fructose-6-phosphate. The pathway leads to formation of uridine diphosphate (UDP)-N-acetyl-glucosamine, which becomes incorporated into the side chains of

Weight loss (if overweight/obese)

Weight maintenance

Exercise Aerobic (general fitness) Resistance (muscle bulk and function) Strengthening (periarticular muscles)

Diet Suitable calorie intake Reasonable protein content ? Vitamin C ? Essential FA (n-3)

General measures Accident/injury prevention Sensible footwear Taping or bracing affected joint Orthotics (joint alignment) Walking aids (stick etc.)

Capsaicin for pain relief

Glucosamine sulphate chondroitin sulphate

Viscosupplementation (hyaluronic acid)

Figure 9.1 Non-pharmacological management of OA. FA, fatty acid.

09 Non-pharmacological treatment

49

proteins and lipids. There is concern that increased ux through this pathway (including by glucosamine supplementation) may worsen insulin resistance and increase susceptibility to diabetes complications. 3 Some of the properties of mature cartilage depend on the charge of sulphate moieties of chondroitin sulphate. Dietary sulphate largely comes from high-protein foods, and low sulphate intake or low serum levels of sulphate have been associated with increased risk of OA. Blinn et al.11 have shown that prolonging the overnight fast or ingestion of calories without protein leads to lower serum levels of sulphate.

Conclusion
Non-pharmacological management should be considered at the outset before drug treatments are initiated (Figure 9.1). The most important factors are weight management, exercises to increase general tness and to increase strength around affected joints, measures to decrease stresses on joints, and improvement of the alignment of joints if need be. Current evidence favours the use of glucosamine sulphate (usually with chondroitin sulphate) and viscosupplementation with hyaluronic acid products. A number of nutritional factors, including antioxidant vitamins and essential fatty acids, are almost certainly important, but routine supplementation is not currently recommended.

Further Reading
1 Roddy E, Doherty M. Changing life-styles and osteoarthritis: what is the evidence? Best Pract Res Clin Rheumatol 2006; 20: 8197. 2 Felson DT, Zhang Y, Hannan MT et al. Risk factors for incident radiographic knee osteoarthritis in the elderly: the Framingham Study. Arthritis Rheum 1997; 40: 72833. 3 Towheed TE, Maxwell L, Anastassiades TP et al. Glucosamine therapy for treating osteoarthritis. Cochrane Database Syst Rev 2005; CD002946. 4 Clegg DO, Reda DJ, Harris CL et al. Glucosamine, chondroitin sulphate, and the two in combination for painful knee osteoarthritis. New Engl J Med 2006; 354: 795808. 5 Zochling J, March LM, Lapsley H, Cross M, Tribe K, Brooks P. Use of complementary medicines for osteoarthritis a prospective study. Ann Rheum Dis 2004; 63: 54954. 6 Bellamy N, Campbell J, Robinson V, Gee T, Bourne R, Wells G. Viscosupplementation for the treatment of osteoarthritis of the knee. Cochrane Database Syst Rev 2006; CD005321. 7 Sun SF, Chou YJ, Hsu CW et al. Efcacy of intra-articular hyaluronic acid in patients with osteoarthritis of the ankle: a prospective study. Osteoarthritis Cartilage 2006; 14: 86774. 8 Karatosun V, Unver B, Gocen Z, Sen A, Gunal I. Intra-articular hyaluranic acid compared with progressive knee exercises in osteoarthritis of the knee: a prospective randomized trial with long-term follow-up. Rheumatol Int 2006; 26: 27784. 9 Chu SC, Yang SF, Lue KH et al. Glucosamine sulfate suppresses the expressions of urokinase plasminogen activator and inhibitor and gelatinases during the early stage of osteoarthritis. Clin Chim Acta 2006; 372 : 16772.

50

02 Osteoarthritis
10 Buse MG. Hexosamines, insulin resistance, and the complications of diabetes: current status. Am J Physiol Endocrinol Metab 2006; 290: E18. 11 Blinn CM, Biggee BA, McAlindon TE, Nuite M, Sibert JE. Sulphate and osteoarthritis: decrease of serum sulphate levels in an additional 3-h fast and a 3-h glucose tolerance test after an overnight fast. Ann Rheum Dis 2006; 65: 12235.

P R O B L E M

10 Drug Treatment

Case History
Mrs GT is 67 years old. Since she developed osteoarthritis (OA) ve years ago she has tried hard to control symptoms with diet, exercise and nutritional supplements. She is increasingly frequently requiring analgesics. She wants to discuss drug treatments for her OA. Her general health is good and she has no history of dyspepsia. Do non-steroidal anti-inammatory drugs (NSAIDs) have an advantage over simple analgesia? How should drug therapy be approached? What are the prospects for disease-modifying drugs?

Background
The rst line of treatment for OA should always be non-pharmacological. When drug therapy is required, paracetamol in doses of up to 4 g/day is the usual rst-line treatment. Zhang et al.1 conducted a meta-analysis of published controlled trials of simple analgesia for OA. The pain response to paracetamol was assessed along with the Western Ontario and McMaster Universities (WOMAC) OA index score, a score that focuses largely on functional symptoms. Combining data from the four trials identied, the pain response was classied as weak, while there was minimal functional improvement. Further trial data conrm that the effect of paracetamol is modest, while there is clear evidence from numerous trials that NSAIDs have at least a modest benet in alleviating pain and Atlas Medical Publishing Ltd

10 Drug treatment

51

improving function.2 Paracetamol should be used only with caution in patients with liver disorders, but it seldom (except in overdose) causes hepatic dysfunction. Compared with aspirin and NSAIDs, paracetamol is relatively safe in patients with renal impairment. If paracetamol is of limited efcacy, do more potent analgesics have greater effect? The best-studied agent is tramadol. In addition to its opioid action, it is a weak central inhibitor of noradrenaline and serotonin uptake. There is no question from available trials that tramadol is moderately effective at decreasing pain and improving function (by WOMAC score) in patients with OA.3 A total daily dose of 200300 mg is usual (given in divided doses), and the drug may be combined with paracetamol either to increase effect or to minimize the dose of each agent. Side effects include dizziness, somnolence, headache and constipation, but the drug is generally very safe, as well as being cheap. There is a range of alternatives including dextropropoxyphene and pentazocine. More potent opiates do not offer any advantage in terms of symptom relief and are more likely to cause side effects including dependence. The efcacy of tramadol is comparable to that of lower doses of NSAIDs or cyclooxygenase-2 (COX-2) inhibitors. Topical NSAIDs may be as effective as systemic administration.2 Since only up to 5% of the topical dose is absorbed systemically, risk of severe adverse drug reactions is substantially lower than with oral dosing. Side effects are usually limited to the site of administration localized itching, burning or skin rash. The only concern with topical NSAIDs from trials is that their benet may be relatively short-lived. OA is not primarily an inammatory disorder. However, inammation does contribute to symptoms and to disease progression. For knee OA in particular, intraarticular corticosteroid injection can be useful. The agents used are prednisolone, methylprednisolone, betamethasone and triamcinolone. The treatment leads to shortterm symptom relief but does not modify the long-term progression of the disease. From assessment of nearly 30 trials,4 the treatment is clearly superior to placebo for pain relief. It is comparable in effect to other pharmacological treatments for pain, but varying effects on functional improvement have been reported. There is a risk of introducing infection into the joint, and the treatment is not useful where there is substantial effusion in the joint. The focus of treatment in OA has been on symptom relief, but there is increasing interest in development of drugs that may modify the progress of the disease, and in approaches to reconstructing the cartilage matrix.5,6 Diacerein is an antagonist to interleukin (IL)-1b that has already been demonstrated to have some efcacy in controlled clinical trials. Other anti-inammatory approaches include antagonists to tumour necrosis factor (TNF)-a. It appears most likely that local administration of disease-modifying agents may be preferred to avoid the systemic effects of immunosuppression. Local inhibitors of nitric oxide synthase (NOS) have the potential to decrease activation of cytokines and the COX-2 enzyme system. Colchicine has been proposed as a means of decreasing local inammation within the joint. Both bisphosphonates and oestrogen treatment (in women) may impact favourably on the increased bone turnover that occurs around affected joints. Decreasing the turnover of the cartilaginous matrix is one of the goals of therapy. Twenty-eight matrix metalloproteinase (MMP) enzymes have now been identied and are key targets for disease-modifying drugs. Of these drugs, the tetracyclines including doxycycline and minocycline are the most widely studied agents. The joint space is a relatively easy target for locally administered gene therapy.

52

02 Osteoarthritis Amongst gene targets studied have been the insulin-like growth factor-1 (IGF-1) gene and the gene for the IL-1 receptor antagonist. Experimental tissue-engineering approaches include the use of mesenchymal stem cells, autologous cartilage plugs, implantation of autologous chondrocytes and use of articial cartilage matrix. Although unlikely to be as potent as pharmaceuticals and tissue-engineering approaches, there is considerable interest in the potential role of diets and supplements. These approaches are cheap, safe and popular with patients. Evidence relating to their effects and modes of action is accumulating. Various polyphenols, including the catechin group, have been studied because of their anti-inammatory and chondroproliferative properties. The best-known supplement is green tea (epigallocatechin gallate), which is widely used as a herbal remedy. There is also considerable interest in ginger extracts and avocado/soybean non-saponiables.

Diet Exercise Joint support and alignment

(Glucosamine chondroitin)

Paracetamol

Maximum dose 4 g/day

Local measures Capsaicin cream NSAIDs Methylsalicylate

Tramadol

Low GI risk NSAID

High GI risk Add PPI Misoprostol COX-2

More potent opioids or analgesia NSAID

Intra-articular treatment Steroids Hyaluronan

Consider surgery

Figure 10.1 Drug treatments for OA. GI, gastrointestinal; PPI, proton pump inhibitor.

10 Drug treatment

53

Recent Developments
1 Doxycycline has been proposed as a disease-modifying drug for OA. Amongst its actions in the joint, the drug inhibits breakdown of type XI collagen and decreases activity of collagenase and NOS. A 30-month trial of doxycycline in women with radiologically proven knee OA7 showed 40% less reduction in the knee joint space in treated patients compared with placebo. While doxycycline appeared to slow the progression of joint disease in this study, its effects on symptom control were disappointing. 2 Nuclear factor-k-beta (NF-kB) is a transcription factor that plays an important part in modulating immunological and inammatory responses. When the NF-kB pathway is activated, there is increased expression of inammatory mediators and proteins involved in the regulation of apoptosis.8 The NF-kB pathway is inhibited by corticosteroids and a number of other immunomodulatory agents. There is a search for more specic agents. The pathway is involved in the pathogenesis of OA not only through regulation of inammation but also through its involvement in regulation of cartilage turnover.

Conclusion
A scheme for managing drug therapy in OA is proposed in Figure 10.1. Paracetamol is the most widely used analgesic and is often effective in alleviating pain without a major risk of side effects. The potential benets of local treatments and more potent analgesics (including tramadol) are often forgotten before proceeding to NSAIDs. Progress is being made with disease-modifying drugs on a number of fronts. The most promising approaches are those that may downregulate the local increases in inammation and cartilage turnover.

Further Reading
1 Zhang W, Jones A, Doherty M. Does paracetamol (acetaminophen) reduce the pain of osteoarthritis? A meta-analysis of randomised controlled trials. Ann Rheum Dis 2004; 63: 9017. Bannwarth B. Acetaminophen or NSAIDs for the treatment of osteoarthritis. Best Pract Res Clin Rheumatol 2006; 20: 11729. Cepeda MS, Camargo F, Zea C, Valencia L. Tramadol for osteoarthritis. Cochrane Database Syst Rev 2006; 3: CD005522. Bellamy N, Campbell J, Robinson V, Gee T, Bourne R, Wells G. Intraarticular corticosteroid for treatment of osteoarthritis of the knee. Cochrane Database Syst Rev 2006; CD005328. Fajardo M, Di Cesare PE. Disease-modifying therapies for osteoarthritis: current status. Drugs Aging 2005; 22: 14161. Baker CL, Ferguson CM. Future treatment of osteoarthritis. Orthopedics 2005; 28 (4 Suppl): s22734.

2 3 4 5 6

54

02 Osteoarthritis
7 Brandt KD, Mazzuca SA, Katz BP et al. Effects of doxycycline on progression of osteoarthritis: results of a randomized, placebocontrolled, double-blind trial. Arthritis Rheum 2005; 52: 201525. Roman-Blas JA, Jimenez SA. NF-kb as a potential therapeutic target in osteoarthritis and rheumatoid arthritis. Osteoarthritis Cartilage 2006; 14: 83948.

P R O B L E M

11 NSAIDs Gastric Side Effects and Protection

Case History
Mr JS is a 58-year-old carpenter. He was diagnosed as having rheumatoid arthritis at the age of 40 years. He is a smoker and is prone to dyspepsia. He does not currently require disease-modifying drugs. However, he does have pain and stiffness and is not able to work comfortably without symptomatic relief. To what extent do non-steroidal anti-inammatory drugs (NSAIDs) cause gastrointestinal symptoms? Does the presence of dyspeptic symptoms inuence the choice of agent? What strategies are available for gastric protection?

Background
The NSAIDs are an important class of drug with analgesic, antipyretic and anti-inammatory activities. They are among the most common drugs prescribed. NSAIDs account for around 3% of the total United States (US) drugs market, and it is estimated that 5%10% of the population use NSAIDs on a regular basis (Box 11.1). In the US during 2001, 70 million prescriptions were issued for NSAIDs and 30 billion doses were bought over the counter. NSAIDs are weak acids (pKa 35) that are highly lipid soluble and readily absorbed from the stomach. In the plasma they are highly protein bound (>95%, mainly to albumin). They are metabolized and conjugated in the liver and mainly excreted in the urine. Atlas Medical Publishing Ltd

11 NSAIDs gastric side effects & protection

55

Box 11.1 Uses of NSAIDs Osteoarthritis Rheumatoid arthritis Inammatory spondyloarthropathies (ankylosing spondylitis, reactive arthritis, psoriatic arthritis) Post-operative pain Acute gout Soft tissue injuries (including sports injuries) Dysmenorrhoea Renal colic NSAIDs reversibly inhibit the cyclooxygenase (COX) enzyme, which catalyses the rst step in the pathway that converts arachidonic acid to key regulatory prostaglandins. The enzyme exists in two isoforms: COX-1 is constitutively expressed in most tissues while COX-2 is an inducible enzyme, the expression of which is upregulated at sites of inammation (Figure 11.1). The gastric side effects of NSAIDs relate to inhibition of COX-1 in the gastric mucosa, where prostaglandins increase gastric mucous and bicarbonate secretion, decrease gastric acid secretion and stimulate mucosal blood ow. COX-2 is not

Membrane phospholipids

Phospholipase A2

Arachidonic acid

COX-1 constitutive/inducible

COX-2 constitutive/inducible

PGs GI cytoprotection Platelet aggregation Renal function (blood flow)

PGs Inflammation Fever Pain Headache Carcinogenesis Kidney, stomach, uterus CNS and spine Endothelium

Figure 11.1 Prostaglandin generation by COX-1 and COX-2. CNS, central nervous system; GI, gastrointestinal; PG, prostaglandin.

56

02 Osteoarthritis

Box 11.2 Risk factors for gastric side effects High dose of NSAID Use of non-selective NSAID First three months of treatment Age >50 years Previous ulcer history Use of multiple agents Concurrent use of aspirin, warfarin or steroids Previous ulcer history Helicobacter pylori infection General disability

normally expressed in the gastric mucosa. The benecial, anti-inammatory actions of NSAIDs are mediated through inhibition of COX-2. NSAIDs vary greatly in their relative COX-1 and COX-2 inhibitory activity. Aspirin, sulindac and piroxicam have much greater selectivity for COX-1. The mechanism of aspirin action differs from that of other NSAIDs in that it acts by irreversibly acetylating the active site of COX-1. Meloxicam, the rst of the selective COX-2 inhibitors, is three times more potent on COX-2 compared with COX-1. COX-2 is not inhibited by aspirin but is downregulated by glucocorticoids. NSAIDs are generally very safe and well tolerated. However, because of the number of patients using them, adverse reactions are frequently encountered. By far the most common side effects relate to gastric mucosal injury (Box 11.2). Up to 20% of patients taking NSAIDs suffer gastrointestinal symptoms including heartburn, dyspepsia, nausea, vomiting and abdominal pain. In endoscopic studies, haemorrhagic reactions to the presence of the tablets in the stomach are almost universal, supercial erosions occur in 50% of subjects and 20% of regular users have chronic ulceration (i.e. ulcers that penetrate the muscularis mucosa). There is a variable relationship between symptoms and the changes in the mucosa visible on endoscopy. The gastrointestinal side effects of NSAIDs are a major cause for concern.1 Serious gastric side effects occur in less than 1% of patients. However, this is sufcient to account for 100 000 hospital admissions, contribute to 16 000 deaths and cost $1.5 billion in the US each year. Similarly in the United Kingdom, where there are around four million users with osteoarthritis, use of NSAIDs contributes to 2000 deaths each year. Hooper et al.2 conducted a review comparing gastric protection strategies in subjects receiving NSAIDs. In all, 112 trials with 74 666 participants were included; 248 serious gastrointestinal adverse drug reactions (ADRs) were reported and there were 138 deaths. Histamine H2 receptor antagonists proved to be ineffective at preventing any of the primary outcomes. Proton pump inhibitors signicantly reduced symptomatic ulcers (relative risk [RR] 0.09; 95% condence interval [CI] 0.020.47) and serious complications (RR 0.57; 95% CI 0.360.91). Misoprostol is a prostaglandin analogue that increases mucous protection and decreases acid-induced damage. It may be prescribed alone (Cytotec) or in combination with diclofenac (Arthrotec). A dose of 200 mg/day is usually sufcient, while higher doses may cause diarrhoea, abdominal pain and bloating. Misoprostol decreased the rate of symptomatic ulcers (RR 0.36; 95% CI 0.200.67) and serious gastrointestinal side effects (RR 0.57; 95% CI 0.360.91). Use of COX-2-selective agents was associated with lower rates of symptoms (RR 0.49; 95% CI 0.380.62) and serious gastrointestinal ADRs (RR 0.55; 95% CI 0.380.80). H2 receptor antagonists are effective at preventing symptoms and duodenal ulcers with NSAIDs but probably do not prevent gastric ulcers. They are probably more cost-effective than COX-2-selective drugs and their routine prescription with non-selective COX inhibitors may be justied.3

11 NSAIDs gastric side effects & protection

57

Helicobacter pylori infection is implicated in the pathogenesis of gastric and duodenal ulcers. It is not clear whether there should be routine screening for and eradication of this agent in patients starting NSAIDs. Screening can be undertaken using breath tests, serological testing for antibodies or endoscopy with gastric biopsy. Serology is cheap and convenient but a positive result does not necessarily indicate current infection and does not change after eradication. The Celecoxib Long-term Arthritis Safety Study (CLASS) and the Vioxx Gastrointestinal Outcomes Research (VIGOR) study each enrolled over 8000 patients and conrmed that COX-2 inhibitors were as effective as non-selective agents for patients with arthritis.1 Use of COX-2 inhibitors was associated with lower risk of symptomatic ulcers and serious gastrointestinal side effects. A number of studies have conrmed that COX-2-selective agents reduce the incidence of serious gastrointestinal side effects by 40%50%. Their role in preventing dyspeptic symptoms has been less certain. Spiegel et al.4 recently reviewed 26 published studies comparing COX-2-selective with non-selective agents; the relative risk of dyspeptic symptoms was decreased by 12% and the absolute risk was decreased by 3.7% with a COX-2-selective agent. By contrast, compared with non-selective NSAIDs alone, NSAID plus concurrent proton pump inhibitor led to a relative risk reduction of dyspeptic symptoms of 66% and an absolute risk reduction of 9%. Aspirin and NSAIDs may be associated with decreased risk of cancers, including those of the gastrointestinal tract. Studies in vitro have shown that COX-2 inhibition may decrease the growth of gastric cancers, and COX-2 inhibitors have been suggested for the treatment of pre-cancerous gastric lesions in humans.5 Increased risk of myocardial infarction has been associated with use of high-dose, non-selective NSAIDs and with use of COX-2-selective agents (chapter 12). Renal side effects include salt and water retention, hypertension, acute renal failure, interstitial nephritis, nephrotic syndrome and acute tubular necrosis.6 Renal side effects are more likely when the patient is also taking angiotensin-converting enzyme inhibitors or diuretics. COX-2 is constitutively expressed in the kidney, and patients taking COX-2 inhibitors are as likely to experience renal side effects as patients taking non-selective agents. Other side effects are headache, dizziness, tinnitus and pruritus. Photosensitivity may occur with piroxicam or diclofenac. NSAIDs may cause increased liver enzymes and, occasionally, severe hepatitis.

Recent Developments
1 The role of COX isoenzymes in ulcer healing has recently been investigated in COX-1 and COX-2 knockout mice with induced gastric ulcers.7 The COX-1 enzyme apparently had no part to play in ulcer healing, while ulcer healing was impaired in COX-2 knockout mice or in wild-type mice treated with COX-2 inhibitors. Inducible nitric oxide synthase (NOS-2) and endothelial nitric oxide synthase (NOS-3) were upregulated in healing ulcers, suggesting that these enzymes might play a part in the healing process. Some of the potential benet of selective COX-2 inhibitors in protecting against ulcer formation may be offset by their inhibitory effect on ulcer healing. 2 The proton pump inhibitors are widely used in prevention and treatment of gastric injury induced by NSAIDs and their use is well supported by trial

58

02 Osteoarthritis

Pain (e.g. from arthritis)

Simple analgesia (paracetamol)

NSAID indicated

Correct risk factors if possible Smoking Alcohol excess Stress Use of steroid, warfarin or aspirin

Previous ulcer

No ulcer

H. Pylori (screen and eradicate) Consider endoscopy/barium Estimate risk of GI side effects Warn patient of risks

Low risk

Medium risk

High risk

Lowest possible dose NSAID

COX-2-selective NSAID H2RA

NSAID PPI NSAID misoprostol

Careful follow-up

Figure 11.2 NSAIDs and risk of gastrointestinal side effects. The considerations shown take account of the relative costs of different drugs and combinations as well as their efcacy. COX-2, COX-2-selective agent; H2RA, histamine H2 receptor antagonist (e.g. cimetidine, ranitidine); NSAID, non-steroidal antiinammatory drug (non-selective agent); PPI, proton pump inhibitor.

11 NSAIDs gastric side effects & protection

59

evidence. Several non-acid-dependent mechanisms are also thought to be important in promotion of gastric ulcers. In a recent animal study,8 lansoprazole decreased oxidative stress within the gastric mucosa and this was thought to contribute to its effect in protecting from mucosal damage induced by NSAIDs. 3 The interaction between NSAIDs and H. pylori infection in the causation of gastric and duodenal ulcers is controversial. A recent study compared rates of gastrointestinal symptoms in patients positive and negative for H. pylori and treated with NSAID.9 Furthermore, the positive group was divided, with half being treated with H. pylori eradication therapy and half with placebo. Overall, there was no relationship between dyspeptic symptoms induced by NSAID and H. pylori status. These data do not favour routine screening for and eradication of H. pylori in patients who are being started on NSAIDs.

Conclusion
NSAIDs represent one of the most commonly prescribed groups of drugs. Their use is frequently associated with gastrointestinal symptoms, and serious drug reactions occur in up to 1% of patients annually. The anti-inammatory action of NSAIDs relates to inhibition of the COX-2 isoenzyme, while gastrointestinal side effects are due to inhibition of COX-1. Individual agents vary widely in their relative inhibitory activity for the two isoenzymes. COX-2-selective agents are much less likely than non-selective agents to cause dyspeptic symptoms. However, combining non-selective NSAIDs with proton pump inhibitors or misoprostol probably provides even greater protection. Recent data showing that the COX-2-specic agents in particular are associated with increased risk of myocardial infarction have limited the use of this class of drug and led to the withdrawal of some agents. Management of risk of gastric problems with NSAIDs is summarized in Figure 11.2.

Further Reading
1 2 Fennerty MB. NSAID-related gastrointestinal injury. Evidence-based approach to a preventable complication. Postgrad Med 2001; 110: 8794. Hooper L, Brown TJ, Elliott R, Payne K, Roberts C, Symmons D. The effectiveness of ve strategies for the prevention of gastrointestinal toxicity induced by non-steroidal antiinammatory drugs: systematic review. BMJ 2004; 329: 94858. Elliott RA, Hooper L, Payne K, Brown TJ, Roberts C, Symmons D. Preventing nonsteroidal anti-inammatory drug-induced gastrointestinal toxicity: are older strategies more cost-effective in the general population? Rheumatology 2006; 45: 60613. Spiegel BMR, Farid M, Dulai GS, Gralnek IM, Kanwal F. Comparing rates of dyspepsia with Coxibs vs NSAID+PPI: a meta-analysis. Am J Med 2006; 119: 448.e2736. Jiang XH, Wong BCY. Cyclooxygenase-2 inhibition and gastric cancer. Curr Pharm Des 2003; 9: 22818.

4 5

60

02 Osteoarthritis
6 7 Cheng HF, Harris RC. Renal effects of non-steroidal anti-inammatory drugs and selective cyclooxygenase-2 inhibitors. Curr Pharm Des 2005; 11: 1795804. Schmassmann A, Zoidl G, Peskar BM et al. Role of the different isoforms of cyclooxygenase and nitric oxide synthase during gastric ulcer healing in cyclooxygenase-1 and -2 knockout mice. Am J Physiol Gastrointest Liver Physiol 2006; 290: G74756. Blandizzi C, Fornai M, Colucci R et al. Lansoprazole prevents experimental gastric injury induced by non-steroidal anti-inammatory drugs through a reduction of mucosal oxidative damage. World J Gastroenterol 2005; 11: 405260. Schaeverbeke T, Broutet N, Zerbib F et al. Should we eradicate Helicobacter pylori before prescribing an NSAID? Result of a placebo-controlled study. Am J Gastroenterol 2005; 100: 263743.

P R O B L E M

12 NSAIDs Cardiac Complications


Case History
Mrs JA is a 64-year-old lady who nds that her osteoarthritis symptoms are no longer controlled with simple analgesia. She has suffered from angina since the age of 57. She has never had a myocardial infarction (MI) and her angina is stable. Her exercise tolerance is not governed by angina and would be better if her arthritic symptoms were improved. Would you prescribe her a non-steroidal anti-inammatory drug (NSAID)? Which agent would you choose? What is the current thinking about NSAIDs and the risk of cardiac events?

Background
The cyclooxygenase (COX) enzymes catalyse the rst stage in the formation of prostaglandins and thromboxanes from arachidonic acid. The COX-1 isoenzyme is constitutively expressed in most tissues, while COX-2 expression is induced at sites of inammation. The major side effects of the NSAID group of drugs, including the effects on the stomach, are caused by COX-1 inhibition, while the benecial effects are almost entirely due to COX-2 inhibition. A classication of drugs with NSAID action is shown Atlas Medical Publishing Ltd

12 NSAIDs cardiac complications

61

Box 12.1 The non-steroidal anti-inammatory drugs (NSAIDs) Salicylates Aspirin Benorilate Diunisal 2-arylpropionic acids (profens) Ibuprofen Fenbufen Fenoprofen Naproxen Ketoprofen Flurbiprofen Coxibs Celecoxib (Celebrex) Parecoxib (Dynastat) Lumiracoxib (Prexige) Rofecoxib * Valdecoxib *
* Drug withdrawn from the market.

Arylalkanoic acids Diclofenac Etodolac Indomethacin Sulindac Tolmetin Oxicams Piroxicam Meloxicam Tenoxicam Pyrazolidine derivatives Phenylbutazone Oxyphenbutazone Azapropazone N-arylanthranilic acids Mefenamic acid Tolfenamic acid

in Box 12.1. Paracetamol has analgesic but no anti-inammatory properties. Unquestionably, endoscopic studies have shown that COX-2 inhibitors are less likely than traditional NSAIDs to produce gastric ulceration. In late 2004, emerging data from the Vioxx Gastrointestinal Outcomes Research (VIGOR) and Adenomatous Polyp Prevention on Vioxx (APPROVe) studies demonstrated that risk of MI was increased up to two-fold in patients treated with rofecoxib (Vioxx). This led to withdrawal of the drug and valdecoxib was withdrawn because of similar considerations. Activation of COX-1 in platelets leads to increased thromboxane A2 (TxA2) and consequent platelet aggregation, vasoconstriction and smooth muscle proliferation.1 This pathway is irreversibly blocked by aspirin. Recent studies suggest that non-selective NSAIDs including naproxen and ibuprofen, when given concurrently with aspirin, may interfere with this benecial effect of aspirin and increase coronary events. COX-2 is expressed only at very low levels in healthy blood vessels but is abundantly expressed in atherosclerotic vessels. The enzyme is expressed in smooth muscle cells, macrophages/foam cells and in the endothelium. Secretion of prostacyclin (PGI2) by endothelial cells counteracts the effects of TxA2 released from platelets. This intrinsic protection mechanism is lost when the COX-2 enzyme is inhibited. Figure 12.1 depicts the proposed model of prothrombotic and antithrombotic balance for low-dose aspirin, traditional NSAIDs and selective COX-2 inhibitors. Recent data suggest that the adverse effects noted with rofecoxib are a class effect. A trial with celecoxib2 revealed a dose-dependent increase in fatal and non-fatal cardiovascular events. Deaths from cardiovascular events were three times more likely on 400 mg/day and six times more likely on 800 mg/day. Hazard ratios for any cardiovascular

62

02 Osteoarthritis

NORMAL PGI2 PGI2


Prothrombotic

LOW-DOSE ASPIRIN TxA2 TxA2


Antithrombotic Prothrombotic Antithrombotic

NSAID

SELECTIVE COX-2 ASPIRIN TxA2

PGI2
Prothrombotic

TxA2
Antithrombotic

PGI2
Prothrombotic Antithrombotic

Figure 12.1 Dynamic balance of prostacyclin (PGI2) and thromboxane A2 (TxA2) under the inuence of NSAIDs.

event were 1.5 and 1.9, respectively, for the two doses. Other studies with parecoxib (a pro-drug for valdecoxib) have conrmed an increased cardiovascular risk, even with short-term usage. Non-selective NSAIDs also inhibit COX-2 to a variable extent. Increased cardiovascular risk has been noted in patients taking diclofenac, ibuprofen or naproxen. Comparing COX-2 inhibitors with non-selective NSAIDs is difcult as the latter also have some COX-2-inhibitory activity. It is agreed that naproxen is the preferred drug to compare with selective COX-2 inhibitors. Some of the conditions for which NSAIDs are used are themselves associated with increased cardiovascular risk (e.g. rheumatoid arthritis and systemic lupus erythematosus). The low-grade inammation associated with arthritic conditions may contribute to cardiovascular risk through increasing endothelial dysfunction, decreasing availability of nitric oxide within vessels and increasing the generation of reactive oxygen species. Since the withdrawal of the two coxib drugs at the end of 2004, there has been considerable activity to clarify whether safety concerns should include all selective COX-2 inhibitors and even extend to some non-selective agents. A large Canadian study3 included 113 927 elderly subjects starting COX-2 inhibitors and who did not have a history of MI. Rofecoxib was associated with increased risk of MI. The increased risk with low-dose rofecoxib was offset by concurrent use of aspirin. There was no increased risk associated with celecoxib or meloxicam use. Kearney et al.4 conducted a meta-analysis of trials published between 1966 and April 2005. Patients taking COX-2 inhibitors had increased risk of vascular events. The annual rates of serious cardiovascular events were 1.2% per year for patients taking COX-2 inhibitors and 0.9% per year for patients receiving placebo. There was no difference in event rate between COX-2-treated patients and

12 NSAIDs cardiac complications

63

those treated with non-selective COX inhibitors. There was, however, considerable heterogeneity amongst the latter agents: relative risk of an event (compared with placebo) was 0.92 (95% condence interval 0.671.26) for naproxen, 1.51 (0.962.37) for ibuprofen and 1.63 (1.122.37) for diclofenac. Another meta-analysis of 114 controlled clinical trials5 has reported increased risk of renal toxicity and arrhythmias with rofecoxib but no increase with other coxib drugs.

Recent Developments
1 A Danish study6 has investigated rates of death and re-admission with MI in 58 432 patients discharged following acute MI. There was a trend towards increased hospitalization for use of both COX-2-selective and non-selective agents. The hazard ratios for death were 2.80 (95% condence interval 2.413.25) for rofecoxib, 2.57 (2.153.08) for celecoxib, 1.50 (1.361.67) for ibuprofen, 2.40

Non-pharmacological treatments been fully explored?

Analgesia (paracetamol up to 4 g/day)

Low cardiac risk*

Angina

Previous MI or stroke

NSAID High gastric risk or symptoms Or Add PPI or misoprostol

NSAID aspirin

Stronger analgesia (e.g. tramadol) High gastric risk or symptoms Naproxen aspirin

Add PPI or misoprostol COX-2

Review need for NSAID regularly (every 3 months)

Figure 12.2 Use of NSAIDs in patients with cardiovascular risk. * Measure lipids, blood glucose and blood pressure; it is useful to use a cardiovascular risk calculator. COX-2, COX-2-selective agent; NSAID, nonselective anti-inammatory drug; PPI, proton pump inhibitor.

64

02 Osteoarthritis (2.092.80) for diclofenac and 1.29 (1.161.43) for other NSAIDs. The risk of death was related to dose of drug taken. 2 A nationwide Finnish study7 examined the use of NSAIDs in 33 309 patients admitted with their rst MI. These subjects were age- and gender-matched with controls. The apparent risk with NSAIDS for rst MI was 1.40 (95% condence interval 1.331.48). No difference was noted between non-selective agents, semiselective COX-2 inhibitors or selective COX-2 inhibitors. There was no suggestion that any of the drugs was protective against MI. 3 A study to compare etoricoxib with diclofenac is under way.8 This study in patients with rheumatoid arthritis or osteoarthritis should answer the question of whether the risk associated with use of COX-2 inhibitors is any greater than that for conventional NSAIDs. The study has enrolled nearly 35 000 patients.

Conclusion
Over 30 million people worldwide take NSAIDs regularly. Recent studies have shown that use of these drugs is associated with increased risk of MI. If non-pharmacological means and simple analgesia fail to control the above patients symptoms, then it is reasonable to consider use of NSAIDs. It is probably best, initially, to use one of the conventional agents and naproxen is the drug of choice, the dose and duration of exposure being kept to a minimum. There is an argument for using higher-dose aspirin to overcome the effects of COX-2 inhibition. A scheme for considering NSAID use is presented in Figure 12.2. There is considerable heterogeneity of cardiovascular risk amongst both COX-2selective and non-selective NSAIDs, although it seems likely that increased risk is related to the ability of the individual drug to inhibit the COX-2 isoenzyme.

Further Reading
1 2 Hermann M, Ruschitzka F. Coxibs, non-steroidal anti-inammatory drugs and cardiovascular risk. Intern Med J 2006; 36: 30819. Solomon SD, McMurray JJ, Pfeffer MA et al; Adenoma Prevention with Celecoxib (APC) Study Investigators. Cardiovascular risk associated with celecoxib in a clinical trial for colorectal adenoma prevention. New Engl J Med 2005; 352: 107180. Lvesque L, Brophy J, Zhang B. The risk for myocardial infarction with cyclooxygenase-2 inhibitors: a population study of elderly adults. Ann Intern Med 2005; 142: 4819. Kearney PM, Baigent C, Godwin J, Halls H, Emberson JR, Patrono C. Do selective cyclooxygenase inhibitors and traditional non-steroidal anti-inammatory drugs increase the risk of atherothrombosis? Meta-analysis of randomised trials. BMJ 2006; 332: 13028. Zhang J, Ding EL, Song Y. Adverse effects of cyclooxygenase 2 inhibitors on renal and arrhythmia events: meta-analysis of randomised trials. JAMA 2006; 296: 161932. Gislason GH, Jacobsen S, Rasmussen JN et al. Risk of death or reinfarction associated with the use of selective cyclooxygenase-2 inhibitors and nonselective nonsteroidal anti-inammatory drugs after acute myocardial infarction. Circulation 2006; 113: 290613.

3 4

5 6

13 Joint replacement surgery


7

65

Helin-Salmivaara A, Virtanen A, Vesalainen R et al. NSAID use and the risk of hospitalization for rst myocardial infarction in the general population: a nationwide case-control study from Finland. Eur Heart J 2006; 27: 165763. Cannon CP, Curtis SP, Bolognese JA, Laine L. Clinical trial design and patient demographics of the Multinational Etoricoxib and Diclofenac Arthritis Long-term (MEDAL) study program: cardiovascular outcomes with etoricoxib versus diclofenac in patients with osteoarthritis and rheumatoid arthritis. Am Heart J 2006; 152: 23745.

P R O B L E M

13 Joint Replacement Surgery


Case History
Mary has been passing on your advice on preventing osteoarthritis (OA) to the members of the local bowls club. The information may have come a little late for George who has severe right hip OA. He is now aged 62 years, is moderately overweight at 89 kg (body mass index 28.7 kg/m2) and has asked about a hip replacement. Will weight reduction delay Georges need for surgery? What are the indications for a knee or hip replacement? What are the options for joint replacement and what is their expected lifespan?

Background
In the United States and United Kingdom (UK), the number and rate has doubled for knee replacements and tripled for hip replacements over the past decade. Future projections are that >750 000 of these procedures will be performed per year by the year 2030. A critical variable in these estimations is the low proportion of individuals who on the basis of symptoms warrant joint replacement but remain unwilling to consider the procedure.

Weight reduction
Given that obesity is modiable by conservative treatment such as weight loss, its importance in reducing the incidence of large joint OA is often advocated, yet determining the impact prospectively of weight reduction is more problematic.1 The effect of adult weight change on risk for total hip replacement resulting from primary OA was studied prospectively in a large Norwegian cohort whose weight had been recorded over a mean 14-year Atlas Medical Publishing Ltd

66

02 Osteoarthritis period from age mid-30s to late 40s. With an average follow-up of nine years, there was no association of weight slope, absolute weight change or relative weight change between screenings with later total hip replacement.2 A number of studies have shown that after large joint replacement weight increases, from a small amount in the pre-operatively non-obese patient to an average 4.8 kg gain in the pre-operatively obese.3,4 Post-operative weight gain has been documented for both hip and knee replacement, with younger hip-replacement patients gaining a signicant amount of weight.5 Obesity therefore needs to be treated as an independent disease that increases the risk of requiring major joint replacement, and is adversely impacted by surgery that is aimed at reducing pain and increasing a persons exercise ability.

Exercise
Exercise is benecial for people with arthritis, and pre-operative functional status is positively related to post-operative functional status. Inclusion of both pre-operative and post-operative strength training in a total hip replacement programme improves physical function at 12 and 24 weeks post-operatively.6 The optimal type and the amount of exercise plus the timing of the pre-operative intervention need to be established.

Indications for hip or knee replacement


These indications are summarized in Figure 13.1. Concerns about the timing of surgery surface for two primary classes of patients: the younger, active candidate and the older candidate. For the younger (less than 60 years of age), active patient, the long-term survival of the prosthesis is the major concern. While there are several studies on long-term outcome, most of these describe prosthesis survival for the more typical older and less active individual. Furthermore, as prosthesis design and materials improve, the newer prostheses will have longer survival. For the less active, older patient in whom long-term prosthesis survival is less of a concern, some information may be provided by comparing outcomes in patients with poor pre-operative symptoms versus those with better pre-operative symptoms as proxies for late versus early surgery. When followed at 6 and 24 months post-operation, those with poorer pre-operative status had poorer outcomes. Patients need to appreciate the small but real risk of perioperative complications and, for younger patients, the potential for revision surgery. However, they also should be appraised that likelihood of functional benet is higher if they enter into surgery with a better functional status.

Perioperative preparations and complications


Rheumatoid arthritis (RA) patients also benet from joint replacement, and in these individuals their immunosuppressant therapy needs to be considered. Weekly methotrexate is the mainstay of most RA regimes, and can be continued up to the time of surgery and recommenced upon return of baseline uid balance and renal function. In a prospective, randomized study of post-operative infection and surgical complications in 388 RA patients in the UK who underwent elective orthopaedic surgery, one group continued methotrexate and the other group ceased methotrexate two weeks pre-operatively and recommenced treatment two weeks post-operatively.6 Complication rates were compared between groups and with 228 RA patients not on methotrexate and undergoing elective orthopaedic surgery. No increase in the duration of early post-operative

13 Joint replacement surgery

67

Indications:

Pain limiting function despite conservative therapy Nocturnal pain disrupting sleep Understanding of potential complications

Contraindications: Severe neurological or cognitive impairment that will impair rehabilitation Vascular disease/unstable angina/anaesthetic risk Chronic sepsis

Plain X-ray

Confirms significant joint pathology

No or mild X-ray changes

Review to exclude periarticular source of pain

Orthopaedic referral

Figure 13.1 Hip and knee joint replacement surgery.

infections or rate of complications was found. Patients who continued methotrexate had signicantly fewer adverse events than either of the other two groups (P <0.003). Additionally, those who did cease methotrexate had a higher incidence of disease ares in the six weeks following surgery than those who continued the medication. The risk of death is approximately 1% in the 90 days following hip replacement and slightly less for knee replacement. Pulmonary embolism and myocardial infarction occur within 90 days in 1% of patients. Dislocation occurs in 3% of total hip replacements. Deep joint infection occurs in about 0.4% of total knee replacements and 0.2% of total hip replacements. While dislocation and infection remain potential complications over the longer term, the risk is much higher in the rst 90 days than subsequently.

How long will it last?


Long-term success of total hip arthroplasty is dependent on prosthetic component xation and the amount of wear and debris generated by the bearing surface. Advances in

68

02 Osteoarthritis both femoral cementing techniques and the design of cemented stems have resulted in near-perfect (98%) survivorship at ten years and good survivorship (93%) at 25 years.7 Comparable survival rates have been reported using cementless techniques for the femoral component. In the acetabular component, ten-year survival rates are similar for cemented and cementless techniques (approximately 95%98%); however, at 15 years, cementless technology supersedes cemented (85%95% versus 70%95%). Cemented total knee arthroplasty (TKA) is the current gold standard, with consistent long-term (1014 years) survival rates of 94%98%. Whilst some cementless TKA techniques have demonstrated good long-term survival, the majority have not reliably resulted in bone ingrowth. Long-term survival rates and functional abilities are comparable in cruciate-retaining and cruciate-substituting prostheses. To improve patient satisfaction and function, implants have an increased arc of exion that may approach 150 degrees of knee exion. Recent literature searches and meta-analyses suggest that resurfacing the patella at TKA likely improves outcomes and long-term, pain-free patella function.

Recent Developments
1 Ogonda et al.8 randomized patients to a standard 16 cm incision or a minimal (10 cm) incision for their hip replacement. They found that although the standard incision group had a higher estimated blood loss, there was no difference in mean units of transfusion or haematocrit at discharge. There were also no signicant differences found in post-operative pain, timed 10 m walk on the second post-operative day, length of hospital stay or functional outcome. 2 Although TKA is currently the gold standard for knee joint replacement, unicompartmental arthroplasty has emerged as a suitable option for advanced medial compartment osteoarthritis, with outcomes comparable with those for TKA. The procedure is generally performed using a minimally invasive approach and leads to more rapid recovery, minimal bone loss, less pain and early discharge compared to TKA. The 1015-year survival rates for unicompartmental knee arthroplasty are high and range from 95%98%.9

Conclusion
Total joint replacement should not be regarded as an intervention of last resort and should be offered to patients when their quality of life is compromised. Pre-operative functional status is the most signicant predictor of post-operative functional status. Surgery should be considered when symptoms warrant it and when the intervention will contribute to a persons post-operative life experience. As procedures, total hip and knee joint replacements (Figure 13.2) are costly, but total hip replacement is among the most cost-effective interventions in medicine. Long-term follow-up is available on older prostheses and relief of pain and restoration of function can be expected in over 90% of patients, with a complication rate that is rare and a mortality rate of <1%.

13 Joint replacement surgery


(A)

69

(B)

Figure 13.2 Total hip and knee joint replacements. (A) An articial hip joint with metallic femoral stem and ultra-high-molecular-weight polyethylene acetabular cup. An articial knee joint in situ, with metallic femoral component and ultra-high-molecular-weight polyethylene bearing. (Both gures redrawn with permission from Fisher 2004.10).

70

02 Osteoarthritis

Further Reading
1 Powell A, Teichtahl AJ, Wluka AE, Cicuttini FM. Obesity: a preventable risk factor for large joint osteoarthritis which may act through biomechanical factors. Br J Sports Med 2005; 39: 45. 2 Flugsrud GB, Nordsletten L, Espehaug B, Havelin LI, Meyer HE. Weight change and the risk of total hip replacement. Epidemiology 2003; 14: 57884. 3 Donovan J, Dingwall I, McChesney S. Weight change 1 year following total knee or hip arthroplasty. ANZ J Surg 2006; 76: 2225. 4 Aderinto J, Brenkel IJ, Chan P. Weight change following total hip replacement: a comparison of obese and non-obese patients. Surgeon 2005; 3: 26972. 5 Heisel C, Silva M, dela Rosa MA, Schmalzried TP. The effects of lower-extremity total joint replacement for arthritis on obesity. Orthopedics 2005; 28: 1579. 6 Grennan DM, Gray J, Loudon J, Fear S. Methotrexate and early postoperative complications in patients with rheumatoid arthritis undergoing elective orthopaedic surgery. Ann Rheum Dis 2001; 60: 21417. 7 Jones DL, Westby MD, Greidanus N et al. Update on hip and knee arthroplasty: current state of evidence. Arthritis Rheum 2005; 53: 77280. 8 Ogonda L, Wilson R, Archbold P et al. A minimal-incision technique in total hip arthroplasty does not improve early post-operative outcomes. A prospective, randomized, controlled trial. J Bone Joint Surg Am 2005; 87: 70110. 9 Yang KY, Wang MC, Yeo SJ, Lon NN. Minimally invasive unicondylar versus total condylar knee arthroplasty early results of a matched pair comparison. Singapore Med J 2003; 416: 55962. 10 Fisher J. Surgery for arthritis: total hip and knee joint replacement. ARC Topical Reviews 2004, Number 3.

S E C T I O N

T H R E E

03

Rheumatoid Arthritis
14 15 16 17 18 19 20 Causes Laboratory and imaging investigations Managing rheumatoid arthritis at onset Evaluating the response to treatment Pregnancy and rheumatic diseases Diet and arthritis Polyarthritis in the elderly

P R O B L E M

14 Causes

Case History
Steves rheumatoid arthritis (RA) has been managed for the past four years with a combination of methotrexate, hydroxychloroquine and sulphasalazine. He is rheumatoid factor (RF) positive and has signicant joint damage. He wishes to explore with you why he may have developed RA. His daughter, aged 24 years, has recently been told that she is RF positive. What genetic factors inuence the development or severity of RA? What environmental factors are known to be involved? How does being RF positive inuence the likelihood of developing RA?

Background
RA is a chronic inammatory disease affecting the synovium and leading to joint damage and absorption of adjacent bone. Peak age of onset is in the fth decade and females are Atlas Medical Publishing Ltd

72

03 Rheumatoid Arthritis two to three times more likely to be affected, although the sex distribution becomes less apparent with increasing age. Apart from the disability caused by the disease, it also has a signicant impact on life expectancy, with RA patients living on average three to ten years less than unaffected individuals. The overall incidence of the condition is 2050 per 100 000 per year. This may have decreased since the 1960s. The prevalence varies in different countries:1 b 0.5%1.1% in Northern Europe and North America b 0.3%0.7% in Southern Europe and other warmer countries b 0.5% in developing countries In the pre-clinical phase of RA, immunoglobulin G (IgG) may be elevated and RF (polyclonal antibodies directed at the Fc fragment of IgG) and anti-cyclic citrullinated peptide (anti-CCP) antibodies may be detected. Citrullination is a post-translational modication of arginine by peptidylarginine deaminase enzymes. Specic diseasemodifying treatment is not indicated at this early phase of the disease.

Genetics of RA
The importance of genetic factors is clear from family studies: prevalence is 2%12% in rst-degree relatives of RA sufferers i.e. approximately ten times that of the background population. Concordance rate is 10%30% in monozygotic twins and 5%10% in same-sex dizygotic twins. The genetic component of RA has been studied by genomewide linkage analyses and by study of individual candidate genes.2 The major susceptibility gene for RA is the DRB1 gene located in the class II histocompatibility region on chromosome 6p. Risk of RA is associated with the alleles HLA-DRB1*0401, DRB1*0404, DRB1*0405, DRB1*0408, DRB1*0101 and DRB1*0102. These alleles have similar or identical sequences at the third hypervariable region in the b chain between amino acid positions 70 and 74. According to the shared epitope (SE) hypothesis, this sequence is directly involved in the immunopathogenesis of RA, although the precise mechanism is not clear. The human leukocyte antigen (HLA) component accounts for around 30% of the genetic risk. The susceptibility alleles linked to RA are present in around 70% of patients but also in 40% of the general population. Thus genotyping is not currently useful in clinical practice. Other genes associated with RA include: b Tumour necrosis factor (TNF)-a located in the class II part of the histocompatibility complex on 6p. Polymorphisms inuence the level of TNF-a expression b The TNF-a receptor type 2 gene (TNFR2) located at 1p36. The 196R allele of this gene is linked with RA b Peptidylarginine deaminases (PADIs), also located at 1p36. These enzymes catalyse the citrullination of arginine residues and, therefore, determine the level of citrullinated peptides b SLC22A4 (solute carrier family 22 A4) at chromosome 5q31 also implicated in Crohns disease b PTNP22 (tyrosine phosphate non-receptor type 22) at 1p13 also linked with type 1 diabetes and autoimmune thyroid disease

14 Causes

73

Environmental factors
These triggers may begin to operate long before clinically apparent disease develops. The pre-clinical phase of RA may last for up to 1520 years. During this phase the prevalence of autoantibody markers increases amongst susceptible individuals as the onset of clinical disease approaches. IgA rheumatoid factors are probably the rst markers to appear in most patients. Neither RF nor anti-CCP antibodies are entirely sensitive, but together the markers are reasonably specic 85% to 90% for RF and 98% for anti-CCP. Markers of inammation including total IgG, erythrocyte sedimentation rate (ESR) and C-reactive protein (CRP) may be increased during the clinical phase. The patient may experience symptoms of malaise and fatigue and intermittent joint symptoms (palindromic rheumatism). Risk factors for RA are summarized in Figure 14.1. Environmental triggers may be more important than genetic factors.3 Some studies have shown much higher association than this in families but it is difcult to disentangle genetic inuences from social and other environmental inuences.

Susceptibility

Genetic

Family history Other autoimmune disease HLA-DRB1 SE etc. Female gender

Subclinical

Environmental

Smoking Infections (EBV etc) Low UV (vitamin D) Aggravating

Diet

Protective

Disease onset

Occupation

Omega-3 Red meat Mediterranean Caffeine ? diet Fruit Vitamin C Agriculture Postal and print Silica Mineral oil

Progression

Smoking Aging Oestrogen withdrawal Decreased androgen levels Periodontal disease

Figure 14.1 Development of rheumatoid arthritis. EBV, EpsteinBarr virus; UV, ultraviolet light.

74

03 Rheumatoid Arthritis Smoking is probably the most potent environmental trigger, with smokers at two- to four-fold increased risk of RA. The risk is dose dependent with heavy smokers being at higher risk than light smokers. A number of mechanisms could account for the link between smoking and RA these include the immunomodulatory effect of tobacco components, oxidative stress and oestrogenic effects, and the effect of smoke in promoting peptide citrullination. Smoking also increases the risk for anti-double-stranded DNA (anti-dsDNA)-positive systemic lupus erythematosus. Smoking is associated with more severe rheumatoid disease: erosive arthropathy, rheumatoid nodules and vasculitis. The Nurses Health Study was a large cohort study investigating the effect of aspirin and vitamin E in preventing cardiac disease. In a prospective study of 103 818 women from this cohort followed from 1976 to 2002,4 there were 680 new cases of RA. Women with a history of ten pack-years or more were at increased risk, and the risk of RA was related both to the intensity and duration of smoking. The increased risk persisted for up to 20 years after cessation of smoking. Oestrogen exposure in women has a protective effect. This is consistent with increased risk of the disease after the menopause and the slight protective effect of using hormone replacement therapy. It is not clear why the prevalence of RA is so much higher in women compared with men. As with other autoimmune diseases, pregnancy is protective, with the incidence during pregnancy being 70% lower than that for age- and sex-matched controls. However, there is a rebound increase after pregnancy with risk increased by as much as ve-fold in the months after delivery. Breast-feeding also appears to increase risk. This may be due to the low-oestrogen state or to pro-inammatory actions of prolactin. The relationship between sex steroid status and susceptibility to autoimmune disease is complex.5 The inuence of social, employment and educational status does not seem to be great in RA. The disease is more prevalent in urban populations, but this could be partly due to migration of patients with chronic RA to metropolitan centres. Certain occupations do seem to have increased risk: men who work in agriculture and in the paper and transport industries, and female printer and postal workers, appear to be at increased risk. Workers exposed to silica dust (e.g. from drilling or crushing rock) are at up to three-fold increased risk. Mineral oil exposure has been noted to increase risk of a number of autoimmune diseases including RA. Low intake of fruit and vitamin C relates to increased risk, while high intake of red meat carries a high risk. Regular consumption of oily sh (a source of omega-3 fatty acids) is protective, as is adherence to a Mediterranean diet. Low vitamin D status is probably a risk factor for RA, but there is no evidence that vitamin D supplementation protects against the disease. Some immune-mediated diseases, particularly type 1 diabetes and multiple sclerosis, are more common in latitudes where there is lower exposure to ultraviolet light.6 While vitamin D status is the most plausible explanation for this association, other metabolic factors may equally be involved. The notion of an infectious trigger for RA is appealing. Evidence in favour of this includes the decreasing incidence of the condition and the associations with previous blood transfusion and pet ownership. There is no evidence for a seasonal incidence of the condition. Amongst candidate organisms are EpsteinBarr virus (EBV), rubella, parvovirus and Borrelia burgdorferi. Of these, the strongest evidence is with EBV, the glycoprotein of which has cross-reactivity with the HLA-DRB1 SE. Increased incidence and

14 Causes

75

levels of EBV DNA have been reported in peripheral blood mononuclear cells and synovial uid of RA patients and the virus is well documented to have immunomodulatory properties.

Recent Developments
1 Two recent studies7,8 have pooled data from the United Kingdom and United States to demonstrate that there are important susceptibility loci at chromosomes 6q and 16p. There appears to be an interaction between these loci and HLA determinants. Further study of the genes at these loci may shed more light on the pathogenesis of RA. 2 In a recent Swedish casecontrol study,9 positive immunostaining for citrullinated peptides was found in the cells from bronchoalveolar lavage of smokers. Furthermore, there was a strong association between smoking status and the presence of anti-citrulline antibodies. There was a positive interaction between anticitrulline autoimmunty and the presence of SE in the HLA-DR region. Individuals who smoked and had double copies of the HLA-DR SE had a 21-fold increased risk of RA. 3 In a mouse model of RA, oestrogen deciency by oophorectomy accelerated the development of arthritis and increased levels of RF, anti-dsDNA and anti-collagen antibodies.10 4 Periodontal disease is mainly caused by infection with Gram-negative anaerobes. These lead to increased production of cytokines that damage gingival connective tissue and increase resorption of dental alveolar bone. Marotte et al.11 have shown that the presence of HLA-DR SE correlates with increased periodontal bone destruction. Furthermore, periodontal bone destruction strongly correlated with wrist bone resorption in patients with RA.

Conclusion
RA has a signicant genetic component but perhaps not as strong as was once thought. The most powerful genetic inuence is the HLA-DRB1-related SE. Of known environmental triggers, smoking is the most powerful. Women are more prone to the disease but low oestrogen status appears to increase susceptibility. The presence of RF is not absolutely specic for the disease but is generally highly predictive. The antibody does occur in a proportion of the general population and may occur in other autoimmune connective tissue disorders. A long pre-clinical phase with positive RF and anti-CCP antibodies, sometimes with mild and variable symptoms, is common in RA.

Further Reading
1 Alamanos Y, Drosos AA. Epidemiology of adult rheumatoid arthritis. Autoimmun Rev 2005; 4: 1306.

76

03 Rheumatoid Arthritis
2 Dieude P, Cornelis F. Genetic basis of rheumatoid arthritis. Joint Bone Spine 2005; 72: 5206. 3 Oliver JE, Silman AJ. Risk factors for the development of rheumatoid arthritis. Scand J Rheumatol 2006; 35: 16974. 4 Costenbader KH, Feskanich D, Mandl LA, Karlson EW. Smoking intensity, duration, and cessation, and the risk of rheumatoid arthritis in women. Am J Med 2006; 119: 503.e19. 5 Cutolo M, Lahita RG. Estrogens and arthritis. Rheum Dis Clin North Am 2005; 31: 1927. 6 Ponsonby AL, Lucas RM, van der Mei IAF. UVR, vitamin D and three autoimmune diseases multiple sclerosis, type 1 diabetes, rheumatoid arthritis. Photochem Photobiol 2005; 81: 126775. 7 Etzel CJ, Chen WV, Shepard N et al. Genome-wide meta-analysis for rheumatoid arthritis. Hum Genet 2006; 119: 63441. 8 John S, Amos C, Shepard N et al. Linkage analysis of rheumatoid arthritis in US and UK families reveals interactions between HLA-DRB1 and loci on chromosomes 6q and 16p. Arthritis Rheum 2006; 54: 148290. 9 Klareskog L, Stolt P, Lundberg K et al. A new model for an etiology of rheumatoid arthritis: smoking may trigger HLA-DR (shared epitope)-restricted immune reactions to autoantigens modied by citrullination. Arthritis Rheum 2006; 54: 3846. 10 Yoneda T, Ishimaru N, Arakaki R et al. Estrogen deciency accelerates murine autoimmune arthritis associated with receptor activator of nuclear factor-kB ligandmediated osteoclastogenesis. Endocrinology 2004; 145: 238491. 11 Marotte H, Farge P, Gaudin P, Alexandre C, Mougin B, Miossec P. The association between periodontal disease and joint destruction in rheumatoid arthritis extends the link between the HLA-DR shared epitope and severity of bone destruction. Ann Rheum Dis 2006; 65: 9059.

15 Laboratory and imaging investigations P R O B L E M

77

15 Laboratory and Imaging Investigations


Case History
Joan is a 52-year-old schoolteacher who you have diagnosed with rheumatoid arthritis (RA). You have discussed a range of treatment options with her and also discussed with her a number of investigations. She is keen to know about the monitoring and investigation required for different medications and what information will be gained from other scheduled investigations. What investigations should be undertaken at the onset of RA? What investigations help with monitoring the activity of the condition? How should follow-up be planned?

Background
Making a diagnosis of RA
Joan has an inammatory arthritis; it is established as a chronic problem, having extended beyond eight to ten weeks, which makes a post-viral arthropathy unlikely. The American College of Rheumatology classication for RA requires four of seven criteria to be met, with the clinical features of arthritis being present for at least six weeks: b Morning stiffness of greater than one hour duration b Objective evidence of joint inammation, such as soft tissue swelling or uid in 3 of 14 dened joint areas (right or left proximal interphalangeal [PIP], metacarpophalangeal [MCP], wrist, elbow, knee, ankle and metatarsophalangeal joints) b At least one of the joint areas demonstrating inammation must be in the hands b Simultaneous involvement of the same joint area bilaterally in at least one pair b Rheumatoid nodules b A positive rheumatoid factor (RF) using a method that is positive in <5% of normal subjects b Typical changes of RA in hand and wrist X-rays, which must include erosions or unequivocal periarticular osteopenia RA is one of the most common autoimmune diseases, affecting 1% of the population. Other diagnoses, such as psoriatic arthritis or systemic lupus erythematosus (SLE), could present with the same clinical scenario and the history and examination undertaken will have considered these in the differential diagnosis. Atlas Medical Publishing Ltd

78

03 Rheumatoid Arthritis

Investigations for RA
RF is a polyclonal antibody predominantly of the immunoglobulin M (IgM) and IgG classes, which targets the Fc region of the IgG. Whilst commonly thought to have diagnostic signicance by both doctors and patients, it has a very poor predictive value, being present in only 70% of RA cohorts and also present in 5% of the normal population. RF positivity increases with age and is positive in any circumstance with prolonged antigen stimulation, so that chronic infections such as malaria and tuberculosis account for the bulk of RF positivity globally. Similarly in individual patients, age and chronic sepsis need to be considered in addition to the pattern of joint disease before attributing a positive RF with a diagnosis of RA. RF positivity associates with increased joint erosions and vasculitis. Radiographs of the hands and wrists, feet and cervical spine were historically undertaken at baseline and then one- to two-yearly to monitor for erosive disease. Fortunately, the model of therapy has changed so that the goal is suppression of inammation as soon as possible after diagnosis is established; this is combined with the knowledge that joint erosions detected by magnetic resonance imaging (MRI) or musculoskeletal ultrasound occur up to 18 months before their detection on plain X-ray. In Joans case, radiographs will add little, demonstrating the soft tissue swelling that was observed on clinical examination and therefore reinforcing the clinical pattern noted. In later disease, different patterns of involvement can be noted for psoriasis, SLE and RA. Psoriasis classically develops pencil in cup erosions of both PIP and distal interphalangeal joints with the proximal phalanx narrowing to the sharpened pencil, burrowing into the cup-like, widened distal phalanx. SLE is almost always a non-erosive disease. X-ray changes in RA mirror the joints clinically involved, primarily affecting the hands, the PIP, MCP and wrists. Soft tissue swelling occurs early, with periarticular osteopenia noted, and subsequently there is symmetrical joint space narrowing. Erosions when they occur begin at the site of synovial lining of the joint capsule reecting onto the cartilage/periosteum junctions, so that they occur slightly distant from the joint surface. Joans erythrocyte sedimentation rate (ESR) and serum C-reactive protein (CRP) will both be increased and will act as a means to monitor therapy, with the goal being their normalization. Both ESR and CRP are measures of acute phase proteins produced in the liver in response to monocyte-derived cytokines. CRP testing is more expensive than ESR but is the preferred measurement for monitoring success in suppressing joint inammation, particularly in RA. The situation is more complex in SLE, with some individuals having a cytokine pattern resulting in an elevated CRP, and others whose disease pattern is not associated with a rise in CRP.1 The blood lm commonly shows an inammatory thrombocytosis, with more established disease showing a normochromic normocytic anaemia; this can eventually mimic the hypochromic microcytic picture of iron deciency, due to the inammatory process inhibiting the utilization of iron stores. Concomitant therapies such as non-steroidal anti-inammatory drugs (NSAIDs) may cause genuine iron deciency through gastrointestinal blood loss. Corticosteroids induce a neutrophil leukocytosis. The presence of lymphopenia in Joan would have caused a reconsideration of the diagnosis of SLE. Clinical examination should include urinalysis, with the presence of proteinuria or leukocytes warranting further microscopy and evaluation. Proteinuria would suggest glomerular inammation. If SLE remains part of the differential diagnosis, an antinuclear antibody (ANA) test can be requested. A signicant positive result is a titre of 1:160

15 Laboratory and imaging investigations

79

or greater, and again in itself has poor diagnostic or prognostic value unless interpreted as part of the clinical presentation. Evaluation of electrolytes, renal function and liver function should be undertaken. NSAIDs may exacerbate hyperkalaemia and impair renal function, with several of the disease-modifying agents notable for their potential to cause hepatitis with elevation of transaminases.

Safety monitoring of RA medications


A guide to the safety monitoring for the agents used for treatment of RA is shown in Table 15.1. Methotrexate, alone or in combination with other agents, is the basis of most RA regimes. Biological agents that target specic cytokines such as tumour necrosis factor (TNF) and interleukin-1, or that block cell-to-cell signalling (such as cytotoxic lymphocyte-associated antigen linked to immunoglobulin [CTLA4-Ig; abatacept]) are becoming increasingly used in those who do not respond quickly to standard therapy. Screening for hepatitis B and C is recommended, although a positive result is not an absolute contraindication to therapy. Recent studies have suggested an enhanced hepatitis C antiviral effect of interferon and ribavirin in the setting of concomitant etanercept.
Table 15.1 Guide to safety monitoring of commonly used antirheumatic medications
NSAIDs
Baseline BP, oedema, K , creatinine
+

Methotrexate

Sulphasalazine

Leunomide
Chest X-ray, hepatitis B & C serology, FBC, LFT, chest auscultation

Anti-TNF therapy
Chest X-ray, hepatitis B & C serology; screen for latent TB as per local guidelines

Chest X-ray, FBC, LFT hepatitis B & C serology, FBC, LFT, chest auscultation FBC, LFT at four- to six- FBC, LFT monthly 3, weekly intervals; then three-monthly chest auscultation at each clinical visit

Periodic

K+ and creatinine at one week; BP and oedema at each clinical visit

FBC, LFT at four- to As clinically indicated six-weekly intervals; chest auscultation at each clinical visit

BP, blood pressure; FBC, full blood count/screen; K+, serum potassium; LFT, liver function tests; TB, tuberculosis.

Liaison between specialist and primary care practitioner


A close collaboration and correspondence ow is required. The management plan will be developed between Joan and her specialist, but much of its implementation will be with her general practitioner. They will need to consider the impact that any new illness will have on her RA management, and also the impact of her RA management on intercurrent illness as well as preventative activities such as vaccinations. Common points of ambiguity are who will undertake prescribing and who will monitor and communicate with Joan about any abnormalities detected on periodic screening. Joan should be encouraged to attend her general practitioner regularly and this eventually should allow alternating visits between the two physicians, with specialist attendance four- to six-monthly.

Recent Developments
1 Detection of antibodies to cyclic citrullinated peptides (CCP) has a sensitivity similar to RF, but specicity is much higher in the range of 95%99%.2,3 In patients with early undifferentiated arthritis, the presence of anti-CCP antibodies has an odds

80

03 Rheumatoid Arthritis ratio of 38 for subsequent development of RA, and is an adverse prognostic marker for both clinical disease and radiological progression. Repeated measurement currently has no clinical utility, with therapeutic intervention leading to only modest reduction in levels that do not correlate with clinical indicators. 2 Anti-TNF therapy appears to protect against joint damage but may have little inuence on systemic inammatory markers such as ESR and CRP.4 High-sensitivity CRP measurement can generate specic values lower than the previous threshold in RA subjects with a normal CRP; subjects with high-sensitivity CRP values 28 mg/l had higher disease activity scores, less remission rates and greater depression and helplessness than those with values <2 mg/l. High-sensitivity CRP also performs consistently better than ESR at predicting other disease activity variables.5 A patients joint count is, not surprisingly, a good marker of activity and subsequent prognosis. In subjects with early arthritis of less than one years duration, a joint count of greater than seven actively diseased joints accurately predicted subsequent performance and function.6 3 Detailed radiological imaging and scoring has been undertaken in trials of biological therapy, which suggest that progression can be halted and that joint erosions can mature with regrowth of cortical bone and inlling. Plain radiographs of the hands and feet remain quick and affordable imaging modalities, but erosions can be detected by more sensitive imaging techniques around 18 months before they are detected on plain lms. Imaging modalities that can be used include ultrasound, MRI and computed tomography (CT). Most erosions at the wrist will be detected by both CT and MRI, although CT will detect more erosions particularly at the metacarpal bases, possibly because of its ability to delineate cortical bony margins.7 High-resolution ultrasound requires good apposition of the probe and an ability to access the bone surface. As a consequence, wrist examination is suboptimal, with MCP joints 2 and 5 exhibiting better correlation with MRI than ultrasound of MCP 3 and 4, which can only be accessed in two planes.8

Conclusion
A clear and positive approach is needed as you discuss the investigations that will assist in conrming the diagnosis, and in determining disease activity and its current impact on function. The shared care between specialist and primary care physician is aimed at inhibition of the inammatory process and therefore halting the local and systemic impact of the disease. Involving the patient in the goal setting at the start is helpful in gaining their understanding of the strategies you are using to bring the disease under control. A clear goal of the absolute minimum number of tender and swollen joints, lowering of inammatory markers and improvement in a patients quality of life is critical. Antirheumatic medications are highly effective in bringing disease under control, and that should always be the opening when discussing any new medications: to clearly put the benets so that this can be weighed against the potential adverse events, which themselves are discussed in the context of how to minimize them and how to detect them early. RA remains a chronic disorder that, while not curable, is eminently treatable with an increasing range of options and exible delivery systems to maintain a high quality of life in nearly all patients.

15 Laboratory and imaging investigations

81

Further Reading
1 Liou LB. Different monocyte reaction patterns in newly diagnosed, untreated rheumatoid arthritis and lupus patients probably confer disparate C-reactive protein levels. Clin Exp Rheumatol 2003; 21: 43744. Riedemann JP, Muoz S, Kavanaugh A. The use of second generation anti-CCP antibody (anti-CCP2) testing in rheumatoid arthritis a systematic review. Clin Exp Rheumatol 2005; 23 (5 Suppl 39): S6976. Zendman AJW, van Venrooij WJ, Pruijn GJM. Use and signicance of anti-CCP autoantibodies in rheumatoid arthritis. Rheumatology 2006; 45: 205. Smolen JS, van der Heijde DM, St Clair EW et al. Predictors of joint damage in patients with early rheumatoid arthritis treated with high-dose methotrexate with or without concomitant iniximab: results from the ASPIRE trial. Arthritis Rheum 2006; 54: 70210. Dessein PH, Joffe BI, Stanwix AE. High sensitivity C-reactive protein as a disease activity marker in rheumatoid arthritis. J Rheumatol 2004; 31: 10957. Gerber LH, Furst G, Yarboro C, el-Gabalawy H. Number of active joints, not diagnosis, is the primary determinant of function and performance in early synovitis. Clin Exp Rheumatol 2003; 21 (5 Suppl 31): S65-70. Perry D, Stewart N, Benton N et al. Detection of erosions in the rheumatoid hand; a comparative study of multidetector computerized tomography versus magnetic resonance scanning. J Rheumatol 2005; 32: 25667. Szkudlarek M, Klarlund M, Narvestad E et al. Ultrasonography of the metacarpophalangeal and proximal interphalangeal joints in rheumatoid arthritis: a comparison with magnetic resonance imaging, conventional radiography and clinical examination. Arthritis Res Ther 2006; 8; R52.

3 4

5 6

82

03 Rheumatoid Arthritis

P R O B L E M

16 Managing Rheumatoid Arthritis at Onset

Case History
Jason, aged 48 years, has returned for his second visit with suspected rheumatoid arthritis (RA). He has a three-month history of increasing pain involving his hands, wrists and feet. He is a non-smoker. You note active synovitis affecting most of his proximal interphalangeal joints and metacarpophalangeal joints, both wrists and his metatarsophalangeal joints. Erythrocyte sedimentation rate (ESR) is 63 mm/h, serum C-reactive protein (CRP) 26 mg/l. He has strongly positive rheumatoid factor (RF) and elevated anti-cyclic citrullinated peptide (anti-CCP) antibodies. What medications would you recommend to Jason? Will providing Jason with educational material and support impact on his RA?

Background
To impact on the morbidity of RA, long-term management of patients is required, based not only on drug treatment, but also on non-pharmacological approaches such as physiotherapy and psychosocial support. The pharmacological treatment of RA is still problematical and as yet there are no reliably curative or disease-remitting therapies, although considerable gains have been made recently with the advent of biological therapies. However, RA often progresses to disability. It is hoped that the full application of the older disease-modifying antirheumatic drugs (DMARDs) together with the newer DMARDs and biological agents can improve this outlook. First-line pharmacological therapy for all of the inammatory arthritic diseases includes the non-steroidal antiinammatory drugs (NSAIDs). These are useful symptomatically but have no clear effect on the progression of RA, with virtually all patients requiring additional treatment with a DMARD.

Omega-3 oil supplementation


Inammation contributes to a range of acute and chronic human diseases and is characterized by the production of inammatory cytokines, arachidonic acid-derived eicosanoids (prostaglandins, thromboxanes, leukotrienes), reactive oxygen species and adhesion molecules. Long-chain omega-3 fatty acids eicosapentaenoic acid and Atlas Medical Publishing Ltd

16 Managing rheumatoid arthritis at onset

83

docosahexaenoic acid have anti-inammatory properties when taken in doses greater than 2.7 g daily and are of proven benet in RA, providing reduced morning stiffness, decreased tender joint count and a lessened need for NSAIDs. At sufciently high intakes, long-chain n-3 polyunsaturated fatty acids (PUFAs), as found in oily sh and sh oils, decrease production of inammatory eicosanoids, cytokines and reactive oxygen species and decrease the expression of adhesion molecules. Long-chain n-3 PUFAs act both directly (e.g. by replacing arachidonic acid as an eicosanoid substrate and inhibiting arachidonic acid metabolism) and indirectly (e.g. by altering the expression of inammatory genes through effects on transcription factor activation). Long-chain n-3 PUFAs also give rise to a family of anti-inammatory mediators termed resolvins. The required dose of omega-3 sh oil can be taken using a large number of sh oil capsules (more than nine standard sh oil capsules containing oil from eviscerated sh bodies [30% omega-3] daily); however, the cost can be prohibitive. A less expensive alternative is to take about 1015 ml of bottled sh body oil daily. The palatability and hence long-term acceptance of sh oil can be improved by using a two-glass technique. This involves oating the dose of sh oil on 30 ml of fruit or vegetable juice in a small glass. The contents are swallowed without contact with the lips, avoiding the sh oil taste. This is followed by slowly sipping a further 40 ml of juice (from a separate glass) to rinse the mouth. Reux of the sh oil can be avoided by taking it immediately before a solid meal and without any extra uid to encourage mixing of the sh oil with the food and its transport from the stomach into the small intestine. Passage of the oil from the stomach can also be facilitated by lying on the left side for 15 minutes after the dose, or by dividing the daily dose and having each half immediately before the morning and evening meals.1

DMARDs
A recent change in the use of DMARDs is that they are generally commenced as soon as a diagnosis of an inammatory arthritis is made, in order to have the maximal effect on slowing the progress of the disease.2 The rationale for early initiation of therapy is the demonstration of rapid functional decline, and evidence that many of the deleterious effects of RA, such as erosions of bone, occur within the rst one to two years of disease. This approach contrasts to the older disease-management regimens in which DMARDs were only commenced on the appearance of bony erosions on joint X-rays. Many patients with RA and other inammatory arthritic diseases are also administered intra-articular and/or low-dose oral corticosteroids. They are frequently administered as bridging therapy until the effect of a DMARD is fully established. In some patients they are used long term with a DMARD, in order to satisfactorily suppress the disease. Corticosteroids may also be used alone for disease control in the elderly if this can be achieved with a daily dose equivalent of 7.5 mg prednisolone or less.

Biological agents
These are inhibitors of either tumour necrosis factor (TNF) or interleukin-1. These drugs produce rapid control of RA in a high percentage of cases, and joint destruction may be halted and even reversed in some patients. There are, however, potential problems with the biological agents, particularly the activation of infections, as well as very high nancial costs.

84

03 Rheumatoid Arthritis The most recent strategy to optimize treatment of RA has been the introduction of initial combination therapy. This approach is based on the experience of oncologists, with the combination of different mechanisms of action leading to additive or synergistic effects without signicantly increasing toxicity. Studies in which one arm receives intermediate- to high-dose corticosteroids have demonstrated a clinical superiority that becomes less noticeable, however, as the steroid dose reduces or is withdrawn. Interestingly, in these studies a radiographic benet does continue to be seen for several years, suggesting that the initial, rapid clinical and laboratory improvement does retard or halt the structural damage that is otherwise occurring.3 The rst-line choices of a single agent in RA consist of methotrexate, leunomide and sulphasalazine. Methotrexate is currently the gold standard DMARD, with doses accelerated to achieve 20 mg/week within 68 weeks of commencement, with folic acid supplementation of 5 mg/week. Objective outcomes need to be recorded including swollen and tender joint counts, inammatory markers (ESR, CRP) and a quality of life/functional index. Failure to achieve remission of disease activity after a maximum of 34 months of therapy should result in an escalation or switching of therapy. An algorithm of treatment is suggested in Figure 16.1.

Patient education
Patient education is thought to be benecial in helping patients cope with their disease and cooperate with its complex management. The focus is to teach patients to adjust their daily activities as dictated by disease symptoms. In addition to teaching patients what they should do, patients are also instructed on how to approach situations and to make adjustments that are appropriate for each individual and their needs. However, it is not clear which educational interventions are most effective in improving health status for patients with chronic disease. A systematic review of randomized controlled trials focused on the effects of patient education on pain, functional disability, joint counts, patient and physician global assessment, affect scores and measures of acute phase reactants.4 The quality of studies identied was not very high, with issues of randomization and concealment impacting adversely on their quality. At rst follow-up there is a small but signicant benecial effect, particularly for functional disability. The magnitude of the effect is small, however, similar to that gained from the use of the weaker DMARDs. The duration of effect is also brief, being lost at follow-up 314 months later. Therefore strategies to preserve and extend the initial benecial effects of education over a longer time need to be explored.

Recent Developments
1 The clinical response to treatment of RA can be assessed by the proportion of patients who have a 70% or greater improvement according to seven clinical and laboratory measures of disease activity; this is designated as having an American College of Rheumatology (ACR)-70 response. ACR-70 responses were found in 19%21% of patients receiving methotrexate monotherapy in trials in early RA but in 33%40% of those receiving TNF inhibitors with methotrexate. In established RA, ACR-70 responses occurred in fewer than 5% of patients receiving methotrexate monotherapy but in 10%27% of those receiving the

16 Managing rheumatoid arthritis at onset

85

Establish RA diagnosis or RF unclassified oligoarthritis/polyarthritis or RF unclassified oligoarthritis/polyarthritis duration

3 months

Document baseline disease activity

Patient education on disease and its therapy Commence analgesia, NSAID as needed Omega3 oil supplementation Consider low-dose steroid Start DMARD and safety monitoring

Methotrexate 2025 mg/wk plus folate

Evaluate at 1012 weeks

Adequate response

Inadequate response

Periodic review

Combination therapy Biological agents

Figure 16.1 Antirheumatic therapy in early RA. (Adapted with permission from the American College of Rheumatology Subcommittee on Rheumatoid Arthritis Guidelines 20025 and Smolen et al. 2005.6)

combination. TNF inhibitors substantially reduce the erosive damage assessed on X-ray and magnetic resonance imaging. They also decrease the self-assessed disability measured using the Health Assessment Questionnaire and improve the quality of life assessed using SF-36.7 2 Cigarette smoking remains the most important lifestyle risk factor and one that is modiable. Males are three times more likely to develop RA if they smoke, with the risk increasing 13-fold in both sexes for heavy smokers (4150 pack-years). The inuence of smoking is substantially greater in those with a background HLA-DRB1

86

03 Rheumatoid Arthritis shared epitope genetic susceptibility. Stopping smoking can reduce this risk, although the benet is not seen until at least ten years after quitting, when the increased risk halves.8

Conclusion
Early recognition of RA and prompt disease suppression with DMARDs and prednisolone is the key to the initial improvement of patients quality of life and to minimizing subsequent progressive joint damage. Methotrexate retains its key role in the early treatment of RA, and whilst there remains controversy over unequivocal demonstration of the superiority of combination therapy, several studies do support the concept. Some of the best evidence in support of combination therapy is that of TNF blockers when combined with methotrexate, in which the combination is superior to monotherapy with either agent with respect to clinical, functional and radiographic outcome. The strategy of aiming for tight disease control with rapid switching of regimens if patients do not improve when assessed against measurable and recorded outcome is also a key advance. The literature also suggests that the addition of intermediate- to high-dose prednisolone to DMARDs, whether as monotherapy or combination therapy, is benecial if introduced early in the course of RA.

Further Reading
1 2 Cleland LG, James MJ, Proudman SM. Fish oil: what the prescriber needs to know. Arthritis Res Ther 2006; 8: 20210. Nell VP, Machold KP, Eberl G, Stamm TA, Uffmann M, Smolen JS. Benet of very early referral and very early therapy with disease-modifying anti-rheumatic drugs in patients with early rheumatoid arthritis. Rheumatology 2004; 43: 90614. Smolen JS, Aletaha D, Keystone E. Superior efcacy of combination therapy for rheumatoid arthritis: fact or ction? Arthritis Rheum 2005; 52: 297583. Riemsma RP, Taal E, Kirwan JR, Rasker JJ. Systematic review of rheumatoid arthritis patient education. Arthritis Rheum 2004; 51: 104559. American College of Rheumatology Subcommittee on Rheumatoid Arthritis Guidelines. Guidelines for the management of rheumatoid arthritis. Arthritis Rheum 2002; 46: 32846. Smolen JS, Aletaha D, Machold KP. Therapeutic strategies in early rheumatoid arthritis. Best Pract Res Clin Rheumatol 2005; 19: 16377. Scott DL, Kingsley GH. Tumor necrosis factor inhibitors for rheumatoid arthritis. N Engl J Med 2006; 355: 70412. Oliver JE, Silman AJ. Risk factors for the development of rheumatoid arthritis. Scand J Rheumatol 2006; 35: 16974.

3 4 5 6 7 8

17 Evaluating the response to treatment P R O B L E M

87

17 Evaluating the Response to Treatment

Case History
Charles and Dianna are a married couple who have been under your care for some years. Charles is aged 43 years with a 20-year history of ankylosing spondylitis (AS), and Dianna is aged 48 years and has had rheumatoid arthritis (RA) for eight years. You have noted that they are increasingly restricted in their activities. As part of your ongoing commitment to care, you want to implement a method of recording the impact of your treatment. What measures are available for monitoring disease activity? How can function and quality of life be quantied?

Background
The benet of any therapeutic intervention must outweigh the cost, with cost most easily measured in monetary terms. More difcult to measure is the cost to the individual in terms of convenience of use, monitoring and adverse effects both actual and potential. Similarly it can often be difcult to measure benet, and this particularly applies to rheumatic disease, where a hard end-point such as mortality is rarely the outcome measure. Patients with rheumatic disease experience pain and impaired function, but over time they adapt to these circumstances and symptom-reporting alters, so that quietly and without complaint function is lost. RA and AS are contrasting diseases. In Table 17.1 the demographic and clinical characteristics are compared, providing insight into the different outcome measures that need to be evaluated for each disease process.
Table 17.1 Characteristics of RA and AS
Rheumatoid arthritis
Gender Age Peripheral disease Spinal disease
F, female; M, male.

Ankylosing spondylitis
M>F Late teens onwards Unusual, with occasional involvement of hip and shoulder Ascending spinal restriction with impaired posture

F>M Middle age onwards Predominant with involvement of hand and feet Cervical inammation

Atlas Medical Publishing Ltd

88

03 Rheumatoid Arthritis Outcome measures vary according to the situation and purpose for which they are being evaluated, often differing signicantly when being used in routine clinical practice and research trials. A pragmatic approach is required if they are to be utilized by the average practitioner who either does not have specialist knowledge or lacks specialist equipment. In broad terms, the outcomes of interest in rheumatic disease are: b Laboratory evidence of inammation recorded as erythrocyte sedimentation rate or serum C-reactive protein b Clinical joint inammation recorded as the number of tender or swollen joints on examination b Clinical examination of spinal function b Patients self-reported experience of pain, fatigue or functional consequence of their disease b Radiological progression

Laboratory markers of disease activity


Laboratory investigations, by themselves, can be misleading. It is not unusual for a person to have normal inammatory markers despite a small number of inamed peripheral joints; similarly, elevated markers may be caused by incidental illness such as urinary tract infection. RA, with its distinct synovial inammation, often leads to elevated inammatory markers, whereas the spondyloarthropathies (in which enthesial inammation at the tendonperiosteal interface predominates) have much lower and often normal values.

Physical evaluation of disease activity


Counting a patients tender and swollen joints is a quick and easy way of both examining the patient and evaluating the extent of their disease activity. While you could examine every joint, examining the easy-to-access 18 joints of upper limbs and knees has been shown to be a valid measure of disease activity (Figure 17.1). The joints examined are the interphalangeal joint of the thumb, proximal interphalangeal joint of the ngers, the metacarpophalangeal joints, wrists, elbows, shoulders and knees totalling 18 joints. The method of examination is to palpate the joint in two directions applying sufcient pressure to blanch the distal third of the nail bed of your thumb to determine separately the presence of swelling and tenderness. The tender joint count (TJC) and swollen joint count (SJC) can be recorded both within the clinical record and in correspondence to other practitioners (i.e. SJC = 4/18, TJC = 6/18). This allows a quick analysis for trends over time and also provides an indicator of absolute disease severity. A count of six to eight indicates moderate severity, sufcient for entry into many trials of new therapeutic agents. Assessing spinal function requires a fuller examination and exposure of the patient but does allow serial evaluations, which are valid in detecting progressive loss of function and potentially act as patient motivation for their ongoing exercise programme. An index of spinal movement the British Ankylosing Spondylitis Metrology Index (BASMI) provides a score of relative severity for any outcome and allows serial comparison. Tables 17.2 and 17.3 describe both the scoring system and method of measurement relating to

17 Evaluating the response to treatment

89

Figure 17.1 Mannequin showing the 28 easily accessible joints for evaluation in routine practice (highlighted).

Table 17.2 BASMI scoring system


Score
Tragus to wall (cm) Lumbar exion (cm) Cervical rotation (degrees) Lumbar side exion (cm) Intermalleolar distance (cm)

0
<15 >4 >70 >10 >100
1

1
1530 24 2070 510 70100

2
>30 <2 <20 <5 <70

Adapted with permission from Irons and Jeffries 2004.

90

03 Rheumatoid Arthritis

Table 17.3 Obtaining BASMI measurements


Measure
Tragus to wall

Starting position
Standing with back to wall, outer edges of feet 30 cm apart Outer edges of feet 30 cm apart. Examiner marks midpoint of line level with iliac crests; a second point is marked 10 cm above and a third 5 cm below Patient supine on bed

Method
Patient draws chin in as far as possible. Examiner measures distance between tragus and the wall with a rigid ruler Patient exes forward from waist with knees fully extended. Distance between upper and lower marks is measured, and then 15 cm is subtracted Patient rotates head as far as possible. Angle between neutral position and rotation is measured with goniometry Measure from tip of middle nger to oor with rigid 1 m ruler. Patient side-exes without forward exion or knee exion. Remeasure from tip of middle nger to oor. Calculate difference between two measurements Keeping knees straight, patient moves legs as far apart as possible. Distance between medial malleoli is measured

Lumbar exion (modied Schober)

Cervical rotation (performed for each side) Lumbar side exion (performed for each side)

As for tragus to wall

Intermalleolar distance Patient lies supine on oor

Modied with permission from Irons and Jeffries 2004.1

AS. The ability to comment to a patient on the progress of their disease is important, and can also be a motivating force for compliance with interventions, particularly exercise and physiotherapy.

Evaluating quality of life


As medical practitioners, it is readily acknowledged that for many patients physicians are not able to cure them of their disease but aim instead to improve their quality of life. The latter has many domains, and in practice physicians are primarily aiming to inuence those areas related to a persons health that impact on their global well-being socalled health-related quality of life. Even within this more circumscribed area there are instruments that measure this in a generic way, thus allowing the same instrument to be used in different disease states providing comparisons that may be used to support health resource distribution decisions. Disease-specic instruments also exist and often provide greater insights into an individual disease and may respond quicker to interventions for that illness. Some self-reported questionnaires available for RA and AS are listed in Table 17.4, and can be accessed via internet search.
Table 17.4 Self-reported health-related quality of life questionnaires
RA
Health Assessment Questionnaire (HAQ) RA Quality of Life (RAQOL)

AS
AS Quality of Life (ASQOL) BAS Disease Activity Index (BASDAI) BAS Functional Index (BASFI)

BAS, British Ankylosing Spondylitis.

17 Evaluating the response to treatment

91

An important domain to consider in patients with RA and AS is that of fatigue. Unlike normal tiredness, fatigue is chronic and typically is not related to over-exertion and is not relieved by rest. Fatigue is a frequent complaint in many chronic diseases and can be assessed via visual analogue scales and also by dedicated questionnaires. As an outcome it has a strong relationship with the other joint symptoms in RA and AS, and is negatively inuenced by sleep disorders. Regular physical activity helps alleviate fatigue and improves quality of life.

Recent Developments
1 There are conicting data regarding survival in patients with AS. A broad population-based study from Rochester, Minnesota, showed no difference in mortality between men with AS and the general population. Other studies indicate that mortality amongst AS patients seen in specialist referral centres is higher standardized mortality ratio of 1.7 with a linear relationship between mortality and disease severity. Furthermore, mortality in AS may be related to disease duration. Mortality risk ratios relative to the general male population are 4.0 for gastrointestinal disease, 1.3 for circulatory diseases and 1.2 for cerebrovascular disease. Smoking is a dose-dependent risk factor for the development of atherosclerosis, but it is unknown whether or not AS patients smoke more than the general population. However, smoking is associated with worse clinical, functional and radiological outcomes in AS.2 2 The inammatory processes in AS may affect various structures of the heart; the most characteristic conditions are conduction defects and aortic insufciency and, less commonly, pericarditis, cardiomyopathy and mitral valve disease. Conduction disturbances may occur in AS due to inammation and brosis of the interventricular septum thereby affecting the atrioventricular node. Aortic insufciency develops because the inammatory process affects the aortic wall directly behind and above the sinuses of Valsalva. This leads to scarred, brotic, thickened and shortened aortic valve cusps and to a dilated aortic root, resulting in aortic regurgitation. The occurrence of conduction disturbances in patients with AS varies from 1% to 33%, and of aortic insufciency from 1% to 10%, and increases with age, disease duration and the presence of peripheral arthritis. Impaired ventricular relaxation leading to diastolic dysfunction is being increasingly recognized in AS, even in subjects with clinically mild disease. Yildirir and colleagues examined 88 AS patients and 31 healthy controls and found that diastolic function of the left ventricle was signicantly disturbed in AS patients compared to controls. 3 3 It is now recommended that three domains be assessed in the clinic for therapeutic responses in RA: patient-reported measures of physical function and/or global disease activity; physician-reported measures of physical function and/or global disease activity; and imaging of the hands and/or feet on a biannual basis.4 In practice, a 10 cm visual analogue scale can be used by both patient and physician for the global disease activity. Radiographic images should be compared over time, and may require the assistance of your local radiologist. Measures of improvement for individually relevant physical activities need to be dened for each patient.

92

03 Rheumatoid Arthritis

Conclusion
A range of practical and relevant measures exists for measuring meaningful outcomes in patients with arthritis particularly AS and RA. These neatly combine laboratory, objective clinical and subjective physician and patients measurements to provide a multidomain assessment that provides information on current disease activity and its impact on the persons quality of life, and allows you to evaluate the utility of interventions that the patient is receiving.

Further Reading
1 Irons K, Jeffries C. The Bath Indices. Outcome Measures for use with Ankylosing Spondylitis Patients. National Ankylosing Spondylitis Society, East Sussex, United Kingdom, 2004. http://www.nass.co.uk/bath_indices.htm [Accessed November 2007] Doran MF, Brophy S, MacKay K, Taylor G, Calin A. Predictors of longterm outcome in ankylosing spondylitis. J Rheumatol 2003; 30: 31620. Yildirir A, Aksoyek S, Calguneri M, Oto A, Kes S. Echocardiographic evidence of cardiac involvement in ankylosing spondylitis. Clin Rheumatol 2002; 21: 12934. Zatarain E, Strand V. Monitoring disease activity of rheumatoid arthritis in clinical practice: contributions from clinical trials. Nat Clin Pract Rheumatol 2006; 2: 61118.

2 3 4

P R O B L E M

18 Pregnancy and Rheumatic Diseases


Case History
Jane is a 32-year-old lawyer who has been treated for rheumatoid arthritis (RA) for the past four years. Her symptoms are well controlled on methotrexate 20 mg/week and diclofenac 50 mg bd as needed. She and her partner would like to start a family. She wishes to discuss the effect a pregnancy would have on her arthritis and vice versa. What are the benecial and adverse effects of pregnancy on RA? Does she need to change her treatment? Will RA affect her chances of becoming pregnant?

Atlas Medical Publishing Ltd

18 Pregnancy and rheumatic diseases

93

Background
RA tends to improve in pregnancy, with 80% of patients enjoying virtually complete remission. The disease remains active in the remainder but only worsens in a small minority. Most causes of inammatory polyarthritis (IP) follow this pattern. A small proportion of patients with systemic lupus erythematosus (SLE) experience ares during pregnancy. These are now less common with more effective therapy before pregnancy and with planning of pregnancy. Previously, pregnancy ares of SLE may have been largely due to withdrawal of drugs in patients with disease that was still quite active. Prolactin is a pro-inammatory hormone and may contribute to some cases of IP worsening in pregnancy and in the post-partum. RA is an autoimmune inammatory disease in which CD4+ T lymphocytes have a prominent role in the underlying pathological process. CD4+ T cells can be divided into two subsets: T helper 1 cells (Th1) and T helper 2 cells (Th2). Th1 cells produce interferon-g, interleukin (IL)-2, tumour necrosis factor (TNF)-b and IL-12. Activation of this cell type and dominance of the Th1 cytokine pattern facilitates cell-mediated immunity. The Th2 cell pattern leads to secretion of IL-4, IL-10 and IL-13, which facilitates humoral immunity. Of the two T-cell patterns, the Th1 cell-mediated immunity pattern is more consistent with active autoimmune diseases. In general terms, pregnancy is associated with a shift towards a Th2 humoral pattern of immunity.1,2 Furthermore, circulating antagonists to Th1-type cytokines are increased, both during normal pregnancy and in pregnancies of patients with autoimmune diseases. These antagonists include interleukin1 receptor antagonist (IL-1ra) and the soluble receptors for TNF and IL-6 (sTNFR and sIL-6R). The cytokine pattern in pregnant patients with rheumatic diseases does not differ markedly from that of normal pregnant women. Increased levels of oestrogen, androgens and progesterone may all contribute to the shift away from the pre-pregnant Th1 pattern, and this leads to decreased cell-mediated immunity. Impairment of antigen-presenting function and neutrophil activity may also be relevant, as these are the initial steps that lead to normal activation of lymphocytes. It is noteworthy that remission of inammatory conditions is more likely if the mother and foetus are dissimilar in HLA determinants, suggesting that the more active pregnancy-related immunosuppression occurs in women where there is the most marked HLA disparity between themselves and their foetus.

Fertility in rheumatic diseases


Women with RA are more likely to be nulliparous or to have a smaller family than their non-RA counterparts. There is, however, no evidence of a general decrease in fertility among patients with RA.3 Their general debility and drug treatment (including steroids) may decrease sexual desire and the frequency of intercourse. They may also be concerned about potential effects of disease-modifying antirheumatic drugs (DMARDs) on their baby, and about their ability to deal with pregnancy and to care for a young child. Concerns about worsening symptoms during pregnancy and afterwards may also play a part. Paradoxically, while normal women often experience marked decrease in physical function and increase in symptoms (including pain) during pregnancy, many women with RA feel better and experience less pain. There is an increased rate of fetal loss related either to the underlying disease states or to drug treatments, including non-steroidal anti-inammatory drugs (NSAIDs).

94

03 Rheumatoid Arthritis Cyclophosphamide is rarely needed in the management of RA but is used in the management of severe SLE, particularly for nephritis or vasculitis. Its use in women over the age of 35 years generally renders them infertile, with its effects on younger women being more variable. The effects of cyclophosphamide on the ovary can be diminished by use of a high-oestrogen oral contraceptive preparation or with gonadotrophin-releasing hormone agonists or antagonists. Alternatively, oocyte recovery with cryopreservation can be used. In men, sperm count is decreased with methotrexate, cyclophosphamide or sulphasalazine. The count usually recovers when the drug is stopped but, if there is concern, sperm can be stored. Fertility treatment is possible for women with RA and SLE care should be taken with SLE as hormone treatments may increase disease activity.

Disease during pregnancy


The risk of increased activity of SLE may be less than was once thought. With modern treatment, more women start pregnancy with the disease in remission. Also, we have become comfortable with using some of the disease-modifying drugs in pregnancy so that the disease can be effectively treated throughout. Flares of lupus during pregnancy are generally mild. However, disease activity in pregnancy increases risk of intrauterine death, fetal growth retardation, pre-eclampsia and premature rupture of membranes leading to early delivery. Lupus ares in pregnancy are less likely if the disease is well controlled at conception. Unplanned pregnancies, however, can lead to worsening of skin rash, increased joint symptoms, mucosal ulceration, anaemia and thrombocytopenia, hypertension and proteinuria, renal casts and increasing levels of anti-double-stranded DNA (anti-dsDNA) antibodies with decreased complement (C3 and C4). The risk of thromboses, particularly venous thromboses, is increased two- to fourfold during normal pregnancy and in the six weeks post-partum, and is particularly high after Caesarean section. The risk of both venous and arterial thrombosis is increased in women with antiphospholipid syndrome (APS), and all women with lupus or a history of pregnancy loss should be screened by testing for lupus anticoagulant and anti-cardiolipin antibodies. APS is present in 37% of patients with SLE, but can occur in other autoimmune disease and in the absence of autoimmune disease.4 APS also increases the risk of pre-eclampsia, premature labour and HELLP (Haemolytic anaemia, Elevated Liver enzymes, Low Platelets). The effect of pregnancy on the spondyloarthropathies is more variable the peripheral manifestations (arthropathy and uveitis) tend to improve, while spinal symptoms often worsen. Vasculitides are relatively rare and there are few systematic studies during pregnancy. Like SLE, these conditions may are up (particularly in the rst and second trimesters). Patients with Behets disease are at increased risk of thrombosis in pregnancy. Lupus nephritis and renal vasculitis can be difcult to distinguish from preeclampsia. The latter is more likely to be associated with hypertension and improves rapidly following delivery. Systemic sclerosis does not appear to be at particular risk of progressing during pregnancy.

Effect on the foetus


Transplacental passage of immunoglobulin G (IgG) antibodies, including anti-SSA/Ro and anti-SSB/La, can lead to neonatal lupus syndrome and congenital heart block (CHB). The former can cause a transient photosensitive rash, hepatic dysfunction and bone

18 Pregnancy and rheumatic diseases

95

marrow depression. CHB occurs in the offspring of mothers who are anti-SSA/Ro positive, and most infants born with this complication require a pacemaker before the age of one year. It is rare, occurring in only 2% of infants of such mothers, and in 1 in 22 000 of the general population. Ideally, all young women with an inammatory arthropathy such as SLE or RA should be screened for lupus anticoagulant, anti-cardiolipin antibodies and anti-SSA/Ro prior to them considering a family, or they should be tested early in their pregnancy. The rash of neonatal lupus resolves over the rst three to ve months of life, as the maternally derived IgG is cleared from the babys circulation. SLE disease activity at conception and the presence of antiphospholipid antibodies are the major determinants of fetal outcome. Fetal loss occurs in up to 30% of patients with active lupus. All inammatory arthropathies increase the risk of premature delivery. However, with modern obstetric monitoring and neonatal management, the risk to the child has been markedly decreased in recent years.

Drugs
None of the drugs used in IP (except simple analgesics) is entirely safe during pregnancy. Methotrexate, cyclophosphamide, azathioprine, gold and cyclosporine should all be stopped at least three and preferably six months before pregnancy. Leunomide has a very prolonged functional half-life (several months) due to its enterohepatic recirculation and is also very teratogenic. Prior to its prescription to a young woman, it is advisable to discuss the time frame of possible pregnancies and to clearly state that pregnancy should not occur within two years of ceasing the medication, unless the drug has been washed out using cholestyramine and a plasma drug level obtained that conrms elimination. NSAIDs should be stopped if possible before pregnancy as they may increase risk of early pregnancy loss. Later adverse effects on the infant include premature closure of the ductus arteriosus with resulting pulmonary hypertension, cutaneous and intracranial bleeding and impaired renal function leading to oligohydramnios. If NSAIDs are required in pregnancy, older agents with a short half-life are best indomethacin, ibuprofen and diclofenac. Corticosteroids are relatively safe in pregnancy if the dose is kept below the equivalent of 20 mg/day prednisolone. If it is considered necessary to treat mother and foetus, then dexamethasone or betamethasone should be used. Otherwise, hydrocortisone, cortisone or prednisolone should be used; less than 10% of these are delivered to the foetus (the remainder that crosses the placenta is inactivated by 11bhydroxysteroid dehydrogenase). Side effects of steroid therapy mainly affect the mother and include hypertension, oedema, pre-eclampsia and gestational diabetes. The most extensively documented teratogenic effect is with methotrexate, which can also induce miscarriage even with normal foetuses. The drug may cause a range of congenital abnormalities including limb anomalies, craniofacial anomalies and a range of nervous system anomalies including anencephaly and hydrocephalus. Sulphasalazine has been extensively used throughout pregnancy in patients with both RA and inammatory bowel disease and is considered safe. Where immunosuppression is required, azathioprine is a reasonable choice. The foetus cannot convert it to its active metabolite, 6-mercaptopurine, and it is therefore relatively free of teratogenic effect. It has, however, been associated with growth retardation and premature rupture of membranes.

96

03 Rheumatoid Arthritis

Recent Developments
1 In a national (United States) study with access to data from over four million deliveries, Chakravarty and colleagues5 showed that, compared with the background pregnant population, women with RA or SLE were more likely to develop a hypertensive disorder, spend more time in hospital and to have a Caesarean delivery. The prevalence of hypertensive disorders was highest in patients with SLE and was comparable in these patients to the prevalence of hypertensive disorders in gestational diabetes (GDM). There was increased risk of premature rupture of membranes and of intrauterine growth retardation in both RA and SLE. The Caesarean section rate was not as high as in patients with GDM. 2 There are fewer data available on Sjgrens syndrome (SS) in pregnancy than for RA or SLE. A recent casecontrol study6 investigated reproductive problems in patients with SS. Complaints of vaginal dryness were found twice as commonly as in controls and affected over 50% of SS patients. The previously documented association between SS and endometriosis was conrmed. Fertility was not decreased in SS, but women chose to have fewer children because of their disease.

Conclusion
The patient should stop methotrexate at least three months before pregnancy, which should be carefully planned wherever possible. Careful monitoring of the condition in the months before pregnancy should be achieved, which will determine the level of therapy required. Clearly the latter should be kept to a minimum before and during pregnancy. One would not expect a major reduction in fertility in the above patient.

Further Reading
1 2 3 4 5 Gordon C. Pregnancy and autoimmune diseases. Best Pract Res Clin Rheumatol 2004; 18: 35979. stensen M, Frger F, Villiger PM. Cytokines and pregnancy in rheumatic disease. Ann NY Acad Sci 2006; 1069: 35363. Katz PP. Childbearing decisions and family size among women with rheumatoid arthritis. Arthritis Rheum 2006; 55: 21723. Tincani A, Bompane D, Danieli E, Doria A. Pregnancy, lupus and antiphospholipid syndrome (Hughes syndrome). Lupus 2006; 15: 15660. Chakravarty EF, Nelson L, Krishnan E. Obstetric hospitalizations in the United States for women with systemic lupus erythematosus and rheumatoid arthritis. Arthritis Rheum 2006; 54: 899907. Haga HJ, Gjesdal CG, Irgens LM, stensen M. Reproduction and gynaecological manifestations in women with primary Sjgrens syndrome: a case control study. Scand J Rheumatol 2005; 34: 458.

19 Diet and arthritis P R O B L E M

97

19 Diet and Arthritis

Case History
Michelle is 37 years old and has had rheumatoid arthritis (RA) for four years. She is compliant with her medication and has achieved near-remission but does not like taking tablets. She has a body mass index of 27.6 kg/m2 and is seeking advice on whether there are any effective diets or supplements. What is the effect of being overweight or obese on rheumatic diseases? Are there pro- or anti-inammatory components of the diet? What simple, risk-free advice might she be given? Is there a place for food supplements?

Background
Pharmacological measures to improve symptoms and drugs to modify disease activity have transformed the management of rheumatic diseases. Dietary measures have a much less certain place but emerging evidence is beginning to point to a role in disease prevention in patients who are at high risk of developing the diseases, and in controlling symptoms or preventing relapse in patients who have diagnosed disease. There is increasing pressure on health professionals to be knowledgeable about dietary factors as patients are increasingly informed. The increased predisposition to osteoarthritis amongst patients who are either overweight or obese is now well documented, as is the importance of various dietary components in RA.

Diet as a susceptibility factor for RA


The cause of RA is unknown but is likely to involve both genetic susceptibility and environmental factors such as diet, and may involve interplay of both factors. Available data suggest that at least one-third of RA susceptibility is explained by environmental variation. The possibility that diet may play a role in both the onset and persistence of RA is a frequently reported experience and there is a growing literature in support of this hypothesis.1 A large prospective study was performed in the United Kingdom using RA cases registered from the Norfolk Arthritis Register. A number of dietary factors emerged as being of possible relevance.2 Low fruit and vitamin C intake was associated with a doubling in the risk for RA, with the highest daily intake of the antioxidants b-cryptoxanthin and zeaxanthin having the lowest risk for RA. It was suggested that one glass of freshly Atlas Medical Publishing Ltd

98

03 Rheumatoid Arthritis squeezed orange juice per day would be sufcient to offer protection. A high intake of red meat had a modest risk for RA, which is independent of any lowering of fruit and vegetable intake, and may be explained by red meat providing a dietary source of omega-6 oils. The reported effects of decaffeinated/caffeinated coffee, tea or total caffeine consumption are internally inconsistent and conicting, making a real effect unlikely.2 Alcohol consumption and the risk of developing RA has only been studied in women. The results are conicting, at best suggesting a modest protective effect in women consuming the most alcohol.1

Fruit, vegetables and antioxidants


Monocytes, macrophages and granulocytes at the site of an inammatory process generate reactive oxygen species. These, in turn, lead to increased production of inammatory mediators. This process, which is partly mediated by increased local nitric oxide production, can potentially be inhibited by antioxidants, which scavenge reactive oxygen species.3 Interestingly, in patients with RA, blood markers of antioxidant nutritional status are lower than in normal controls. Fruit and vegetables are the main source of antioxidants in the diet, including retinoids, vitamins C and E, carotenoids, selenium and lutein. Lower intake of vitamin C is associated with increased risk of inammatory polyarthritis.4

Essential fatty acids and the Mediterranean diet


The Mediterranean diet is characterized by less red meat and more sh, with olive oil as the primary fat source, an abundance of plant food and moderate wine consumption. High levels of omega-3 fatty acids may protect against inammation by substituting for omega-6 fatty acids. Polyunsaturated fatty acids contain two or more double bonds, with the omega-3 or omega-6 classication according to the site of the double bond proximal to the methyl terminus. Mammals do not possess the enzyme necessary to introduce double bonds at the 3 or 6 positions they are therefore essential fatty acids and must be obtained from the diet. The Western diet is abundant in omega-6 fatty acids due to the abundance of linoleic acid in soybean, safower, sunower and corn oil.5 Fish oil contains the omega-3 fatty acids eicosapentaenoic acid (EPA) and docosahexaenoic acid (DHA). These compete with arachidonic acid for cyclooxygenase (COX)-1, COX-2 and lipoxygenase enzymes, and their metabolism leads to incorporation of anti-inammatory prostanoids and leukotrienes in cell membranes. Olive oil is rich in oleic acid, which can be metabolized to eicosatrienoic acid, similar to omega-3 polyunsaturated fatty acid from sh oil. Stamp et al.5 have extensively reviewed the role of dietary oils in RA. In combination with non-steroidal anti-inammatory drugs (NSAIDs)/aspirin, omega-3 oil produced a novel family of bioactive lipids known as resolvins, named for their antiinammatory effect and tendency to resolve inammation at the site of study. Fish oil supplementation reduced expression of the antigen-presenting class II HLA-DR molecule on human monocytes, with a reduced ability to present antigen after culture with EPA and DHA. The risk of RA has been documented to be lower in those who consume higher amounts of olive oil or oily sh.6 These observations are consistent with several randomized controlled trials showing modest antirheumatic benet of omega-3 fatty acid supplementation, including reduced early morning stiffness, tender joint count and use of NSAIDs (Box 19.1).

19 Diet and arthritis

99

Box 19.1 Effects of omega-3 oils on the manifestations of RA


b b b b

Modest clinical improvement when used with standard pharmacotherapy Deterioration in symptoms after discontinuation Ability to decrease or cease NSAIDs and hence reduce associated gastrointestinal and cardiovascular risk Reduce sudden cardiac death

Dietary supplements and RA


In all studies of sh oils, the benets were observed against a background of standard therapy of NSAIDs, disease-modifying agents and immunomodulators. Collectively, these studies have established that sh oil supplementation that delivers 2.6 g or more per day of the long-chain fatty acids EPA plus DHA reduces symptoms after a latent period of three months. The latency can be reduced with higher doses of sh oil. For antiinammatory effect it is recommended to take 2.64.0 g of EPA and DHA per day. The required dose of sh oil can be taken either as sh oil capsules or from bottled sh body oil, with patient preference relating to cost and palatability. In addition to oil supplements, patients should be encouraged to review their background diet so as to minimize omega-6 oil intake and maximize omega-3 intake (Box 19.2). Elemental diets provide food in the most basic forms i.e. free amino acids, medium chain triglycerides and small sugars and are proposed as being hypoallergenic. General benet has not been evident in several trials. Vegetarian and vegan diets have been reported to improve symptoms. Many of the studies commence with a fast with resultant improvement in clinical and biochemical parameters, which is not always sustained in the longer term when caloric intake in its modied form is reintroduced. Fasting has been shown to reduce both clinical and laboratory markers of inammation, possibly by redirecting scarce resources away from inammatory pathways and reducing disease activity but compromising immune defences. It is important to ensure that sufcient calories and nutrients are being provided to ensure well-being. Elimination diets seek to remove foods that are putatively allergenic from the diet. Studies demonstrate that a proportion of patients improve when putative allergenic foods are eliminated, with worsening of symptoms at some point when foods are sequentially reintroduced. Box 19.2 Diet in rheumatoid arthritis
b

b b

Diet as a risk factor Low fruit, vitamin C, b-cryptoxanthin, zeaxanthin High meat Diet as a protective or therapeutic factor High consumption of oily sh, olive oil, cooked vegetables Conicting data Caffeine, decaffeinated coffee, tea Vitamin D and dairy products Doubtful benet Elemental diet

100

03 Rheumatoid Arthritis

Recent Developments
1 Turmeric has been used for centuries in Ayurvedic medicine as a treatment for inammatory disorders including arthritis. In an animal model of arthritis, a curcuminoid-containing turmeric extract demonstrated efcacy in preventing joint swelling and destruction as determined clinically, histologically and by measurement of bone mineral density.7 The putative mechanism of action was further elucidated by analysis of turmerics effect on articular transcription factor activation and microarray analysis of articular gene expression. Treatment in vivo prevented local activation of nuclear factor-k-beta (NF-kB) and the subsequent expression of NF-kB-regulated genes mediating joint inammation and destruction, including chemokines, COX-2 and receptor activator of NF-kB ligand (RANKL). Consistent with these ndings, inammatory cell inux, joint levels of prostaglandin E2 and periarticular osteoclast formation were all inhibited by turmeric extract. Other researchers using curcumin have documented antiproliferative, anti-inammatory and immunosuppressive activities, as identied by inhibition of neutrophil activation, synoviocyte proliferation and angiogenesis.8 2 Polyphenols are secondary metabolites of plants and are generally involved in the defence mechanism against ultraviolet radiation and insects, but have antiinammatory activity when consumed. Classied into four groups avonoids, stillbenes, lignas and phenolic acids these naturally occurring plant compounds can have potent effects on reducing chronic disease in animal models.9 Flavanols (one of the avonoids) have been extensively studied, and there is an estimated daily consumption of 2025 mg/day in the United States. The potential mechanisms of their anti-inammatory activities include inhibition of COX and lipoxygenase enzymes, as well as nitric oxide synthase, and inhibition of the transcription factors NF-kB and PPARs (peroxisomal proliferator-activated receptors). In collageninduced arthritis (a mouse model of RA, which is triggered by tumour necrosis factor [TNF]-a) the oral administration of avonoids improves the arthritis even when the disease has become established.10 Dietary sources of avonoids include tea, red wine, fruits, vegetables and legumes. Flavanones are in citrus, isoavones in soy products, anthocyanidins in wine and bilberry, and avans are in apples and tea. Citrus avonoids are found in citrus fruits, rutin in buckwheat, epigallocatechin gallate in green tea and naringenin in grapefruit. Oligomeric proanthocyanidins are found in grape seeds and skins. Quercetin is found in onions, tea and apples. Polyphenols are the most abundant group of compounds in fresh tea leaves and are found in green and black tea beverages as 30%42% and 3%10% of the total dry matter, respectively. Many medicinal plants contain bioavonoids, such as ginkgo biloba, hawthorn and Chinese skullcap.

Conclusion
Does my diet affect my arthritis? is a question familiar to all rheumatologists and primary care practitioners. One-third to three-quarters of RA patients believe that food plays an important role in their symptom severity, and up to half will have tried dietary manipulation to improve their quality of life. The most consistent link between diet and

19 Diet and arthritis

101

Table 19.1A Omega-3 friendly foods


FATS AND OILS Use either canola, sunola or olive oil spread, mayonnaise and salad dressing instead of your usual oil products Flaxseed oil can be used in salad dressings or mixed with sunola or olive oil and used in baking BREADS AND CEREALS Choose breads, crispbreads and cereals that have added linseed NUTS Macadamias, hazelnuts, cashews and almonds can be enjoyed every day in limited amounts (one handful) Peanuts (and peanut paste) and pistachios one handful 23 times a week MEAT Use lean cuts of meat FISH Choose oily sh, i.e. mullet, athead, snook, trevally, fresh tuna, canned pink salmon, canned sardines FRUITS AND VEGETABLES Eat freely CONVENIENCE FOODS Frozen chips (these are pre-cooked in canola oil) McCain Healthy Choice; Straight Cut or Country Style Birds Eye Steak House, Crinkle Cut, Home Style, French Fries, Flavour Crisp (Seasoned Crunchy Wedges only) SNACK FOODS & CONFECTIONERY Most potato crisps are cooked in omega-6 fats, which may prevent the omega-3 fats going into your body Kettle Chip Company chips are cooked in sunola oil FAST FOODS Pizza Haven and Fasta Pasta use canola oil
Based on a pamphlet developed by Dr E Mantzioris, Dr MJ James and Prof LG Cleland, Royal Adelaide Hospital, Australia.

Table 19.1B Foods to avoid


FATS AND OILS Avoid all other oils, cooking sprays, dressings, mayonnaise, lards, dairy blends, drippings, cooking margarines and spreads NUTS & LEGUMES Brazil nuts, pecan nuts, pine nuts, walnuts, sesame seeds (tahini paste, houmous), sunower seeds FISH Packaged frozen sh such as sh ngers CONVENIENCE FOODS All other varieties of frozen chips. Avoid those fruits or vegetables that are canned in dressings (e.g. potato salad) SNACK FOODS & CONFECTIONERY Health food bars sesame bars, nut bars, muesli bars

arthritis to date remains that of polyunsaturated omega-3 oils. Additional potential mechanisms are under exploration, and it is prudent advice to any patient with inammatory polyarthritis that they aim to alter their omega-3/omega-6 balance (Table 19.1). This should occur in parallel with the use of single and preferably combination diseasemodifying agents. There is limited evidence for benet of restrictive or elimination diets and any benets are likely to be individual and not transferable to the next patient.

102

03 Rheumatoid Arthritis Research into the dietary polyphenols may open up new areas of dietary manipulation in the treatment of chronic inammatory disease.

Further Reading
1 Pattison DJ, Harrison RA, Symmons DPM. The role of diet in susceptibility to rheumatoid arthritis: a systematic review. J Rheumatol 2004; 31: 131019. 2 Oliver JE, Silman AJ. Risk factors for the development of rheumatoid arthritis. Scand J Rheumatol 2006; 35: 16974. 3 Pattison DJ, Symmons DPM, Young A. Does diet have a role in the aetiology of rheumatoid arthritis? Proc Nutr Soc 2004; 63: 13743. 4 Pattison DJ, Silman AJ, Goodson NJ et al. Vitamin C and the risk of developing inammatory polyarthritis: prospective nested casecontrol study. Ann Rheum Dis 2004; 63: 8437. 5 Stamp LK, James MJ, Cleland LG. Diet and rheumatoid arthritis: a review of the literature. Semin Arthritis Rheum 2005; 35: 7794. 6 Choi HK. Dietary risk factors for rheumatic diseases. Curr Opin Rheumatol 2005; 17: 1416. 7 Funk JL, Frye JB, Oyarzo JN et al. Efcacy and mechanism of action of turmeric supplements in the treatment of experimental arthritis. Arthritis Rheum 2006; 54: 345264. 8 Jackson JK, Higo T, Hunter WL, Burt HM. The antioxidants curcumin and quercetin inhibit inammatory processes associated with arthritis. Inamm Res 2006; 55: 16875. 9 Yoon J-H, Baek SJ. Molecular targets of dietary polyphenols with anti-inammatory properties. Yonsei Med J 2005; 46: 58596. 10 Kumazawa Y, Kawaguchi K, Takimoto H. Immunomodulating effects of avonoids on acute and chronic inammatory responses caused by tumor necrosis factor alpha. Curr Pharm Des 2006; 12: 42719.

20 Polyarthritis in the elderly P R O B L E M

103

20 Polyarthritis in the Elderly

Case History
John is 69 years old and presents because of pain in his shoulders and hands. He is unable to get out of bed with ease or dress himself. His symptoms started suddenly ve weeks ago and he thought they would get better with tablets he bought from the chemist. What are the common forms of polyarthritis in the elderly? Is there evidence that osteoarthritis can be modied by the use of medication? What factors should be considered in the treatment of arthritis in the elderly?

Background
Arthritis and rheumatism are major physical and psychological burdens in the elderly population. As most forms of arthritis are chronic, the combination of increasing lifespan, cumulative arthritis incidence and concomitant disease and treatments creates the almost inevitable experience of arthritis. Approximately two-thirds of elderly people experience sufcient symptoms each day to warrant regular use of non-steroidal antiinammatory drugs (NSAIDs). Symptoms are sufcient to disrupt the sleep or leisure activities of one-third of the elderly population, with nearly all these people using at least one medical, complementary or self-care strategy. Arthritis is the most prevalent chronic condition among adults over the age of 65 years (48.9 per 100 adults), followed by hypertension (40.3 per 100 adults) and heart disease (28.6 per 100 adults). The psychological burden of arthritis is often underestimated.

Osteoarthritis
Osteoarthritis (OA) is the commonest condition to affect synovial joints and the single most important cause of locomotor disability. Although not an inevitable consequence of aging, OA is strongly related to age, which may represent cumulative insult to the joint, possibly aggravated by decline in neuromuscular function or senescence of homeostatic repair mechanisms. OA is uncommon and multiple joint OA is rare in persons aged less than 45 years. Prevalence of OA varies in different populations, but on average affects 60%70% of those aged over 45 years. The prevalence of radiographic OA exceeds that of clinical OA, and is almost universal in the elderly population in distal interphalangeal joints and knees. The prevalence of symptomatic OA also increases with age, 15% of those aged 55 years having symptomatic knee OA. There is a pronounced female preponderance after the age of 55 years for severe radiographic OA. This has suggested that Atlas Medical Publishing Ltd

104

03 Rheumatoid Arthritis oestrogen deciency may play a role in aetiology. Occupation predisposes to OA of specic joints (e.g. hip of farmers). The best-characterized OA risk factor is obesity, with OA in the knee and to a lesser extent in the hip correlating with higher body weights. OA is a condition of synovial joints characterized by focal cartilage loss and an accompanying reparative bone response. It is a slowly progressing inammatory process and not simply wear and tear. The nal common pathway of OA is cartilage degradation. Brandt and Mazzuca1 reviewed nine clinical trials purporting to demonstrate disease modication in OA, with their focus on placebo-controlled randomized clinical trials in which joint space narrowing (JSN) was used as an outcome. They also examined the effects of the purported disease-modifying osteoarthritis drugs (DMOADs) on joint pain (Table 20.1). Whilst some agents have shown possible efcacy, trial design has been poor with heterogeneous populations and insufciently sensitive outcome measures. No therapy has yet been approved as having either chondroprotective or structure-modifying properties in OA.
Table 20.1 Effect of disease-modifying osteoarthritis drugs (DMOADs)
Intervention
GI. Sulp 1500 mg GI. Sulp 1500 mg Ch. Sulp 800 mg Ch. Sulp 800 mg Doxycycline 200 mg Risedronate 5/15 mg IA Hyaluronan Diacerein 100 mg

Joint
K K K K K K K H

Study duration (months)


36 36 24 12 30 12 12 36

JSN
  ? ?   ? 

Sx control
?      ? 

Ch. Sulp, chondroitin sulphate; GI. Sulp, glucosamine sulphate; H, hip; IA, intra-articular; JSN, joint space narrowing; K, knee; Sx, symptom.

The Glucosamine/chondroitin Arthritis Intervention Trial (GAIT)2 involving 1500 overweight subjects with symptomatic and radiographic knee OA, found that neither glucosamine nor chondroitin alone or in combination were better than placebo for symptom control. The internal active control in this study was celecoxib and this was signicantly better than placebo for symptom control. A potential explanation for this lack of clinical effect of glucosamine is the differences between the serum levels obtained in vivo after standard doses of glucosamine and the doses used in the precursor loading of cultured human chondrocytes studied in vitro. In clinical studies, a single dose of glucosamine sulphate was detected in 17 of 18 subjects at 3045 minutes after administration, with peak concentrations occurring at 90180 minutes. The detected serum concentration range of 211 mmol/l would contribute only 2% of galactosamine incorporation into chondroitin sulphate in the in vitro experiments in human chondrocytes.3

Rheumatoid arthritis
The onset of rheumatoid arthritis (RA) is often abrupt in the elderly, and although the classical peripheral symmetrical small joint polyarthritis develops, shoulder symptoms

20 Polyarthritis in the elderly

105

may predominate initially and prominent hand oedema may obscure the diagnosis. Lateonset RA (LORA) has been reported to have a more equal gender distribution, an acute onset and prominent elevation of erythrocyte sedimentation rate (ESR). It is also characterized by disabling morning stiffness and marked pain, predominantly in the upper extremities. The physical examination is of pronounced synovitis of the shoulders and the wrists as well as the metacarpophalangeal joints and proximal interphalangeal joints, with marked limitation of motion and severe soft tissue swelling. Conditions that may mimic and cause diagnostic confusion with LORA are remitting seronegative symmetric synovitis with pitting oedema (RS3PE) and polymyalgia rheumatica (PMR) (Table 20.2). A diagnosis of PMR or LORA can be made in the same patient at different stages of the same illness depending on the predominant clinical manifestation at the time. RS3PE is an acute onset, bilateral symmetrical synovitis predominantly of the wrist, carpus, small hand joints and exor digitorum sheaths, associated with marked oedema of the dorsum of the hand. Patients are persistently rheumatoid factor (RF) negative and respond rapidly to low-dose steroids. Cantini et al.,4 in their prospective ve-year study of PMR and RS3PE, found no demographic, clinical or immunogenetic differences. No patient developed RA, and they concluded that PMR and RS3PE constitute a spectrum within the same disease.
Table 20.2 Differences and similarities between LORA, PMR and RS3PE
LORA
Age of onset Sex ratio F:M Mode of onset Predominant joint pattern Distal oedema RF positivity HLA association Course Response to low-dose steroid Over 60 years 1:1 Acute or subtle Peripheral joints in upper limbs and shoulders May be present 50% DR4 Severe in RF+ Mild in RF Poor in RF+ Good in RF Long-term steroids 24 years Giant cell arteritis in 10%15% Good

PMR
Over 55 years 5:1 Acute Shoulders, lower back, hips, knees May be present Negative

RS3PE
Over 60 years 1:4 Acute Hands, wrists, shoulders, knees Present Negative ? B7 Self-limited 12 years Good

Modied with permission from Canoso JJ. Rheumatology in Primary Care. WB Saunders Co., Philadelphia, 1997.

In general, the therapeutic approach in LORA is similar to that for younger patients. Consideration needs to be given, however, to concomitant illnesses and the elderly patients reduced physiological reserve, and the ability of the liver and kidneys to metabolize medications. NSAIDs increase the risk of gastrointestinal bleeding and can impair renal function, with the resultant salt and water retention impacting on hypertension and cardiac function. The mortality for gastrointestinal bleeding increases rapidly in later life, with risk factors for bleeding being age over 65 years, past history of peptic ulcers or

106

03 Rheumatoid Arthritis bleeds, concomitant steroids/anticoagulants/antiplatelet agents and general frailty. Risk factors for renal failure include age over 65 years, hypertension, cardiac failure and concomitant diuretics or angiotensin-converting enzyme inhibitors. The evidence that selective cyclooxygenase-2 inhibitors offer meaningful gastroprotection in an elderly cohort is doubtful, and any positives are outweighed by adverse impact on renal and cardiac function. In the past, low-dose corticosteroids have been promoted for use in the elderly due to their rapid onset of action. This strategy is acceptable if the goal is rapid suppression of the inammatory process with the steroids acting as a bridge to a longer-term alternative agent allowing the withdrawal of steroids. The danger is if the alternative agent is not introduced and/or the patients continue to receive steroids in the long term. Even lowdose steroids (<7.5 mg prednisolone per day) are associated with reduction in bone mineral density, hyperglycaemia, weight gain and hypertension.

Gout
Prolonged hyperuricaemia is a necessary prerequisite for clinical gout, but fortunately gout only develops in a small percentage of patients with chronic hyperuricaemia. In men, uric acid accumulation begins after puberty, with the rst attacks usually occurring after the age of 3540 years. In women, oestrogen facilitates renal excretion of urate, such that the age at which clinical gout occurs is older; occurrence is often the combination of post-menopausal status and, frequently, thiazide diuretic use, and more recently the concomitant use of low-dose aspirin. In the elderly, joints previously damaged (mostly by OA) facilitate seeding of uric acid crystals through exposed collagen and glycosaminoglycans. Age-related and concomitant illness impacts on renal function to further enhance uric acid retention, leading to more frequent and more severe attacks of gout. Eventually, the patient may have chronic polyarticular gout, which no longer resembles the acute, self-limited illness, and is easily confused with a polyarthritis such as RA. Colchicine is related to the cytotoxic agents vincristine and vinblastine, and is renally excreted. Its use in the elderly is often associated with diarrhoea and risk of bone marrow suppression. Oral, intramuscular and intra-articular steroids are well suited to suppressing an acute attack of gout. In the longer term, allopurinol remains the agent of choice in the elderly, as it does not rely on preserved renal function and does not increase the risk of renal calculi. It is, however, renally excreted, with any dosage reduction either based empirically on renal function, or more specically based on the serum measurement of the active allopurinol metabolite, oxypurinol.

Conclusion
OA, gout, pseudogout, RA and PMR are commonly encountered in the elderly population. Diagnostic confusion may occur because of age-related effects on diagnostic tests such as ESR, RF and antinuclear antibodies. The onset of scleroderma, systemic lupus erythematosus or idiopathic polymyositis is uncommon in the elderly. Increasing lifespan is impacting both on overall prevalence and incidence of the rheumatic diseases in the elderly community. The combination of aging, cumulative morbidities and polypharmacy makes the elderly particularly susceptible to drug-related adverse events.

20 Polyarthritis in the elderly

107

This, however, should not detract from the same overall goal of improving an individuals quality of life and functioning.

Further Reading
1 2 3 Brandt KD, Mazzuca SA. Lessons learned from nine clinical trials of disease-modifying osteoarthritis drugs. Arthritis Rheum 2005; 52: 334959. Clegg DO, Reda DJ, Harris CL et al. Glucosamine, chondroitin sulfate, and the two in combination for painful knee osteoarthritis. N Engl J Med 2006; 354: 795808. Biggee BA, Blinn CM, McAlindon TE, Nuite M, Silbert JE. Low levels of human serum glucosamine after ingestion of glucosamine sulphate relative to capability for peripheral effectiveness. Ann Rheum Dis 2006; 65: 2226. Cantini F, Salvarini C, Olivieri I et al. Remitting seronegative symmetrical synovitis with pitting oedema (RS3PE) syndrome: a prospective follow up and magnetic resonance imaging study. Ann Rheum Dis 1999; 58: 2306.

S E C T I O N

F O U R

04

Systemic Lupus Erythematosus, Sjgrens Syndrome and Scleroderma


21 22 23 24 25 26 27 Antinuclear factor SLE risk factors and diagnosis Monitoring and managing SLE Sjgrens syndrome Raynauds phenomenon Assessing and treating scleroderma Immunosuppressive drugs

P R O B L E M

21 Antinuclear Factor

Case History
Mary is 32 years old and presents with chronic fatigue and general malaise. She has two young children aged two and ve years. Her pregnancies were uncomplicated. Her mother suffers from autoimmune thyroid disease. Her erythrocyte sedimentation rate (ESR) is slightly elevated at 25 mm/h and the immunology laboratory reports that her serum is antinuclear antibody (ANA) positive at a titre of 1:320. What is the clinical signicance of the positive ANA? What factors will aid the interpretation of this result? Are there other tests that should be requested?

Atlas Medical Publishing Ltd

110

04 Systemic Lupus Erythematosus, Sjgrens Syndrome and Scleroderma

Background
ANA is detected by indirect immunouorescence on xed and permeabilized Hep2 cells derived from a human laryngeal carcinoma. ANA is positive in the majority of patients with systemic lupus erythematosus (SLE) but is not specic for SLE, frequently being positive in other connective tissue diseases. ANA may also be detected in autoimmune thyroid and liver diseases. It may become positive in patients with infections and inammatory disorders including inammatory bowel disease. A small proportion of the normal population, particularly the elderly, are also ANA positive. ANA results are reported as titres the maximum dilution at which the antibody can be detected by immunouorescence.1 Beginning at 1:40 dilution, serial doubling dilutions are analysed until the observer no longer detects the presence of antinuclear uorescence (i.e. 1:40, 1:80, 1:160, 1:320, 1:640 dilutions and so on). Often reported as positive at a titre of 1:40, clinically meaningful positives begin at 1:160. Although higher titres are more likely to indicate the presence of an underlying disease, repeated measurements are generally not useful in tracking the activity of underlying diseases. Different patterns of staining, indicating which antibody may be present, have long been recognized but are now of less clinical signicance as specic markers for connective tissue diseases have been described: Homogeneous: Due to anti-double-stranded DNA (anti-dsDNA), anti-histone (seen in drug-induced lupus) or anti-nucleosome (histoneDNA complex). Membranous: Antibodies to membrane components including lamins A, B and C. Speckled: Antibodies directed at non-histone antigens. Coarse speckles anti-Sm (Smith antigen) and anti-uracil-rich 1 ribonucleoprotein (anti-U1RNP); ne speckles antiSSA/Ro or anti-SSB/La. Anti-centromere: Characteristic of limited form of systemic sclerosis (CREST syndrome). Nucleolar: Anti-DNA topoisomerase I (Scl-70), a marker for systemic sclerosis. The Hep2000 cell line is increasingly used for ANA testing as it contains a proportion of Ro antigen-transfected cells and identies both ANA and anti-Ro antibodies. Sensitivities and specicities for various disease states are shown in Table 21.1. Positive ANA should always be interpreted in the light of the clinical picture. Extractable nuclear antigens are antibodies directed against small ribonuclear proteins and include anti-Sm, anti-U1RNP, anti-SSA/Ro and anti-SSB/La. With modern diagnostic techniques, it is debatable whether SLE can be diagnosed in the absence of antinuclear antigens although older literature suggests that up to 5%10% of cases may be autoantibody negative. Of 263 patients referred with a presumptive diagnosis of SLE, 29% were ANA positive but did not have autoimmune disease, and half of these had received treatment with corticosteroids at dosages as high as 60 mg/day.2

Anti-dsDNA
DNA was the rst of the nuclear antigens to be identied (in 1957). High avidity immunoglobulin G (IgG) antibodies directed at double-stranded DNA are highly specic for SLE. The antigenicity of DNA is increased when it is complexed to histones or other nucleosome proteins. The level of anti-dsDNA is a reasonable marker of disease activity, particularly where there is renal involvement. Antibodies against the comple-

21 Antinuclear factor

111

Table 21.1 ANA in connective tissue diseases


Disease
Systemic lupus erythematosus Scleroderma Secondary Raynauds Polymyositis/dermatomyositis Sjgrens syndrome Rheumatoid arthritis

Sensitivity (%)
96 85 64 61 48 41
3

Specicity (%)
57 54 41 63 52 56

Adapted with permission from Habash-Bseiso et al. 2005.

ment component C1q may also be useful. The antibodies are not only markers for the disease but directly participate in the disease process through immune complex deposition, cross reactivity with other antigens and promotion of immune reactions within tissues. A variety of methods have been described for detection including the Farr assay, Crithidia luciliae immunouorescence and a range of enzyme-linked immunosorbent assays (ELISAs).

Anti-SSA/Ro and anti-SSB/La


These were rst described as antibodies against determinants in salivary glands of patients with Sjgrens syndrome. They are directed against 52 kDa and 60 kDa Ro proteins, La or to one of a number of RNA particles. Anti-Ro occurs in some patients with SLE, mixed systemic sclerosis (SSc)/SLE syndrome, primary biliary cirrhosis and in 15%20% of patients with polymyositis/dermatomyositis (PM/DM). In SLE, it has been associated with non-erosive (Jaccouds) arthropathy and with late onset of the disease (at age >50 years). Anti-La is most commonly found in Sjgrens syndrome. In SLE, it generally occurs with anti-Ro and together they are a marker for low risk of renal disease. AntiLa is also found in patients with hypergammaglobulinaemia or cryoglobulinaemia. Anti-Ro is found in patients with complement C2 or C4 deciency. Anti-Ro IgG crosses the placenta and is responsible for congenital heart block in the offspring of mothers with SLE. Heart block and tachyarrhythmias occur with increased frequency in mixed connective tissue disease (MCTD), SSc and PM/DM, where they are often associated with the presence of anti-Ro.4

Anti-Sm and anti-RNP


Anti-Sm shows high specicity for SLE but is only found in 5%30% of patients, with fewer Caucasian patients and up to 30% of black patients being positive. Anti-RNP is positive in 25%47% of SLE patients, and is a particularly good marker for MCTD. Although the two autoantibodies are highly associated, they may have different clinical associations: antiRNP in patients with SLE and Raynauds phenomenon and typically with milder renal involvement; anti-Sm, along with high anti-dsDNA, is a marker for more severe renal involvement. Anti-Sm is directed at seven proteins that form the core of U1, U2, U4 and U5 small nuclear ribonucleoprotein particles. Anti-RNP is directed at 70 kDa, A and C proteins in the core of U1RNA particles. Detection is by counterimmunoelectrophoresis,

112

04 Systemic Lupus Erythematosus, Sjgrens Syndrome and Scleroderma immunoblotting or ELISA. Anti-Sm is technically difcult to detect and more than one method may be required. Anti-Sm may arise by molecular mimicry with an EpsteinBarr virus, while cross-reactivity with inuenza B has been suggested for anti-RNP.

Anti-Jo-1
Initially described in 1980 in a patient with interstitial pulmonary brosis (John P), these antibodies are highly specic for inammatory myopathies. Anti-Jo-1 is directed at the histidyl-tRNA synthetase. Autoantibodies against six of the 20 aminoacyl-tRNA synthetases have now been described. The antibody occurs in 20%30% of patients with PM, but in 60%70% where there is concurrent pulmonary brosis. DM is more frequently associated with malignancy than PM, but anti-Jo-1 is present in 5%10% of cases. Anti-Jo-1 often occurs along with anti-Ro52. The association of inammatory myositis, interstitial pulmonary brosis and Raynauds phenomenon has been called antisynthetase syndrome. Detection of anti-Jo-1 is by counterimmunoelectrophoresis, immunoblotting or ELISA.

Anti-Scl-70
These are antibodies against the enzyme topoisomerase I. They are rarely found in healthy individuals and are reasonably specic for SSc, particularly where there is diffuse cutaneous involvement or interstitial pulmonary brosis. They are detectable in about 40% of patients with SSc, and have also been described in patients with localized scleroderma, eosinophilia myalgia syndrome and graft-versus-host disease.

Anti-hnRNP
Antibodies to heterogeneous nuclear ribonucleoproteins (hnRNPs) have only relatively recently come to prominence and are positive in 35% of rheumatoid arthritis patients, 20%30% of patients with SLE and in 40% with MCTD. The hnRNPs are abundant nuclear proteins involved in RNA processing in association with RNA polymerase II. The most common anti-hnRNPs are antibodies against the A1 protein (a marker for Raynauds phenomenon associated with SLE), the A2/B complex (associated with erosive arthritis in SLE) and K protein. Anti-RA33 is thought to be a relatively specic marker for rheumatoid disease. The above autoantibodies should only be sought if ANA is positive and there is a strong clinical suspicion of the related disease. The associations between autoantibodies and the various disease states is summarized in Table 21.2.

Recent Developments
1 Environmental factors including drugs and infections are important triggers for the development of ANA. In a recent study,5 ANA (measured by ELISA) was found in 10.9% of men and 12.2% of women. Records of infectious diseases in early life were available. Participants who had mumps, rubella or a diarrhoeal illness later in life were more likely to be ANA positive. 2 Smoking is another environmental trigger for rheumatoid arthritis, SLE, autoimmune thyroid disease and multiple sclerosis. Generation of reactive oxygen

21 Antinuclear factor

113

Table 21.2 Autoantibodies in connective tissue diseases


SLE
ANA dsDNA Ro La Sm RNP Jo-1 Scl-70 hnRNP +++ ++++ ++ + ++ ++ + ++

MCTD
++ ++ ++ + +++ + ++

SS
++ ++ + + + +

SSc
+++ ++

RA
+ ++

PM/DN
++ ++ +++

Raynauds
++ + ++ + +

The higher number of + signs, the greater utility in diagnosis for each syndrome. This does not imply that the markers are useful for monitoring disease activity. ANA, antinuclear antibody; dsDNA, double-stranded DNA; MCTD, mixed connective tissue disease; PM/DM, polymyositis/dermatomyositis; RA, rheumatoid arthritis; RNP, ribonucleoprotein; SLE, systemic lupus erythematosus; SS, Sjgrens syndrome; SSc, systemic sclerosis.

species in smokers may lead to exposure of potential autoantigens or may alter antigens rendering them more immunogenic. In the study by Freemer et al.,6 current smokers had a higher risk for anti-dsDNA positivity than never smokers (odds ratio [OR] 4.0; 95% condence interval [CI] 1.610.4). Current smokers had a higher risk than previous smokers (OR 3.0; 95% CI 1.37.1). 3 Recent technical advances both in immunouorescence techniques7 and in multiplex technologies8 are not only allowing for more standardized testing but are increasing throughput and decreasing cost. Multiplex technologies include precoating nitrocellulose strips with multiple antigens or using polystyrene beads coated with antigen. In the latter case, the presence of autoantibodies to specic antigens can be quantied by uorescence-activated cell sorting. A line blot immunoassay has been developed for the simultaneous detection of multiple antibodies to extractable nuclear antigens.9 4 Endometriosis affects 5%15% of women and is characterized by growth of endometrial tissue in extrauterine sites. An association with autoimmunity has long been postulated. In a recent study, circulating ANA was found in 46 out of 112 women with laparoscopically proven endometriosis.10 Laminin-1 is a major component of basement membranes. Autoantibodies to laminin-1 determinants have been associated with infertility and recurrent abortion, and the incidence of such antibodies is increased in endometriosis.11

Conclusion
ANA positivity does not necessarily signify that the patient has an underlying connective tissue disorder. However, the above patient has a high titre of ANA and some symptoms that would be consistent with an autoimmune disorder. History and examination should focus on the presence or otherwise of joint symptoms, skin lesions, renal involvement

114

04 Systemic Lupus Erythematosus, Sjgrens Syndrome and Scleroderma

Positive ANA

Signs and symptoms of Arthropathy Skin lesion Lung involvement (fibrosis) Nervous system involvement

Check renal and hepatic function

? Increased inflammatory markers (ESR and CRP) ? Normocytic anaemia

Titre

1:80

Titre 1:801:320

Titre

1:320

? Clinical significance

Possible clinical significance

Likely to be significant

Ix further only if strong clinical suspicion

If normal SLE unlikely

dsDNA C3 and C4 levels

Screen for ENA

Measure specific autoantibodies

Interpret results with clinical picture

Specific diagnosis and treatment

Figure 21.1 Investigation of a patient with positive ANA. ENA, extractable nuclear antigen; Ix, investigation.

21 Antinuclear factor

115

and central nervous system problems. Increased levels of inammatory markers (ESR and serum C-reactive protein [CRP]) would be consistent with an active autoimmune condition. SLE should be excluded by measurement of anti-dsDNA and complement C3 and C4. Levels of the latter would be lower than normal in a patient with active SLE. Management of the patient with positive ANA is summarized in Figure 21.1.

Further Reading
1 Muro Y. Antinuclear antibodies. Autoimmunity 2005; 38: 39. 2 Narain S, Richards HB, Satoh M et al. Diagnostic accuracy for lupus and other systemic autoimmune diseases in the community setting. Arch Int Med 2004; 164: 243541. 3 Habash-Bseiso DE, Yale SH, Glurich I, Goldberg JW. Serologic testing in connective tissue diseases. Clin Med Res 2005; 3: 1903. 4 Lazzerini PE, Capecchi PL, Guideri F, Acampa M, Galeazzi M, Laghi Pasini F. Connective tissue diseases and cardiac rhythm disorders: an overview. Autoimm Rev 2006; 5: 30613. 5 Edwards CJ, Syddall H, Goswami R, Dennison EM, Cooper C. Infections in infancy and the presence of antinuclear antibodies in adult life. Lupus 2006; 15: 21317. 6 Freemer MM, King TE, Criswell LA. Association of smoking with dsDNA autoantibody production in systemic lupus erythematosus. Ann Rheum Dis 2006; 65: 5814. 7 Tsiakalou V, Tsangaridou E, Polioudaki H et al. Optimized detection of circulating antinuclear envelope autoantibodies by immunouorescence. BMC Immunol 2006; 7: 209. 8 Binder SR. Autoantibody detection using multiplex technologies. Lupus 2006; 15: 41221. 9 Damoiseaux J, Boesten K, Giesen J, Austen J, Tervaert JWC. Evaluation of a novel lineblot immunoassay for the detection of antibodies to extractable nuclear antigens. Ann NY Acad Sci 2005; 1050: 3407. 10 Dias JA, de Oliveira RM, Abrao MS. Antinuclear antibodies and endometriosis. Int J Gynaecol Obstet 2006; 93: 2623. 11 Inagaki J, Kondo A, Lopez LR, Shoenfeld Y, Matsuura E. Pregnancy loss and endometriosis. Pathogenic role of anti-laminin-1 autoantibodies. Ann NY Acad Sci 2005; 1051: 17484.

116

04 Systemic Lupus Erythematosus, Sjgrens Syndrome and Scleroderma

P R O B L E M

22 SLE Risk Factors and Diagnosis

Case History
Jenny, aged 21 years, presented with fatigue, a photosensitive rash on her arms and face, recurrent mouth ulcers and sore joints. She smokes 20 cigarettes per day and consumes very little alcohol. Her medications are the oral contraceptive and minocycline for acne. On what basis could you make a diagnosis of systemic lupus erythematosus (SLE)? What further investigations should be undertaken? What environmental or inherited risk factors predispose to SLE?

Background
Diagnosis of SLE
SLE is a disease characterized by the production of a variety of antibodies against nuclear components that causes inammation and injury to multiple organs. It primarily affects women from the late teens onwards with a peak incidence between the ages of 15 and 45 years. In European populations the prevalence is 1264 cases per 100 000, and it is three to ve times more prevalent in African-American and Afro-Caribbean women. Recent studies in human populations and animal models have associated elements of the innate immune system and abnormalities in the immature B lymphocyte receptor repertoires with disease initiation. A variety of cytokines, most notably type 1 interferons (IFN-a, IFN-b), play an important role in pathogenesis. The American College of Rheumatology (ACR) criteria for the classication of SLE (Table 22.1) act as an aide-memoire to the multisystem nature of SLE and to the investigations that should be undertaken. Patients with SLE almost always suffer from debilitating fatigue, with the majority having associated weight loss and fever. Three-quarters of patients with SLE have mucocutaneous disease in the form of buttery rash, oral ulcers, alopecia and Raynauds phenomenon; purpura, vasculitis or urticaria occur less often. The majority (60%) experience arthralgia or arthritis in a pattern that may mimic rheumatoid arthritis (RA). Renal disease and haematological problems occur in approximately onethird of patients; cardiac, neuropsychiatric and gastrointestinal problems occur in 20%. Lack of antinuclear antibody (ANA) at a titre of 1:160 or higher makes SLE very unlikely. Remaining patients are often positive for anti-Ro antibodies. False-positive ANA results increase with age, and up to 5% of healthy individuals may be ANA positive. Specic ANA immunouorescence patterns such as anti-centromere, anti-nucleolar or proliferating cell nuclear antigen may be reported and require no further characteriza Atlas Medical Publishing Ltd

22 SLE risk factors and diagnosis

117

Table 22.1 American College of Rheumatology (ACR) classication criteria for SLE1
1. Malar rash b Fixed erythema over the malar eminences, sparing the nasolabial folds 2. Discoid rash b Erythematous patch with keratotic scaling and follicular plugging 3. Photosensitive skin rash 4. Oral ulcers 5. Arthritis b Non-erosive joint inammation in 2+ peripheral joints 6. Pleuritis or pericarditis 7. Renal disorder b Proteinuria >0.5 g/24 h, or cellular casts 8. Neurological disorder b Seizures or psychosis (not explained by drugs or metabolic changes) 9. Haematological disorder b Haemolytic anaemia, leukopenia, lymphopenia, thrombocytopenia 10. Immunological disorder b Anti-DNA antibody, anti-Sm antibody, antiphospholipid antibody 11. Antinuclear antibody

tion. Sera with either a homogeneous or rim pattern on ANA testing should undergo antidouble-stranded DNA (anti-dsDNA) testing, and a speckled ANA pattern should undergo testing for antibodies to extractable nuclear antigens. Anti-dsDNA antibodies are the most specic autoantibody; however, they are not very sensitive, occurring in only one-half of lupus patients during the course of their disease. Anti-dsDNA antibodies may also be found in autoimmune hepatitis and chronic infections such as syphilis, subacute bacterial endocarditis and parasitic infection.2 The presence of anti-Sm antibodies is specically identied in the ACR criteria, even though low titres have been reported in other diseases. Anti-Sm is very rare in normal individuals and is virtually pathognomonic for SLE. It is found in 5%30% of SLE patients and is more prevalent in black Americans.

Environmental and genetic interaction in the aetiology


Several genetic loci have been associated to SLE in multiplex families using genome-wide linkage studies. Alleles within the major histocompatibility complex (Fcg receptors, immunoglobulin receptor homologues, cell signalling molecules, cytokines/ chemokines, complement, opsonins etc.) have been found to be associated with SLE. Concordance rate in monozygotic twins is <60%, invoking a signicant environmental contribution to the disease. SLE, as already noted, is characterized by a wide range of autoantibodies that target intracellular, cytoplasmic, cell-surface and plasma antigens. Autoantibodies to DNA and histones individually and together (as the nucleosome complex) have been found in serum of individuals years before the onset of disease, and progressive accumulation of the autoantibodies precedes the onset of disease. An understanding of the mechanism that allows B cells to be exposed to and subsequently to develop antibodies to nuclear antigens that are shielded from immune surveillance is critical to our understanding of the pathogenesis of SLE. A common link between many of the antigens that are targeted in SLE is that they are translocated to the cell surface as nuclear blebs during apoptosis. The current dogma is that clearance of intact dying cells prevents secondary necrosis of apoptotic cells and hence release of nuclear blebs containing potential autoantigens.

118

04 Systemic Lupus Erythematosus, Sjgrens Syndrome and Scleroderma

Table 22.2 Drugs implicated in drug-induced lupus3


Denitely capable of DIL b Hydralazine, procainamide, isoniazid, methyldopa, chlorpromazine, quinidine, minocycline
b

Possibly inducing lupus Sulphasalazine, anticonvulsants, antithyroid drugs, statins, b-blockers, interferon-a, penicillamine, uorouracil, thiazide diuretics Suggested to induce lupus Multiple antibiotics, oestrogens and oral contraceptives, captopril, calcium channel blockers, etanercept, iniximab

However, under special circumstances, apoptotic cells may trigger an immune response, which may activate T and B cells and the formation of autoantibodies.

Drug-induced lupus
There are various kinds of lupus: (a) SLE; (b) discoid lupus (inammatory scarring skin lesions); (c) subacute cutaneous lupus (non-scarring, non-atrophy producing photosensitive dermatosis); and (d) drug-induced lupus (DIL). Drugs responsible for developing DIL can be divided into three groups (Table 22.2).3 There are no symptoms pathognomonic for DIL; however, some features are common. Patients often present with mild lupus-like symptoms, which can develop as early as one month after initiation of treatment or can be delayed as long as a decade. Men are as frequently affected as women, and Caucasians are affected up to six times more frequently than black patients and may have a more severe illness. Features common in DIL are fever, arthralgia, pleuritis, pericarditis, mild cytopenia, anaemia and elevated erythrocyte sedimentation rate. Hypocomplementaemia and clinical features of malar or discoid rash, photosensitivity, oral ulcers, alopecia and renal or neurological disorder are very uncommon. Anti-histone antibodies, especially IgG anti-([H2A-H2B]-DNA) are positive in 75%95% of DIL patients, but also occur in 75% of SLE patients. There is usually an absence of anti-dsDNA and anti-extractable nuclear antigen (anti-ENA) antibodies, with a high frequency of anti-single-stranded DNA (anti-ssDNA) antibodies. Amongst the medications denitely capable of inducing lupus, procainamide is currently the drug most frequently associated with DIL. Between 2% and 21% of patients receiving hydralazine also develop the disease, the greatest risk being with doses >200 mg/day. The link between minocycline and DIL became evident when minocycline became used for the treatment of acne and RA. A casecontrol prospective study found an eight-fold increased risk for current users, and DIL has been individually conrmed by rechallenge. Developing any time between three months and six years after initiation, minocycline DIL (in contrast to classical DIL) has a female preponderance. Typical is a symmetrical polyarthritis with early morning stiffness, often with fever and myalgia. A wide range of cutaneous manifestations are reported: vasculitis, livedo reticularis, erythema nodosum and subcutaneous nodules. Positive ANA is found in >80% of patients. Hepatic damage is often associated but prognosis of minocycline DIL is generally good, with rapid resolution of all aspects including chronic active hepatitis on withdrawal of the drug. The high incidence of SLE in females during their reproductively capable years has prompted the association with female sex hormones. However, it remains controversial as to whether the addition of exogenous oestrogens on a background of endogenous

22 SLE risk factors and diagnosis

119

hormones can induce or worsen lupus. It had been suggested that both oral contraceptive pills (OCPs) and hormone replacement therapy (HRT) could be used without changing disease activity if patients were normotensive, non-smoking, with only stable to moderate disease and were antiphospholipid antibody negative.4 The Safety of Estrogens in Lupus Erythematosus National Assessment (SELENA) trials comprised two separate randomized controlled trials of low-dose synthetic oestrogen and progestin OCPs,5 and HRT6 in women with inactive or stable SLE disease activity. The twelve-month rates of severe ares and for mild or moderate ares were almost identical for the group receiving OCPs and for the placebo group, demonstrating safe use of low-dose OCPs in SLE patients with stable disease. In the HRT trial severe ares were rare, with a twelve-month severe are rate of 0.081 for the HRT group and 0.049 for the placebo group. Mild to moderate ares were signicantly increased in the HRT group: 1.14 ares/person-year for HRT vs 0.86 are/person-year for placebo (P <0.01), with an increased probability of any type of are by twelve months in the HRT group (P <0.01). Adding a short course of HRT is associated with a small risk for increasing the natural are rate of lupus.

Recent Developments
1 Whereas genetic information (with the exception of somatic mutations) is homogeneous in an organism regardless of cell type, epigenetic modications are characteristic of different cell types and play a role in dening the transcriptome, which determines the identity of each cell type.7 The epigenetic framework could explain several disease characteristics, including age dependence and quantitative nature, and the mechanism by which environment modulates genetic predisposition to disease. Abnormalities in the DNA methylation system aberrantly increase or decrease gene expression, which has been implicated in SLE. Support for this concept is the global hypomethylation of T cells from patients with active SLE and the hypomethylating effect of drugs used to induce SLE such as procainamide, hydralazine and 5-azacytidine (Table 22.3). In the case of histone modication, histone-deacetylating drugs skew gene expression of CD40L, interleukin (IL)-10 and interferon-g. Alteration of T cellB cell interactions has been proposed as the common pathogenic mechanism that leads to disease. However, the recognition of dendritic cells (DCs) as efcient stimulators of B and T cell lymphocytes, as well as key controllers of immunity and tolerance, has led to the hypothesis that SLE may be driven by inappropriate DC activation. Two mechanisms have been described: elevated functional levels of interferon,8 and defective complement-mediated clearance of apoptotic cells (Figure 22.1).9 2 Monocytes from SLE patients have enhanced antigen-presenting ability and act like myeloid dendritic cells; serum from SLE patients rapidly differentiates normal monocytes into cells with DC morphology and function. A candidate serum cytokine that may be responsible for this maturation is type 1 interferon (IFN-1). Elevated levels of IFN-1 have been found in the serum of SLE patients. Administration of exogenous IFN-1 as therapy for chronic viral infection or malignancy is associated with autoantibody production in up to 34% of recipients and autoimmune complications in 4%19%.10 Mature DCs activate cytotoxic T cells, leading to cell death and the generation of nucleosomes that can be captured and presented by DCs

120

04 Systemic Lupus Erythematosus, Sjgrens Syndrome and Scleroderma

Table 22.3 Epigenetic changes in SLE


Epigenetic change
Methylation of cytosine in CpG dinucleotides

Consequence
Repression of transcription promoters Altered nuclear architecture

Disease association
Autoreactivity via T cell response to sub-threshold antigenic stimuli, and overexpression of adhesion receptor LFA-1 CD70 overexpression leading to excess B cell stimulation in SLE Repression of tumour suppressor genes leading to malignant transformation Hypomethylation causing chromosomal instability Failed transcriptional regulation of imprinted genes (e.g. PraderWilli syndrome)

Histone modication b Lysine acetylation b Lysine methylation, serine phosphorylation

Transcriptional activity Variable transcriptional effect Altered nuclear architecture

Complement deficiency

Viral infection

IFN mutations

Clearance of apoptotic cells

Type 1 interferon

Monocyte

Apoptosis and nuclear blebs

Myeloid DC

Loss of tolerance and autoantibody production

Figure 22.1 Decreased complement and increased interferon (IFN) predispose to autoantibody production. DC, dendritic cell.

generated in the presence of IFN-1. Together with IL-6, IFN-1 promotes the differentiation of mature B cells into plasma cells, which are long-lived autoantibody producers. Thus, the effect of IFN-1 on DCs, B cells and T cells could explain the breakdown of tolerance to nuclear antigens, and the subsequent autoantibody secretion and immune complex formation characteristic of SLE. 3 In humans there is a very strong association between deciencies of the complement components C1 and C4 and the presence of SLE. It has been hypothesized9 that complement deciency leads to impaired clearance of dying cells and allows prolonged exposure of nuclear blebs (the putative source of autoantigens). The same authors expanded the theory to include any mechanism that impairs clearance of apoptotic cells, with evidence for C1q, C4, IgM, macrophage phagocytosis and tissue transglutaminase. In the presence of adequate complement levels, apoptotic cells are rapidly taken up by immature DCs before the release of intracellular material can

22 SLE risk factors and diagnosis

121

activate them, yet at the same time allowing for the nuclear material to be processed by the DC in its immature form leading to tolerance.

Conclusion
SLE is one of the great masqueraders in medicine, with multiple varied presentations. Despite its potential to be a devastating illness, in the majority of cases the presentation is often a poorly dened illness with subjective complaints greater than any observed abnormalities. Many of the classication criteria listed in Table 22.1 develop over time and are not cumulative, such that a detailed previous history needs to be undertaken, particularly as many of the features such as mouth ulcers or photosensitivity may not subsequently be recalled spontaneously. It is in this setting that the clinical challenge is to consider the diagnosis of SLE, to investigate appropriately, but then not to over-rely on the presence of an antinuclear antibody to make the diagnosis.

Further Reading
1 Hochberg MC. Updating the American College of Rheumatology revised criteria for the classication of systemic lupus erythematosus. Arthritis Rheum 1997; 40: 1725. 2 Kurien BT, Scoeld RH. Autoantibody determination in the diagnosis of systemic lupus erythematosus. Scand J Immunol 2006; 64: 22735. 3 Sarzi-Puttini P, Atzeni F, Capsoni F, Lubrano E, Doria A. Drug-induced lupus erythematosus. Autoimmunity 2005; 38: 50718. 4 Kreidstein S, Urowitz MB, Gladman DD, Gough J. Hormone replacement therapy in systemic lupus erythematosus. J Rheumatol 1997; 24: 214952. 5 Petri M, Kim MY, Kalunian KC et al. Combined oral contraceptives in women with systemic lupus erythematosus. N Engl J Med 2005; 353: 25508. 6 Buyon JP, Petri MA, Kim MY et al. The effect of combined estrogen and progesterone hormone replacement therapy on disease activity in systemic lupus erythematosus: a randomized trial. Ann Intern Med 2005; 142: 95362. 7 Ballestar E, Esteller M, Richardson BC. The epigenetic face of systemic lupus erythematosus. J Immunol 2006; 176: 71437. 8 Pascual V, Farkas L, Banchereau J. Systemic lupus erythematosus: all roads lead to type 1 interferons. Curr Opin Immunol 2006; 18: 67682. 9 Cook HT, Botto M. Mechanisms of disease: the complement system and the pathogenesis of systemic lupus erythematosus. Nat Clin Pract Rheumatol 2006; 2: 3307. 10 Borg FAY, Isenberg DA. Syndromes and complications of interferon therapy. Curr Opin Rheumatol 2007; 19: 616.

122

04 Systemic Lupus Erythematosus, Sjgrens Syndrome and Scleroderma

P R O B L E M

23 Monitoring and Managing SLE

Case History
Jane, having contacted the Arthritis Foundation and looked on the internet, realises that systemic lupus erythematosus (SLE) is a multisystem disorder. She wishes to discuss the potential problems that she may face in the future. What are the most frequent manifestations of SLE? What monitoring should she undergo? What are the current treatments for SLE?

Background
The term lupus was rst used in the early 1800s, and is derived from the Latin term for wolf (Canis lupus). It was applied to the skin condition lupus vulgaris to imply an appearance of the skin that looked to be torn off as a wolf might have done. Subsequently, the Latin term for redness erythematosus was appended. The clinical features are represented in Figure 23.1. Almost ubiquitous is the presence of fatigue, weight loss and intermittent fevers. After constitutional symptoms, arthralgia and mucocutaneous manifestations are the most common, and in various forms account for ve of the eleven American College of Rheumatology (ACR) criteria for the classication of SLE. Joint pain with or without objective inammation is the most common reason for patients to present to their doctor. Most commonly, this presents as a small joint peripheral arthritis, which also involves wrists and knees and in many respects resembles rheumatoid arthritis. Both can be multisystem in manifestation, and nodules similar to rheumatoid nodules may be present in up to 10% of patients. It is unusual for patients with SLE to develop any erosive changes. The buttery rash in a malar distribution with sparing of the nasolabial folds is the pathognomonic rash of SLE. Photosensitivity occurs in around one-half of SLE patients, and warrants reminding of the importance of sun avoidance and sunblock, with light in the 360400 nm spectrum inducing both rash and systemic exacerbations. Other rashes are less common. Alopecia varies in its pattern from more hair in my brush or shower plug hole to distinct patches of hair loss, particularly if there have been scarring discoid lesions of the scalp. Immunosuppressive agents may lead to hair thinning. Progressive renal disease in SLE is often asymptomatic and should be sought by urinalysis for blood and protein and serial serum creatinine measurements. Any abnormality warrants further evaluation including 24-hour urine collection, microscopy of urinary sediment and a low threshold for review by a renal physician. Atlas Medical Publishing Ltd

23 Monitoring and managing SLE

123

Alopecia Rash Oral ulcer 75%

Depression Psychosis 12%20%

Pleurisy Lung disease 15%30%

Pericarditis Cardiac disease 5%20%

Renal 40%70%

Arthritis 5%75%

Raynauds 35%50%

Gastrointestinal 15%25%

Figure 23.1 Clinical features of SLE.

Pleural manifestations have been reported in 30%60% of patients, and include symptomatic pleurisy, effusions and pleural rubs. Pleurisy is usually not associated with pleural rub, and the differential diagnosis includes primary pleural serositis and secondary serositis due to underlying infection, or pulmonary infarct (particularly in those with thrombophilia). Pleural effusions may be found incidentally on chest X-ray and are usually small. When sufciently large to drain, an exudate is present with a normal glucose level.

Monitoring SLE
SLE aficts African-Americans three times more frequently than their EuropeanAmerican counterparts, and they are considered to have a poorer prognosis. Some of the

124

04 Systemic Lupus Erythematosus, Sjgrens Syndrome and Scleroderma different socioeconomic, demographic, psychological and behavioural variables are highly correlated. The LUMINA (Lupus in minorities: nature versus nurture) cohort study resulted in a series of papers, with a recent report on predictors of disease activity over time.1 The cohort of 554 subjects included 18% Texan Hispanics, 17% Puerto Rican Hispanics, 36% African-Americans and 29% Caucasians. Univariate analysis conrmed an odds ratio of 3 between Texan Hispanic or African-American ethnicity and Caucasian, and higher disease activity over time. These characteristics remained signicant in a multivariate model. High disease activity was independently associated with lack of health insurance, abnormal illness-related behaviour and poor social support, and was negatively associated with age.

Laboratory monitoring
Early detection of renal disease is a major goal (Table 23.1), as the survival curves are vastly different for those with and without renal disease. Specic symptoms are not observed by the individual until there is advanced nephrotic syndrome or renal failure. Urinalysis is an extremely cheap and highly sensitive approach to detect haematuria and proteinuria, and can then be followed by a 24-hour urine collection and quantitative microscopy. Regular serum creatinine measurement is recommended. Routine haematology is undertaken for the detection of anaemia, leukopenia and thrombocytopenia. Interpretation of results should take into account the effect of treatment, with steroids elevating neutrophil counts and decreasing lymphocyte counts, and immunosuppressants exerting cytotoxic effects. In the LUMINA study, the immunological variable identied as being independently associated with high levels of disease activity was the presence of anti-double-stranded DNA (anti-dsDNA) antibodies.1 Previously these antibodies had been associated with disease activity and with lupus nephritis but not concomitantly with ares, as it is proposed that as the immune complexes are deposited in tissues their circulating levels decrease. Increasing levels of anti-dsDNA antibodies, which then fall, may herald exacerbations of lupus nephritis or other organ involvement. Whilst this remains controversial,

Table 23.1 Monitoring of SLE patients


History
Fatigue Mucocutaneous features (oral ulcers, photosensitive rash) Chest pain, SOBOE Arthritis Urinary features

Examination
Vitals BP, pulse, temperature Cardiorespiratory rubs, effusions and brosis Arthritis number and severity Skin rash

Laboratory investigations
Urinalysis for proteinuria, haematuria Urine microscopy for casts Full blood screen Renal function C-reactive protein

Preventative screening Smoking cessation; body mass index goal <25 kg/m2 Monitor BP (target of <130/80 mmHg). Treat if >140/90 mmHg Fasting lipids (target LDL-cholesterol <2.6 mmol/l. Treat if >34 mmol/l Fasting glucose, and homocysteine Anti-Ro, anticardiolipin and lupus anticoagulant prior to planned pregnancy Anti-dsDNA antibodies
BP, blood pressure; LDL, low-density lipoprotein; SOBOE, shortness of breath on exertion.

23 Monitoring and managing SLE

125

Hypertension

Hyperhomocysteinemia

Hyperlipidaemia

Diabetes

Inflammation CRP, TNF Cardiovascular disease in SLE

Lipid oxidation

Prothrombosis (anticardiolipin antibodies)

? SLE treatment (steroid therapy)

Smoking

Figure 23.2 Potential factors for cardiovascular disease in SLE. Adapted with permission from Frostegrd 2005.2

some authors have recommended corticosteroid therapy to prevent ares in SLE patients manifesting this pattern. If an increasing titre of anti-dsDNA antibody is detected, careful examination should be undertaken.

Preventative monitoring
With modern drugs, the survival rate for SLE has dramatically improved, as has survival from acute disease ares. This has, however, revealed a later mortality due to cardiovascular disease (CVD). The predisposition of younger women to SLE results in premenopausal women with SLE having a 50-fold increased risk of myocardial infarction. Autopsy and angiographic studies demonstrate increased prevalence of atherosclerotic lesions in SLE patients. Measurements of intimamedia thickness (IMT) of the carotid artery can be used as a surrogate measure of atherosclerosis; it is not clear from studies of IMT in patients with SLE whether this marker is increased in the condition. Crosssectional studies reveal carotid plaques in one-third of women with lupus. There remains uncertainty as to whether atherosclerosis in SLE is increased in a general distribution or whether it is primarily an increased risk of localized plaque. Traditional risk factors for CVD should be sought and monitored in patients with SLE, but research is highlighting a number of other risk factors (Figure 23.2).2 The suppression of inammation is the aim of SLE therapy, and this can be assessed by both the erythrocyte sedimentation rate (ESR) and serum C-reactive protein (CRP). In

126

04 Systemic Lupus Erythematosus, Sjgrens Syndrome and Scleroderma SLE the profound hypergammaglobulinaemic response leads to chronic elevation of the ESR, leaving CRP to be the better marker of inammation. Systemic inammation accelerates the atherosclerotic process. In the general population, increased levels of CRP are associated with increased risk of CVD. In one of the LUMINA reports, CRP was a specic risk factor for CVD: the median serum CRP in all patients with vascular events was approximately three times that in those without vascular events. This nding, however, also raises the conundrum of corticosteroids, which on the one hand are very potent inhibitors of inammation, yet in long-term high doses do contribute to adverse cardiovascular outcomes. It is possible that low doses of corticosteroids that optimize suppression of inammation may have a net benecial cardiovascular effect.3 Guidance and support in the cessation of smoking is advised, as smoking promotes both atheroma and ares of lupus. A recent meta-analysis showed that smoking increased the likelihood of SLE (odds ratio 1.5), most notably with antibodies to dsDNA.4,5 The riskbenet ratio of low-dose aspirin (60100 mg/day) favours its use in all patients with SLE.6 All SLE patients who have dened risk for CVD should receive low-dose aspirin, as should those patients with both SLE and either a positive immunoglobulin G anticardiolipin antibody or lupus anticoagulant.

Treatment of SLE
Modiable lifestyle factors include sun avoidance, stopping smoking, exercise, diet (with emphasis on altering the omega-3:omega-6 balance and reducing salt intake if hypertensive) and weight management. Specic interventions may be required for the treatment of dyslipidaemia, diabetes and hypertension. The threshold for intervention is lower due to the adverse effect of SLE itself. In practice, sun avoidance needs to be constantly reinforced wide-brimmed hats, clothing that provides sun protection and the regular application of quality sunblock. The availability of a range of cosmetic-grade moisturisers and make-up containing sunblock has aided the acceptability of this advice. Non-steroidal anti-inammatory drugs and selective cyclooxygenase-2 (COX-2) inhibitors are frequently used for soft tissue symptoms and control of arthralgia and arthritis in the general population, and have been used in SLE, although they are not usually specically registered for this population group. They should, however, be used with caution due to risks of induction of hypertension, peripheral oedema and reduced renal function. Most regulatory agencies have stated that COX-2 inhibitors should not be used in patients who have, or are at major risk of, CVD. Hydroxychloroquine (200400 mg/day) is useful for skin rashes, joint pain and oral ulcers. It also has a weak antithrombotic action. Doses of <6 mg/kg/day are associated with a very low risk of retinopathy, and physicians are advised to discuss with a local ophthalmologist as to a monitoring regime. Higher doses can be used in the short term for patients with ongoing photosensitivity. While its mechanism of action remains uncertain, it does alter antigen processing by affecting the pH of the phagolysosome, and may also regulate innate immunity via Toll-like receptors. A teratogenic effect was suspected but seems to have been ruled out,7 and its use is safe and effective in pregnancy. Dehydroepiandrosterone (DHEA) has been advocated for the treatment of SLE, providing a reduction in fatigue. DHEA is a sex-hormone precursor produced by the adrenal glands and is converted to either oestrogen or androgen in different peripheral tissues.

23 Monitoring and managing SLE

127

Randomized placebo-controlled trials support a modest benecial effect when doses of 200 mg daily are used, but this is outweighed by side effects of hirsutism, acne, hypertension and adverse lipid prole.8 It is tempting to prescribe oral corticosteroids to the patient who is miserable with symptoms of SLE. However, once commenced, it is very hard to keep good control on the dose and to cease therapy. Corticosteroid toxicity includes hyperglycaemia, hypercholesterolaemia, hypertension, osteoporosis, glaucoma, weight gain, skin fragility, myopathy and avascular necrosis. As an adjunct and steroid-sparing agent, azathioprine is one of the immunosuppressants most commonly used in SLE (chapter 27). A purine analogue, its effect is to inhibit DNA synthesis, and in doses of 1.52.5 mg/kg/day it has benet in preventing serious lupus complications. Drug hypersensitivity occurs in a small percentage of patients, resulting in systemic illness, rash, acute hepatitis and abdominal pain. Monitoring of bone marrow, liver function and lipase is recommended at four- to sixweekly intervals. As a cytotoxic agent it has been generally recommended that azathioprine not be used in pregnancy although the evidence for teratogenicity is limited and the benecial effects of disease control on fertility, fetal outcome and the health of the mother may warrant its continuation during pregnancy.

Recent Developments
1 SLE nephritis is characterized by immune complex-mediated glomerular and tubulointerstitial inammation, which leads to chronic renal insufciency in up to 30% of affected patients. Cyclophosphamide is the principal treatment of diffuse glomerulonephritis (World Health Organization [WHO] class IV). Cyclophosphamide given as 6 500 mg infusions followed by oral azathioprine was as effective as the initial high-dose cyclophosphamide National Institutes of Health (NIH) regime.9 The evolution of mycophenolate mofetil (MMF) from a transplant rejection modier to a broader immunosuppressant application has shown that 23 g/day is as effective as cyclophosphamide in inducing and maintaining remission in those with mild to moderate renal disease. MMF has a better safety prole than cyclophosphamide, avoiding haemorrhagic cystitis, ovarian failure and the longerterm cancer risks. A systematic review and meta-analysis found that a complete or partial (66%80%) response was more frequent with MMF than with cyclophosphamide, with one additional complete or partial response for every eight patients treated.10 2 Central nervous system involvement has been described in up to 50% of lupus patients. Cognitive dysfunction and headaches are the most common features, affecting 60%80% of patients. Aseptic meningitis, demyelination, myelopathy, acute confusional states or psychosis are rare. The prevalence of depression is no higher in SLE than in other chronic debilitating diseases.11 Psychosis in lupus is characterized by delusions or hallucinations; it is uncommon in SLE and occurs in only 2%5% of patients receiving high-dose (1 mg/kg/day) corticosteroids. Secondary causes such as drug use or drug withdrawal, metabolic and electrolyte derangements and sepsis should always be excluded before concluding that it is lupus psychosis.

128

04 Systemic Lupus Erythematosus, Sjgrens Syndrome and Scleroderma

Conclusion
SLE is challenging both to diagnose and to treat successfully. With its protean manifestations, milder forms are being increasingly recognized or suspected due to the increasing availability of autoantibody testing. There have been major advances in therapy. Immunosuppressants such as MMF have led to signicant gains in the treatment of nephritis. Even newer therapies are on the horizon, with the monoclonal antibody rituximab providing dramatic and long-lasting remissions in patients who were previously unresponsive. The remaining challenges are in improving overall quality of life, reducing the impact of fatigue and protecting patients from cardiovascular deaths as they survive the initial insult of SLE.

Further Reading
1 Alarcn GS, Calvo-Aln J, McGwin G et al; LUMINA study group. Systemic lupus erythematosus in a multiethnic cohort: LUMINA XXXV. Predictive factors of high disease activity over time. Ann Rheum Dis 2006; 65: 116874. 2 Frostegrd J. SLE, atherosclerosis and cardiovascular disease. J Intern Med 2005; 257: 48595. 3 Hall FC, Dalbeth N. Disease modication and cardiovascular risk reduction: two sides of the same coin? Rheumatology 2005; 44: 147382. 4 Costenbader KH, Kim DJ, Peerzada J et al. Cigarette smoking and the risk of systemic lupus erythematosus: a meta-analysis. Arthritis Rheum 2004; 50: 84957. 5 Freemer MM, King TE, Crisswell LA. Association of smoking with dsDNA autoantibody production in systemic lupus erythematosus. Ann Rheum Dis 2006; 65: 5814. 6 Wahl DG, Bounameaux H, de Moerloose P, Sarasin FP. Prophylactic antithrombotic therapy for patients with systemic lupus erythematosus with or without antiphospholipid antibodies: do the benets outweigh the risks? Arch Int Med 2000; 160: 20428. 7 Costedoat-Chalumeau N, Amoura Z, Duhaut P et al. Safety of hydroxychloroquine in pregnant patients with connective tissue diseases: a study of one hundred thirty-three cases compared with a control group. Arthritis Rheum 2003; 48: 320711. 8 Sibilia J. Treatment of systemic lupus erythematosus in 2006. Joint Bone Spine 2006; 73: 5918. 9 Houssiau FA, Vasconcelos C, DCruz D et al. Early response to immunosuppressive therapy predicts good renal outcome in lupus nephritis: lessons from long-term followup of patients in the Euro-Lupus Nephritis Trial. Arthritis Rheum 2004; 50: 393440. 10 Moore RA, Derry S. Systematic review and meta-analysis of randomised trials and cohort studies of mycophenolate mofetil in lupus nephritis. Arthritis Res Ther 2006; 8: R182. 11 Bruns A, Meyer O. Neuropsychiatric manifestations of systemic lupus erythematosus. Joint Bone Spine 2006; 73: 63945.

24 Sjgrens syndrome P R O B L E M

129

24 Sjgrens Syndrome

Case History
You have been asked to see Wilma, aged 52 years, because she may have Sjgrens syndrome (SS). Wilma has a teenage family of three and works part-time in the family dry cleaning business. Recently she has become fatigued. Her optician recommended drops for dry eyes and she complains of mouth dryness. Investigations have been normal apart from a 1:160 positive antinuclear antibody (ANA). What are the potential causes of sicca syndrome? How would you investigate? Assuming she has SS, how should she be followed-up and managed?

Background
After rheumatoid arthritis (RA), SS is the second most common autoimmune rheumatic disease.13 It is named after Henrik Sjgren, who described a series of cases in 1933. SS affects 0.5% of the population and is nine times more common in women. SS may affect up to two million people in the United States. There are two peaks of onset one after menarche and a second following menopause. The cardinal symptoms are dryness of the eyes (keratoconjunctivitis sicca) and mouth (xerostomia) secondary to autoimmune disease of the exocrine glands (lacrimal and salivary). The international consensus criteria for diagnosis of primary SS are summarized in Box 24.1. Diagnosis of secondary SS can be made when there is an established connective tissue disease, at least one symptom of sicca syndrome and at least two objective tests to conrm the diagnosis. SS associated with RA may produce more lacrimal than oral symptoms. Secondary SS may occur in up to 40%50% of patients with RA, 10%30% of those with SLE and may also occur in systemic sclerosis, polymyositis and polychondritis. Tests to measure tear and saliva production should be carried out when the patient is not taking anticholinergic drugs. At least 50% of glandular tissue needs to be lost before patients begin to experience symptoms. The following should be considered in a differential diagnosis: radiation to the head and neck, sarcoidosis, infection (human immunodeciency virus [HIV], human T-lymphotropic virus type 1 and hepatitis C) and graft-versus-host disease. In SS, the exocrine glands are inltrated by CD4+ helper T lymphocytes, which trigger activation of cytotoxic T cells and B cells with consequent production of autoantibody and destruction of glandular tissue. Anti-SSA/Ro or anti-SSB/La are present in a high proportion of patients and may have a diagnostic sensitivity comparable with that of Atlas Medical Publishing Ltd

130

04 Systemic Lupus Erythematosus, Sjgrens Syndrome and Scleroderma

Box 24.1 International consensus criteria for diagnosis of SS Ocular symptoms (at least one) Persistent dry eyes every day for at least three months Recurrent sensation of sand or gravel in the eyes Use of a tear substitute more than three times per day Oral symptoms (at least one) Persistent dry mouth every day for at least three months Recurrent feeling of swollen salivary glands Need to drink liquid in order to swallow dry food Objective evidence of dry eyes (at least one) Schirmers test Rose-Bengal Lacrimal gland biopsy with focus score >1 (at least 50 lymphocytes per lobule with four lobules assessed) Objective evidence of salivary gland involvement (at least one) Salivary gland scintigraphy Parotid sialography Unstimualted whole sialometry (1.5 ml per 15 minutes) Laboratory abnormality (at least one) Anti-SSA or anti-SSB ANA Immunoglobulin M (IgM) rheumatoid factor Positive biopsy of a minor salivary gland Focus score >1 (see above) At least four of the above six criteria should be satised for a positive diagnosis of primary SS salivary gland biopsy.4 ANA is positive in the majority, but is not detectable in up to 25% of patients. Other autoantibodies may also be detected, particularly in secondary SS.

Dry eyes
Over 85% of patients with SS present with sicca syndrome. Decreased tear production can be demonstrated using Schirmers test, where a strip of lter paper is placed under the lower eyelid. The degree of wetting of the paper over ve minutes is measured. There is an age-related decrease in tear formation, probably contributed to by androgen or oestrogen deciency. Subjects who habitually use computer screens may complain of dry eyes because of decreased blinking, particularly if they work in a low-humidity environment. Not only is the volume of tears decreased in sicca syndrome, but there is also decreased tear content of lysozyme and lactoferrin. Management may include use of articial tears, topical immunosuppressives (corticosteroid, cyclosporin) and punctal occlusion.

Dry mouth
Patients with dry mouth are at risk of oral candidiasis, dental caries and ssuring of the tongue and gums. Decreased saliva ow is easily demonstrated using sialometry the

24 Sjgrens syndrome

131

patient discharges his/her saliva into a vessel for 15 minutes and the amount of saliva is weighed. Flow rates of <0.1ml/min are abnormal. Differential diagnosis of dry mouth is presented in Box 24.2. Salivary ow is stimulated by acetylcholine acting through muscarinic M1 and M2 receptors. Drugs with anticholinergic action (e.g. tricyclic antidepressants) should be avoided where possible. Muscarinic secretagogues (pilocarpine or cevimiline) may improve salivary ow. Patients should avoid alcohol and smoking if possible, and not use alcohol-containing mouthwashes. Careful attention should be paid to oral and dental hygiene. Antibacterial mouthwashes (e.g. chlorhexidine) may be useful. Chewing sugar-free gum or use of articial saliva may provide temporary relief. Fluoridereleasing preparations including dental varnishes are often used. A topical preparation of interferon is being tried although, as with decreased lacrimation, the glands are largely destroyed by the time symptoms develop and immunomodulatory treatment is likely to be of limited use.

Extraglandular manifestations
These are extremely variable but can pose a considerable threat to health. Skin lesions include palpable and non-palpable purpura, alopecia, vitiligo and an increased incidence of cutaneous lymphoma. Arthralgia is typically symmetrical and non-erosive, although erosive arthralgia can occur. Muscular pains can produce a picture like bromyalgia or polymyalgia, but inammatory polymyositis can occur. Gastrointestinal manifestations include swallowing problems related to decreased saliva, atrophic gastritis, abnormal liver tests and autoimmune liver disease. The lungs may be affected by interstitial pneumonitis and there is an increased risk of pulmonary lymphoma. Pericarditis and pulmonary hypertension can occur. Renal problems, including interstitial nephritis, may occur with SS or as a result of immunosuppressives. Cranial and peripheral neuropathies Box 24.2 Differential diagnosis of xerostomia Sjgrens syndrome b primary b secondary Salivary duct aplasia or atresia Salivary duct stones Sialadenitis b bacterial (staphylococcal, streptococcal) b viral (e.g. mumps) Irradiation or chemotherapy Dehydration, mouth breathing Diabetes (mellitus or insipidus) Hypercalcaemia Hyperlipoproteinaemia Autonomic neuropathy LambertEaton syndrome Drugs Anticholinergics Tricyclic antidepressants Antihistamines Benzhexol, benztropine Entacapone Loperamide Diuretics Smoking Alcohol Marijuana Taste disturbances Head/neck trauma Sarcoidosis Amyloidosis Cirrhosis

132

04 Systemic Lupus Erythematosus, Sjgrens Syndrome and Scleroderma are described. Autoimmune thyroiditis is very common. The chronic and disabling symptoms may lead to anxiety, depression or other psychological problems. There is a 40-fold increase in the risk of lymphoma in SS. Manifestations may include enlarging parotids, hypergammaglobulinaemia and hepatosplenomegaly. The presence of systemic features may necessitate the prescription of diseasemodifying drugs. Hydroxychloroquine (68 mg/kg/day) is useful where there is myalgia or arthralgia. Visceral manifestations may respond to low-dose corticosteroids (prednisolone <15 mg/day). Other immunosuppressives used include azathioprine (12 mg/ kg/day), methotrexate (7.525 mg/week) and cyclophosphamide (0.51 g/m2/day). Leunomide or cyclosporin are useful in refractory cases.

Recent Developments
1 Both sarcoidosis and SS affect salivary and lacrimal glands and they share many systemic features. The presence of autoantibodies obviously favours SS, while hilar lymphadenopathy or hypercalcaemia would point towards sarcoidosis. A number of cases have now been identied where the two conditions coexist.5 While this may be coincidence, it could point to the presence of common aetiological factors. 2 Geographical distribution and seasonality of onset suggest that there could be an infectious trigger. Amongst the leading candidates are Coxsackie viruses, which appear to have a predilection for exocrine glands. Recently, Coxsackie virus B4 sequences have been identied in the salivary glands of SS patients.6 Subclinical infection may trigger localization of helper lymphocytes in the salivary glands leading to lymphocytic and dendritic cell activation. 3 Anti-centromere antibodies are characteristic of limited forms of scleroderma but can also be detected in some patients with SS. The molecular determinants are a group of centromere proteins (CENP-A, -B, -C, -D, -E, -F, -G and -H). Antibodies to CENP-B or CENP-C have been described in about one-half of patients with limited scleroderma and one-fth of those with SS.7 By contrast, antibodies to CENP-H may be more specic for SS, and may identify a distinct group separate from those who are positive for anti-Ro and anti-La.8 4 The difculty in dening SS as a clinical entity has led to a quest for more specic serological markers. Antibodies to the protein a-fodrin were described about ten years ago. a-fodrin is an intrinsic membrane protein that binds to calmodulin, actin and microtubules, and is involved in regulation of exocytosis. A recent study9 has conrmed that these antibodies are present in a small proportion of a large series of SS patients, and may be positive in some who are negative for anti-Ro and anti-La.

Conclusion
Normally, up to 500 ml of saliva is produced daily. SS is an autoimmune condition, and investigations should conrm diminished lacrimal and salivary secretion. Ideally, autoimmune markers should be positive, and evidence of lymphocytic inltration should be demonstrated on a biopsy of lacrimal or salivary gland. SS may be primary or occur in association with an underlying connective tissue disease. If the disease is thought to be primary, careful inquiry should be made about systemic symptoms and involve-

24 Sjgrens syndrome

133

4 out of 6 positive

Dry eyes Decreased lacrimation (test) Dry mouth Decreased saliva (test) Autoantibody (Ro, La, ANA) Positive salivary/lacrimal biopsy

Underlying CT disease

Primary SS

Secondary SS

Dry eyes Artificial tears Punctal closure Local immunosuppressives Exclude other causes Dry mouth Dental/oral hygiene Pilocarpine, cevimiline Artificial saliva

Systemic/visceral involvement

Low-dose prednisolone

Azathioprine, cyclophosphamide or methotrexate

Leflunomide or cyclophosphamide

Figure 24.1 Investigation and management of SS. CT, connective tissue.

ment of organs other than the exocrine glands. Treatment is mainly symptomatic and topical (Figure 24.1), with immunosuppressive treatment being reserved for patients with involvement of other organs including the lung and kidney.

Further Reading
1 Derk CT, Vivino FB. A primary care approach to Sjgrens syndrome. Postgrad Med 2004; 116: 4959.

134

04 Systemic Lupus Erythematosus, Sjgrens Syndrome and Scleroderma


2 3 4 5 6 7 Fox RI. Sjgrens syndrome. Lancet 2005; 366: 32131. Fox RI, Liu AY. Sjgrens syndrome in dermatology. Clin Dermatol 2006; 24: 393413. Kessel A, Toubi E, Rozenbaum M, Zisman D, Sabo E, Rosner I. Sjgrens syndrome in the community: can serology replace salivary gland biopsy? Rheumatol Int 2006; 26: 3379. Ramos-Casals M, Brito-Zeron P, Garcia-Carrasco M, Font J. Sarcoidosis or Sjgren syndrome? Clues to dening mimicry or coexistence in 59 cases. Medicine 2004; 83: 8595. Triantafyllopoulou A, Moutsopoulos HM. Autoimmunity and coxsackievirus infection in primary Sjgrens syndrome. Ann NY Acad Sci 2005; 1050: 38996. Gelber AC, Pillemer SR, Baum BJ et al. Distinct recognition of antibodies to centromere proteins in primary Sjgrens syndrome compared with limited scleroderma. Ann Rheum Dis 2006; 65: 102832. Hsu TC, Chang CH, Lin MC, Liu ST, Yen TJ, Tsay GJ. Anti-CENP-H antibodies in patients with Sjgrens syndrome. Rheumatol Int 2006; 26: 298303. Ruiz-Tscar JL, Lpez-Longo FJ, Snchez-Ramn S et al. Prevalence of IgG anti-a-fodrin antibodies in Sjgrens syndrome. Ann NY Acad Sci 2005; 1050: 21016.

8 9

P R O B L E M

25 Raynauds Phenomenon
Case History
Rachel is aged 14 years and presents because she has poor circulation. Her mother describes Rachels ngers as going white, blue and then red when the weather is cold or when Rachel has been exercising. Her mother is concerned about this and wants to know what it may mean for Rachels future health. What is Raynauds phenomenon? How would you determine whether it is a marker for future disease? What treatment options exist for management?

Background
In 1862, a medical student Maurice Raynaud described the phenomenon that bears his name; it is characterized by transient cessation of blood ow to the digits of the hands or Atlas Medical Publishing Ltd

25 Raynauds phenomenon

135

Box 25.1 Triphasic colour changes of Raynauds phenomenon White: excessive vasoconstriction and cessation of regional blood ow Blue: cyanosis due to residual blood desaturation Red: hyperaemia as the attack subsides and blood ow is restored feet (Box 25.1). Occasionally other structures are involved such as the tip of the nose, an earlobe, nipple or even tongue. Raynauds phenomenon occurs in 3%5% of the population, although symptoms have been reported in 10% of women and 8% of men. The prevalence is higher in women, with onset generally between menarche and menopause, and the severity is greater over this period. Between 80% and 90% of Raynauds phenomenon is termed primary and has no identiable cause or associated disease. Secondary Raynauds phenomenon is most often found in association with an underlying rheumatic disease (Box 25.2). There is some evidence for a genetic predisposition, especially in those with early onset (age <40 years). Box 25.2 Primary and secondary Raynauds phenomenon Primary Raynauds phenomenon An uncomplicated and usually benign condition, often with a long-standing history, beginning in adolescence, with no or minimal tissue damage. Secondary Raynauds phenomenon Associated with underlying connective tissue disease (systemic sclerosis or mixed connective tissue disease), with potentially disabling ulceration or tissue necrosis. Onset is usually after age 3040 years. Less common causes are use of vibrating tools, paraproteinaemias, cryoglobulinaemia, large blood vessel disease and drugs (b-blockers, bleomycin, cisplatin, fentanyl, nicotine). The Practice Points (Box 25.3) identify a number of factors that can be evaluated in predicting the likelihood of the Raynauds presentation being the rst indicator of a more serious disease. Of the four points listed, the absence of distorted nailfold capillaries is the most predictive of a benign outcome. Even patients with isolated Raynauds with nailfold capillaroscopy abnormalities and/or antinuclear antibody have only a 10%15% likelihood of developing a connective tissue disease during long-term follow-up. Box 25.3 Practice Points Raynauds phenomenon as a precursor of an underlying connective tissue disease such as scleroderma or systemic lupus erythematosus is very unlikely with: b b b b Absence of tissue damage such as pulp scarring or atrophy Normal erythrocyte sedimentation rate Negative antinuclear antibody Absence of distortion of nailfold capillaries observed with an ophthalmoscope set at 40+ dioptre

136

04 Systemic Lupus Erythematosus, Sjgrens Syndrome and Scleroderma

Table 25.1 Pathogenetic factors


Increased vasoconstriction
Vascular factors Endothelin-1 Angiotensin II Thromboxane A2 Activation of a2-adrenoreceptors

Reduced vasodilation
NO and prostacyclin

Neural factors

Calcitonin gene-related peptide

Circulating factors Platelet-derived thromboxane and serotonin Oxidative stress damaging endothelium Flow factors
NO, nitric oxide.

Reduced brinolysis predisposing to brin deposition and vascular obstruction Increased viscosity and reduced red cell deformability

Pathogenesis
The association with female gender suggests a hormonal inuence. Increased vascular reactivity also occurs in the pre-ovulatory part of the menstrual cycle. A large epidemiological study found no association between Raynauds phenomenon and smoking, although severity may be worse in smokers. The pathogenesis of Raynauds phenomenon has been reviewed by Herrick1 and by Boin and Wigley.2 The overall model of aetiology is an imbalance of vasoconstriction over vasodilation. This results from changes in the vascular endothelium and smooth muscle, neural mediators of vascular tone and circulating platelet-derived mediators. These factors are in turn moderated by the level of physical activity, ambient temperature, emotional state and direct trauma or inammation of the vessels. Table 25.1 summarizes the multiple inuences that underlie the pathogenesis.

Non-pharmacological therapy
Education and introduction of conservative management are needed for all patients (Table 25.2). Avoidance of cold exposure is advocated, including frozen food sections of supermarkets. Patients should maintain or try and increase the temperature of the chest and abdomen to enhance peripheral vasodilation as a heat dissipation measure. Thermal biofeedback (TBF) is a psychophysiological technique in which subjects are trained to control peripheral vasoconstrictor responses and to acquire voluntary hand-warming skills. Initially taught with feedback such as peripheral temperature or blood ow, it is
Table 25.2 Treatment strategies
Non-pharmacological therapy
Avoid cold, emotional stress, smoking Biofeedback

Pharmacological therapy
Antiplatelet agents Vasodilation Calcium channel blockers Sildenal/tadalal Intravenous/oral prostanoids L-arginine supplementation Vasoconstriction Angiotensin II receptor antagonists ACE inhibitors Bosentan Serotonin reuptake inhibitors a1/a2-adrenergic blockers

ACE, angiotensin-converting enzyme.

25 Raynauds phenomenon

137

hypothesized that subsequent hand-warming can occur without the feedback instrument. A systematic review of eight randomized controlled trials (RCTs) highlighted major shortcomings in study design, particularly in the teaching of TBF and taking account of environmental factors.3 However, the authors concluded that TBF treatment was efcacious. Cervical spinal cord stimulation has been proposed for severe Raynauds phenomenon, but remains controversial and is not yet supported by good clinical trials. Lowlevel laser therapy, applied to the ngers and dorsum of the hand for 30 minutes in ve sessions over three weeks, signicantly decreased the frequency and severity of attacks in a sham-controlled double-blind trial. The mechanism of action remains unexplained.

Pharmacological treatment
The sympathetic nervous system mediates a tonic vasoconstrictive effect on the vascular wall, with noradrenaline-induced vasoconstriction mediated by a1 and a2 adrenoreceptors. The a2 receptors are more important in the regulation of digital vascular tone. Trials of nonselective sympathetic blockers have not been successful. Prazosin, an a1 receptor antagonist, showed a modest benet over placebo, but probably insufcient to balance side effects. An a2 receptor antagonist has been developed, and in an RCT reduced the frequency of Raynauds attacks in scleroderma-associated Raynauds, but not in primary Raynauds patients. Calcium channel blockers (CCBs), particularly of the dihydropyridine class, are widely prescribed for Raynauds phenomenon. A meta-analysis evaluated the effectiveness of CCBs in primary Raynauds and noted an average reduction of three to ve attacks per week and a one-third reduction in severity.4 Nifedipine is the most studied and has benets over placebo and nicardipine. Adverse effects of CCBs include ushing, headache, oedema and tachycardia. Prostacyclin is a potent, short-lived vasodilator that also has antiproliferative effects on smooth muscle and reduces platelet aggregation. Intravenous iloprost decreases the number and duration of vasospastic episodes, but its use usually requires central-line administration and is complicated by cost, hypotension and parotid and menstrual pain. Serotonin is a potent platelet-derived vasoconstrictor, and it is therefore counterintuitive to use selective serotonin reuptake inhibitors (SSRIs) as they would be expected to increase the plasma concentration of serotonin. Despite this, several case reports describe the use of SSRIs in Raynauds phenomenon, with both benecial and worsening outcomes. Fluoxetine decreases the frequency and severity of vasospastic episodes and increases the rate of rewarming after a cold challenge. Angiotensin II receptor antagonists and angiotensin-converting enzyme inhibitors have both shown effects on endothelial function and remodelling. On this basis, their use in Raynauds phenomenon is being explored. The frequent ndings of microthrombin in secondary Raynauds and evidence of platelet activation suggest a potential benet from antiplatelet therapy or anticoagulation. Two controlled trials that studied aspirin and dipyridamole in patients who had scleroderma failed to show a benet; however, many clinicians recommend low-dose aspirin unless there is a contraindication.

Recent Developments
1 Sildenal is a selective inhibitor of cyclic guanosine monophosphate (cGMP)-specic phosphodiesterase type V; its use leads to an increase in cGMP and thus it

138

04 Systemic Lupus Erythematosus, Sjgrens Syndrome and Scleroderma potentiates the effect of nitric oxide-mediated activation of guanylate cyclase. cGMP causes a decrease in intracellular calcium, resulting in vascular smooth muscle relaxation and dilatation. Lichtenstein was the rst to successfully use sildenal for Raynauds phenomenon in a group of ten patients.5 Fries et al.6 reported a crossover study of 16 patients with secondary Raynauds who had failed to respond to at least two conventional vasodilatory agents. Active treatment was sildenal 50 mg bd and the frequency and duration of symptoms of Raynauds phenomenon decreased signicantly, leading to improved well-being of the patients. In this study, capillary blood ow was severely impaired and sometimes hardly detectable, with sildenal leading to a >400% increase in ow velocity. Despite its short half-life (four hours), the data on capillary ow and symptom control suggest a longer functional effect of sildenal. Further renements in phosphodiesterase inhibition are on the horizon, with the longer-acting tadalal (half-life 17 hours) having been used in a patient with secondary Raynauds who was resistant to sildenal.7 2 Cilostazol (100 mg bd) is a phosphodiesterase type III inhibitor that in a crossover trial showed no change in symptoms and only a very modest increase in owmediated dilation of the brachial artery in patients with primary and sclerodermarelated Raynauds.8 3 Bosentan is a non-selective endothelin A/B antagonist. In a trial of scleroderma patients with digital ulcers, bosentan almost halved the development of new ulcers compared to placebo during the four-month trial.9 However, there was no benet on healing of existing ulcers, and self-assessed Raynauds severity was similar between active and placebo therapy. In a case report of severe Raynauds phenomenon, benecial effects of bosentan in healing of gangrenous digits were observed. A reduction of CD146 cells a population of mainly endothelial cells was noted, suggestive of reduced vascular injury. A concomitant increase in CD34 cells which are stem cells containing endothelial progenitors suggested increased vascular repair.10

Conclusion
Cold-induced vasospasm is common in the teenage years and for the majority of patients does not require medical review. For those patients presenting to a doctor, it is helpful to have a scheme to identify those at risk of developing a connective tissue disease. The majority of patients will full the criteria for primary Raynauds phenomenon and require no further investigation. Avoidance of cold and emotional triggers is important, as once vasospasm has occurred its resolution can be protracted and painful. A range of treatments is under investigation, although at this time the benet:risk ratio is marginal for most.

Further Reading
1 Herrick AL. Pathogenesis of Raynauds phenomenon. Rheumatology 2005; 44: 58796. 2 Boin F, Wigley FM. Understanding, assessing and treating Raynauds phenomenon. Curr Opin Rheumatol 2005; 17: 75260. 3 Karavidas MK, Tsai P-S, Yucha C, McGrady A, Lehrer PM. Thermal biofeedback for primary Raynauds phenomenon: a review of the literature. Appl Psychophysiol Biofeedback 2006; 31: 20316.

26 Assessing and treating scleroderma

139

4 Thompson AE, Pope JE. Calcium channel blockers for primary Raynauds phenomenon: a meta-analysis. Rheumatology 2005; 44: 14550. 5 Lichtenstein JR. Use of sildenal citrate in Raynauds phenomenon: comment on the article by Thomson et al. Arthritis Rheum 2003; 48: 2823. 6 Fries R, Shariat K, von Wilmowsky H, Bhm M. Sildenal in the treatment of Raynauds phenomenon resistant to vasodilatory therapy. Circulation 2005; 112: 29805. 7 Baumhaekel M, Schefer P, Boehm M. Use of tadalal in a patient with secondary Raynauds phenomenon not responding to sildenal. Microvasc Res 2005; 69: 1789. 8 Rajagopalan S, Pfenninger D, Somers E et al. Effects of cilostazol in patients with Raynauds syndrome. Am J Cardiol 2003; 92: 131015. 9 Korn JH, Mayes M, Matucci Cerinic M et al. Digital ulcers in systemic sclerosis: prevention by treatment with bosentan, an oral endothelin receptor antagonist. Arthritis Rheum 2004; 50: 398593. 10 Dunne J, Dutz J, Shojania K, Ng B, van Eeden S. Treatment of severe Raynauds phenomenon with bosentan in a patient with systemic sclerosis. Rheumatology 2006; 45: 91112.

P R O B L E M

26 Assessing and Treating Scleroderma


Case History
Mary is 42 years old and presents with increasing stiffness of her hands. Since her midthirties she had noted painful blanching of her ngers in cold water or when handling cold objects. Last winter she had ulcers on her ngertips and she had indigestion most of this year. She has bilateral sclerodactyly. Her blood pressure (BP) is 145/93 mmHg and she is easily fatigued. What clinical and laboratory features will assist you in the diagnosis? What treatments should be used for scleroderma?

Atlas Medical Publishing Ltd

140

04 Systemic Lupus Erythematosus, Sjgrens Syndrome and Scleroderma

Background
Scleroderma is a rare condition with a prevalence of approximately 1 in 5000, and an annual incidence of 1 in 50 000 to 1 in 300 000. Strictly, scleroderma denotes disease limited to the skin and subcutaneous tissues (localized scleroderma, or morphea), and disease involving the internal organs is systemic sclerosis (SSc). Morphea is characterized by brosis of the skin and adjacent structures. Morphea can be differentiated from SSc by the distribution of lesions (no sclerodactyly or perioral involvement), absence of Raynauds phenomenon and/or periungual telangiectasia.1 The generalized form of disease can be divided into limited cutaneous SSc/scleroderma and diffuse cutaneous SSc/scleroderma; limited cutaneous disease does not extend beyond the elbow and knee, although the face may be involved. In diffuse cutaneous disease, the skin lesions involve the more proximal limb and trunk. Diffuse scleroderma is more common in African-Americans, black patients in Africa and the Choctaw Indian. Genome-wide scanning has identied susceptibility loci on three different non-HLA chromosomes: brillin on chromosome 15, SPARC (secreted protein acidic and rich in cysteine) on chromosome 5 and topoisomerase 1 on chromosome 20. The characteristics of diffuse and limited cutaneous scleroderma are presented in Table 26.1. CREST is often but not always used as a synonym for limited cutaneous scleroderma, and dates from the early 1960s. CREST patients have three or more of the ve syndromic features: b b b b b Calcinosis Raynauds phenomenon oEsophageal dysmotility Sclerodactyly Telangiectasia

CREST can be considered with limited cutaneous SSc. Wollheim proposed that the ve CREST components were not meant to serve as criteria of a special subset of scleroderma, and features of CREST occur with time irrespective of other disease characteristics.2
Table 26.1 Characteristics of diffuse and limited scleroderma
Diffuse cutaneous scleroderma Limited cutaneous scleroderma

History of Raynauds phenomenon with onset within one year Skin involvement restricted to hands, face, forearm, feet Skin sclerosis proximal to the elbow, and may involve trunk Prominence of calcinosis and telangiectasia Tendon friction rubs may occur Nailfold capillary dilation, aneurysm and dropout Early onset of pulmonary, renal and diffuse gastrointestinal involvement Often anti-topoisomerase 1 antibodies (anti-Scl-70); rarely anti-centromere antibodies Anti-RNA polymerase III Ten-year survival: 54%62% Nailfold capillary dilation, aneurysm and dropout Delayed but often severe pulmonary hypertension Anti-centromere antibodies common especially with calcinosis and telangiectasia Ten-year survival: 69%89%

26 Assessing and treating scleroderma

141

Ninety-ve per cent of patients with scleroderma have a positive antinuclear antibody (ANA) test, but specic patterns are not reliably diagnostic. Both anti-centromere and anti-topoisomerase 1 (anti-Scl-70) have a low sensitivity, and a diagnosis of scleroderma is not excluded by a negative result. Anti-centromere antibodies favour a diagnosis of limited cutaneous scleroderma. Anti-topoisomerase 1 antibodies are only 30% sensitive, but are highly specic for SSc and correlate with interstitial lung disease. The skin capillaries can be visualized at the nailfold using an ophthalmoscope or a dermoscope, or in the research situation using video capture. A range of abnormalities have been described including dilatation and aneurysm formation, dropout, haemorrhage and budding. Three-quarters of scleroderma patients have abnormalities of their nailfold capillaries, and the presence of normal capillaries is the strongest negative predictor of scleroderma in patients with Raynauds phenomenon. There appears to be no nailfold capillary difference between limited cutaneous and diffuse cutaneous scleroderma. The initial complaint of limited cutaneous scleroderma is Raynauds phenomenon, whereas patients with diffuse cutaneous scleroderma often present with generalized hand swelling, skin thickening or arthralgias with or without Raynauds phenomenon. They may initially complain of tight, puffy ngers that occurs in the morning but then lasts all day. This is the oedematous stage, with oedema usually non-pitting and painless. This usually progresses to a thickened, tight indurative stage over months to years, with the skin becoming shiny and adherent to the underlying subcutis. Dermal thickening and epidermal thinning lead to loss of dermal appendages, with hair loss and absent sweating. Mottled hyper- and hypopigmentation may be noted. Facial involvement may lead to a pinched nose, tightened lips and a mouse-like appearance. Finally, there is an atrophic stage in which the dermis softens and thins but remains rmly bound down to the subcutaneous fat. With limited cutaneous scleroderma, clusters of dilated tortuous capillaries and venules are noted. Joint pain, immobility and contractures are the result of brosis around tendons and other periarticular structures. Contractures of the hand are most common, but large joint contractures may occur. Some patients have palpable and/or audible deep tendon friction rubs due to brinous tenosynovitis. Skin ulcers are of two forms: digital tip ulcers secondary to ischaemia, and ulceration over bony prominences where the skin is contracted and tight and susceptible to trauma. Reduced vascularity perpetuates poor healing. Both limited and late diffuse cutaneous scleroderma may develop calcinosis at areas of pressure such as the nger pads, olecranon bursa, extensor surface of forearm, buttocks and around the patella. The gastrointestinal (GI) tract is the second most commonly affected organ, with involvement in 75%90% of cases. The hallmark effect is smooth muscle brosis, leading to dysmotility, with 80% of patients developing oesophageal dysfunction. Impaired gastric emptying and decreased lower oesophageal sphincter tone allow reux with heartburn and dysphagia. In the longer term, Barretts oesophagus and strictures may develop, and the risk of adenocarcinoma of the oesophagus is increased. Dysmotility occurs throughout the GI tract and can cause gastroparesis, small bowel bacterial overgrowth and malabsorption, pseudo-obstruction, severe constipation and megacolon. Pulmonary involvement occurs in more than 70% of cases and is the most common cause of SSc-related death. Pulmonary brosis is common and can cause severe restrictive lung disease. Chest X-ray shows lower lobe interstitial thickening in a reticular nodular pattern. High-resolution computed tomography (HRCT) may show ground-glass

142

04 Systemic Lupus Erythematosus, Sjgrens Syndrome and Scleroderma appearance, representing alveolitis, or brotic honeycombing with traction bronchiectasis. Pulmonary function testing can serially document changes in lung volumes with restriction and increasing impairment of gas transfer. Readily detected by right heart catheterization and echocardiograph, pulmonary artery hypertension (PAH) occurs in 20%40% of patients with SSc. The ve-year cumulative survival is 10% in those with PAH compared with 80% in those without. PAH can develop early, particularly in limited cutaneous SSc, and can occur without pulmonary brosis. In diffuse cutaneous SSc, development of pulmonary brosis may lead to secondary PAH. The patient develops dyspnoea disproportionate to any lung ndings, and may have accentuation of the pulmonary component of the second heart sound. A clue to PAH is disproportionate reduction in the carbon monoxide diffusion capacity relative to the forced vital capacity (FVC). Renal disease occurs almost only in those with diffuse cutaneous disease. Scleroderma renal crisis, which could result in rapid death or dialysis, is essentially a thing of the past due to the use of angiotensin-converting enzyme (ACE) inhibitors. The crisis occurs most often in patients with early diffuse cutaneous SSc in the context of rapid skin progression. It is characterized by accelerated hypertension presenting as headache, shortness of breath, seizures, retinal haemorrhage, left heart failure and renal failure.

Prognosis
Italian and French-Canadian studies report increasing mortality with increasing skin involvement using a diffuse, intermediate and limited classication. A ve-year study from the United Kingdom (UK) identied advanced age, diffuse cutaneous disease, higher skin score, elevated erythrocyte sedimentation rate (ESR) and signs of organ involvement as poor prognostic factors in univariate analysis. However, logistic regression analysis identied proteinuria, elevated ESR and reduced carbon monoxide diffusion capacity as the most important factors.3,4 PAH has a devastating impact on survival and outcome, because the progressive elevation of pulmonary artery pressure leads to right ventricular failure, arrhythmias and death. The mean survival of untreated patients with primary PAH is 2.8 years from diagnosis. In untreated and treated patients with SScassociated PAH, mean survival is less than one year in those with severe disease. Several manifestations are associated with specic antibodies: interstitial lung disease with anti-topoisomerase 1 (Scl-70), and anti-centromere antibodies with digital necrosis and delayed pulmonary hypertension independent of pulmonary brosis. Anti-RNA polymerase III antibodies in limited cutaneous scleroderma correlate with severe skin and renal involvement. U3-RNP antibodies, including brillarin, occur in both limited and diffuse disease and correlate with pulmonary hypertension, cardiac involvement and myositis. In a multicentre study of 1012 Italian patients, ten-year actuarial survival was 72% in patients with and 81% in those without anti-centromere antibody, and 73% in patients with ANA.5 Overall, ANA is of limited help in predicting prognosis.

Pathophysiology and treatment


Three abnormalities have been identied: 1 Fibroblast dysfunction leading to increased production and deposition of extracellular matrix, particularly collagen

26 Assessing and treating scleroderma

143

2 Vascular abnormality leading to tissue hypoxia 3 An immune response manifest as altered T- and B-lymphocyte function and production of autoantibodies Endothelial damage and vascular dysfunction occur early, with activated lymphocytes secreting transforming growth factor-b (TGF-b) causing endothelial injury and upregulation of the major histocompatibility complex, adhesion molecules and connective tissue growth factor (CTGF). TGF-b maintains broblasts in an activated state, promotes collagen and matrix protein production and decreases synthesis of collagen-degrading matrix metalloproteinases. CTGF is an important growth factor in brosis, triggering angiogenesis and the structural organization of connective tissue. Endothelial damage results in the increased production of endothelin (ET)-1 and impaired prostacyclin release, resulting in imbalance between vasodilators and vasoconstrictors. ET combined with platelet-derived factors changes the vascular smooth muscle cell into a myobroblast, with proliferation of these narrowing the vessel lumen. Later in the disease process the inammatory vasculopathy subsides and brosis dominates, with skin thickening, interstitial lung brosis, and GI and renal disease. Skin: The natural history of skin involvement is quite variable, with striking improvement occurring without intervention. Cyclophosphamide, mycophenolate mofetil and methotrexate are all used, but only methotrexate has shown a statistical benet in controlled trials. Kidney: Renal crises are treated with ACE inhibitors titrated to achieve a systolic BP reduction of 1020 mmHg over 24 hours. Continuous low-dose prostacyclin improves BP control and renal blood ow. Creatinine clearance is monitored and ACE inhibitors maintained even during dialysis. There is a prolonged period of potential renal recovery. Refractory hypertension may require calcium channel blockers or a-adrenergic blockers. Pulmonary brosis: The greatest evidence of efcacy is available for cyclophosphamide. Recent trials include: 1 A UK study of 45 SSc patients who received low-dose prednisolone and six infusions of cyclophosphamide monthly, followed by oral azathioprine or placebo. At one year this did not demonstrate improvement in the primary endpoints (FVC and diffusing capacity for carbon monoxide [DLCO]) or secondary endpoints (HRCT appearance and dyspnoea scores).6 2 A United States study of 158 SSc patients with interstitial lung disease who received oral cyclophosphamide (2 mg/kg/day) or placebo for one year and were followed for one additional year.7 The mean difference in FVC at 12 months was 2.5% favouring cyclophosphamide, which was maintained at 24 months. The clinical benet was modest, aiding skin scores, dyspnoea and health-related quality of life. Pulmonary hypertension: Early diagnosis is essential to optimize outcome. Mortality and morbidity of PAH has been improved by the use of long-term oxygen therapy, diuretics and digoxin for symptom relief, and anticoagulation. Intravenous prostacyclin leads to a rapid improvement in cardiac output, exercise capacity and survival but is expensive and associated with adverse effects. Gastrointestinal: Oesophageal dysmotility occurs in the majority and requires regular use of proton pump inhibitors, sometimes with prokinetic agents. Monitoring for

144

04 Systemic Lupus Erythematosus, Sjgrens Syndrome and Scleroderma Barretts oesophagus is indicated. Broad-spectrum antibiotic and prokinetic agents may aid midgut disease. Stool frequency can be improved with frequent, small, low-bre meals, with alternating aperients and antidiarrhoeals.

Recent Developments
1 ET is a key mediator in the pathology of PAH and also SSc, participating in brotic, hypertrophic and inammatory processes. The oral dual ET-A/ET-B receptor antagonist bosentan has proven benet in patients with severe PAH. In a six-month open-label prospective study, eight patients with PAH and pulmonary brosis related to SSc received bosentan 62.5 mg bd for four weeks followed by maintenance dosing of 125 mg daily.8 Mean six-minute walking distance increased from 72 m at baseline to 192 m after three months, and to 203 m after six months. Six of eight patients responded with improvement in functional class. 2 The two pivotal trials of bosentan included 52 patients with SSc and other connective tissue diseases. During a four-month trial, mean treatment effect on six-minute walking distance was 37 m. In the long-term extension, survival was 82% after one year and 67% after two years.9 Bosentan has also been used in connective tissue disease patients with PAH who were non-responsive to prostanoid therapy.10 Thirteen patients (primarily with SSc) were treated for one year. A progressive improvement in exercise capacity was noted, with half improving their six-minute walking distance. 3 Scleroderma/SSc and chronic graft-versus-host disease (GVHD) resemble each other in tissue distribution (skin, lung, oesophagus), lymphocytic infiltration in affected tissues and tissue fibrosis. It has also been suggested that SSc may be a form of GVHD.11 GVHD occurs when foreign immunocompetent cells in grafted or transfused tissue react with the recipients cell-surface antigens. To induce GVHD, the graft must contain immunocompetent cells, the host must appear foreign to these cells and the host must be incapable of mounting an effective response and destroying them. The hypothesis of microchimaerism underlying SSc was postulated in 1989. As an aetiological factor, microchimaeric cells could be from previous pregnancies, or from maternal origin in the case of nulliparous women or male patients. The mere presence of these cells in peripheral blood is therefore not sufficient to cause SSc and a second, unknown activation step is postulated. Microchimaeric cells are absent from the skin of normal subjects, and are present in higher numbers in the normal skin of SSc subjects compared with involved skin.12 4 Statins have more widespread effects than just lowering cholesterol (Figure 26.1). There appear to be substantial differences amongst the statins as to their impact on these alternate pathways.13 The potential of this approach has been trialled in an open-label study of atorvastatin 10 mg/day for twelve weeks.14 Raynauds phenomenon improved, indicated by reduced subjective scoring. A two- to eightfold increase in circulating endothelial precursors was noted, although not to normal levels. A reduction in the angiogenic factors vascular endothelial growth factor and basic broblast growth factor was observed.

Statins

Block HMG-CoA reductase CEPs Leukocyteendothelial adhesion TGF-

Collagen gene expression

Mevalonate

Endothelial repair and neovascularization

Inflammatory chemokines and cytokines

Fibrosis

LDL Farnesyl PP Geranylgeranyl PP

26 Assessing and treating scleroderma

VEGF

Endothelial apoptosis

Altered gene expression

Intimal proliferation eNOS expression ROS

145

Figure 26.1 Benecial effects of statins in scleroderma. CEP, circulating endothelial precursors; eNOS, endothelial nitric oxide synthase; HMG-CoA, 3-hydroxy-3methylglutaryl coenzyme A; LDL, low-density lipoprotein; PP, pyrophosphate; ROS, reactive oxygen species; TGF-b, transforming growth factor-b; VEGF, vascular endothelial growth factor.

146

04 Systemic Lupus Erythematosus, Sjgrens Syndrome and Scleroderma

Conclusion
Scleroderma is a rare disease of unknown cause, which is diagnosed essentially on clinical grounds. To date, most interventions have been of little clinical utility, except proton pump inhibitors for reux disease and ACE inhibitors for renal crisis. The challenges for the future are in the early diagnosis of and effective intervention for pulmonary brosis and pulmonary hypertension. Use of ET receptor antagonists has shown it is possible to intervene with an oral medication, but the gains in either morbidity or mortality are modest. A holistic approach is required, which includes a broad treatment strategy aimed at reducing vascular complications of this disorder.

Further Reading
1 Chung L, Lin J, Furst DE, Fiorentino D. Systemic and localized scleroderma. Clin Dermatol 2006; 24: 37492. 2 Wollheim FA. Classication of systemic sclerosis. Visions and reality. Rheumatology 2005; 44: 121216. 3 Bryan C, Knight C, Black CM, Silman AJ. Prediction of ve-year survival following presentation with scleroderma: development of a simple model using three disease factors at rst visit. Arthritis Rheum 1999; 42: 26605. 4 Meyer O. Prognostic markers for systemic sclerosis. Joint Bone Spine 2006; 73: 4904. 5 Ferri C, Valentini G, Cozzi F et al. Systemic sclerosis: demographic, clinical, and serologic features and survival in 1,012 Italian patients. Medicine 2002; 81: 13953. 6 Hoyles RK, Ellis RW, Wellsbury J et al. A multicentre, prospective, randomized, double-blind, placebo-controlled trial of corticosteroids and intravenous cyclophosphamide followed by oral azathioprine for the treatment of pulmonary brosis in scleroderma. Arthritis Rheum 2006; 54: 396270. 7 Tashkin DP, Elashoff R, Clements PJ et al. Cyclophosphamide versus placebo in scleroderma lung disease. N Engl J Med 2006; 354: 265566. 8 Ahmadi-Simab K, Hellmich B, Gross WL. Bosentan for severe pulmonary arterial hypertension related to systemic sclerosis with interstitial lung disease. Eur J Clin Invest 2006; 36 (Suppl 3): 448. 9 Denton C, Humbert M, Rubin L, Coghlan J, Black C. Dual endothelin receptor antagonism in pulmonary arterial hypertension related to systemic sclerosis. Eur J Clin Invest 2005; 35 : I72. 10 Cozzi F, Montisci R, Marotta H et al. Bosentan therapy of pulmonary arterial hypertension in connective tissue disease. Eur J Clin Invest 2006; 36 (Suppl 3): 4953. 11 Jimenez SA, Artlett CM. Mirochimerism and systemic sclerosis. Curr Opin Rheumatol 2005; 17: 8690. 12 Sawaya HHB, Jimenez SA, Artlett CM. Quantication of fetal microchimeric cells in clinically affected and unaffected skin of patients with systemic sclerosis. Rheumatology 2004; 43: 9658. 13 Kuwana M. Potential benets of statins for vascular disease in systemic sclerosis. Curr Opin Rheumatol 2006; 18: 594600.

27 Immunosuppressive drugs

147

14 Kuwana M, Kaburaki J, Okazaki Y, Yasuoka H, Kawakami Y, Ikeda Y. Increases in circulating endothelial precursors by atorvastatin in patients with systemic sclerosis. Arthritis Rheum 2006; 54: 194651.

P R O B L E M

27 Immunosuppressive Drugs

Case History
Mrs TJ is 55 years old and developed sicca syndrome after the menopause at age 51 years. Her serum is positive for antinuclear antibody (ANA), anti-Ro and anti-La. She has symptoms of general malaise, arthralgia and myalgia, and a recent chest X-ray suggests that she is developing pulmonary inltrates. Inammatory markers (erythrocyte sedimentation rate and serum C-reactive protein) are elevated. She has been taking prednisolone for the past six months but this has only partly relieved her symptoms. What factors inuence the choice of immunosuppressive agent? How should she be monitored? Is immunosuppression becoming safer and more effective?

Background
Immunosuppressive treatment is a large part of rheumatological practice. It is the cornerstone for many of the immune-mediated connective tissue disorders including rheumatoid arthritis (RA), systemic lupus erythematosus (SLE), systemic sclerosis, vasculitides and Sjgrens syndrome. Because of the widespread use of immunosuppressive agents, the generalist and the general practitioner should understand current usage of these drugs and how they are monitored.

Glucocorticoids
These bind to glucocorticoid response elements in the genome, thus regulating activity of a broad range of genes. They downregulate expression of pro-inammatory cytokines including interleukin (IL)-1 and IL-6. Decreased IL-2 production from T cells is associated with decreased lymphocyte proliferation. Cytotoxic and phagocytic cells are inhibited. Increased activity of the inhibitor of k-beta (Ikb) decreases activity of the key Atlas Medical Publishing Ltd

148

04 Systemic Lupus Erythematosus, Sjgrens Syndrome and Scleroderma regulatory pathway involving nuclear factor-k-beta (NF-kb) with consequent downregulation of components of the immune response and increased lymphocyte apoptosis. Between 1 in 400 and 1 in 200 of the population (higher in the elderly) is taking chronic steroid medication, usually prednisolone. One of the major indications for use of other immunosuppressive drugs is to minimize exposure to steroids and to protect the patient from long-term side effects. The latter include hypertension, hyperglycaemia, osteopenia, cataracts, gastric ulceration, impaired wound healing and susceptibility to infection. Topical agents (inhaled, rectal or skin) should be used where possible.

Antiproliferative and antimetabolic drugs


Azathioprine (Imuran) is a purine derivative that has been used as an antimetabolite since the early 1960s. It is used in transplant patients and in treatment of RA. Side effects include bone marrow suppression, susceptibility to infections (especially varicella and herpes simplex viruses), hepatotoxicity, alopecia and increased risk of malignancies. Methotrexate is used for psoriasis and RA as well as in treatment of malignancies. It inhibits dihydrofolate reductase, therefore decreasing purine synthesis and progression through the G1 and S phases of the cell cycle. This may explain its cytotoxic effects, with its anti-inammatory effects related to increasing adenosine levels. Side effects include bone marrow suppression, interstitial pneumonitis and increased liver enzymes with occasionally more severe hepatic impairment. A full blood count (FBC) and liver function tests (LFTs) should be monitored monthly and adequate contraception used. Cyclophosphamide is used particularly in SLE and in vasculitic conditions including Wegeners granulomatosis. Side effects include leukopenia, haemorrhagic cystitis and increased risk of transitional cell carcinoma, infertility, alopecia and increased risk of lymphoma. Leunomide is used in active RA, particularly in those unresponsive to methotrexate, for whom leunomide can be used alone or in combination with methotrexate. Side effects are diarrhoea, alopecia, severe skin reactions, pneumonitis, hypertension and neurological toxicity. FBC and LFTs should be monitored monthly for the rst six months and then two-monthly. Blood pressure should be monitored. Adequate contraception is essential, with leunomide having a long practical half-life due to enterohepatic circulation. Mycophenolate mofetil is hydrolysed to mycophenolic acid in the body, and inhibits T- and B-cell proliferation through its inhibitory effect on the enzyme inosine monophosphate dehydrogenase (IMPDH). It may cause bone marrow suppression, pneumonitis and severe diarrhoea. Sirolimus (Rapamune) binds to the 12 kDa FK506-binding protein (FKBP-12) and inhibits the protein kinase mammalian target of rapamycin (MTOR), a key protein in cellular proliferation. The drug is mainly used in transplant patients and in drug-eluting stents for patients with coronary artery disease.

Calcineurin inhibitors
Calcineurin is a Ca2+/calmodulin-dependent protein phosphatase involved in T-cell differentiation and proliferation. The enzyme catalyses dephosphorylation of the nuclear factor of activated T lymphocytes (NFAT), which is subsequently translocated to the nucleus and increases expression of pro-inammatory cytokines including IL-2.

27 Immunosuppressive drugs

149

Cyclosporin (by interacting with cyclophilin) and tacrolimus (by interacting with FKBP12) inhibit the phosphatase activity of calcineurin. Cyclosporin is a cyclic peptide of eleven amino acids, which is secreted by the fungus Beauveria nivea. Its predominant effect is inhibiting T-cell responses. It can be given intravenously (Sandimmune) or orally (Neoral). Its main uses are in transplant, RA and psoriasis. For transplant patients, it is usually combined with steroids and either azathioprine or mycophenolate mofetil. In RA, it is usually reserved for patients who do not respond to rst- or second-line disease-modifying antirheumatic drugs (DMARDs) including methotrexate. With the introduction of anti-tumour necrosis factor (antiTNF) agents, cyclosporin is being used less. Nephrotoxicity is the most common severe side effect. The drug may also worsen hypertension, hyperglycaemia and hyperuricaemia. Side effects also include tremor, hirsutism and gum hyperplasia. Monitoring to keep the plasma concentration of the drug in the range 50100 ng/ml should be undertaken. Tacrolimus (Prograf) is a macrolide antibiotic secreted by Streptomyces tsukubaensis. It can be given orally or systemically, and is mainly used for transplant patients. Nephrotoxicity and neurotoxicity are relatively common with its use.

Antibody treatments
Lymphocyte immune globulin and antithymocyte globulin have been used in immunosuppression since the 1960s. To avoid the potential problems of using polyclonal sera, monoclonal or monospecic antibody treatments have been developed. These include: Anti-CD3 Anti-CD25 Anti-CD52 Anti-TNF-a Directed at the e chain of the T-cell receptor Daclizumab and basiliximab, directed at the IL-2 receptor Alemtuzumab Iniximab, etanercept, used in refractory RA Efalizumab

Anti-lymphocyte function-associated antigen-1 (anti-LFA-1) Anti-CD20 Rituximab, B-cell depleting antibody

Other drugs
Hydroxychloroquine (Plaquenil, 200400 mg/day) is an antimalarial drug used in RA and SLE. It may be particularly useful for skin lesions in the latter. It is usually well tolerated but can cause macular damage. Patients should have an ophthalmological assessment at baseline and six-monthly. Sulphasalazine is used in RA and in seronegative spondyloarthropathies. It may cause gastrointestinal symptoms, headache, rash (including photosensitivity) and discolouration of soft contact lenses. Because of potential bone marrow toxicity, a FBC should be measured every two weeks for the rst three months, then three-monthly. LFTs should be monitored monthly for the rst three months then six-monthly. Use of immunosuppressive drugs has transformed management of many nonmalignant diseases. They should only be used where they are clearly indicated (Box 27.1) and with an appropriate programme of drug monitoring. An algorithm for monitoring of the most commonly prescribed drugs is presented in Figure 27.1. Increasingly with

150

04 Systemic Lupus Erythematosus, Sjgrens Syndrome and Scleroderma newer drugs including cyclosporine, mycophenolate mofetil, sirolimus and tacrolimus therapeutic monitoring of drug levels is being undertaken.1 Box 27.1 Use of immunosuppressive drugs b b b b b b b Underlying diagnosis well established Less toxic alternatives have failed The disease poses a severe threat to health Patient is informed of the risks and benets Infection and malignancy excluded No possibility that the patient will become pregnant Arrangements for monitoring and treating complications in place

Recent Developments
1 The eld of pharmacogenomics is developing rapidly. Expression of levels of key enzymes varies considerably between individuals, and this may explain why some patients fail to respond to drugs while others are very sensitive and more susceptible to side effects. Levels of thiopurine methyltransferase may be useful in predicting response to azathioprine.2 Those with high levels of the enzyme are at risk of being underdosed, and patients with low enzyme levels may be more susceptible to side effects including bone marrow suppression. In a similar fashion, variations in the expression of methylene tetrahydrofolate reductase may be important determinants of the response to methotrexate.3 2 It is not clear whether elderly onset RA behaves differently from that which has an earlier onset. In the recent study by Tutuncu et al.,4 elderly patients with RA had comparable duration and comparable indices of severity compared with younger patients. They were, however, less likely to receive as aggressive treatment; this may be partly due to under-recognition of the severity of the disease particularly if there are comorbidities or to concern about potential side effects. 3 Much of the stimulus for development and clinical trials with newer agents has come from the transplant eld. Examples of the way practice is changing are the increasing use of mycophenolate mofetil for lupus nephritis5 (where previously cyclophosphamide was the drug of choice) and the increasing use of cyclosporin for autoimmune diseases.6 Renal toxicity may limit the use of this drug by rheumatologists but it certainly may prove very useful in life-threatening situations where other drugs have failed. Amongst biological therapies, the TNF-a antagonists have found the widest use in RA. The maximum effect of biological therapies in RA is likely to be seen when they are combined with cytotoxic treatments such as methotrexate.7

Conclusion
The range of immunosuppressive drugs is increasing, as is trial evidence relating to their use in individual diseases. The choice of agents depends on the clinical picture

27 Immunosuppressive drugs

151

Baseline FBC, LFTs, U/E and CXR

FBC

LFTs

U/E

Weekly C*, A, M

2-weekly C, A, M

Weekly C, A, M

Monthly M

Monthly C*, A, M

Monthly C*, A, M

3-monthly A

3-monthly A, M

2-monthly A, M

? Stop WBC 1.5 109/l Pl 100 109/l

? Stop Transaminases 3 upper limit of normal

? Stop Creatinine >30% above baseline

Figure 27.1 Therapeutic monitoring of common immunosuppressives. Low white blood cell (WBC) count, increased liver enzymes or increasing creatinine may necessitate temporarily or permanently stopping the treatment a decision best made by the specialist responsible for starting it. * monitor at this frequency for the duration of treatment; A, azathioprine; C, cyclophosphamide; CXR, chest X-ray; FBC, full blood count; LFT, liver function text; M, methotrexate; Pl, platelet; U/E, urea and electrolytes.

and diagnosis, along with an assessment of the likely benefits and potential risks. Corticosteroids are often used in the first instance but the tendency now is to use lower doses, and thus they are often combined with other drugs. Methotrexate, azathioprine and cyclophosphamide have been used for many years but still have a major place in management. Monitoring of FBC, hepatic function and renal function is the most important aspect generally. Use of newer drugs and biological agents is increasing, although this is still limited by cost and clinical experience. With appropriate monitoring and choice of regimens, the treatment of autoimmune connective tissue disease with agents that suppress the immune system is becoming safer and more effective.

Further Reading
1 Holt DW, Armstrong VW, Griesmacher A, Morris RG, Napoli KL, Shaw LM. International Federation of Clinical Chemistry/International Association of Therapeutic Drug Monitoring

152

04 Systemic Lupus Erythematosus, Sjgrens Syndrome and Scleroderma


and Clinical Toxicology working group on immunosuppressive drug monitoring. Ther Drug Monit 2002; 24: 5967. 2 3 Patel AA, Swerlick RA, McCall CO. Azathioprine in dermatology: the past, the present, and the future. J Am Acad Dermatol 2006; 55: 36989. Hughes LB, Beasley TM, Patel H et al. Racial or ethnic differences in allele frequencies of single-nucleotide polymorphisms in the methylenetetrahydrofolate reductase gene and their inuence on response to methotrexate in rheumatoid arthritis. Ann Rheum Dis 2006; 65: 121318. Tutuncu Z, Reed G, Kremer J, Kavanaugh A. Do patients with older-onset rheumatoid arthritis receive less aggressive treatment? Ann Rheum Dis 2006; 65: 12269. Dooley MA. Mycophenolate mofetil: what role in the treatment of lupus? Lupus 2006; 15: 17982. Ponticelli C. Cyclosporine: from renal transplantation to autoimmune diseases. Ann N Y Acad Sci 2005; 1051: 5518. Heiberg MS, Rdevand E, Mikkelsen K et al. Adalimumab and methotrexate is more effective than adalimumab alone in patients with established rheumatoid arthritis: results from a 6-month longitudinal, observational, multicentre study. Ann Rheum Dis 2006; 65: 137983.

4 5 6 7

S E C T I O N

F I V E

05

Vasculitic Syndromes
28 29 30 Vasculitic disease Giant cell arteritis and polymyalgia rheumatica Behets syndrome

P R O B L E M

28 Vasculitic Disease

Case History
Mrs SR attends your clinic with a rash over her legs. She is aged 32 years and for the last six months has been unwell, with intermittent fevers, loss of appetite and fatigue. Recent blood tests show elevated erythrocyte sedimentation rate (ESR; 83 mm/h) and C-reactive protein (CRP; 46 mg/dl). Today she has palpable purpura on her lower legs. Urinalysis is positive for blood and protein. What are the clinical clues to vasculitis? What investigations will assist with a precise diagnosis? How should the condition be treated and monitored?

Background
The vasculitides are an important group of disorders in which an immune reaction affects the walls of the blood vessels.1,2 The immune dysfunction may be triggered by drugs or by an underlying autoimmune disease, but often an underlying cause is not identied. Manifestations vary between diseases and from person to person. The following are common: general malaise, arthralgia, normocytic anaemia, skin lesions (palpable purpura or necrotic ulcers), renal dysfunction and neurological dysfunction (particularly wrist drop and ankle drop). Differential diagnosis includes drug reactions (antibiotics, amphetamines, ergot derivatives), infections (human immunodeciency virus [HIV], Atlas Medical Publishing Ltd

154

05 Vasculitic Syndromes subacute bacterial endocarditis), malignant disease and antiphospholipid syndrome. Classication of vasculitides, shown in Box 28.1, is based on the size of blood vessels affected. Individually, the diagnoses, with the exception of giant cell arteritis (GCA), are relatively rare. Collectively (excepting GCA), there is an incidence of 2040 per million of the population per year. There is considerable geographical variation and also a seasonal variation in incidence. The latter suggests an infectious trigger and exposure to Mycoplasma pneumoniae has been linked with risk of GCA. The latter is more common in Caucasians than all the other diagnoses put together (around 170 per million of the population per year). Cutaneous vasculitis may occur in isolation or as part of an underlying systemic disorder.3 In each case of suspected vasculitis, a careful search for multiorgan involvement should be undertaken.

Small vessel vasculitis


The vessels involved are the venules, capillaries and arterioles of the skin, kidney and gastrointestinal (GI) tract. The hallmark lesion is palpable purpura, which is a slightly raised, non-blanching lesion that typically affects the lower legs. Gut involvement causes colicky abdominal pain and may lead to melaena. HenochSchnlein purpura is common in children after viral or streptococcal infection. The blood vessels are affected by immunoglobulin A (IgA)-containing immune complex deposition. Apart from the purpura, abdominal pain and arthralgia, renal involvement develops in up to 40%, with progression to chronic nephritis in 10% of cases. The condition is generally self-limiting and immunosuppressive treatment is not usually required. Cryoglobulins are antigenantibody complexes that precipitate in the cold. In addition to the other common manifestations of small vessel vasculitis, they may be associated with serious liver and pulmonary disease. Cryoglobulinaemia occurs in one-third of patients with hepatitis C infection. Cryoglobulins should be sought in all patients with a clinical picture of small vessel vasculitis, and all positive patients should be tested for hepatitis C.

Small to medium vessel vasculitis


These diseases generally affect the skin, upper respiratory tract, lungs and kidneys. Wegeners granulomatosis often begins with apparently innocent symptoms including Box 28.1 Classication of vasculitis Small vessels HenochSchnlein purpura b Mixed cryoglobulinaemia
b

Medium vessels Polyarteritis nodosa b Kawasaki syndrome


b

Small to medium vessels b Wegeners granulomatosis b ChurgStrauss syndrome b Microscopic polyangiitis

Large vessels b Giant cell arteritis b Takayasus arteritis

28 Vasculitic disease

155

sinusitis, nasal ulcers, otitis media and hearing loss (70% of cases). Pulmonary inltrates or nodules develop in 85% of cases, and 80% of patients have renal involvement with a focal necrotizing glomerulonephritis that frequently progresses to renal failure. Histologically, there is necrotizing vasculitis of medium-sized vessels with extravascular granulomata. Cytoplasmic anti-neutrophil cytoplasmic antibody (c-ANCA) is present in 80% of patients and perinuclear ANCA (p-ANCA) in 20%, with at least 90% of patients overall being ANCA positive. The disease is usually treated with prednisolone and cyclophosphamide. Microscopic polyangiitis is a variant of polyarteritis where patients present with pulmonary haemorrhage and glomerulonephritis. ChurgStrauss syndrome presents with allergic rhinitis and asthma. Eosinophilia, and inltrates of the lungs and GI tract account for the features of the disease. Up to 75% of patients have palpable purpura or skin nodules. It affects the kidneys in up to 50% of cases. Immunosuppressives are frequently required.

Medium vessel vasculitis


Patients with polyarteritis nodosa typically present with fever, myalgia and arthralgia. Renal disease develops in up to 60% of cases, with cardiac and GI involvement in 40% (may lead to bowel infarction), and central or peripheral nervous system involvement in 20%. Angiography reveals multiple microaneurysms of vessels, with beading and tapering of the vessel lumen. Biopsy of affected tissues reveals focal necrosis of vessel walls. p-ANCA is positive in 20% of cases and c-ANCA in 10%. The disease is associated with hepatitis B and carries a worse prognosis in this circumstance. Treatment is with prednisolone and cytotoxic agents, with plasma exchange used for extreme cases. Vidarabine and interferon are useful where there is concurrent hepatitis B infection. Kawasaki syndrome typically affects children presenting with fever, conjunctival infection, red tongue, arthralgia and lymphadenopathy. Fatal coronary arteritis can develop if the syndrome is untreated. Aspirin and intravenous immunoglobulin is the mainstay of treatment.

Large vessel vasculitis


GCA almost exclusively occurs in patients over the age of 50 years and is frequently accompanied by polymyalgia rheumatica. Visual loss may arise from central retinal artery occlusion, anterior ischaemic optic retinitis or occipital cortex infarction. Claudication or ischaemia of the tongue is a frequently overlooked but fairly common symptom. Temporal artery biopsy should be carried out where possible and will reveal lymphocytic and giant cell inltration with fragmentation of the internal elastic lamina. Treatment with prednisolone usually sufces, with doses tailored according to the ESR. Occasionally, other immunosuppressive agents are required. Takayasus arteritis (pulseless disease) usually affects young women who present with fever, arthralgia and myalgia and have high ESR. Treatment is with prednisolone and other immunosuppressive agents. Transluminal angioplasty or other mechanical revascularization may be required where there is risk of infarction. Patient demographics and the pattern of organ involvement provide clues to the underlying diagnosis (Figure 28.1). A full blood count will often show a normocytic

156

05 Vasculitic Syndromes anaemia, particularly where there is systemic disease. Urea and creatinine should be carefully monitored in all cases so that renal involvement can be identied early. ESR and CRP will almost invariably be elevated and are useful to monitor disease activity and response to treatment. Liver tests should be monitored, not only because of potential hepatic involvement in vasculitis, but also because of potential side effects of treatment. Detection of antinuclear antibody and rheumatoid factor (RF) may indicate the presence of an underlying connective tissue disorder. Apart from rheumatoid disease, RF may also be positive in patients with Sjgrens syndrome, and is more likely to be positive in patients with hepatitis B infection. Patients with both vasculitis and antiphospholipid antibodies are recognized. Anti-neutrophil cytoplasmic antibodies (ANCA) are more specic markers for certain vasculitic disorders (Table 28.1). c-ANCA are antibodies primarily directed at the enzyme proteinase-3. The perinuclear pattern (p-ANCA) is much

Child: (Any HSP; Asian Kawasaki)

Demographics

Young adult: (Middle Eastern Behets; Asian Takayasus)

Elderly: (GCA mainly Caucasian)

Palpable purpura: Small- to medium-sized vessels involved Skin Ulcers or ischaemia: PAN, CSS, Wegeners

Muscle

Pain

stiffness: GCA

Joints

Swollen/inflamed: PAN, CSS, Wegeners

Neuropathy

PAN, CSS, Wegeners, cryoglobulinaemia

Ischaemia: PAN, Takayasus Renal Glomerulonephritis: MPA, Wegeners, cryoglobulinaemia, CSS, HSP

Figure 28.1 Clinical pointers as to underlying disease state. The pattern and nature of organ involvement give clues as to the likely underlying diagnosis. CSS, ChurgStrauss syndrome; GCA, giant cell arteritis; HSP, HenochSchnlein purpura; MPA, microscopic polyangiitis; PAN, polyarteritis nodosa.

28 Vasculitic disease

157

less disease specic, and antibodies are directed at a number of determinants including myeloperoxidase, lactoferrin, elastase, cathepsin G and lysozyme. c-ANCA is also positive in many patients with crescentic or necrotizing glomerulonephritis. ANCA (predominantly p-ANCA) is positive in a proportion of patients with antiglomerular basement membrane disease, inammatory bowel disease, autoimmune hepatitis, sclerosing cholangitis and Felty syndrome. ANCA has traditionally been detected by indirect immunouorescence techniques but modern enzyme-linked immunosorbent assays (ELISAs) with specic antigens provide a more sensitive test. Cryoglobulins should be sought. Patients should be screened for hepatitis B and C, as well as HIV. Low levels of the complement components C3 and C4 are consistent with systemic lupus erythematosus, and may be increased in other inammatory conditions. Chest X-ray will reveal the presence of pulmonary nodules or inltrates. Biopsy of affected tissue may be required. Punch biopsy is suitable for most skin lesions, but excision biopsy is required where large vessel vasculitis is suspected. The characteristic change of leukocytoclastic angiitis is seen in small to medium vessel disease. Treatment depends on the diagnosis and severity of the disease. Simple measures for skin lesions include avoiding contact with lesions (loose clothing, bedding etc.), elevating the affected part, analgesia and non-steroidal anti-inammatory drugs. The mainstay is immunosuppression, usually initially with high-dose steroids with the later addition of other immunosuppressive agents including cyclophosphamide, azathioprine and methotrexate. Cyclosporine and mycophenolate mofetil may be used in more serious cases, while plasmapheresis and intravenous immunoglobulin may also be useful in resistant cases. Induction and maintenance phases of treatment require different approaches. For some patients, colchicine or dapsone are useful to maintain remission. Finally, biological therapies may be required for the most serious cases.4 These include the antiCD20 (anti-B cell) antibody rituximab, and the antitumour necrosis factor (TNF)-a agents: iniximab, adalimumab and etanercept.
Table 28.1 Major diseases associated with ANCA
Proteinase-3 (c-ANCA)
Wegeners granulomatosis Microscopic polyangiitis ChurgStrauss syndrome 80% 30% 30%

Myeloperoxidase (p-ANCA)
10% 60% 30%

Figures show percentage of cases with each condition that has the ANCA subtype.

Recent Developments
1 Recent evidence5 suggests that ANCA antibodies are directly involved in the vasculitic disease process. Their action helps promote neutrophil activation and adhesion, facilitating interaction with cytokine-primed vascular endothelium. They may arise from molecular complementarity the antibodies being directed against peptides encoded by the antisense strands of key epitopes. This phenomenon has

158

05 Vasculitic Syndromes been invoked to explain the relationship between ANCA positivity and exposure to infectious agents. Possible genetic determinants of ANCA-positive vasculitic syndromes have also been identied: these include increased carrier frequency of the Z allele of a1-antitrypsin (SERPINA1) and certain alleles of Fc receptors. Environmental triggers include exposure to silica, solvents, asbestos and pesticides. Drug triggers include the antithyroid drugs propylthiouracil and carbimazole, phenytoin, cocaine and allopurinol. 2 Data to support the use of the antibody therapies (rituximab, iniximab and etanercept) in vasculitic conditions, particularly Wegeners granulomatosis, are accumulating.6 The role of TNF-a in many of these disease processes means that iniximab and etanercept may nd broader usage. Even in GCA, which is generally very responsive to steroids, biological therapies may nd a use where the individual is susceptible to side effects of steroids. Interferon (a and b) has shown promise for treatment of ChurgStrauss syndrome. 3 The recent interest in atherosclerosis as an inammatory disease may give rise to novel therapies for common vascular disorders including coronary heart disease and stroke.7 Vascular inammation is known to play a prominent role in the development of atherosclerotic plaques and their subsequent rupture, giving rise to acute vascular events. Therapeutic strategies to decrease this vascular inammation include use of statins, cyclooxygenase inhibition, blockade of the reninangiotensin system and inhibition of pro-inammatory cytokines. Proposed biological approaches include immunization against the protein component of low-density lipoproteins (apoB-100).

Conclusion
Vasculitic disease is extremely variable in its presentation. The most common cutaneous manifestation is palpable purpura. The pattern of organ distribution and the nature of the lesions give clues to the underlying disease process. In all cases, the potential for involvement of major organ systems (lung, liver, heart, kidneys and nervous system) should be considered. The lesions are accompanied by symptoms of systemic inammation and inammatory markers are increased. Once diagnosis is established, or before in severe cases, treatment should be started. This is mainly with immunosuppressive agents. The acute and maintenance phases of treatment require different approaches. Biological treatments, including antibodies to B cells and TNF-a, are making a major impact on the management of difcult and refractory cases.

Further Reading
1 2 3 Roane DW, Griger DR. An approach to diagnosis and initial management of systemic vasculitis. Am Fam Physician 1999; 60: 142130. Scott DGI, Watts RA. Systemic vasculitis: epidemiology, classication and environmental factors. Ann Rheum Dis 2000; 59: 1613. Carlson JA, Cavaliere LF, Grant-Kels JM. Cutaneous vasculitis: diagnosis and management. Clin Dermatol 2006; 24: 41429.

29 Giant cell arteritis and polymyalgia rheumatica


4 5 6 7

159

Foster R, Rosenthal E, Marques S, Vounotrypidis P, Sangle S, DCruz D. Primary systemic vasculitis: treatment of difcult cases. Lupus 2006; 15: 1437. Bosch X, Guilabert A, Font J. Antineutrophil cytoplasmic antibodies. Lancet 2006; 368: 400418. Chan AT, Flossmann O, Mukhtyar C, Jayne DRW, Luqmani RA. The role of biologic therapies in the management of systemic vasculitis. Autoimm Rev 2006; 5: 2738. Stoll G, Bendszus M. Inammation and atherosclerosis: novel insights into plaque formation and destabilization. Stroke 2006; 37: 192332.

P R O B L E M

29 Giant Cell Arteritis and Polymyalgia Rheumatica

Case History
Mrs FM is a 74-year-old woman who complains of headaches, general malaise and stiffness of the upper arms and shoulders. The symptoms have gradually increased over four weeks. She is generally healthy and takes no medication apart from paracetamol for her mild osteoarthritis. Is temporal artery biopsy required to diagnose giant cell arteritis (GCA)? What is current thinking on the aetiology of GCA and polymyalgia rheumatica (PMR)? For how long is she likely to require steroid treatment? How should the condition be monitored?

Background
GCA is the most common vasculitic disease and affects medium- to large-sized arteries. GCA presents with headache, tenderness over the temporal artery or scalp, jaw claudication and sometimes visual loss. The latter develops quickly and is frequently permanent, making urgent diagnosis and treatment mandatory. PMR is a closely related condition that usually presents with pain and stiffness of the shoulder muscles, often accompanied by general malaise, fever and weight loss. PMR occurs in about 50% of patients with Atlas Medical Publishing Ltd

160

05 Vasculitic Syndromes GCA; of patients with PMR, around 10% will have GCA. The prevalence of PMR is around 1015 per 1000 and women are affected twice as often as men. GCA is more common in colder climates, is less common than PMR and has an incidence of about 20 per 100 000 per year. Both GCA and PMR are very uncommon under the age of 50 years.1,2 GCA and PMR are much more common in Caucasians, particularly those of Scandinavian descent. The fact that there is a racial predisposition and that familial clustering has been described points to a genetic component to the aetiology. Alleles of HLADRB1*04 have been commonly implicated. Amongst infectious triggers that have been considered are Mycoplasma pneumoniae, parvovirus B19 and parainuenza virus type 1.3 In GCA, inammation in the vascular wall leads to structural changes including intimal hyperplasia, fragmentation of the internal elastic laminae and luminal occlusion. The disorder in the vessel wall is driven by activated T cells and macrophages. T cells enter the vessel wall through the vasa vasorum, become activated through contact with antigen and produce a range of pro-inammatory cytokines including interferon-g. These cytokines lead to macrophage homing and activation. Shoulder pain and stiffness are the most common symptoms of PMR. Systemic symptoms include fever, mood changes, sweating, anorexia and weight loss. Erythrocyte sedimentation rate (ESR) is usually greater than 50 mm/h and there is often mild normochromic anaemia. Muscles are not strikingly tender. There is often some limitation of movement due to muscle stiffness, and secondary increase in joint pains is common. Differential diagnosis includes bromylagia, rheumatoid arthritis, polymyositis and osteoarthritis. Some patients develop swelling of the distal digits, carpal tunnel syndrome and tenosynovitis. GCA can affect any medium- to large-sized artery but most commonly affects the internal and external carotid arteries. Headache has often been present for several weeks

Table 29.1 Features of GCA


Frequency (%)
Headache Temporal artery thickened, pulseless or tender Malaise, anorexia or weight loss Jaw claudication Polymyalgia rheumatica Scalp tenderness Retinal ischaemia Visual loss Fever Dysphagia Cerebrovascular accident Recent onset of peripheral arterial disease
Adapted from Gonzalez-Gay et al. 2006.
1

85 73 61 41 40 33 23 13 10 5 2 2

29 Giant cell arteritis and polymyalgia rheumatica

161

before the patient presents. Depression, malaise, weight loss and fever often accompany the diagnosis, even in the absence of clinically apparent PMR. GCA is unusual when the ESR is less than 50 mm/h. The temporal artery may be thickened, tender and lack pulsation. Eyes should be examined, looking for loss of vision, an afferent papillary defect and ocular movement disorders (diplopia). Fundoscopy should be carried out to look for signs of ischaemic retinopathy. Table 29.1 lists the common features of GCA. The initial investigation should be ESR, which is seldom normal and frequently greater than 100 mm/h. The normal upper limit for ESR is estimated as: Men: Women: Age (years) 2 Age (years) + 10) 2

Up to 20% of patients with PMR have normal ESR. Serum C-reactive protein (CRP) is also usually elevated. CRP and ESR values are generally very strongly correlated. Other laboratory ndings include normochromic anaemia, thrombocytopenia and elevated liver enzymes. There may be a mild leukocytosis, and thrombocytosis is recognized. Temporal artery biopsy is the cornerstone of diagnosis and should be considered even where symptoms appear to strongly support the diagnosis. Before embarking on prolonged steroid therapy, a precise diagnosis is highly desirable. If the initial biopsy is negative, and symptoms are highly suggestive, biopsy of the contralateral temporal artery should be considered. Positive biopsy shows inammatory inltrate with disruption of the internal elastic lamina. The inammation is focal and segmental and granulomatous with multinucleate giant cells. The biopsy should be performed at the most symptomatic site and should ideally be at least 2.5 cm long. Temporal artery biopsy is not usually carried out in patients with what appears to be uncomplicated PMR. Newer diagnostic techniques are colour-coded duplex ultrasonography and positron emission tomography (PET). Both have a high sensitivity for detecting vascular disease, but changes are often not specic. Magnetic resonance imaging (MRI) angiography is also emerging as a useful tool. There are no documented associations with circulating autoantibodies. Prednisolone is the rst-line treatment. An initial dose of 1020 mg/day is usually sufcient for patients with PMR. Response is usual within two weeks, and after one month the dose can begin to be tapered down. The usual duration of therapy is two to three years. GCA usually requires prednisolone 4060 mg/day. Initiation should not be delayed while the results of investigations, including temporal artery biopsy, are awaited. Visual symptoms seldom develop after steroid therapy has been commenced. If present, or allowed to develop, visual symptoms usually do not recover. Severe, refractory or visual symptoms can be treated with intravenous methylprednisolone (1 g daily for three consecutive days). Symptoms respond to prednisolone within days and the dose can be tapered after a month. At six months, a dose in the range 7.510 mg/day is usual. Treatment for two to three years is generally required. Alternate day steroid regimens or adjuvant methotrexate are of uncertain benet. Flare-up of disease activity may occur with both PMR and GCA as the steroid dose is reduced below 7.5 mg/day. These ares can be managed by temporarily increasing the steroid dose. Safely decreasing and stopping prednisolone requires clinical judgement supported by the patients account of symptoms and measurement of inammatory markers. For refractory cases or those at high risk from corticosteroid side effects, methotrexate, azathioprine and iniximab are the agents with the best evidence for benet.

162

05 Vasculitic Syndromes As well as anti-inammatory therapy, antiplatelet therapy (e.g. low-dose aspirin) should be considered. As steroid treatment is likely to be ongoing for at least two years, baseline bone mineral density measurements should be carried out. Those with osteopenia or who are otherwise at high risk of osteoporosis should have prophylactic calcium (1000 mg/day) and vitamin D (800 Units/day) supplements. Those with osteoporosis should be considered for bisphosphonate treatment (chapter 44). Patients should be screened periodically for impaired glucose tolerance/diabetes, and their blood pressure should be monitored for the duration of treatment.

Recent Developments
1 Epidemiological studies are difcult because PMR and GCA are generally managed in primary care. A recent study4 documented the incidence in the United Kingdom between 1990 and 2001. During this period, the incidence of PMR increased from 6.9 to 9.3 per 10 000 person-years. There was no parallel increase in GCA, which remained static at 2.2 per 10 000 person-years. 2 Beyond its association with HLA-DRB1*04, remarkably little is known about the genetic basis of GCA. An association with alleles of the Fcg receptors has been proposed.5 These are important in immune regulation. Improved knowledge of the genetic factors in pathogenesis may help to identify susceptible individuals. 3 Study of the genes expressed in temporal artery biopsies may also give clues to disease pathogenesis and help to identify likely responders to treatment. Cid et al.6 reported increased expression of the chemokine monocyte chemoattractant protein-1 (MCP-1; also called CCL2) in temporal artery biopsies of patients with GCA. Furthermore, expression of CCL2 correlated with systemic measures of inammation, and the highest level of expression was seen in those with relapsing disease. 4 While ESR and CRP are highly correlated with each other and with a positive temporal artery biopsy, this is not invariable.7 The sensitivity of ESR alone may be no more than 76%, while positive CRP has a sensitivity of 97.5% alone and 99% when combined with ESR. Neither is specic and values of ESR and CRP should be interpreted in the light of the clinical picture and other investigations. 5 Lee et al.8 conducted a retrospective chart review and demonstrated that 16.2% of GCA patients in the study who were taking antiplatelet drugs suffered an ischaemic event compared with 48% of those who were not taking antiplatelet therapy. There is no compelling evidence to start all GCA patients on aspirin or clopidogrel at present but these interventions appear to pose little risk and may have considerable benet. 6 Pulsed intravenous methylprednisolone is used for initial therapy in those at high risk from an ischaemic event. Using this treatment initially may actually decrease steroid requirements subsequently. In a recent small study,9 10 of 14 patients treated initially with intravenous methylprednisolone were taking 5 mg/day prednisolone at 36 weeks, compared with only 2 out of 13 patients treated conventionally (P = 0.003). Even without intravenous steroid, most agree that high-dose steroid to bring the condition under control quickly allows a decrease in steroid exposure over succeeding months. 7 It is not known why older people are uniquely susceptible to PMR and GCA. Dehydroepiandrosterone (DHEA) is the most abundant circulating steroid hormone and levels decline with age. This decline has been associated with a range of

29 Giant cell arteritis and polymyalgia rheumatica

163

involutional changes including decreased cognitive function, decreased muscle bulk and a decline in bone health. Low levels of DHEA have been reported in patients with PMR or GCA plus PMR.10 There is currently no evidence to support routine treatment with DHEA in PMR or GCA.

Conclusion
As the number of elderly people is increasing, it is likely that GCA and PMR will be encountered more commonly in practice. Prompt diagnosis and effective treatment are

Muscle pain and stiffness

Consider other causes: Statin treatment Inflammatory myopathy Osteoarthritis Rheumatoid arthritis
Prednisolone 1020 mg/day*

High ESR and CRP

Headache or temporal artery symptoms/signs ESR 80 mm/hour Constitutional symptoms Poor response to low-dose steroids

Temporal artery biopsy

Negative ? Contralateral biopsy ? Another diagnosis

Positive Prednisolone 4060 mg/day*

* Taper dose of prednisolone down after one month if there is satisfactory response Monitor for: Hyperglycaemia Hypertension Osteoporosis (consider prophylaxis)

Figure 29.1 Diagnosis and management of PMR and GCA.

164

05 Vasculitic Syndromes essential not only to alleviate symptoms but also to prevent permanent disability from blindness and other serious vasculitic complications. Many clinicians diagnose and manage GCA without recourse to temporal artery biopsy. However, most experts agree that biopsy is indicated where there is a high suspicion of GCA. Given that the condition requires prolonged steroid treatment, it seems prudent to make a denitive diagnosis wherever possible. Diagnosis and management of PMR and GCA are summarized in Figure 29.1. The aetiology remains unknown but there appears to be some genetic inuence, and amongst environmental factors a variety of infectious triggers have been proposed. Steroid treatment is usually required for at least two years. Careful monitoring of the clinical response and inammatory markers is required to minimize exposure to steroids. Low-dose aspirin and osteoporosis prophylaxis should be considered, and all patients should be monitored for potential side effects of corticosteroids.

Further Reading
1 Gonzalez-Gay MA, Garcia-Porrua C, Miranda-Filloy JA, Martin J. Giant cell arteritis and polymyalgia rheumatica: pathophysiology and management. Drugs Aging 2006; 23: 62749. 2 Carroll SC, Gaskin BJ, Danesh-Meyer HV. Giant cell arteritis. Clin Experiment Ophthalmol 2006; 34: 15973. 3 Nordborg E, Nordborg C. Giant cell arteritis: epidemiological clues to its pathogenesis and an update on its treatment. Rheumatology 2003; 42: 41321. 4 Smeeth L, Cook C, Hall AJ. Incidence of diagnosed polymyalgia rheumatica and temporal arteritis in the United Kingdom, 19902001. Ann Rheum Dis 2006; 65: 10938. 5 Morgan AW, Robinson JI, Barrett JH et al. Association of FCGR2A and FCGR2A-FCGR3A haplotypes with susceptibility to giant cell arteritis. Arthritis Res Ther 2006; 8: R109. 6 Cid MC, Hoffman MP, Hernandez-Rodriguez J et al. Association between increased CCL2 (MCP-1) expression in lesions and persistence of disease activity in giant-cell arteritis. Rheumatology 2006; 45: 135663. 7 Parikh M, Miller NR, Lee AG et al. Prevalence of a normal C-reactive protein with an elevated erythrocyte sedimentation rate in biopsy-proven giant cell arteritis. Ophthalmology 2006; 113: 18425. 8 Lee MS, Smith SD, Galor A, Hoffman GS. Antiplatelet and anticoagulant therapy in patients with giant cell arteritis. Arthritis Rheum 2006; 54: 33069. 9 Mazlumzadeh M, Hunder GG, Easley KA et al. Treatment of giant cell arteritis using induction therapy with high-dose glucocorticoids: a double-blind, placebo-controlled, randomized prospective clinical trial. Arthritis Rheum 2006; 54: 331018. 10 Narvez J, Bernad B, Daz Torn C et al. Low serum levels of DHEAS in untreated polymyalgia rheumatica/giant cell arteritis. J Rheumatol 2006; 33: 12938.

30 Behets syndrome P R O B L E M

165

30 Behets Syndrome

Case History
A 26-year-old woman presents with subacute onset of fever and arthralgia and has developed oral and genital ulceration within the past two weeks. There is no family history of note. She has recently returned from a holiday in a subtropical region but there is no evidence of an infectious disease. Her previous health has been excellent and she takes no medications apart from the oral contraceptive pill. How can the diagnosis of Behets syndrome be established? What treatment options are available? What is her likely prognosis?

Background
Behets syndrome (AdamantiadesBehets disease) is a relapsing and remitting vasculitic condition of unknown aetiology, the manifestations and severity of which vary widely from patient to patient. The condition typically runs a more severe course in men and in those of either gender who develop it at an earlier age (<25 years). The condition was rst described by Hulusi Behet in 1937, and is most common in patients of Mediterranean or Asian origin. The common features are summarized in Table 30.1.1,2

Table 30.1 Features of Behets syndrome


Oral aphthous ulceration Genital ulceration Anterior uveitis Acne Erythema nodosum Skin ulceration Arthralgia Vasculitis: Arteritis Thrombophlebitis Aneurysm formation 98% 80% 50% Retinal vasculitis

Non-destructive arthritis Cerebral lesions Pulmonary lesions

Gastrointestinal ulceration Epididymitis

Atlas Medical Publishing Ltd

166

05 Vasculitic Syndromes The most common feature is oral ulceration. The ulcers vary in size but are typically small and occur in crops on the gums, inside of the mouth and on the tongue. Genital ulcers also occur in the majority, and frequently leave residual scarring when they have healed. Uveitis causes blurred vision, discomfort (particularly when looking at bright lights) and redness. Retinal vasculitis may lead to blindness if left untreated. Joint symptoms most commonly affect the knees, wrists, ankles and knees. Central nervous system involvement may cause headache, neck stiffness (if the meninges are involved), features of encephalitis (including fever, confusion, impaired consciousness, ts and coma) and focal neurological signs and symptoms. The disease usually starts in the 20s and 30s. It is more common in men in prevalent areas but in other countries, such as the United States, it occurs slightly more commonly in women. The aetiology appears to involve both genetic and environmental factors. The most prominent pathological feature is involvement of the small blood vessels with either vasculitis or thrombosis. Abnormalities at the vascular endothelium appear to be responsible for initiating the disease. Immunoglobulin M (IgM) anti-endothelial antibodies have been described. These may be, in part, directed at the endothelial enzyme a-enolase, a component of the glycolytic pathway. Cross-reaction of these antibodies with proteins of other organisms has been proposed to account for some of the environmental component of the aetiology. These organisms include Saccharomyces cerevisiae, Streptococcus pneumoniae and Candida albicans. There is markedly increased expression of cytokines and their receptors in affected tissues. The role of autoimmunity is uncertain there are no specic immune markers. Diagnosis is made clinically and may be conrmed by biopsy of affected tissue. Treatment depends on the severity and manifestations of the disease. There is no specic treatment. Milder cases only require symptomatic therapy (e.g. mouthwashes for oral ulceration or non-steroidal anti-inammatory drugs for arthralgia). Genital ulceration can be treated with azathioprine, cyclosporine, colchicine or thalidomide. The latter has to be used with care in female patients because of its teratogenicity. Biological agents may be required in severe cases of orogenital ulceration. Agents with proven effect include interferon-a, etanercept and iniximab. Uveitis or retinal vasculitis can be sightthreatening and require prompt treatment. The usual rst-line approach is corticosteroids plus one of the cytotoxic agents (azathioprine or cyclophosphamide). Cyclosporine or interferon-a may be used for refractory cases. Anticoagulation should be considered in those with severe thrombophlebitis. There is a risk of haemorrhage from aneurismal or vasculitic lesions (particularly in the lungs).

Recent Developments
1 Behets syndrome sufferers have a high prevalence of psychological disorders and quality of life is impaired. The latter particularly relates to fatigue, arthralgia and mucocutaneous lesions.3,4 Effective management of the disorder requires that psychological and psychosocial factors are taken into account. 2 The genetic contribution is not at all well understood. The best-characterized marker is HLA-B51, but a wide variety of other genetic polymorphisms have been studied. Recently, the tumour necrosis factor (TNF)-a-1031C allele has been linked with susceptibility to Behets syndrome.5 Other workers have found no relationship with

30 Behets syndrome

167

polymorphisms of the TNF-a gene.6 Other candidate genes that have been studied include those for the solute carrier SLC11A1 and the interleukin-18 gene. No association with polymorphisms of these genes was found, although levels of IL-18 are particularly high in patients with Behets syndrome. The disease may be a disorder of the interaction between the innate immune system and bacterial commensals or pathogens. A recent study found no link with polymorphisms for the genes for Toll-like receptors and no link with leptin gene polymorphism.7,8 High levels of leptin have been described in Behets syndrome. One positive link described recently has been a relationship with the Val16 allele for the superoxide dismutase gene in Japanese patients.9 3 Erdem et al.10 have reported increased insulin resistance in Behets syndrome patients. This is not surprising since systemic inammation and endothelial dysfunction are both forerunners of insulin resistance. Other vascular risk factors in Behets patients include impaired endothelium-dependent vasodilatation and increased circulating levels of homocysteine.11 Recently, increased platelet activation has been described and may contribute to the prothrombotic state.12

Conclusion
The aetiology of Behets syndrome is still relatively poorly understood. The diagnosis is made on clinical grounds. There is no specic serological marker. Measures of systemic inammation including serum C-reactive protein and cytokine levels may be useful in monitoring disease activity. Biopsy of affected tissue may help to establish the diagnosis. There is no specic treatment. The cornerstone of its management is symptomatic measures along with judicious use of corticosteroids and other immunosuppressive agents. The typical course is a relapsing and remitting one, but the disease does not usually lead to premature mortality. Vascular disorders and neurological involvement are the major threats but only occur in a minority of patients.

Further Reading
1 Kalayciyan A, Zouboulis C. An update on Behets disease. J Eur Acad Dermatol Venereol 2007; 21: 110. 2 Yazici H, Fresko I, Yurdakul S. Behets syndrome: disease manifestations, management, and advances in treatment. Nat Clin Pract Rheumatol 2007; 3: 14855. 3 Bodur H, Borman P, Ozdemir Y, Atan C, Kural G. Quality of life and life satisfaction in patients with Behets disease: relationship with disease activity. Clin Rheumatol 2006; 25: 32933. 4 Mumcu G, Inanc N, Ergun T et al. Oral health related quality of life is affected by disease activity in Behets disease. Oral Dis 2006; 12: 14551. [Erratum appears in Oral Dis 2006; 12: 356] 5 Akman A, Sallakci N, Coskun M et al. TNF-alpha gene 1031 T/C polymorphism in Turkish patients with Behets disease. Br J Dermatol 2006; 155: 3506.

168

05 Vasculitic Syndromes
6 Ates A, Kinikli G, Dzgn N, Duman M. Lack of association of tumor necrosis factor-alpha gene polymorphisms with disease susceptibility and severity in Behets disease. Rheumatol Int 2006; 26: 34853. 7 Bacanli A, Sallakci N, Yavuzer U, Alpsoy E, Yegin O. Toll-like receptor 2 Arg753Gln gene polymorphism in Turkish patients with Behets disease. Clin Exp Dermatol 2006; 31: 699701. 8 Aydin F, Kara N, Senturk N et al. Lack of association between leptin G2548A gene polymorphism and Behets disease. J Eur Acad Dermatol Venereol 2007; 21: 6871. 9 Nakao K, Isashiki Y, Sonoda S, Uchino E, Shimonagano Y, Sakamoto T. Nitric oxide synthase and superoxide dismutase gene polymorphisms in Behet disease. Arch Ophthalmol 2007; 125: 24651. 10 Erdem H, Dinc A, Pay S, Simsek I, Turan M. Peripheral insulin resistance in patients with Behets disease. J Eur Acad Dermatol Venereol 2006; 20: 3915. 11 Kayikioglu M, Aksu K, Hasdemir C et al. Endothelial functions in Behets disease. Rheumatol Int 2006; 26: 3048. 12 Akar S, Ozcan MA, Ates H et al. Circulated activated platelets and increased platelet reactivity in patients with Behets disease. Clin Appl Thromb Hemost 2006; 12: 4517.

S E C T I O N

S I X

06

Back and Specic Joint Problems


31 32 33 34 35 36 37 38 39 Acute back pain Chronic back pain Psoriatic arthritis Asymptomatic hyperuricaemia Gout acute attack and beyond Pseudogout investigation and management Joint and bone infections Viral arthritis Rheumatological complications of diabetes

P R O B L E M

31 Acute Back Pain

Case History
You have been asked to do a home visit to see Bob. He is a 46-year-old storeman who is moderately overweight (body mass index 32 kg/m2) and who had a sudden onset of low back pain yesterday when moving a box of parts in the storeroom. What features enable you to plan investigation and management? What are the important prognostic features?

Background
Low back pain (LBP) is common, with an annual incidence of 2%5%, and 70%90% of the population will experience an episode of LBP at some point. Prevalence increases with age until 65 years and decreases thereafter. LBP is the most common work-related injury and is second only to the common cold as a cause of absence from work (Table Atlas Medical Publishing Ltd

170

06 Back and Specic Joint Problems

Table 31.1 Risk factors for LBP


Strongest: Previous history of back pain Strong: Poor job satisfaction Emotional distress Manual labourer involving heavy lifting Prolonged sitting or standing

Moderate: Vibration tool use Smoking Obesity Poor physical tness

31.1). Fortunately, 90% of individuals with acute LBP improve within four to eight weeks, and only 5% of patients develop persistent or chronic LBP lasting longer than three months. For practical purposes, LBP is bounded by the level of the 12th thoracic vertebra and the gluteal folds, and pain may radiate into the leg. The cause of LBP is unknown in at least 80% of cases (Table 31.2) LBP is classied as non-specic LBP or simple backache, nerve-root (Box 31.1) or spinal nerve compromise, or potentially serious spinal pathology (including infection, cancer, fracture, inammatory back pain and cauda equina syndrome). In the retrospective study of Deyo et al.,1 of
Table 31.2 Causes of LBP
97% Mechanical LBP 80% Idiopathic 20% Prolapsed intervertebral disc Lumbar spondylosis Spondylolisthesis Spinal stenosis Fracture Scheuermanns disease Non-mechanical spinal pain Inammatory spondyloarthritis

1%

1%2% Visceral disease Infection Neoplasia

Table 31.3 Red ags of LBP


b b b b b b b b b b

Weight loss, fever, night sweats Nocturnal pain History of malignancy Recent or current infection Acute onset <20 or >55 years Constant or progressive pain Bilateral or alternating symptoms Neurological or sphincter disturbance Morning stiffness Claudication or peripheral ischaemia

31 Acute back pain

171

Box 31.1 Indicators for nerve root problems3


b b b b b

Unilateral leg pain > low back pain Radiates to foot or toes Numbness and paraesthesia in same distribution Straight leg-raising induces more leg pain Localized neurology in one nerve root

subjects presenting with acute LBP, 4% had compression fractures and 1% had cancer, infection, inammatory disorders or cauda equina syndrome. Another cohort of patients with acute LBP had an even smaller percentage of serious pathology.2 Sinister causes of back pain are relatively rare and suspicious features are listed in Table 31.3. Investigations are not usually indicated in simple backache. Where investigation is required, it should be aimed at conrming a specic pathological process. Cauda equina syndrome should be suspected when leg pain that includes several spinal nerve levels is accompanied by widespread motor and/or sensory weakness, and when there is associated bladder or bowel dysfunction.

Specic pathology leading to acute LBP


The spine is commonly described as a three-joint complex, consisting of the intervertebral disc anteriorly and the two facet joints posterolaterally. These border the triangularshaped spinal canal (Figure 31.1). Deformity of the three-joint complex, anterior or posterior subluxation (spondylolisthesis) of a vertebral body, as well as deformity of the posterior longitudinal ligament and ligamentum avum can lead to nerve-root impingement in the lower spine or spinal cord compression at higher levels. The spinal cord ends at approximately the level of lumbar vertebrae L1/L2 so that neurological manifestations in LBP syndromes are lower motor neurone lesions affecting peripheral nerve roots. The most common site of disc prolapse is the L4/L5 disc where a posterolateral protrusion impinges on the L5 nerve root. A centrally placed bulge may affect multiple lower nerve roots bilaterally, causing a cauda equina syndrome. The combined pathologies of facet joint hypertrophy/osteophyte formation, disc bulges and deformity of the long ligaments stenoses the spinal canal and presents with pseudoclaudication, sciatica and dysfunction of the cauda equina.

Intervertebral disc

Normal intervertebral disc, facet joints and exiting nerve root

Central disc bulge and posterolateral bulge with ligamentum flavum hypertrophy

Facet joint hypertrophy

Figure 31.1 Nerve root pain.

172

06 Back and Specic Joint Problems

Prolapsed intervertebral disc


The intervertebral disc comprises the inner nucleus pulposus and the outer annulus brosus. The nucleus is a proteoglycan matrix that must remain hydrated to act as a shock absorber. The overlapping zig-zag alignment of the layers of collagen bres within the annulus provides resistance to bulging when load is applied. The outer third of the annulus is innervated such that mechanoreceptors respond to distortion.

Spinal fracture
In the older patient, particularly female, presenting with sudden-onset localized back pain after minor trauma, osteoporotic compression fracture should be suspected. Fracture in younger people without major trauma is a rare cause of LBP. Lateral spinal views for compression fractures, or oblique views, will identify established or major changes on X-ray. Isotope bone scan or single photon emission computed tomography (SPECT) imaging may be more sensitive.

Cancer
Four clinical features have been proposed to be most predictive of malignancy as a cause of LBP: 1 2 3 4 Previous history of cancer Aged 50 years or older Failure of conservative therapy Weight loss

An erythrocyte sedimentation rate of >50 mm/h also leads to magnetic resonance imaging (MRI) investigation.

Infection
A history of immunosuppression or risk factors for breaches in the normal barriers to bacteraemia should be elicited, coupled with an examination looking for septic foci. Culture and microbial analysis of relevant tissue should be undertaken. In 40% of cases of spinal osteomyelitis there is haematogenous spread from an identiable extraosseous source, most commonly genitourinary, skin or respiratory. The most common organism is Staphylococcus aureus, with Gram-negative organisms in the elderly or intravenous drug users being common.

Inammatory LBP
It has been proposed that amongst patients with chronic LBP, up to 5% have inammatory back pain due to spondyloarthropathy (i.e. ankylosing spondylitis, reactive arthritis, psoriatic arthritis or inammatory bowel disease).

Management of acute LBP


Patients benet from non-steroidal anti-inammatory drugs (NSAIDs) to relieve pain, and should be encouraged to remain active (Box 31.2). Medications such as muscle relaxants may help but can cause drowsiness. Although the contribution of psychosocial factors to chronic LBP is appreciated, successful interventions aimed at patients with increased risk are yet to be identied. A

31 Acute back pain

173

Box 31.2 Summary of treatment guidelines for acute LBP3


b b b b

Reassure patient of favourable prognosis Advise to remain active, and discourage bed rest Do not prescribe specic back exercises Prescribe regular medication consisting of: b Paracetamol b NSAIDs Consider benet of muscle relaxants

general practitioner-implemented intervention focused on the identication of psychosocial prognostic factors and discussing these with the patient, setting specic goals for reactivation and providing an educational booklet had no effect on any outcome measure.4

Outcome of LBP
The prognosis of chronic LBP is poorly documented, with cohorts varying as to their composition and adequacy for dening an underlying aetiology of pain. Socioeconomic factors including litigation, unemployment benets and educational opportunities also impact on outcome. Poor prognostic factors include fear avoidance behaviour, leg pain and low job satisfaction. However, it is consistently reported that over 90% of patients improve by six to twelve weeks. Refshauge and Maher5 have explored this further; the ndings are a reminder that not all patients with LBP present for medical care (and that this may not be inuenced by severity of pain), and that others return to work despite ongoing pain. Recurrences of LBP are common, with an estimated three-quarters of patients having a recurrence within one year. The severity, however, is usually less and does not always lead to a new visit to the doctor.

Recent Developments
1 In the majority of patients with ankylosing spondylitis there is a ten-year delay between onset of symptoms and diagnosis. In about 95% of patients, the rst symptoms of spondyloarthritis develop before the age of 45 years and this should be the target group for screening. To help reduce the delay between symptom onset and diagnosis, it has been proposed that patients with LBP of duration greater than three months should be screened for the presence of inammatory back pain, the presence of HLA-B27 and for evidence of sacroiliitis.6 If one or more of these parameters is conrmed, patients should be referred to a rheumatologist. 2 New criteria have been proposed by Rudwaleit et al.7 to assist in dening inammatory LBP. The four parameters are (a) morning stiffness duration >30 min, (b) improvement with exercise but not with rest, (c) awakening during the second half of the night and (d) alternating buttock pain. For classication purposes, two or more criteria have a sensitivity of 70% and specicity of 81%. As the number of parameters increases, the likelihood ratio also increases, from 0.25 for no parameters to 2.3 for two parameters and to 12.0 for three or more parameters.7

174

06 Back and Specic Joint Problems

Conclusion
The majority of patients with LBP require either no medical intervention or only conservative therapies consisting of advice, simple analgesics and continuation of daily activities. Considering, however, its prevalence and impact on health economy there are still a large number of therapies offered but not of proven benet. Critical appraisal of features identied in history-taking and examination is required to provide a more accurate and specic diagnosis, and hence provide guidance on investigation, treatment and prognosis.

Further Reading
1 2 Deyo R, Rainville J, Kent D. What can the history and physical examination tell us about low back pain? JAMA 1992; 268: 7605. McGuirk B, King W, Govind J, Lowry J, Bogduk N. Safety, efcacy, and cost effectiveness of evidence-based guidelines for the management of acute low back pain in primary care. Spine 2001; 26: 261522. Koes BW, van Tulder MW, Thomas S. Diagnosis and treatment of low back pain. BMJ 2006; 332: 14304. Jellema P, van der Windt DAWM, van der Horst HE, Twisk JWR, Stalman WAB, Bouter LM. Should treatment of (sub)acute low back pain be aimed at psychosocial prognostic factors? Cluster randomised clinical trial in general practice. BMJ 2005; 331: 8490. Refshauge KM, Maher CG. Low back pain investigations and prognosis: a review. Br J Sports Med 2006; 40: 4948. Sieper J, Rudwaleit M. Early referral recommendations for ankylosing spondylitis (including pre-radiographic and radiographic forms) in primary care. Ann Rheum Dis 2005; 64: 65963. Rudwaleit M, Metter A, Listing J, Sieper J, Braun J. Inammatory back pain in ankylosing spondylitis: a reassessment of the clinical history for application as classication and diagnostic criteria. Arthritis Rheum 2006; 54: 56978.

3 4

5 6 7

32 Chronic back pain P R O B L E M

175

32 Chronic Back Pain

Case History
Jolene is a 43-year-old woman who has become rather depressed and angry about her painful back, which has been present for two years. She was a typist until her chair slipped out from under her and she landed heavily on her sacrum. Despite the best efforts of health practitioners she still experiences severe pain that no one can explain. She is frustrated at the number of doctors who have seen her and left her with the impression there is nothing abnormal to nd. Are there strategies for successful management of chronic low back pain (LBP)? When should we investigate further?

Background
In patients with chronic LBP the terms non-specic or mechanical LBP are used with an implication that an anatomical or pathological basis is understood. In reality, the aetiology remains unknown for the majority of cases. Observation is used to determine alteration in posture, muscle wasting, changes in the physiological lordosis and the effects of movement on spinal alignment. Palpation can be undertaken to assist in evaluating movement at a segmental level, and also to identify local tenderness. Dynamic procedures are used to elicit alteration in symptoms to a manoeuvre that is aimed at identifying the cause of symptoms. A systematic review analysed examination procedures used in the assessment
Table 32.1 Reliability of clinical examination
Observation Judging lordosis: two quality studies indicating reliability, two against Detecting lateral spinal shift: low reliability Evaluating abnormal posture or movement: low reliability Timed endurance testing: high reliability in low-quality studies Palpation Evaluating muscle tension or spasm: conicting evidence Existence of a xation or manipulative lesion: low reliability Identifying spinal level: low reliability Instability testing: conicting evidence Symptom response Pain response to repeated movement: conicting to low reliability Pain on palpation and trigger points: low reliability

Atlas Medical Publishing Ltd

176

06 Back and Specic Joint Problems of LBP.1 Methodological aws were identied in the majority of studies, but the results remain interesting. Palpation-based assessment was of low reliability, with moderate reliability of some examination procedures based on symptom response (Table 32.1). Abnormalities on X-ray and magnetic resonance imaging (MRI) are poorly associated with the occurrence of non-specic LBP. Abnormalities found when imaging people without back pain are just as prevalent as those found in patients with LBP, and it needs to be remembered that radiological abnormalities of degeneration and spondylolysis have been reported in 40%50% of people without LBP. With a prevalence of such magnitude, it has been suggested that this background prevalence of abnormalities be included in radiological reports.2 Computed tomography and MRI are equally effective in diagnosing lumbar disc herniation and stenosis, but only have meaning if the presentation had suggested this as the likely diagnosis and the imaging ndings remain compatible with clinical ndings.

Conservative treatment
Exercise and intensive multidisciplinary pain-treatment programmes are effective (Box 32.1). Evidence supports use of cognitive behaviour therapy, analgesics, antidepressants, non-steroidal anti-inammatory drugs (NSAIDs), back schools and spinal manipulation. However, the effects are usually only small and short term. Unfortunately, many commonly used interventions lack sufcient evidence of benet. No evidence supports using interventions such as steroid injections, lumbar supports and traction. Box 32.1 Guidelines for the treatment of chronic LBP4,5
b b b b b b

Recommend cognitive behavioural therapy, supervised exercise, brief educational interventions and multidisciplinary (biopsychosocial) treatments Advise patient to remain active Short-term use of NSAIDs and weak opioids Consider net benet of muscle relaxants, back schools, antidepressants and manipulation Avoid passive treatment such as ultrasound and shortwave diathermy Invasive treatments such as facet joint injections generally not recommended

Invasive treatment
A review of the efcacy of surgery and invasive interventions for the treatment of LBP and sciatica indicated that facet joint, epidural, trigger point and sclerosant injection have not clearly been shown to be effective.3 No sound evidence is available for the efcacy of spinal stenosis surgery, although surgical discectomy may be considered in those with sciatica due to lumbar disc prolapse who have failed to respond to conservative management. Randomized controlled trials (RCTs) comparing fusion surgery with conservative treatment show conicting results.

Recent Developments
1 Sciatica resulting from disc herniation resolves in the majority of patients within four weeks. Of fteen RCTs of epidural steroid injection, nine showed no benet, with six

32 Chronic back pain

177

showing short-term benet that did not last longer than one month. Valat has reviewed an additional three studies,6 in which patients received three epidural injections at between two-daily (one study) and three-weekly (two studies) intervals. Three-weekly methylprednisolone (80 mg) did not alter the primary outcome measure (the Oswestry Disability Questionnaire). Alternate daily prednisolone (50 mg) also had no effect. Three-weekly triamcinolone (80 mg) plus 10 ml bupivacaine led to signicant improvement in leg pain and self-reported function at three weeks, which was not sustained to six weeks or at one year. 2 Martell and colleagues reviewed the prevalence of opioid use in chronic LBP, its effectiveness and the prevalence of substance use disorders.7 Meta-analysis of four studies assessing the efcacy of opioids compared with placebo or a non-opioid control did not show reduced pain with opioids. Meta-analysis of ve studies directly comparing efcacy of different opioids showed there was no reduction in pain. On the important end-point of substance use disorders, there was a lifetime prevalence of substance use disorders of 1:2 to 1:3, with the estimate of the prevalence of current substance use disorder as high as 43%. It has to be noted that none of the trials extended beyond four months. While short-term use of opioids may be benecial, the value beyond four months is unclear, with substance use disorders common and aberrant medication-taking behaviours documented in a quarter of cases. 3 In another recent meta-analysis, the efcacy of psychological interventions was evaluated in patients with non-malignant chronic LBP.8 Outcomes included pain intensity, emotional functioning, physical functioning, participant rating of global improvement, healthcare use, pain medication and compensation status. Positive effects of psychological interventions, contrasted with various control groups, were noted for pain intensity, pain-related interference, health-related quality of life and depression. Cognitive behavioural and self-regulatory treatments were found to be efcacious. Multidisciplinary approaches that included a psychological component were also noted to have short-term effects on pain interference and positive longterm effects on return to work.

Conclusion
Chronic LBP is common and produces a signicant burden on healthcare resources, the individual and their caring practitioners. Treatment guidelines provide much-needed information on what works and what does not, but the meta-analyses underpinning these recommendations also identify the weaknesses in our studies to date. The apparent benet of psychological interventions should be borne in mind, and should be invoked in the maintenance of an ongoing supportive and empathetic relationship with the patient who has chronic LBP.

Further Reading
1 May S, Littlewood C, Bishop A. Reliability of procedures used in the physical examination of non-specic low back pain: a systematic review. Aust J Physiother 2006; 52: 91102.

178

06 Back and Specic Joint Problems


2 3 4 5 6 7 Roland M, van Tulder M. Should radiologists change the way they report plain radiography of the spine? Lancet 1998; 352: 22930. van Tulder MW, Koes B, Seitsalo S, Malmivaara A. Outcome of invasive treatment modalities on back pain and sciatica: an evidence-based review. Eur Spine J 2006; 15: s8292. Koes BW, van Tulder MW, Thomas S. Diagnosis and treatment of low back pain. BMJ 2006; 332: 14304. Airaksinen O, Brox JI, Cedrashi C et al. European guidelines for the management of chronic nonspecic low back pain. Eur Spine J 2006; 15 (Suppl 2): S192300. Valat J-P. Epidural corticosteroid injections for sciatica: placebo effect, injection effect or antiinammatory effect? Nat Clin Pract Rheumatol 2006; 2: 51819. Martell BA, OConnor PG, Kerns RD et al. Systematic review: opioid treatment for chronic back pain: prevalence, efcacy, and association with addiction. Ann Intern Med 2007; 146: 11627. Hoffman BM, Papas RK, Chatkoff DK, Kerns RD. Meta-analysis of psychological interventions for chronic low back pain. Health Psychol 2007; 26: 19.

P R O B L E M

33 Psoriatic Arthritis

Case History
Alice is 26 years old and has had psoriasis since age 18. This has been treated with topical preparations and she currently has only mild skin disease. She presents with pain and swelling in the small joints of her hands and reports pains in her back and right knee over the past few months. Her mother, aged 56, has psoriasis with associated arthritis. Is it likely that she is developing psoriatic arthritis (PsA)? How relevant is the positive family history? What steps should be taken to diagnose psoriatic arthritis? What is the treatment and prognosis?

Atlas Medical Publishing Ltd

33 Psoriatic arthritis

179

Background
Psoriasis is a common disorder of keratinocyte growth and differentiation affecting up to 2% of the population. The skin lesions are red, scaly, raised plaques typically found on the scalp, elbows and knees. Onset is usually in adolescence or early adulthood although a late-onset form is recognized. Prevalence estimates for psoriatic arthritis range from 10% to 40% of all cases of psoriasis. Clinically signicant arthritis affects about one in six patients with psoriasis.1,2 There is a decrease in the granular layer of keratinocytes, which are not fully differentiated and do not stack normally. Psoriasis is considered to be an organ-specic autoimmune disease that is predominantly T-cell mediated. The lesions are heavily inltrated with lymphocytes, monocytes and neutrophils. There is activation of antigen-presenting cells and endothelial cells, and blood vessels at the base of the lesions are hyperplastic. The T-cell response is a type 1 reaction with interleukin (IL)-12 and IL-23 production leading to increased interferon-g and tumour necrosis factor (TNF)-a, with consequent activation of inammatory pathways through signal transducer and activator of transcription-1 (STAT1). There is decreased local expression of the anti-inammatory cytokine IL-10. Platelet-derived growth factor (PDGF) and vascular endothelial growth factor (VEGF) contribute to the intense inammatory and vascular reaction. The normal keratinocyte maturation cycle spans 30 days, and this is truncated to only four days in psoriatic lesions. There is a strong genetic component in the aetiology of psoriasis. It is common in Caucasians, much less common in those of Asian descent (prevalence 0.1%) and very rare in individuals of African descent. Up to one-third of patients have a family history of the condition and 8% have an affected rst-degree relative. Concordance rate in dizygotic twins is 15%30% and in monozygotic twins is around 65%. At least ve susceptibility loci have been identied for psoriasis by genome-wide scanning studies.3 The most studied locus (PSORS1) is in the HLA region of chromosome 6p. Susceptibility is linked with a haplotype that includes the HLA-Cw0602 allele. However, non-HLA determinants in this region are also thought to be important (including polymorphisms of TNF-a), and there has been particular interest in the gene for corneodesmin (CDSN), which codes for a protein involved in keratinocyte maturation. The PSORS2 locus at chromosome 17q25 contains several candidate genes including SLC9A3R1, NAT9 and RAPTOR (the p150 target of rapamycin). Other loci are PSORS3 (4q35), PSORS4 (1q21) and PSORS5 (3q21). PSORS2 and PSORS4 contain genes that are also involved in susceptibility to atopic dermatitis. There is relatively little known about the genetics of PsA. The CARD15 gene at chromosome 16q has been suggested as a candidate, and is also involved in susceptibility to Crohns disease. PsA characteristically begins after ve to ten years of psoriasis, but may present at the same time as the skin lesions or even precede them. PsA is an important determinant of quality of life and disease burden associated with psoriasis. Studies have been hampered by lack of a specic biomarker, thus placing reliance on clinical scoring systems. The simplest of these is presented in Box 33.1. It should not be assumed that all arthritis in patients with psoriasis is PsA other conditions should be carefully excluded. At onset, there is pain, redness and stiffness which usually only affects a few joints in the rst instance, with development of the typical symmetrical polyarthropathy over time. Involvement of the distal interphalangeal (DIP) joints is usual, and helps to distinguish

180

06 Back and Specic Joint Problems

Box 33.1 Scoring system for diagnosis of PsA PsA can be diagnosed when the patient has an inammatory arthropathy (articular, spine enthesis) plus 3 from the following: 1 Active psoriasis, a personal or family history of psoriasis; current active psoriasis scores 2 2 Nail dystrophy pitting, onycholysis, hyperkeratosis 3 Rheumatoid factor (RF) negative 4 Active or previous dactylitis 5 Juxta-articular new bone formation on X-ray of the hands and feet (not osteophytes)
All features score 1 if present, except active psoriasis, which scores 2. Adapted from Taylor et al. 2006.4

Box 33.2 Comparison between PsA and RA PsA Small joint involvement DIP involvement Spine, sacroiliac joint involvement Enthesitis Synovitis Subcutaneous nodules Skin lesions RF positive Anti-cyclic citrullinated peptide (anti-CCP) antibodies Inammatory markers Family history ++ +++ ++ +++ ++ +++ + ++ +++ RA +++ + + +++ ++ + +++ +++ +++ +

When fully expressed, both diseases can cause a symmetrical, deforming polyarthropathy, which predominantly affects small joints of the hands and feet.

PsA from rheumatoid arthritis (RA) (Box 33.2). PsA is classied with the spondyloarthritides and spine involvement occurs in up to 40% of cases. One important characteristic of PsA is the involvement of entheses the points at which tendons, ligaments and joint capsules insert into bone. Enthesitis most frequently affects the plantar fascia, Achilles tendon and the ribs and pelvis. Another feature is dactylitis, where a whole digit is inamed because of a combination of enthesitis and synovitis. These changes are best diagnosed on a fat-suppressed magnetic resonance imaging scan. Five different forms of PsA are recognized (often reecting the stage to which the disease has progressed): asymmetrical; symmetrical; predominantly DIP joint involvement; spondylitis; and arthritis mutilans. The latter is the most severe form, affecting mainly the small joints of the hands and feet and causing considerable deformity. It affects less than 5% of patients with PsA. Spondylodiscitis is another serious, though fortunately rare, complication. SAPHO syndrome (Synovitis, Acne, Pustulosis, Hyperostosis and Osteitis) is an unusual overlapping syndrome. PsA is a serious diagnosis with the minority of cases running an entirely benign course.

33 Psoriatic arthritis

181

Inflammatory polyarthritis

Active psoriasis

Previous psoriasis F/H psoriasis

Exclude RA Nodules RF* Anti-CCP

Predominantly DIP** Spinal, SI joint Dactylitis Nail dystrophy

PsA

Symptomatic Physiotherapy NSAIDs

DMARDs

Biological therapies

Figure 33.1 Diagnosis and management of PsA. * RF and anti-CCP are positive in a proportion of patients with PsA. ** One of these is sufcient to make a diagnosis of PsA in the presence of active psoriasis; two should be present to make the diagnosis when there is a previous or family history (F/H) of psoriasis. anti-CCP, antibodies to cyclic citrullinated peptides; DIP, distal interphalangeal; DMARDs, diseasemodifying antirheumatic drugs; NSAID, non-steroidal anti-inammatory drug; PsA, psoriatic arthritis; RF, rheumatoid factor; SI, sacroiliac.

Diagnosis and treatment is summarized in Figure 33.1. Management of PsA overlaps with that of the skin condition, particularly where systemic disease-modifying drugs are used. For mild cases, physical treatments (exercises, splinting etc.) along with nonsteroidal anti-inammatory drugs where needed sufce. Intra-articular steroid is useful for large joint inammation. Systemic steroids are generally avoided because of the risk of are-up of skin lesions when the steroid is stopped. Methotrexate is the most commonly used disease-modifying drug for the skin condition and is also useful for joint involvement. As for RA, sulphasalazine, chloroquine, hydroxychloroquine, azathioprine and leunomide may be useful. Mycophenolate mofetil has also been used. The involvement of T cells in the pathogenesis of PsA has stimulated the use of cyclosporine and tacrolimus (both anti-T-cell agents), although their use is often limited by renal toxicity. Finally, biological agents are increasingly used for patients with severe disease. These

182

06 Back and Specic Joint Problems include IL-2 receptor antagonists, anti-T-cell agents (alefacept and efalizumab) and TNF-a antagonists (etanercept, iniximab and adalimumab).

Recent Developments
1 Disturbed angiogenesis is a key feature of psoriasis, with increased local production of angiogenic peptides. Butt et al.5 have investigated polymorphisms in the genes for VEGF, broblast growth factor (FGF)-1 and FGF-2, and epidermal growth factor (EGF). Increased prevalence of a polymorphism of the VEGF gene was demonstrated in patients with PsA. 2 High levels of RF in a patient with symmetrical polyarthritis supports the diagnosis of RA. However, RF positivity is not uncommon in inammatory or immune disorders other than RA, and does occur in a minority of patients with PsA. Anti-CCP antibodies are more specic for RA, but have also been described in 5%10% of patients with clinical PsA.6 3 Self-assessed quality of life and health status are directly linked to disease activity in PsA.7 The number of joints either involved or deformed has a direct bearing. Morning stiffness, level of inammatory markers and duration of disease are also important. With increasing duration of disease, the activity of the disease becomes a less important determinant of health status. 4 Evidence is accumulating that disease-modifying drugs for PsA, including the newer biological agents, are not only clinically effective but that their use also brings cost benets.8 This is leading to earlier and more aggressive treatment. PsA is a progressive condition and evidence of radiological joint damage begins to appear as the number of joints involved increases.9 There are strong arguments for making the diagnosis early and limiting the progression of a disease that can be devastating and disabling. 5 Psoriasis and PsA are associated with deterioration in cardiovascular risk prole.10 This is probably due to the widespread inammation that accompanies these disorders. Thus dyslipidaemia, hyperglycaemia, endothelial dysfunction and oxidative stress are all increased, as may be the impact of other conventional cardiovascular risk factors such as smoking and obesity. Recently, patients with PsA have been shown to have increased carotid intimamedia thickness a direct correlate with increased risk of vascular disease.11

Conclusion
PsA is a fairly common accompaniment to the skin lesions of psoriasis. It appears likely that the above patient is developing PsA, but other causes of arthritis should be considered. The family history is highly relevant. Several genetic determinants of psoriasis have now been identied. Environmental triggers for the disease are relatively poorly understood at present. It is not clear to what extent the genetic predisposition for PsA is separate to that of the skin disease. Diagnosis of PsA is largely clinical. There is no specic biological marker. Initial treatment is usually symptomatic, including the use of antiinammatory drugs. Systemic corticosteroids are not usually administered. There is

33 Psoriatic arthritis

183

increasing evidence favouring the early use of disease-modifying drugs, including the newer biological agents. PsA is a serious disease, often progressive, and associated with impaired function and quality of life as well as with increased cardiovascular risk.

Further Reading
1 Myers WA, Gottlieb AB, Mease P. Psoriasis and psoriatic arthritis: clinical features and disease mechanisms. Clin Dermatol 2006; 24: 43847. 2 Lowes MA, Bowcock AM, Krueger JG. Pathogenesis and therapy of psoriasis. Nature 2007; 445: 86673. 3 Bowcock AM, Cookson WOCM. The genetics of psoriasis, psoriatic arthritis and atopic dermatitis. Hum Mol Genet 2004; 13: R4355. 4 Taylor W, Gladman D, Helliwell P, Marchesoni A, Mease P, Mielants H. Classication criteria for psoriatic arthritis: development of new criteria from a large international study. Arthritis Rheum 2006; 54: 266573. 5 Butt C, Lim S, Greenwood C, Rahman P. VEGF, FGF1, FGF2 and EGF gene polymorphisms and psoriatic arthritis. BMC Musculoskelet Disord 2007; 8: 17. 6 Inanc N, Dalkilic E, Kamali S et al. Anti-CCP antibodies in rheumatoid arthritis and psoriatic arthritis. Clin Rheumatol 2007; 26: 1723. 7 Husted JA, Tom BD, Farewell VT, Schentang CT, Gladman DD. A longitudinal study of the effect of disease activity and clinical damage on physical function over the course of psoriatic arthritis. Arthritis Rheum 2007; 56: 8409. 8 Bansback NJ, Ara R, Barkham N et al. Estimating the cost and health status consequences of treatment with TNF antagonists in patients with psoriatic arthritis. Rheumatology 2006; 45: 102938. 9 Bond SJ, Farewell VT, Schentag CT, Gladman DD. Predictors for radiological damage in psoriatic arthritis: results from a single centre. Ann Rheum Dis 2007; 66: 3706. 10 Wakkee M, Thio HB, Prens EP, Sijbrands EJG, Neumann HAM. Unfavorable cardiovascular risk proles in untreated and treated psoriasis patients. Atherosclerosis 2007; 190: 19. 11 Kimhi O, Caspi D, Bornstein NM et al. Prevalence and risk factors of atherosclerosis in patients with psoriatic arthritis. Semin Arthritis Rheum 2007; 36: 2039.

184

06 Back and Specic Joint Problems

P R O B L E M

34 Asymptomatic Hyperuricaemia

Case History
Geoffrey is a 43-year-old man who recently applied for life insurance. As part of the application, a biochemical prole was undertaken which, from your pathology service, includes a serum uric acid level. Geoffreys result was 0.62 mmol/l (normal range 0.150.50 mmol/l). He has neither gout nor renal stones and is generally very healthy. Should his hyperuricaemia be treated? Should he be worried about vascular disease or renal impairment? What lifestyle measures should be recommended?

Background
Uric acid (UA) is the nal breakdown product of purine nucleotides (adenine and guanine) in the body. The nal metabolic steps are hypoxanthine to xanthine, which in turn forms UA. Both these steps are catalysed by the enzyme xanthine oxidase (XO). In most animals, UA is further metabolized to allantoin by urate oxidase (uricase). Allantoin is 510 times more soluble that UA and is readily eliminated by the kidneys. Humans and other primates have a non-sense mutation that results in defective uricase activity and thus higher concentrations of serum urate are reached. UA is a weak acid and in the extracellular uid is predominantly ionized 98% as monosodium urate. Plasma is saturated with UA at a concentration of 0.415 mmol/l (6.8 mg/dl) and above. When the plasma is saturated, UA crystals are liable to form. The precise threshold for this varies slightly because of other physicochemical and environmental inuences. Excretion of UA is almost entirely by the kidney, with urine at pH 5 being virtually saturated with UA. At higher pH, the solubility of UA increases exponentially and urine can contain more UA without risk of crystallization. It is for this reason that urinary alkalinization is used with uricosuric agents. UA can only be formed in tissues that contain XO (liver and the small intestine). About two-thirds of the daily purine load is generated endogenously from turnover of cells, while one-third is derived from diet. Levels are generally higher in men, and increase substantially at puberty. In women, plasma UA increases after the menopause to levels comparable to that of men. Women are at lower risk of gout but, when the saturation threshold for UA is reached, crystallization is equally likely to occur. Increased UA is present in 5%8% of males in the United States, and is higher in some racial groups with particular predisposition to obesity and vascular disease. Atlas Medical Publishing Ltd

34 Asymptomatic hyperuricaemia

185

Ribose-5-P

ATP

ATP

PRPP

GTP

AMP

Inosine monophosphate

GMP

Adenosine

Inosine

Guanosine

Adenine

Hypoxanthine XO Xanthine XO Urate Guanine

Figure 34.1 Purine metabolism. Ethanol, fructose feeding, glycogen storage diseases and severe hypoxia lead to ATP depletion and thus increased inux of AMP and adenosine into the UA synthetic pathway. The last two steps in the above, simplied pathway are catalysed by xanthine oxidase (XO). AMP, adenosine monophosphate; ATP, adenosine triphosphate; GMP, guanosine monophosphate; GTP, guanosine triphosphate; PRPP, 5 phosphoribosyl 1-pyrophosphate; Ribose-5-P, ribose-5-phosphate.

A detailed knowledge of UA metabolism1 is not required by the practising clinician. A simplied scheme is presented in Figure 34.1. This also serves as a reminder of how closely interrelated UA metabolism is with other aspects of intermediary metabolism. UA is almost entirely secreted by the kidneys. The UA transporter URAT1 (SLC22A12) is expressed in the proximal tubule and is a target for drug treatment. In most individuals with high UA, no underlying cause will be found. The clinician, however, should be aware of the large number of clinical associations. When there is an underlying cause, 90% are due to disorders of UA excretion while the remaining 10% relate to increased UA production. The causes of hyperuricaemia are summarized in Box 34.1. The most common contributor to UA overproduction is excessive dietary intake, including foods high in purines. Also, obesity and metabolic syndrome are associated with a marked tendency to increased UA synthesis. Fructose intake increases UA: following cellular uptake, fructose is phosphorylated, consuming adenosine triphosphate (ATP) and generating adenosine diphosphate (ADP). As shown in Figure 34.1, ADP feeds into the UA synthetic pathway, and this is favoured by intracellular phosphate depletion, which can also accompany fructose ingestion. Alcohol ingestion also leads to intracellular ATP depletion and consequent increases in both ADP and adenosine

186

06 Back and Specic Joint Problems

Box 34.1 Causes of hyperuricaemia A: Increased UA production (10%) Obesity Alcohol Myeloproliferative disease Polycythaemia vera Haemolysis Pagets disease Psoriasis Rhabdomyolysis

Glycogen storage diseases (type III, V and VII) Inborn errors of metabolism (including HPRT deciency) B: Decreased UA excretion (90%) Renal impairment Polycystic kidney disease Hypothyroidism Hyperparathyroidism Diabetes insipidus Sarcoidosis Drugs: Starvation Acidosis (ketoacidosis, lactic acidosis) Toxaemia of pregnancy Bartters syndrome Downs syndrome Lead poisoning Aspirin Cyclosporine L-dopa Diuretics Ethambutol Pyrazinamide

monophosphate (AMP). Beer and spirits are more likely to provoke increased UA than wine. A number of rare inborn errors of metabolism increase UA synthesis. The most common of these is deciency of hypoxanthine phosphoribosyltransferase (HPRT), complete deciency of which causes LeschNyhan syndrome (self-mutilation, choreoathetosis and mental retardation). The disorder is X-linked and carrier females are asymptomatic. A partial deciency of the enzyme causes KelleySeegmiller syndrome with the metabolic consequences of increased UA but without central nervous system manifestations. Other inborn errors include activating mutations of the phosphoribosylpyrophosphate synthetase gene and deciencies of adenine phosphoribosyltransferase, adenylsuccinate lyase, myoadenylate deaminase, adenosine deaminase and purine nucleoside phosphorylase. The association of high UA with metabolic syndrome and cardiovascular risk has attracted a great deal of attention.2,3 Increased UA may be a useful marker for insulin resistance. Even in subjects who have not yet developed metabolic syndrome there is a correlation between UA levels and body mass, total and high-density lipoprotein (HDL) cholesterol and triglycerides. As overweight individuals lose weight, UA levels decrease in parallel with serum leptin. Twenty to forty per cent of patients with untreated hypertension have high UA, and this is increased where there is also renal impairment. UA levels almost always increase in evolving renal failure. However, although increased UA is a marker for cardiovascular risk, it is not certain that it is an independent risk factor and one that we should be attempting to modify. To date, there is no direct evidence to suggest that drug treatment to decrease UA protects against cardiovascular events. There is suggestive evidence from the Greek Atorvastatin and Coronary Heart Disease Evaluation

34 Asymptomatic hyperuricaemia

187

Box 34.2 Dietary recommendations for hyperuricaemia and gout


b b b b b b b

Reduce body weight if overweight Avoid extremes of food intake both large, rich meals and fasting can increase UA. Eat small, regular meals Maintain uid (water) intake aim for 23 litres per day, particularly during areups of gout Limit alcohol intake to no more than one drink per day Avoid fatty food fried foods, fatty meats and excessive use of butter or margarine Limit protein intake and focus on proteins that are low in purines (dairy produce) The ideal diet is relatively high in carbohydrate but some high-bre and wholegrain foods are relatively high in purines

The following are high in purines and best avoided: Game meats and birds; liver; kidney; small oil sh (sardines, anchovies, mackerel); seafood (mussels, scallops, prawns); meat extract; yeast supplements The following are relatively high in purines and intake should be limited: Oatmeal (< cup per day); wheat bran and wheat germ (< cup per day); peas, beans, spinach, asparagus, cauliower and mushrooms (< cup per day); poultry, meat, sh and shellsh (12 servings per day); dried beans, lentils and other pulses (maximum 1 cup per day cooked)
2 3 1 4 1 2

study4 and from the Losartan Intervention for Endpoint Reduction (LIFE) study with losartan that decreasing UA may parallel decreasing vascular risk in intervention studies.5 The vast majority of patients with hyperuricaemia do not require treatment. This should be reserved for patients who are either symptomatic or who are perceived to be at particularly high risk. The only widely used drug is allopurinol, which is an inhibitor of XO. All patients with high UA should have the benet of dietary advice (Box 34.2). A purine-free diet may also be used to investigate the underlying cause of hyperuricaemia. On such a diet, normal individuals will excrete less than 3.6 mmol/day (600 mg) of UA in the urine. Those who continue to excrete greater than 4.2 mmol/day almost certainly have a condition that leads to UA overproduction. Uric acid excretion can be increased by alkalinizing the urine with either sodium bicarbonate or acetazolamide.

Recent Developments
1 Fructose feeding of rats leads to development of the metabolic syndrome and also increases circulating UA. In a recent animal study,6 the XO inhibitor allopurinol and the uricosuric agent benzbromarone decreased the effect of fructose feeding on increasing insulin levels, systolic blood pressure and triglycerides, and also prevented weight gain. Furthermore, direct effects of UA-lowering agents on endothelial function were demonstrated by studying the response of isolated aortic rings to acetylcholine. 2 The recommended adult daily allowance for protein is 0.8 g per kilogram ideal body weight. High protein intake is associated with higher UA, which may contribute to

188

06 Back and Specic Joint Problems decline in renal function in patients with chronic kidney disease.7 There is debate about the benet of restricted protein diets in patients with declining renal function. The balance of evidence slightly favours protein restriction as a measure for patients with chronic kidney disease. 3 At present, allopurinol is the only agent in common use for lowering UA. The drug is well tolerated and generally quite effective. Febuxostat is a non-purine XO inhibitor. A recent trial8 with this drug in patients with gout demonstrated that the drug was well tolerated and was more effective than allopurinol in lowering serum UA.

Conclusion
Increased UA predisposes to gout, nephrolithiasis and renal impairment. Current evidence does not favour routinely treating hyperuricaemia in asymptomatic individuals. The exception is for patients that were treated with cytolytic chemotherapy for malignant disease. Increased cellular breakdown in these patients can cause marked hyperuricaemia. Most patients with increased UA do not develop gouty arthritis, and the latter can be effectively treated when it arises. A causative role for UA in the genesis of vascular and renal disease remains to be established. The above patient should be encouraged to maintain a healthy body weight and may consider restricting foods that are known to be high in purines.

Further Reading
1 2 3 4 Choi HK, Mount DB, Reginato AM. Pathogenesis of gout. Ann Intern Med 2005; 143: 499516. Baker JF, Krishnan E, Chen L, Schumacher HR. Serum uric acid and cardiovascular disease: recent developments, and where do they leave us? Am J Med 2005; 118: 81626. Becker MA, Jolly M. Hyperuricaemia and associated diseases. Rheum Dis Clin North Am 2006; 32: 27593. Athyros VG, Elisaf M, Papageorgiou AA et al. Effect of statins versus untreated dyslipidaemia on serum uric acid in patients with coronary heart disease: a subgroup analysis of the GREek Atorvastatin and Coronary-heart-disease Evaluation (GREACE) study. Am J Kidney Dis 2004; 43: 58999. Alderman M, Aiyer KJV. Uric acid: role in cardiovascular disease and effects of losartan. Curr Med Res Opin 2004; 20: 36979. Nakagawa T, Hu H, Zharikov S et al. A causal role for uric acid in fructose-induced metabolic syndrome. Am J Physiol Renal Physiol 2005; 290: F62531. Mandayam S, Mitch WE. Dietary protein restriction benets patients with chronic kidney disease. Nephrology 2006; 11: 537. Becker MA, Schumacher HR, Wortmann RL et al. Febuxostat compared with allopurinol in patients with hyperuricaemia and gout. N Engl J Med 2005; 353: 245061.

5 6 7 8

35 Gout acute attack and beyond P R O B L E M

189

35 Gout Acute Attack and Beyond

Case History
Samuel is a 38-year-old mine worker who had an episode of pain and swelling in his left great toe six months ago. He now presents with a one-week history of right ankle swelling. His serum uric acid (UA) is elevated and aspiration of his right ankle joint conrms that he has acute gout. His general health is good and he takes no medications. What features of the clinical history are important? What aspects of examination may help determine if there is an underlying cause? How should he be treated and followed up?

Background
Gout is the most common inammatory arthritis and usually presents with single joint involvement, with the rst metatarsophalangeal joint affected in over 70% of cases.1 It has been recognized for centuries,2 affects about 1% of the population in developed countries and is becoming more common because of the increasing prevalence of obesity and metabolic syndrome. The prevalence increases with age and is around 4% in those aged 65 years and over. Males are two to three times more commonly affected. Gout is almost unique in being a disease state precipitated by a physicochemical reaction i.e. crystallization of UA when the plasma and synovial uid become saturated with UA at a concentration of around 0.42 mmol/l. Dietary factors are extremely important and frequently neglected. High-purine vegetable foods do not appear to affect UA levels so much as meat-based foodstuffs. The typical attack begins in the early hours of the morning and affects the great toe (podagra), ankle, ngers, elbow or, occasionally, knee. The distribution of joints affected relates to the prevailing temperature in the joints, with UA precipitation more likely to occur in cool joints. A variety of nucleating agents in the joint space including collagen, chondroitin sulphate and proteoglycans may initiate crystallization of UA. Gout is therefore more likely to occur in joints that are already damaged, for example by osteoarthritis. The pain is typically very severe and the affected joint is swollen, red and tender. Differential diagnosis includes septic arthritis, pseudogout and reactive arthritis. An acute attack usually resolves within 710 days, even without specic treatment. However, there will be a recurrence within three years in at least 80% of cases. Tophi are crystal deposits of UA in soft tissue and do not elicit an inammatory response comparable to that found in joints. They typically occur on the ears, hands, feet or elbow. Atlas Medical Publishing Ltd

190

06 Back and Specic Joint Problems The gold-standard investigation is the nding of UA crystals in a joint aspirate in association with elevated plasma UA. In practice, joint aspiration is frequently not undertaken. Plasma UA may increase during an acute attack of gout, but one-third of patients with acute gout have normal plasma UA levels. White cell count may be modestly elevated, while a very high count might suggest septic arthritis. If the full blood count is otherwise normal, a myeloproliferative disorder is unlikely (Box 35.1). Renal function may be impaired in patients with chronic hyperuricaemia, and should be checked in all cases. It may be useful to measure urinary urate concentration, since high levels would preclude the use of uricosuric agents. Hyperglycaemia and dyslipidaemia are frequent amongst patients with increased UA. Hypothyroidism is quite common, and may precipitate gout. X-ray of the affected joint usually only demonstrates the associated soft tissue swelling, but may show erosions around the joint in patients with chronic gout. Box 35.1 Common risk factors for gout Age Male gender Race (black > white) Acute illness with dehydration Increased uric acid Overweight or obese Diet high calorie and high purine Alcohol Proliferative lesions psoriasis, haemolysis, trauma/burns, myeloproliferative disease Drugs most commonly diuretics Initiation of UA-lowering therapy

Management of the acute attack is usually straightforward.35 Conservative measures include rest, elevation of the affected joint and cooling of the joint. It is important to ensure that the patient has adequate analgesia. Non-steroidal anti-inammatory drugs (NSAIDs) are widely used and may be the only treatment required. Colchicine has been used for many years, but should be regarded as a second-line drug. It acts by decreasing phagocytosis of UA crystals by neutrophils, and thus limits the inammatory response. The drug has a narrow therapeutic window and toxic effects include diarrhoea, nausea and vomiting. A dose of 0.5 mg tds is often sufcient. Some would precede this with a loading dose of 1 mg. Up to 6 mg/day can be given, although side effects are almost universal at this dose. It can be given intravenously, but there is a high risk of toxicity. The third line of therapy is corticosteroids, given either systemically or intra-articularly. For the latter, methylprednisolone or triamcinolone are most commonly used. Systemic steroids may be particularly useful where there is multiple joint involvement. A single dose of 80120 mg methylprednisolone may be used, or a course of prednisolone starting with 3060 mg/day. UA-lowering therapy should not be used during the acute attack. Planning of chronic treatment for a patient with high UA is summarized in Figure 35.1. The patient should be given appropriate dietary advice (chapter 34) and, if overweight, advised to lose weight. The two major approaches to pharmacological therapy are decreasing UA production by inhibiting xanthine oxidase (XO), or increasing UA excretion using uricosuric agents. A third approach, not currently widely available, is to

35 Gout acute attack and beyond

191

UA 0.42 mmol/l Confirmed gout (clinically or by joint aspiration)

Treat acute attack Wait for 4 weeks

? Overweight decrease weight Dietary advice (low purine, low fat) Limit alcohol intake Water intake 23 litres/day

Renal function Thyroid function

Plasma lipids Blood glucose

First attack or 2 attacks per year

2 attacks per year UA 0.6 mmol/l

Lifestyle management Monitor UA at 3 months

Failed Allopurinol

Successful

Continue treatment Monitor UA renal function

UA 0.36 mmol/l Decreased attacks

Check urinary urate If not increased

Consider uricosuric agent (e.g. sulphinpyrazone) Losartan if hypertensive Fenofibrate if dyslipidaemic

Figure 35.1 Prophylaxis of gout.

192

06 Back and Specic Joint Problems use uricase preparations (bacterial or fungal) to increase UA breakdown in vivo. Humans lack the enzyme uricase and cannot degrade UA.

Allopurinol
The mainstay of drug treatment for the past 50 years has been the XO inhibitor allopurinol, itself a purine derivative. Generally, it should be reserved for patients who have two or more attacks per year. Unless the UA level is very high, there is no justication for using drug therapy in patients with asymptomatic hyperuricaemia. The drug is well tolerated but reactions may occur in up to 2% of patients. These range from mild urticaria and pruritus to severe hypersensitivity. The latter causes severe (often ulcerating) skin lesions, fever, neutrophilia and often renal impairment. Other rare side effects include leukopenia, thrombocytopenia, peripheral neuropathy and GuillainBarr syndrome. In trials, only as few as 20% of patients treated with allopurinol reach the target serum UA value of 0.36 mmol/l. There are a number of potential drug interactions. Allopurinol inhibits the breakdown of the purine drugs azathioprine and mercaptopurine, and will increase risk of bone marrow toxicity from these drugs. Allopurinol may increase circulating levels of theophylline and increase International Normalized Ratio (INR) in patients taking warfarin. The dose of allopurinol should generally be kept to 300 mg/day or less, and should be decreased in patients with renal impairment (Box 35.2). Allopurinol is the pro-drug for the active metabolite oxypurinol, and many laboratories are able to provide oxypurinol levels as a guide to compliance or for titration of a lower dose in those with renal impairment. Box 35.2 Allopurinol dose vs glomerular ltration rate Estimated creatinine clearance (ml/min) >100 60 40 20 10 <10 Dose 300 mg/day 200 mg/day 150 mg/day 100 mg/day 100 mg every other day 100 mg every third day

Most rheumatologists concur with a costbenet analysis indicating that allopurinol only be commenced if >2 attacks of gout are occurring per year. After the acute attack, it may be useful to commence prophylactic treatment with either a low-dose NSAID (e.g. indomethacin 25 mg bd or diclofenac 25 mg bd) or colchicine 0.5 mg bd. This is continued throughout the introduction and adjustment of allopurinol and for three months afterwards. After one month of prophylaxis and quiet joints, allopurinol 100 mg is commenced in addition to the colchicine/NSAID. At one month the serum UA is measured, and if greater than 0.36 mmol/l, the dose of allopurinol is increased to 200 mg/day whilst continuing on the colchicine/NSAID. This continues monthly until the serum UA is below 0.36 mmol/l or 400 mg/day of allopurinol is reached. Patients are strongly advised that in the event of an acute attack of gout, they must not discontinue or alter their allopurinol dose but rather reinstitute their acute management plan with either an NSAID or

35 Gout acute attack and beyond

193

colchicine and seek medical review. If the aim is tophi dissolution, then the plasma UA level needs to be lowered towards the lower limit of normal. The current recommendation is for lifelong therapy.

Uricosurics
For patients who either do not respond to allopurinol or are intolerant of the drug, options are fairly limited.6 Currently, the second-line approach is to use uricosuric agents. In order to work, uricosurics require near-normal renal function, a sufcient urine volume and a suitable pH to prevent UA stone formation, and are contraindicated in anyone with renal stones or in those with a high urinary excretion of UA. Uricosurics will not work with concomitant salicylates. Probenecid and sulphinpyrazone are oldfashioned drugs that are still available. The more potent uricosuric benzbromarone is not licensed in most countries because of hepatic toxicity, but can be used on a named patient basis in some countries. Losartan should be considered if the patient is hypertensive, and fenobrate if there is dyslipidaemia. Both of these drugs have uricosuric action, as does the NSAID azapropazone. For patients with frequent attacks, or rebound attacks, low-dose colchicine can be used to help prevent recurrences.

Recent Developments
1 Becker et al.7 studied 762 patients with gout and hyperuricaemia randomly assigned to receive 300 mg/day of allopurinol or febuxostat at a dose of 80 mg or 120 mg. Of the latter two groups, 53% and 62%, respectively, reached the primary endpoint a serum UA concentration of 0.36 mmol/l. No difference was noted in the rate of are-up of gout during the 52 weeks of the study. Decrease in the size of gouty tophi was also demonstrated. 2 Martinon et al.8 have provided insight into the molecular basis for the intense inammation that arises as a consequence of UA crystal deposition in joints. Activation of inammatory cascades in phagocytic cells involves the formation of a unit called an inammasome. This is formed on activation of the cells by the complexing of a member of the NALP family of proteins (pyrin domaincontaining proteins sharing structural homology with NODs [nucleotide-binding and oligomerization domain proteins]) with the linking protein ASC (apoptosisassociated speck-like protein) and the enzyme caspase-1. In this study, UA crystals activated NALP3-containing inammasomes. Colchicine, acting upstream from inammasome formation, decreased activation of the pro-inammatory cytokine interleukin-1b.

Conclusion
In the vast majority of patients with gout, there is no single underlying cause. A positive family history could favour an underlying metabolic disturbance. Splenomegaly might suggest an underlying myeloproliferative disorder. Associated conditions are much more frequent than underlying causes. These include renal impairment, obesity, hypertension

194

06 Back and Specic Joint Problems and the metabolic syndrome. Management of the acute attack is with hydration and analgesia, often with the addition of colchicine. In planning ongoing management, careful attention should be paid to lifestyle factors. Allopurinol is the rst-line agent to lower UA, but does not need to be prescribed to all patients who either have high UA or have had an attack of gout.

Further Reading
1 2 3 4 5 Choi H. Epidemiology of crystal arthropathy. Rheum Dis Clin North Am 2006; 32: 25573. Nuki G. Treatment of crystal arthropathy history and advances. Rheum Dis Clin North Am 2006; 32: 33357. Suresh E. Diagnosis and management of gout: a rational approach. Postgrad Med J 2005; 81: 5729. Underwood M. Diagnosis and management of gout. BMJ 2006; 332: 131519. Zhang W, Doherty M, Bardin T et al. EULAR evidence based recommendations for gout. Part II: Management. Report of a task force of the EULAR Standing Committee for International Clinical Studies Including Therapeutics (ESCISIT). Ann Rheum Dis 2006; 65: 131224. Bardin T. Current management of gout in patients unresponsive or allergic to allopurinol. Joint Bone Spine 2004; 71: 4815. Becker MA, Schumacher HR, Wortmann RL et al. Febuxostat compared with allopurinol in patients with hyperuricaemia and gout. N Engl J Med 2005; 353: 245061. Martinon F, Ptrilli V, Mayor A, Tardivel A, Tschopp J. Gout-associated uric acid crystals activate the NALP3 inammasome. Nature 2006; 440: 23741.

6 7 8

36 Pseudogout investigation and management P R O B L E M

195

36 Pseudogout Investigation and Management

Case History
Reg is aged 53 years and presented last week with a four-day history of an acutely swollen and very painful left knee that had occurred without trauma. He was afebrile but had a large knee effusion and Bakers cyst. Aspiration yielded 45 ml of non-viscous blood-stained uid that appeared turbid. The laboratory conrmed sterile synovial uid and a moderate presence of red cells, plus elevation of white cells (of which 60% are neutrophils) and abundant intracellular calcium pyrophosphate crystals. What are the known factors predisposing to pseudogout? How should these be investigated? What management strategies will you employ?

Background
The most frequent manifestation of calcium pyrophosphate dihydrate (CPPD) deposition disease is chondrocalcinosis, the asymptomatic radiographic nding of calcication of articular cartilage or brocartilage. Up to 5% of the population show radiographic evidence of chondrocalcinosis, with the incidence rising with age to 15%40% of those over 60 years of age, and 30%60% of those >85 years old. Because over three-quarters of presenting patients are >60 years old, and most have pre-existing joint damage, it is likely that biochemical changes in aging cartilage favour crystal nucleation. The acute symptomatic presentation of chondrocalcinosis is termed pseudogout. While usually mild, it can lead to quite severe and rapidly destructive arthritis. The presentation is of an inammatory arthropathy with loss of function, early morning stiffness and improvement with activity. Other manifestations include atypical forms of osteoarthritis (OA), severe destruction mimicking neuropathic arthropathy, a symmetrical synovitis similar to rheumatoid arthritis, and calcication of the intervertebral discs and longitudinal spinal ligaments leading to restricted spinal mobility and hence resembling ankylosing spondylitis but without sacroiliitis. Disordered calcication of cartilage and other skeletal tissues occurs commonly among the elderly, yet the reasons for this are poorly understood.1 CPPD deposition is associated with acute attacks of pseudogout, characterized by joint effusions with marked neutrophilia. The release of CPPD crystals into a joint space Atlas Medical Publishing Ltd

196

06 Back and Specic Joint Problems is followed by neutrophilic phagocytosis and subsequent release of potent chemoattractant and inammatory mediators. Pseudogout most commonly involves the knee followed by the wrists, metacarpophalangeal joints, hips, shoulders, elbows and ankles. The joint distribution may provide a clue to CPPD deposition disease, as primary OA rarely involves the metacarpophalangeal joint, wrist, elbow, shoulder or ankle. CPPD crystals, however, may also be found in the synovial uids of patients with primary OA, either alone or in association with basic calcium phosphate crystals. Acute attacks of pseudogout may be precipitated by local events that induce crystal shedding from cartilage into the synovial uid, such as trauma, arthroscopy or intra-articular injection of highmolecular-weight hyaluronic acid as a viscosupplement. Systemic changes affecting calcium concentration such as rapid changes in uid balance, medical illness, commencement of thyroxine or parathyroid surgery can also induce an acute attack. A systemic response to pseudogout is noted in half of patients, with fever, neutrophil leukocytosis and raised inammatory markers. Denitive diagnosis of CPPD deposition disease would require unequivocal identication of weakly positive birefringent rhomboid or rod-shaped CPPD crystals in joint uid or articular cartilage. Aspirated uid is often turbid or blood-stained, with reduced viscosity and a marked neutrophil leukocytosis. Articular cartilage at any site may demonstrate chondrocalcinosis; the classical sites are the triangular ligament of the wrist, the pubic symphysis and the menisci of the knee. It has been proposed that the presence of CPPD crystals in synovial uid combined with radiographic evidence of calcication of the cartilage would make a denitive diagnosis, and either one would make a probable diagnosis. Because joint aspiration or biopsy is impractical in population studies, presence of radiographic chondrocalcinosis is often used in epidemiological and clinical studies.2 There are no population epidemiological studies of pseudogout, although most case series suggest that the mean age at presentation is between the seventh and eighth decades, with a female predominance of 23:1. The prevalence of radiographic chondrocalcinosis increases with age, with the reported prevalence of radiographic chondrocalcinosis in the knee joints of elderly males increasing from <4% in those less than 70 years of age to 27% in those aged over 85 years.3 Only a minority of patients with CPPD arthritis have metabolic or hereditary abnormalities. These include hyperparathyroidism, haemochromatosis, hypophosphatasia and
Table 36.1 Conditions associated with CPPD
Secondary to underlying medical condition b Hyperparathyroidism b Hypophosphatasia b Haemochromatosis b Hypomagnesaemia b Familial hypocalciuric hypercalcaemia b Possibly hypothyroidism b Chronic gout Secondary to underlying cartilage alterations b Aging and OA b Post-meniscectomy b Epiphyseal dysplasia

36 Pseudogout investigation and management

197

ATP

AMP

Hypomagnesaemia

Iron

Hypophosphatasia Alkaline phosphatase

Pyrophosphatase activity Human ANKH mutation Extracellular pyrophosphate

Iron, Fe3

Fe2 Crystal nucleation Hypercalcaemia

Hypomagnesaemia CPPD Disease

Figure 36.1 Metabolic factors predisposing to CPPD disease.

hypomagnesaemia (Table 36.1 and Figure 36.1). Hypothyroidism is probably associated with chondrocalcinosis, but not necessarily pseudogout. The presence of a CPPD arthropathy in a patient younger than 50 years, or in those with orid polyarticular chondrocalcinosis, should lead to investigation for an underlying metabolic disorder. Hereditary CPPD deposition disease has been reported in several ethnic populations, usually with autosomal dominant inheritance, and has early onset and varying penetrance. Whether genetic risk factors are involved in later-life chondrocalcinosis has not been established. Small studies suggest an increased recurrence risk in rst-degree relatives of 11%28%, and while this is likely to be greater than the prevalence in matched, unrelated individuals, this has not formally been established. A comparison of the recurrence rate of chondrocalcinosis of the knee in siblings of index cases and unrelated community members found signicantly increased familiarity for chondrocalcinosis (sibling recurrence risk ratio [SRRR] 2.0); for pyrophosphate arthropathy (chondrocalcinosis and OA combined), SRRR was 2.3.4 This showed that the familiarity of knee chondrocalcinosis in elderly patients is low, as would be expected for a common disease, and that severely affected young cases may provide a more rewarding genetic focus.1

Treatment
The therapeutic options for pseudogout are more limited and less based on an understanding of the underlying metabolic derangement than is the treatment for gout. Symptomatic therapy with non-steroidal anti-inammatory drugs, colchicine, joint aspirations, intra-articular steroids and non-pharmacologic support are the main

198

06 Back and Specic Joint Problems approaches to acute management of the acutely inamed joint and are applied to pseudogout. There are, however, few controlled trials. There is currently no specic treatment to slow or prevent the gradual joint deterioration due to chondrocalcinosis, or the progression of the crystal deposition, other than treatment of any underlying biochemical or metabolic disorders.

Recent Developments
1 Toll-like receptors (TLRs) are a family of receptors with roles in host defence and inammation. They provide a critical step in the innate immune response and are particularly adapted to recognize microbial components; for example, lipopolysaccharide is recognized by TLR4, and peptidoglycan by TLR2. The TLRs share a cytosolic Toll/interleukin-1 receptor domain that transduces upregulation of pro-inammatory genes through activation of nuclear factor-k-beta (NF-kB). TLR2 is expressed constitutively in chondrocytes and is upregulated in cartilage as a consequence of OA. Liu-Bryan et al. demonstrated that CPPD crystals could function as a ligand for TLR2 and mediate signalling to initiate nitric oxide production in chondrocytes.5 2 Two chromosome regions (on chromosomes 5 and 8) have been linked to chondrocalcinosis. The chondrocalcinosis gene on chromosome 5p at the CCAL2 locus has been demonstrated to be the ANKH gene. ANKH is a transmembrane pyrophosphate transporter and dysfunction of the gene causes elevation of intracellular, and reduction in extracellular, pyrophosphates. Mutations of the ANKH gene altering the amino terminal of the protein cause familial autosomal dominant chondrocalcinosis; changes towards the carboxy terminal cause cranial metaphyseal dysplasia.6,7 Cells transfected with ANKH variants have not shown signicant effects on pyrophosphate levels, leaving open the possibility that ANKH mutations cause chondrocalcinosis through effects other than on pyrophosphate transport.7

Conclusion
Chondrocalcinosis and pseudogout remain enigmatic diseases. Currently, the vast majority of cases are termed primary or idiopathic, reecting our lack of knowledge rather than providing a useful classication. Human genome mapping and lessons from animal models have provided genetic clues in our understanding of pyrophosphate metabolism and transport. It is hoped that further analysis of both populations and families with the various forms of CPPD deposition disease will identify both the basis of the disease and effective therapies.

Further Reading
1 2 Zhang Y, Brown MA. Genetic studies of chondrocalcinosis. Curr Opin Rheumatol 2005; 17: 3305. Choi H. Epidemiology of crystal arthropathy. Rheum Dis Clin North Am 2006; 32: 25573.

37 Joint and bone infections


3

199

Felson DT, Anderson JJ, Naimark A, Kannel W, Meenan RF. The prevalence of chondrocalcinosis in the elderly and its association with knee osteoarthritis: the Framingham Study. J Rheumatol 1989; 16: 12415. Zhang W, Neame R, Doherty S, Doherty M. Relative risk of knee chondrocalcinosis in siblings of index cases with pyrophosphate arthropathy. Ann Rheum Dis 2004; 63: 96973. Liu-Bryan R, Pritzker K, Firestein GS, Terkeltaub R. TLR2 signaling in chondrocytes drives calcium pyrophosphate dihydrate and monosodium urate crystal-induced nitric oxide generation. J Immunol 2005; 174: 501623. Williams CJ, Zhang Y, Timms A et al. Autosomal dominant familial calcium pyrophosphate dehydrate deposition disease is caused by mutation in the transmembrane protein ANKH. Am J Hum Genet 2002; 71: 98591. Pendleton A, Johnson MD, Hughes A et al. Mutations in ANKH cause chondrocalcinosis. Am J Hum Genet 2002; 71: 93340.

4 5

P R O B L E M

37 Joint and Bone Infections

Case History
Mr DH is 32 years old and was diagnosed with type 1 diabetes at the age of 8. He presents with a hot, swollen right knee with marked overlying cellulitis. The symptoms developed rapidly over 24 hours. He had been working on his garden the day before. He is febrile (38.3C) with tender lymphadenopathy in his right groin. He has maintained satisfactory glycaemic control by increasing his insulin. His knee is aspirated and the aspirate is purulent with large numbers of Gram-positive cocci. What is the likely organism? Is he likely to have a bone infection (osteomyelitis)? What investigations and treatment are indicated?

Atlas Medical Publishing Ltd

200

06 Back and Specic Joint Problems

Background
Septic arthritis
The diagnosis of joint infection is usually easy. The patient typically presents with a single hot, swollen and tender joint with fever, rigors and systemic upset. The knee is affected in around 50% of cases, the hip and ankle each in 15%, the elbow in 10% and the wrist and shoulders each in 5%. Around 10% of cases are polyarticular, particularly where the underlying infection is gonorrhoea, group B streptococcus, pneumococcus or Gramnegative organisms. Risk factors are summarized in Box 37.1. The incidence of septic arthritis is increasing, partly because of the aging population structure and partly because of antibiotic resistance. Much of the increase is due to methicillin-resistant Staphylococcus aureus (MRSA) and group B streptococci. The average age Box 37.1 Risk factors for septic arthritis Prior joint problems (osteoarthritis, rheumatoid arthritis, gout) Joint surgery, prosthetic joints, injection or arthroscopy Loss of skin integrity (ulcer, psoriasis, eczema) Diabetes Renal failure Cirrhosis Immunosuppression (including use of anti-tumour necrosis factor [TNF]-a therapy) Intravenous drug abuse No underlying risk factor in 20% of cases Box 37.2 Organisms responsible for septic arthritis Organism Staphylococcus aureus Streptococcus pyogenes Streptococcus pneumoniae Haemophilus inuenzae Mycobacterium tuberculosis Escherichia coli Coagulase-negative staphylococci Neisseria gonorrhoeae Streptococcus agalactiae Pseudomonas aeruginosa Neisseria meningitidis Salmonella Other Gram-negative organism Other b-haemolytic streptococci Polymicrobial Other (including fungi)
Adapted from Ross et al. 2003.1

% of cases 44.3 7.6 6.5 4.3 4.2 3.8 3.5 3.2 2.9 1.5 1.2 1.0 4.6 4.3 1.4 5.9

37 Joint and bone infections

201

at onset is 50 years. The elderly, those with comorbidities (including diabetes) and those infected with b-haemolytic streptococci are at increased risk.2 S. aureus is by far the most common organism and joint infection often follows a transient bacteraemia. The range of organisms implicated is very wide (Box 37.2). Unusual infections and at-risk groups include: b Brucellosis related to consumption of unpasteurised milk products. Presents with mono- or oligoarthropathy, often affecting the sacroiliac joint. Diagnosed by blood culture or serology and treated with doxycycline or streptomycin b Human and animal bites may present with mixed organisms (usually including streptococci and staphylococci) or unusual organisms. Rat-bite fever is caused by Streptococcus moniliformis and presents with arthralgia, systemic upset and rash of the palms and soles b Melioidosis, caused by Burkholderia pseudomallei, is an endemic infection in rural South-East Asia and tropical Australia. Prolonged treatment with Bactrim, doxycycline or cephalosporin is required to prevent reactivation b Intravenous drug users. The most prominent organism (60% of cases) in this group is Pseudomonas aeruginosa. The infection may affect joints not usually involved in septic arthritis including the sternoclavicular joint, sacroiliac joints or the pubic symphysis Investigation and management is summarized in Figure 37.1. The patient is usually febrile but peripheral white blood cell count is not always elevated. Inammatory markers (erythrocyte sedimentation rate and, particularly, C-reactive protein [CRP]) are almost always increased. Where possible, the joint should be aspirated; for hip infections, this should be done in theatre. It is not clear whether arthroscopy with joint washout is routinely superior to simple aspiration. Arthroscopy should be carried out if there is a possibility of a foreign body in the joint and where there has been injury to the joint. The white cell count on aspiration of a septic joint is usually greater than 50 000 cells/ml. This level of white cell count can also be seen with crystal arthritis. Blood cultures can also be taken, and the chances of a positive culture from a joint aspirate may be increased if a sample of the aspirate is incubated in a blood-culture bottle. The choice of antibiotic depends on the organism. An initial choice, unless Gram stain of the aspirate suggests otherwise, should include cover against S. aureus. A combination of ucloxacillin 500 mg qds and clindamycin 300 mg qds, both intravenously, is suitable. Generally, the patient should stay on antibiotics for a total of six weeks.

Osteomyelitis
Bone infection is either blood-borne or arises from contiguous spread from a focus of infection.3 Osteomyelitis may be further classied as acute or chronic. An anatomic classication is also used: medullary osteomyelitis is infection limited to medullary content and endosteal surface (equivalent to early haematogenous spread); supercial implies early change from local spread, and necrosis is limited to the exposed surface of bone; localized implies full cortical thickness, but which could be removed without compromising the stability of the bone; diffuse implies a segment of bone is involved and debridement would affect bone stability.

202

06 Back and Specic Joint Problems

Hot, swollen, tender joint

FBC Blood cultures ESR CRP

Aspirate

Arthroscopy irrigation

Aspirate WBC 50 000 per ml No crystals

Gram stain Culture aspirate Gram ve cocci e.g. Gram ve bacilli

Flucloxacillin clindamycin

Other antibiotics e.g. vancomycin

Rest

elevation of joint Analgesia

IV antibiotics until swelling subsided and CRP near normal

Oral antibiotics (6 weeks total antibiotics treatment)

Figure 37.1 Management of septic arthritis. +ve, positive; ve, negative; CRP, C-reactive protein; ESR, erythrocyte sedimentation rate; FBC, full blood count; WBC, white blood cell count.

37 Joint and bone infections

203

Haematogenous spread is the source of infection in about 20% of cases and S. aureus is by far the most common organism. It usually presents with local pain but with relatively little constitutional upset. The vertebral column is a common site, and the lumbar spine is affected in 45% of cases, thoracic spine in 35% and cervical spine in 20%. Patients with diabetes, renal failure or other chronic disease associated with a compromised immune system are at particular risk, with men twice as likely to be affected. Contiguous-focus osteomyelitis without vascular insufciency may follow trauma or surgical procedure (typically one month afterwards). Procedures include internal xation of fracture or insertion of a joint prosthesis. S. aureus is commonly implicated but infections are often mixed and include Gram-negative bacilli and anaerobes. Contiguous-focus osteomyelitis with vascular insufciency is most commonly seen in diabetic patients with poor vasculature, neuropathy and foot ulcers. It typically affects the small bones of the foot and generally requires amputation. The earliest X-ray signs of osteomyelitis in the long bones are periosteal thickening or elevation. Lytic areas may not be apparent for some weeks, when up to 75% of the bone matrix has been lost. Radionuclide scanning with technetium 99m is frequently helpful in localizing osteomyelitis, although the presence of an area of increased uptake does not necessarily equate with bone infection. A gallium scan may be helpful in difcult cases. Indium-labelled leukocyte scans are seldom used in clinical practice and have little to offer over other modalities. Magnetic resonance imaging (MRI) is very helpful, with areas of infection showing as increased intensity on the T2-weighted image. Management should include wound toilet and debridement where appropriate. Antibiotics should be continued for at least six weeks. The antibiotic regimen will depend on the clinical scenario and whether an organism has been isolated. Ciprooxacin, clindamycin, levooxacin and cephalexin are frequently useful. Two separate antistaphylococcal agents are usually prescribed. Outpatient intravenous treatment using a peripherally inserted central catheter is frequently used to avoid a prolonged hospital admission. Hyperbaric oxygen (HBO2) therapy, though not universally available, is a useful adjunct. The oxygen tension in infected bone is very effectively increased with HBO2. Low oxygen tension decreases migration of cells involved in wound healing, including broblasts, and also impairs the bacterial killing ability of phagocytic cells.4

Recent Developments
1 Culture-negative septic arthritis (5%20% of cases) may occur because of organisms that are difcult to detect or because the patient has been partially treated with antibiotics. A variety of markers in serum or synovial uid have been proposed. These include TNF-a, lactic acid, lactate dehydrogenase and procalcitonin. Recent studies, both with children5 and adults,6 conrm that procalcitonin is a highly specic marker for bone and joint infections. Sensitivity is improved by combining the results with CRP level. An alternative is polymerase chain reaction (PCR) screening for bacterial products. Availability of high-throughput techniques should make this method of diagnosis a viable option in the very near future. 2 Gavet et al.7 have compared joint infections in elderly patients with those that occur at a younger age. The range of causative organisms, the distribution of joint involvement and the incidence of polyarticular disease were similar. There was a

204

06 Back and Specic Joint Problems marked increase in mortality with age: death occurred in 0.7% of patients aged under 60 years, in 4.8% of those aged between 60 and 79 and in 9.5% of those aged 80 and older. 3 Spinal infections account for only between 2% and 4% of cases of osteomyelitis.8 Diagnosis is frequently delayed and should be suspected in patients who present with back pain that is not clearly of mechanical origin. Inammatory markers are usually elevated and are extremely useful in following the response to treatment. Imaging is extremely important in making the diagnosis and a combination of plain X-rays, radionuclide scanning and MRI is frequently required. Denitive diagnosis sometimes requires either open or percutaneous biopsy. 4 (18)-F-uorodeoxyglucose-positron emission tomography (FDG-PET) is emerging as a potentially useful tool in the diagnosis of musculoskeletal infections.9 It may be even more accurate than conventional radionuclide scanning for the diagnosis of osteomyelitis, and may be particularly useful where there are metal implants or prostheses. 5 The increasing use of invasive radiological techniques for angiography and lesion biopsy is placing an increasing number of patients at risk of staphylococcal bacteraemia.10 Risk factors for bacteraemia include general debility, being on a haemodialysis programme, having an indwelling vascular catheter and acquiring the bacteraemia outside a hospital setting. The duration of symptoms is also important. When undergoing a procedure, patients should be made aware of the need to report symptoms suggestive of infection promptly.

Conclusion
By far the most likely causative organism in the above case is S. aureus. However, a wide range of other organisms commonly causes septic arthritis. The diagnosis should be conrmed by joint aspiration where possible. The patients history of diabetes is highly relevant and represents a major risk factor. It is unlikely that the patient has osteomyelitis. Treatment should begin with intravenous antibiotics, which should be continued until the infection is fully subsided and inammatory markers have returned virtually to normal. In total, antibiotics should be given for at least four to six weeks. The risk of bone and joint infections increases with age, and they are still associated with considerable mortality, some of which is related to delayed diagnosis and inadequate antibiotic therapy.

Further Reading
1 Ross JJ, Saltzman CL, Carling P, Shapiro DS. Pneumococcal septic arthritis: review of 190 cases. Clin Infect Dis 2003; 36: 31927. 2 Ross JJ. Septic arthritis. Infect Dis Clin North Am 2005;19: 799817. 3 Calhoun JH, Manring MM. Adult osteomyelitis. Infect Dis Clin North Am 2005; 19: 76586. 4 Kawashima M, Tamura H, Nagayoshi I, Takao K, Yoshida K, Yamaguchi T. Hyperbaric oxygen therapy in orthopaedic conditions. Undersea Hyperb Med 2004; 31: 15562.

38 Viral arthritis
5 Butbul-Aviel Y, Koren A, Halevy R, Sakran W. Procalcitonin as a diagnostic aid in osteomyelitis and septic arthritis. Paediatr Emerg Care 2005; 21: 82832.

205

6 Martinot M, Sordet C, Soubrier M et al. Diagnostic value of serum and synovial procalcitonin in acute arthritis: a prospective study of 42 patients. Clin Exp Rheumatol 2005; 23: 30310. 7 Gavet F, Tournadre A, Soubrier M, Ristori JM, Dubost JJ. Septic arthritis in patients aged 80 and older: a comparison with younger adults. J Am Geriatr Soc 2005; 53: 121013. 8 An HS, Seldomridge JA. Spinal infections: diagnostic tests and imaging studies. Clin Orthop Relat Res 2006; 444: 2733. 9 Stumpe KDM, Strobel K. 18F FDG-PET imaging in musculoskeletal infection. Q J Nucl Med Mol Imaging 2006; 50: 13142. 10 Fowler VG, Justice A, Moore C et al. Risk factors for hematogenous complications of intravascular catheter-associated Staphylococcus aureus bacteremia. Clin Infect Dis 2005; 40: 695703.

P R O B L E M

38 Viral Arthritis

Case History
JG is a 45-year-old woman who presents with three days of pain and stiffness in her knees, ankles and hands. She reports fever, mild upper respiratory tract symptoms and accompanying myalgia. There is no past medical history of note. Her white blood cell count is normal but erythrocyte sedimentation rate is elevated at 56 mm/h and C-reactive protein is elevated at 80 mg/l. What other clinical features should be sought? Which viruses commonly give rise to this clinical picture? How should she be investigated and managed?

Background
Viral arthritis is relatively uncommon, accounting for only around 3% of acute polyarthropathies.1,2 It is usually self-limiting. Persistent joint symptoms can occur, particularly in patients who are immunocompromised or have persistent infection. In practice, it is often difcult to be certain of a diagnosis of viral arthritis initially: evidence of recent or past viral infection is very common; the range of viruses that can cause joint symptoms Atlas Medical Publishing Ltd

206

06 Back and Specic Joint Problems is broad; joint tissue is very vascular, especially when inamed, and even the nding of viral particles or DNA in joint tissue cannot alone be taken as evidence for a viral aetiology for the arthropathy. Joint inammation following viral infection can occur by direct damage due to viral replication in the joint (as in rubella) or alteration of the humoral and cell-mediated immune systems (as with hepatitis B and C), and many viruses can predispose to low-level autoimmunity. The latter can lead to autoantibodies including those to double-stranded DNA (dsDNA), rheumatoid factor (RF), SSA (Ro), SSB (La), neutrophil cytoplasmic antigen and cardiolipin. The appearance of low titres of autoantibodies does not necessarily signify that an autoimmune disease has developed. Conversely, many patients with autoimmune diseases have antiviral antibodies, but this does not necessarily signify previous viral infection. An example is the frequency of antibodies to human immunodeciency virus (HIV) p24 core in patients with systemic lupus erythematosus or Sjgrens syndrome (SS). The viruses that most commonly cause viral arthritis are summarized in Table 38.1.

Parvovirus B19 (B19)


Until recently, B19 was thought to be a single species and the only parvovirus to infect humans. In fact, three closely related genotypes (differing by less than 10%) have been identied. Parvovirus is a single-stranded DNA virus encoding two capsid proteins VP1 and VP2 the latter being the major capsid protein. The virus replicates in erythroid precursors. Epidemics of infection occur in late winter and early spring. The virus is spread by respiratory secretions. Symptoms develop after 718 days incubation and include upper respiratory tract symptoms, fever, arthralgia, myalgia and skin lesions. The characteristic skin lesion, erythema infectiosum (slapped cheek), occurs in most children but is characteristically absent in adults. Joint symptoms develop in only 8% of infected children but in 80% of adults. It typically causes symmetrical polyarthopathy, with affected

Table 38.1 Causes of viral arthritis


Parvovirus B19 Rubella Hepatitis C Alphaviruses b Chikungunya b Onyong-nyong b Ross River Virus b Barmah Forest Virus b Sindbis Virus, Pogosta Disease
b b b

Flaviviruses Kunjin Dengue Kokobera

Retroviruses Others Hepatitis B, EpsteinBarr, Varicella Zoster, Coxsackie B4 (Bornholm Disease), Cytomegalovirus

38 Viral arthritis

207

joints being tender and swollen but usually not erythematous and not deformed. Other manifestations include a transient aplastic anaemia, which can also occur in developing foetuses due to transplacental passage of the virus. Infection of a pregnant woman can lead to spontaneous miscarriage or hydrops fetalis. Development of immunoglobulin M (IgM) antibodies, followed by IgG antibodies, is characteristic of acute infection. Isolation of viral DNA may be achieved by hybridization or with polymerase chain reaction. As the pattern of joint involvement resembles that in rheumatoid arthritis (RA), and B19 infection is frequently accompanied by low titre RF, the virus has been considered as a potential aetiological agent for RA. Persistent joint involvement following B19 infection may occur in patients who fail to develop neutralizing titres of antibody, and this may be difcult to distinguish from RA. Intravenous immunoglobulin treatment should be considered for patients with evidence of continued infection.

Rubella
This is the only member of the Rubivirus genus of the Togaviridae family, and is an enveloped, single-stranded, positive-sense RNA virus. Sporadic cases are now typical, with the advent of vaccination programmes, while previously late-winter epidemics were characteristic. After an incubation of 79 days, the patient develops fever, cervical and suboccipital lymphadenopathy and a characteristic maculopapular rash that lasts up to ve days. Petechiae on the soft palate (Forchheimers spots) are characteristic. The vaccines used are live attenuated vaccines and may also cause joint symptoms. The arthralgia of rubella has a typical rheumatoid distribution. Low-level titres of RF are quite common, and RF positivity may conversely give a false positive for rubella IgM. Joint manifestations are much more common in adults, and more likely in females.

Hepatitis C
This is an enveloped, single-stranded RNA avivirus. More than 170 million people worldwide have now been exposed to hepatitis C. The prevalence is increasing, largely amongst intravenous drug users and prison populations. The infection now affects up to 3% of the United States population. Joint symptoms occur in up to 40% of hepatitis Cinfected patients, but arthritis occurs in only 2%. Again the distribution is usually like that found in RA, but erosive changes are absent. Around 20% of patients have mixed cryoglobulinaemia, and this is associated with an asymmetrical, pauciarticular, mediumlarge joint arthritis. The virus has been associated with a range of autoimmune diseases including RA, autoimmune hepatitis, glomerulonephritis and SS. Between 15% and 20% of SS patients show evidence of hepatitis C, often with absent SSA or SSB antibodies. The prevalence of RF positivity is greatly increased in hepatitis C-infected patients, but antibodies to cyclic citrullinated peptides (anti-CCP; a more specic marker for RA) are usually not detected. The presence of severe joint involvement should lead to antiviral therapy being considered. Up to 75% of patients with hepatitis C infection have extrahepatic manifestations, which usually improve with antiviral therapy. The usual treatment is with weekly subcutaneous peginterferon-a for 48 months and twice-daily ribavarin for 24 weeks. Corticosteroids may also be used for arthropathy, as may hydroxychloroquine, and there is some evidence of benet with anti-tumour necrosis factor

208

06 Back and Specic Joint Problems therapy. Hepatitis B infection is not commonly complicated by arthritis unless polyarteritis nodosa develops.

Alphaviruses
There are around 26 members of this genus of the Togaviridae family. The central RNAcontaining nucleoplasmid is surrounded by a lipid bilayer, into which is embedded multiple copies of the two encoded glycoproteins. The viruses are transmitted by mosquitoes, and reservoirs include birds, mammals and marsupials. A careful travel history is essential. Some of the common viruses of this group are listed in Table 38.1. Chikungunya occurs in central Africa, South and East Asia and the West Pacic; Onyong-nyong occurs in central East Africa; Ross River Virus is found in Australia and the West Pacic; Barmah Forest Virus occurs only in Australia; Sindbis Virus occurs in Scandinavia, Northern Russia, Africa and Australia; Pogosta Disease is largely conned to Finland. Symptoms are variable. After 712 days incubation, the patient may develop anorexia, nausea and vomiting, abdominal pain, pharyngitis, headache and photophobia. Facial ushing may give rise to a more generalized maculopapular rash. Joint symptoms a symmetrical, non-deforming polyarthropathy may precede the rash and other symptoms. Diagnosis is made on paired sera 1014 days apart, with increased IgM antibody indicating recent infection. Viral DNA can be persistent in synovial tissue, while detection of viral DNA in blood indicates ongoing or chronic infection. Treatment is symptomatic. Salicylates should be avoided if there is a possibility of dengue infection because of the increased risk of haemorrhagic complications.

Retroviruses
Articular manifestations are common and include arthralgia, spondyloarthropathy, SS (with CD8+ lymphocytic inltration of salivary and lachrymal glands), myopathy, systemic vasculitis and increased risk of septic arthritis. The latter may be caused by a variety of organisms including Staphylococcus aureus, streptococcus, salmonella and atypical mycobacteria. The presentation is often like reactive arthritis with uveitis and skin lesions. Psoriasis is also more common, frequently with joint manifestations. With the advent of highly active antiretroviral therapy in the 1990s, the risk of death from HIV infection decreased, while improved prognosis has been associated with a higher incidence of chronic complications, including those affecting the joints.

Recent Developments
1 A recent serological survey3 in the United Kingdom showed that 25% of young children had been exposed to parvovirus B19, compared with up to 75% of the adult population. Maternal infection could affect up to one in 500 pregnancies, and thus be a signicant contributor to fetal loss. Recent studies46 conrm increased B19 seropositivity in patients with RA. While this does not prove a causative relationship, the nding that viral DNA is present in synovial tissue from a proportion of patients, and that the presence of B19 DNA may precede the onset of clinical RA, adds weight to the hypothesis that B19 infection may contribute to the risk of RA. Furthermore,

38 Viral arthritis

209

cross-reactivity to the VP1 unique region has also been implicated in the development of antiphospholipid antibodies.7 2 There has been recent interest in the RNA-induced silencing complex (RISC).8 It is striking that the viruses that cause arthritis are RNA viruses, and that many of the autoantibodies involved in connective tissue diseases are directed at proteins involved in regulation of RNA replication and expression. RNA interference is an important post-transcriptional regulatory mechanism whereby small RNA species of up to 25 nucleotides direct the interaction between proteins and RNA. Nucleolytic enzymes (Dicer and the Argonaute proteins) are responsible for the generation of the small RNA species, which clearly could be of viral as well as of host origin. 3 Arthritis related to hepatitis C infection could become an increasing problem.9 Recent data conrm that the joint manifestations are not directly due to viral replication, and that less-direct, immune-activating mechanisms must be responsible.10 Understanding of the mechanism of hepatitis C-induced arthritis is important since it may not be inuenced by antiviral treatment but may be amenable to other forms of immune modulation.

Travel history Sexual history Drug usage

Other symptoms and signs CRP, ESR

NSAIDs Range of motion exercises

1st serum sample IgM positive 4-fold increase in Ab titre 2nd serum sample (1014 days) Viral arthritis Serology negative Persistent or worsening symptoms

Consider steroid treatment

Reconsider diagnosis Rheumatoid etc. Lyme disease Reactive arthritis

Figure 38.1 Diagnosis and treatment of viral arthritis. CRP, C-reactive protein; ESR, erythrocyte sedimentation rate; NSAID, non-steroidal anti-inammatory drug.

210

06 Back and Specic Joint Problems

Conclusion
Given the range of viruses that cause clinically signicant human infections, it is surprising how few have been associated with viral arthritis. Diagnosis and treatment are summarized in Figure 38.1. The mainstay of diagnosis is paired serological tests with an increase in antibody titre of four-fold above baseline or development of IgM antibodies indicating recent infection. Treatment is symptomatic, and usually conned to nonsteroidal anti-inammatory drugs and a range of motion exercises. Steroids may be useful for patients who have a conrmed diagnosis and who have mounted a serological response to the infecting agent.11 Prolonged arthritis may occur with parvovirus B19 and with the alphaviruses. For patients with severe, atypical or persistent symptoms, other diagnoses should be considered.

Further Reading
1 Calabrese LH, Naides SJ. Viral arthritis. Infect Dis Clin North Am 2005; 19: 96380. 2 Franssila R, Hedman K. Viral causes of arthritis. Best PractRes Clin Rheumatol 2006; 20: 113957. 3 Vyse AJ, Andrews NJ, Hesketh LM, Pebody R. The burden of parvovirus B19 infection in women of childbearing age in England and Wales. Epidemiol Infect 2007; 135: 135462. 4 Caliskan R, Masatlioglu S, Aslan M et al. The relationship between arthritis and human parvovirus B19 infection. Rheumatol Int 2005; 26: 711. 5 Chen YS, Chou PH, Li SN et al. Parvovirus B19 infection in patients with rheumatoid arthritis in Taiwan. J Rheumatol 2006; 33: 88791. 6 Baskan EB, Yilmaz E, Saricaoglu H et al. Detection of parvovirus B19 DNA in the lesional skin of patients with Behets disease. Clin Exp Dermatol 2007; 32: 18690. 7 Tzang BS, Tsay GJ, Lee YJ, Li C, Sun YS, Hsu TC. The association of VP1 unique region protein in acute parvovirus B19 infection and anti-phospholipid antibody production. Clin Chim Acta 2007; 378: 5965. 8 Jakymiw A, Ikeda K, Fritzler MJ, Reeves WH, Satoh M, Chan EKL. Autoimmune targeting of key components of RNA interference. Arthritis Res Ther 2006; 8: R8795. 9 Sanzone AM, Bgu RE. Hepatitis C and arthritis: an update. Infect Dis Clin North Am 2006; 20: 87789. 10 Tarantino G, Riccio A, Span A et al. HCV infection and chronic arthritis: Does viral replication matter? Hepatol Res 2006; 35: 23841. 11 Mylonas AD, Harley D, Purdie DM et al. Corticosteroid therapy in an alphaviral aarthritis. J Clin Rheumatol 2004; 10: 32630.

39 Rheumatological complications of diabetes P R O B L E M

211

39 Rheumatological Complications of Diabetes

Case History
Mr JT is a 62-year-old man who has had type 2 diabetes for ten years. Glycaemic control is not perfect (glycosylated haemoglobin [HBA1C] 8.2%) on a combination of sulphonylurea and metformin. He presents with stiffness and some pain around his left shoulder. Shoulder movement is limited and he cannot place his hand behind his head. Is his problem likely to be related to diabetes? What might be the underlying mechanism? Are musculoskeletal symptoms increased in patients with diabetes?

Background
The prevalence of diabetes is increasing globally. Currently, around 7% of the adult population is affected, and by 2025 the number of patients in the world with diabetes will have increased to over 300 million. This increase is largely attributed to the aging population structure and the rising prevalence of obesity. It is well established that the presence of diabetes relates to impaired quality of life and functional ability. Much of this is due to the direct metabolic consequences of diabetes, and also to its associated vascular complications. The development of diabetes also predisposes to a variety of musculoskeletal complications. These are often under-recognized in clinical practice. Rheumatological complications of diabetes have been classied according to whether they are due to the metabolic consequences of the condition, to microvascular complications or to underlying aetiological mechanisms.1,2 The problems encountered will be considered according to the affected tissue. Increased uric acid is frequent as part of the metabolic syndrome, possibly making gout more common in patients with diabetes. Osteoarthritis is much more common in obese individuals, who are also more likely to develop diabetes. Nerve entrapment syndromes may also be more common amongst patients with diabetes.

Muscle complications
Statin drugs are increasingly prescribed for the dyslipidaemia of diabetes. Muscle symptoms are relatively common with statin treatment, and severe muscle complications Atlas Medical Publishing Ltd

212

06 Back and Specic Joint Problems including myalgia, myositis and rhabdomyolysis occur in around 1% of treated patients. Diabetic muscular infarction is a rare disorder that usually occurs in patients with type 1 or type 2 diabetes of long standing and is associated with the presence of vascular complications. It presents as pain and swelling, most commonly in the thigh, but can occur in the calf or other muscle groups and may cause symptoms in multiple muscle groups. Conditions that should be excluded are venous thrombosis, abscess, haematoma, traumatic muscle tear and inammatory myositis. The aetiology is not clear but may include poor muscle perfusion from vascular disease, increased thrombotic tendency, metabolic alterations in skeletal muscle and the mechanical demands made on large muscle groups such as those in the thigh.

Ligament, tendon and capsular problems


Lower limb complications of diabetes osteomyelitis, foot ulceration and Charcots arthropathy are more likely to be serious and life-threatening. However, upper limb complications are more common and contribute considerably to the functional disability associated with diabetes. The most common of these are adhesive capsulitis of the shoulder, Dupuytrens disease, limited joint mobility syndrome (cheiroarthropathy) and palmar exor tenosynovitis (trigger nger). These may occur in isolation or associated with each other. They are more common in patients with long-standing diabetes, with poor glycaemic control and with microvascular complications. Increased collagen glycosylation and cross-linking has been implicated in the pathogenesis of these complications. Adhesive capsulitis causes pain and stiffness and limits the range of movement. It occurs in 2%3% of the non-diabetic population but is up to ve times more common in patients with diabetes, who are also more likely to have bilateral involvement. Treatment is with exercises, non-steroidal anti-inammatory drugs (NSAIDs), injected steroids or surgery. In a Finnish study,3 the prevalence of rotator cuff tendonitis and non-specic shoulder pain was 2% and 12%, respectively, in the general population. The risk of symptoms was much higher amongst patients with insulin-requiring diabetes (odds ratio = 8.8). There was a strong association between shoulder symptoms and psychological disturbances. Dupuytrens disease (DD), due to brosis and nodules in the palmar fascia, leads to exion contractures of the digits and occurs particularly in patients with long-standing and poorly controlled diabetes. DD primarily affects the middle and ring ngers. The reported prevalence amongst patients with diabetes varies between 3% and 30%. It is equally common in type 1 and type 2 diabetes. There is no strong relationship between risk of DD and the level of glycaemic control. Generally DD is more common in men but in diabetes the two genders are almost equally affected. Patients with diabetes are less likely to require, or have, surgical treatment. Cheiroarthropathy is usually manifest as an inability to extend the metacarpophalangeal joints fully. The prayer sign is usually positive the patient is unable to atten the hands together. The relationship between limited joint mobility (LJM) and poor diabetes control is not established. LJM may contribute to the development of the diabetic foot, including altering plantar pressures and predisposing to ulceration. The prevalence of LJM may have decreased in recent years, perhaps because of improvements in management.4 Nonetheless it still may occur in up to one in ve patients, and is associated with longer duration of diabetes.

39 Rheumatological complications of diabetes

213

Bone complications
Hyperostosis may present as spondylosis, hyperostosis frontalis interna or calcication of the joints and ligaments. All are much more common in patients with diabetes. Diffuse idiopathic skeletal hyperostosis (DISH) is a condition with excessive new bone formation, particularly in the enthesal region. There is considerable interest in this condition currently as it is associated not only with diabetes but also with other components of the metabolic syndrome including hyperinsulinaemia and increased growth hormone. The relationship between diabetes and low bone mineral density (BMD) has been uncertain. Patients with type 1 diabetes may have decreased BMD from an early stage. Low levels of vitamin D have been documented as a risk factor of diabetes, and this may partly explain the association. Insulin is a growth factor for bone. Insulin deciency in type 1 diabetes has been suggested as a contributor, although patients with type 1 diabetes are generally not insulin decient for long. Patients with type 2 diabetes are generally somewhat protected because of the associated obesity.

Joint disorders
Neuropathic arthropathy (Charcot joints) occurs in 0.1%2.0% of patients with diabetes. It most commonly affects the metatarsophalangeal, tarsometatarsal, ankle and interphalangeal joints, and occurs when sensation is lost but motor function is preserved. Initially affected joints are often swollen, red and tender. Charcot arthropathy is described in a wide range of neurological conditions but diabetes is by far the most common disorder in which it occurs. It frequently occurs alongside diabetic foot ulceration because of the altered foot pressure distribution, which occurs when the joints of the foot are misaligned. The affected bones are usually osteoporotic. The major differential diagnosis is with osteomyelitis. Inammatory markers (higher in bone infection) and magnetic resonance imaging help to distinguish the diagnoses. Treatment is with rest and pressure relief. Bisphosphonate drugs are widely used most commonly a course of intravenous pamidronate. Intranasal calcitonin is also used to decrease bone turnover. Preliminary evidence supports the use of this treatment in patients with neuropathic arthropathy.5

Recent Developments
1 Expression proling of genes potentially involved in the pathogenesis of DD reveals signicant changes in the matrix-degrading matrix metalloproteinases (MMPs).6 These enzymes are also involved in vascular remodelling and in the pathogenesis of the vascular complications. Expression of MMP1, MMP13 and MMP14 were all increased in DD tissue. Another recent study has reported increased activity of MMP2 in DD tissue.7 DD is due to increased broblast activity in affected tissue. As a consequence, there is increased production of matrix and remodelling of extracellular tissues. 2 It now seems certain that risk of osteoporotic fracture is increased amongst patients with insulin-requiring diabetes. In the Troms study,8 over 27 000 patients were followed up for six years. The relative risk (RR) of all non-vertebral fractures was 3.1 for men with type 1 diabetes and the RR for hip fracture was 17.8. For women, the

214

06 Back and Specic Joint Problems RR of hip fracture was 8.9 and 2.0 for type 1 and type 2 diabetes, respectively. The Womens Health Initiative study9 followed more than 93 000 women for seven years. Women with type 2 diabetes were at increased risk of fracture in spite of the fact that BMD was at least as high in diabetic patients as in those without diabetes. There is concern that thiazolidinedione drugs (rosiglitazone and pioglitazone), used in treatment of type 2 diabetes, accelerate bone loss and thus predispose to fracture.10

MUSCLE

Statin-induced myopathy LIGAMENT TENDON CAPSULE

Diabetic muscle infarction

Shoulder Capsulitis Tendonitis

Cheiroarthropathy (Limited Joint Mobility)

Dupuytrens disease

Trigger finger

BONE

Osteoporosis JOINT

Hyperostosis Local Diffuse

Neuropathic arthropathy (Charcots)

Association* Rheumatoid Psoriatic

Figure 39.1 Musculoskeletal complications of diabetes. * Rheumatoid disease and type 1 diabetes share some genetic predisposition but it is uncertain whether the two are associated. There is emerging evidence that psoriasis and its arthropathy are associated with diabetes, but the mechanism for this is not known.

39 Rheumatological complications of diabetes

215

Conclusion
Musculoskeletal complications are common in patients with diabetes mellitus (Figure 39.1). Attention has focused on foot complications, including neuropathic arthropathy. This is not unreasonable since these cause considerable disability and contribute greatly to the cost of managing diabetes. However, diabetes is associated with a wide range of musculoskeletal problems, the commonest of which involve the hand.11 Musculoskeletal complications are not strongly related to the level of glycaemic control, or even to the duration of diabetes. They do correlate with the presence of microvascular complications (retinopathy and neuropathy) and it may well be that microvascular changes in connective tissue are important in their pathogenesis.

Further Reading
1 Crispin JC, Alcocer-Varela J. Rheumatologic manifestations of diabetes mellitus. Am J Med 2003; 114: 7537. 2 Arkkila PE, Gautier JF. Musculoskeletal disorders in diabetes mellitus: an update. Best Pract Res Clin Rheumatol 2003; 17: 94570. 3 Miranda H, Viikari-Juntura E, Heistaro S, Helivaara M, Riihmki H. A population study on differences in the determinants of a specic shoulder disorder versus nonspecic shoulder pain without clinical ndings. Am J Epidemiol 2005; 161: 84755. 4 Lindsay JR, Kennedy L, Atkinson AB et al. Reduced prevalence of limited joint mobility in type 1 diabetes in a UK clinic population over a 20-year period. Diabetes Care 2005; 28: 65861. 5 Bem R, Jirkovsk A, Fejfarov V, Skibov J, Jude EB. Intranasal calcitonin in the treatment of acute Charcot neuroosteoarthropathy: a randomized controlled trial. Diabetes Care 2006; 29: 13924. 6 Johnston P, Chojnowski AJ, Davidson RK, Riley GP, Donell ST, Clark IM. A complete expression prole of matrix-degrading metalloproteinases in Dupuytrens disease. J Hand Surg 2007; 32: 34351. 7 Augoff K, Ratajczak K, Gosk J, Tabola R, Rutowski R. Gelatinase A activity in Dupuytrens disease. J Hand Surg 2006; 31: 16359. 8 Ahmed LA, Joakimsen RM, Berntsen GK, Fnneb V, Schirmer H. Diabetes mellitus and the risk of non-vertebral fractures: the Troms study. Osteoporos Int 2006; 17: 495500. 9 Bonds DE, Larson JC, Schwartz AV et al. Risk of fracture in women with type 2 diabetes: the Womens Health Initiative observational study. J Clin Endocrinol Metab 2006; 91: 340410. 10 Schwartz AV, Sellmeyer DE, Vittinghoff E et al. Thiazolidinedione use and bone loss in older diabetic adults. J Clin Endocrinol Metab 2006; 91: 334954. 11 Ardic F, Soyupek F, Kahraman Y, Yorgancioglu R. The musculoskeletal complications seen in type II diabetics: predominance of hand involvement. Clin Rheumatol 2003; 22: 22933.

S E C T I O N

S E V E N

07

Bone Diseases
40 41 42 43 44 45 46 Osteoporosis prevention and lifestyle management Bisphosphonates for osteoporosis which agent and when? Osteoporosis drugs other than bisphosphonates Male osteoporosis Glucocorticoid-induced osteoporosis Pagets disease of bone Bone complications of renal disease

P R O B L E M

40 Osteoporosis Prevention and Lifestyle Management


Case History
Jane is a t 51-year-old woman whose periods are becoming infrequent. She is concerned about developing osteoporosis as she approaches the menopause. Her mother has recently fractured her hip. Jane has recently had her bone mineral density (BMD) measured, and was told that she has osteopenia. What are the risk factors for osteoporosis? What advice would you give her on preventing osteoporosis? What is the role for calcium and vitamin D supplementation?

Background
Osteoporosis arises from loss of BMD with consequent disruption of bony microarchitecture and increased fracture risk. Osteoporosis is, by denition, present when the T score is 2.5 or less i.e. bone density is 2.5 standard deviations below the estimated Atlas Medical Publishing Ltd

218

07 Bone Diseases peak BMD for the population. Osteopenia is dened as a T score between 1 and 2.5. While both males and females are at risk of fracture in later life, the dramatic decrease in oestrogen at menopause in women means that they are generally at greater risk from an earlier age. BMD is most conveniently measured by dual-energy X-ray absorptiometry (DEXA). Screening of the population with DEXA is not generally recommended but may be justied in women aged over 65 years. Low BMD should always be interpreted in the light of the overall clinical picture and estimated fracture risk. All patients with fragility fractures should be screened for osteoporosis, and treatment should be considered where indicated. Fifty per cent of women and 20% of men will suffer a fragility fracture during their lifetime. Osteoporotic fracture is uncommon below the age of 60 years, and 85% of fractures occur in subjects over the age of 65. Peak bone density is attained in early adult life (around age 30 years); there is a steady decline in BMD thereafter, and this accelerates markedly after the menopause. Individuals with higher peak BMD are better able to withstand the later decline in BMD. At least 50% of variance in peak BMD is genetically determined. Polymorphisms in genes for the vitamin D receptor, collagen 1A1, lowdensity lipoprotein (LDL) receptor-related protein-5 (LRP-5) and the oestrogen receptor may all be determinants of peak BMD. The remainder of the variance in peak BMD is due to environmental factors including nutrition in early life, calcium and vitamin D status and exercise habits. These factors also determine the maintenance of BMD during middle life. At menopause, loss of oestrogen leads to activation of bone-resorbing cytokines including interleukin-1 (IL-1) and tumour necrosis factor (TNF)-a. Osteoclasts are activated through the receptor activator of nuclear factor-k-beta (RANK). The ligand for RANK (RANKL) is expressed on osteoblasts. Osteoprotegerin (OPG) a matrix protein produced by osteoblasts and stromal cells functions as an orphan receptor for RANKL, decreasing its ability to activate RANK on osteoclasts. Declining with age, OPG expression may contribute to development of osteoporosis. Increasing availability of drug treatments over the past 20 years has revolutionized management of patients with osteoporosis. Vitamin D and its analogues, oestrogen, selective oestrogen receptor modulators (SERMs, e.g. raloxifene), bisphosphonates, teriparatide and strontium all increase both trabecular and cortical bone. Data with vitamin D treatment suggest that it may reduce risk of fracture by up to 25%.1 Subclinical vitamin D deciency is common and the major impact of treatment is in patients with suboptimal vitamin D status. The place of oestrogen therapy has also altered, mainly as a result of the Nurses Health Initiative.2 In this large group of healthy post-menopausal women, oestrogen had the predicted benets on bone health but risk of cardiovascular events in women taking combined hormone replacement therapy (HRT) actually increased. Stroke risk increased by 8 per 100 000 person-years; risk of breast cancer increased by a similar amount. HRT is now only recommended for relatively short-term use in women with vasomotor and other menopausal symptoms.3 Bisphosphonates are the rst line of treatment for patients with established osteoporosis. There is some doubt, however, about how long they should be used for, and the typical three to ve years of treatment represents only a fraction of the time that many patients are exposed to risk of osteoporotic fracture. Lifestyle factors and secondary causes of osteoporosis are summarized in Box 40.1. Higher levels of activity, particularly weight-bearing exercise, are well documented to pro-

40 Osteoporosis prevention and lifestyle management

219

Box 40.1 Risk factors for osteoporosis Age Family history Smoking Excessive alcohol intake Prolonged immobility Lack of exercise Glucocorticoid exposure Previous fragility fracture Number of years since menopause Low body weight Low calcium intake Inadequate vitamin D Prolonged amenorrhoea Hyperthyroidism Hyperparathyroidism

tect against loss of BMD. Furthermore, exercise leads to improved muscle tone and function, and thus to lower risk of fall. Lock et al.4 have reviewed the literature on short- to medium-term exercise interventions for patients at high risk. Studies were difcult to compare because of differences in patient cohort, type of intervention and study design. At present, the evidence that short-term exercise interventions protect against fracture is limited. Up to 15% of the adult population, and up to 90% of very elderly subjects, have suboptimal vitamin D status.5,6 This can be checked by measuring plasma 25-hydroxyvitamin D (25[OH]D). A target range of 50100 nmol/l (2040 ng/ml) is generally agreed, and 75 nmol/l (30 ng/ml) is a reasonable threshold below which supplements should be considered. Toxicity is unlikely at levels below 250 nmol/l. The recommended daily allowance of vitamin D is 400 International Units (IU) per day, increasing to 800 IU/day in those at high risk of osteoporosis. Cholecalciferol (vitamin D3) is produced in the skin by the action of ultraviolet-B light in the wavelength range 290315 nm on the precursor 7-dehydrocholesterol. Vitamin D3 is transported in the blood mainly bound to vitamin D-binding protein. Sequential 25- and 1-hydroxylation in the liver and kidney, respectively, lead to formation of active 1,25-dihydroxy-vitamin D. The actions of this hormone include increasing intestinal and renal calcium absorption, as well as skeletal actions. Maintaining plasma calcium in the optimal range suppresses parathyroid hormone (PTH) secretion, therefore decreasing bone turnover. Low levels of vitamin D have also been implicated in autoimmune diseases such as type 1 diabetes and multiple sclerosis, and in malignancies including colon and breast cancers. For routine replacement, cholecalciferol (vitamin D3) should be used. Activated vitamin D analogues (calcitriol and alfacalcidol) should be reserved for use where there is a 1-hydroxylation defect i.e. patients with hypoparathyroidism or renal impairment.

Recent Developments
1 Bisphosphonate treatment decreases the risk of fracture by up to 50% in high-risk groups but only by around 20% in the age group 5059 years, which is at relatively low risk of fracture. A recent costbenet analysis7 conrms that it is not costeffective to routinely treat the younger age group by pharmacological means.

220

07 Bone Diseases 2 In trials of bisphosphonates, patients are routinely supplemented with calcium and vitamin D. Unfortunately, less attention is paid to nutritional factors in routine clinical practice. The recent launch of a combination once-weekly treatment of 70 mg alendronate with 2800 IU (70 mg) cholecalciferol should streamline the treatment of many patients with established osteoporosis.8
Perimenopausal and patient concerned Family history of osteoporosis Other risk factors

Avoid smoking Limit alcohol intake Exercise high intensity if possible Dietary advice calcium and vitamin D intake

Measure BMD

Normal BMD Reassure

Osteopenia or osteoporosis Thyroid function calcium, phosphate, PTH

Consider repeat DEXA in 35 years

Consider HRT if post menopause and has menopausal symptoms

Check 25(OH)D

<50 nmol/l

5075 nmmol/l

>75 nmol/l

800 IU D3/day

Recheck at 6 months

No supplement

Repeat DEXA at 1824 months Further decrease in BMD or fracture Consider drug treatment

Figure 40.1 Managing osteoporotic risk. 25(OH)D, 25-hydroxy-vitamin D; BMD, bone mineral density; DEXA, dual-energy X-ray absorptiometry; HRT, hormone replacement therapy; IU, International Units; PTH, parathyroid hormone.

40 Osteoporosis prevention and lifestyle management

221

3 In a recent study,9 subclinical vitamin D deciency was present in 75.4% of a cohort of elderly women selected from acute hospital admissions. Of those with vitamin D deciency, 36.7% had secondary hyperparathyroidism. Even after supplementation, 35.3% of the cohort still had suboptimal vitamin D status, partly because of imperfect compliance. This argues for more robust means of supplementation i.e. larger doses given as infrequent boluses. 4 A better understanding of the relationship between physical activity and bone physiology may lead to more effective exercise interventions. In a recent study of pre-menopausal women,10 only relatively high-intensity exercise was associated with favourable changes in BMD. Lower levels of exercise may, of course, be benecial for other aspects of health.

Conclusion
Osteoporosis will affect one in three women and one in eight men. Increasing numbers of elderly people mean that the condition will become more prevalent over the next few decades. An algorithm for management of osteoporotic risk in perimenopausal women is shown in Figure 40.1. Adequate calcium and vitamin D status should be ensured. The patient should receive advice on smoking, alcohol consumption and maintaining body weight in a desirable range. There is a relatively limited role for drug treatment in the age group of the above patient. HRT should be reserved for patients with menopausal symptoms, and the duration of therapy kept to a minimum.

Further Reading
1 Sambrook P, Cooper C. Osteporosis. Lancet 2006; 367: 201018. 2 Rossouw JE, Anderson GL, Prentice RL et al. Risks and benets of estrogen plus progestin in healthy postmenopausal women: principal results from the Womens Health Initiative randomized controlled trial. JAMA 2002; 288: 32133. 3 Ettinger B, Harris ST, Kendler D, Kessel B, McClung MR. Management of osteoporosis in postmenopausal women. Menopause 2006; 13: 34067. 4 Lock CA, Lecouturier J, Mason JM, Dickinson HO. Lifestyle interventions to prevent osteoporotic fractures: a systematic review. Osteoporos Int 2006; 17: 2028. 5 Boonen S, Vanderschueren D, Haentjens P, Lips P. Calcium and vitamin D in the prevention and treatment of osteoporosis a clinical update. J Intern Med 2006; 259: 53952. 6 Souberbielle JC, Friedlander G, Kahan A, Cormier C. Evaluating vitamin D status. Implications for preventing and managing osteoporosis and other chronic diseases. Joint Bone Spine 2006; 73: 24953. 7 Sanders KM, Nicholson GC, Watts JJ et al. Half the burden of fragility fractures in the community occur in women without osteoporosis. When is fracture prevention cost-effective? Bone 2006; 38: 694700. 8 Epstein S. The problem of low levels of vitamin D and osteoporosis: use of combination therapy with alendronic acid and colecalciferol (vitamin D3). Drugs Aging 2006; 23: 61725.

222

07 Bone Diseases
9 DeLappe E, McGreevy C, ni-Chadhain N, Grimes H, OBrien T, Mulkerrin E. Vitamin D insufciency in older female community-dwelling acute hospital admissions and the response to supplementation. Eur J Clin Nutr 2006; 60: 100915. 10 Vainionp A, Korpelainen R, Vihril E, Rinta-Paavola A, Leppluoto J, Jms T. Intensity of exercise is associated with bone density change in premenopausal women. Osteopor Int 2006; 17: 45563.

P R O B L E M

41 Bisphosphonates for Osteoporosis Which Agent and When?

Case History
Mrs RC is a 66-year-old woman who has lost 3 cm in height over the past three years. She has not had any other fractures. Her only medication is a diuretic that she takes for hypertension. X-ray reveals a wedge fracture of the T12 vertebra and generalized osteoporosis. A subsequent dual-energy X-ray absorptiometry scan shows the T score for her lumbar vertebrae to be 2.8, and that for her hip to be 2.1. Would you carry out any further investigations? Should she have treatment with a bisphosphonate? If so, which one would you choose and for how long should it be used?

Background
Bisphosphonates (BPs) have found wide usage in patients with metastatic bone disease, myeloma, Pagets disease and osteoporosis. They are stable analogues of inorganic pyrophosphate and bind to hydroxyapatite bone surfaces with high afnity, decreasing bone turnover by inhibiting osteoclastic activity.1 The general structure of BPs is shown in Figure 41.1. The R1 moiety is generally a hydroxyl group (except clodronate, where it is a chloride), while the structure of the R2 side-chain moiety varies. Two classes of BPs have been developed. The older group, which includes etidronate, tiludronate and clodronate, has an R2 group that does not contain nitrogen. Within the osteoclast, BPs Atlas Medical Publishing Ltd

41 Bisphosphonates for osteoporosis which agent and when?

223

OH R1 O P C

OH P OH O

OH R2

Figure 41.1 General structure of bisphosphonates.

become incorporated into non-hydrolysable adenosine triphosphate (ATP) analogues and thus inhibit a number of intracellular processes. The second group of BPs have a nitrogen-containing R2 group and include the more potent, modern drugs such as alendronate, risedronate, pamidronate, ibandronate and zoledronate. These modify the synthesis of isoprenoid compounds by inhibiting the enzyme farnesyl pyrophosphate synthase. They thus modify activity of several guanosine triphosphate (GTP)-binding proteins, inhibiting activity and promoting apoptosis of osteoclasts. The rst clinical use of BPs was in imaging because of the afnity of the drugs for areas of bone with high turnover. BPs are now the mainstay of treatment for established osteoporosis. Over a typical three-year course of treatment, BPs increase spine bone mineral density (BMD) by 40%50% and BMD at the hip by 20%40%. There is clear evidence from trials that this increase in BMD translates into decreased fracture risk, both spinal and at other sites including the hip.2 BPs are indicated for patients in the following categories: b Osteoporosis (T score <2.5) plus fracture(s) b Osteopenia (T score 1 to 2.5) plus fracture(s) b Osteoporosis without fracture but at high risk At present, they are not indicated for patients with uncomplicated osteopenia. One of the theoretical risks of bisphosphonate treatment was that, with prolonged use, they might inhibit bone mineralization. The window between doses that inhibit osteoclast activity and decrease mineralization is relatively narrow for etidronate but much wider for more modern agents. It is also possible that the drugs could increase brittleness of bone by progressively increasing bone mineral content. Patients from the major alendronate trials have now been followed for up to ten years and there is no evidence of long-term side effects or a rebound increase in fractures after the initial three years of treatment. The agents currently recommended for treatment of patients with established osteoporosis are: b Alendronate (Fosamax) can be given in a daily dose of 10 mg or in a weekly dose of 70 mg. The recent availability of a 70 mg/week preparation with 70 mg of cholecalciferol (2800 International Units vitamin D; Fosamax Plus) should prove advantageous, especially for older patients b Risedronate (Actonel) given in a daily dose of 5 mg or in a weekly dose of 35 mg b Ibandronate (Bonviva) given as a single dose of 150 mg once a month BPs should all be taken on an empty stomach. The patient should swallow the tablet with a full glass of water and remain upright for at least 30 minutes. The less frequent

224

07 Bone Diseases dosing schedules (weekly or monthly) have improved compliance since many patients had difculty adhering to the above routine on a daily basis. Intravenous infusion of pamidronate or ibandronate can be used in patients who cannot tolerate oral BPs. Potent drugs such as zoledronate may need to be given as infrequently as once a year. The major uncertainty with bisphosphonate treatment is the optimal duration. Following a three-year course of treatment, markers of bone turnover may be suppressed for up to ve years. Available evidence3 suggests that the drugs continue to be safe and benecial beyond three years. Long-term follow-up data with ibandronate are not available. Two non-placebo-controlled studies with alendronate following patients for up to ten years, and two similar studies with risedronate, have been published. These are continuation studies from the original three-year placebo-controlled trials. In these studies, continued fracture-prevention benet was apparent. There was no evidence of major gastrointestinal toxicity. Osteonecrosis is a very rare side effect and is often related to other factors such as poor dentition, high doses of bisphosphonate or concurrent chemotherapy. Alendronate, risedronate, ibandronate, raloxine, calcitonin, strontium ranelate and teriparatide have all been conclusively shown to prevent vertebral fracture. For nonvertebral fracture, the available evidence shows that alendronate, risedronate, strontium ranelate and teriparatide are effective. Based on currently available data, alendronate and risedronate are the drugs of rst choice for treatment of established osteoporosis in postmenopausal women.4 The drugs are probably entirely comparable in terms of their efcacy, and the safety prole of both is excellent. The efcacy and safety of oral ibandronate given as a single monthly dose has been established in animal, pre-clinical and clinical studies.5,6 The BONE study7 was a threeyear, randomized, double-blind, placebo-controlled trial involving 2946 postmenopausal women with osteoporosis and at least one vertebral fracture. Daily (2.5 mg) and alternate day (20 mg) doses were compared. The rate of new vertebral fractures in the placebo group was 9.6% compared with 4.7% in the daily ibandronate group and 4.9% in the alternate day group. In a post hoc analysis, there was also a reduction in risk of non-vertebral fracture. The MOBILE study8 was a two-year, randomized, parallel group study, and was the rst to examine the efcacy of a monthly dosing regimen. A total of 1609 women with post-menopausal osteoporosis were enrolled. The study conrmed that monthly dosing regimens were superior to daily dosing schedules. The 150 mg single-dose regimen gave the best results in terms of increased BMD.

Recent Developments
1 Osteonecrosis affecting either the mandible or maxilla is a recently described complication of bisphosphonate therapy.9 It occurs particularly in patients who have been exposed to high doses or multiple agents as used in metastatic disease. It presents with swelling, tenderness and pain. Treatment is with analgesia, withdrawing the bisphosphonate and using hyperbaric oxygen. It is thought that exposure of bone in the periodontal space and rapid turnover of bone in patients with dental problems or periodontal disease may predispose to osteonecrosis. Patients starting BPs should have any anticipated dental work carried out before starting bisphosphonate treatment, and be careful about dental hygiene while on treatment.

41 Bisphosphonates for osteoporosis which agent and when?

225

2 Siris et al.10 have examined compliance issues in a database of 35 537 women in the United States who were prescribed BPs. Only 43% of patients were completely rellcompliant and only 20% persisted with bisphosphonate therapy for the entire 24 months of the study. There was a decreased fracture rate in women who complied with, and persisted with, bisphosphonate treatment.

BMD measured by DEXA X-ray spine and other affected areas

Thyroid function, 25(OH)D Deoxypyridinoline

Osteoporosis

Osteopenia fracture

Normal or osteopenia

Raloxifene if osteoporosis predominantly spinal; HRT if menopausal symptoms

Lifestyle treatment Vitamin D

Alendronate or risedronate

Deoxypyridinoline 3 months Repeat DEXA 1218 months Response Continue for 35 years

No response

Oral ibandronate or IV bisphosphonate or teriparatide

If osteoporosis progressing

Stop if BMD restored and no further fracture Monitor deoxypyridinoline 6 monthly Check PTH* DEXA every 1824 months

Figure 41.2 Bisphosphonate treatment of post-menopausal osteoporosis. * Increased parathyroid hormone (PTH) may be a sign that bone turnover is increased and that further treatment is warranted. 25(OH)D, 25-hydroxy-vitamin D; DEXA, dual-energy X-ray absorptiometry; HRT, hormone replacement therapy.

226

07 Bone Diseases

Conclusion
The above patient has osteoporosis by denition. Baseline investigations might include thyroid function, renal function, erythrocyte sedimentation rate and protein electrophoresis to exclude myeloma, and measurement of plasma 25-hydroxy-vitamin D, calcium and parathyroid hormone. Calcium and vitamin D supplementation should be considered if there are grounds for suspecting that the patient may be decient. The rst line of treatment for this woman would be either alendronate or risedronate in a weekly dosing regimen. Progress can be monitored as shown in Figure 41.2. If there is no progress (determined by bone markers or BMD), changing to an alternative bisphosphonate is suggested (e.g. monthly oral ibandronate). Teriparatide is useful in refractory cases. There is considerable uncertainty about the optimal duration of bisphosphonate therapy. Typically, three to ve years is recommended but this may depend on demonstrated benet and perceived risk of stopping the treatment.

Further Reading
1 Russell RGR. Bisphosphonates: from bench to bedside. Ann N Y Acad Sci 2006; 1068: 367401. 2 Sambrook P, Cooper C. Osteoporosis. Lancet 2006; 367: 201018. 3 Liberman UA. Long-term safety of bisphosphonate therapy for osteoporosis: a review of the evidence. Drugs Aging 2006; 23: 28998. 4 Iwamoto J, Takeda T, Sato Y. Efcacy and safety of alendronate and risedronate for postmenopausal osteoporosis. Curr Med Res Opin 2006; 22: 91928. 5 Epstein S. Ibandronate treatment for osteoporosis: rationale, preclinical, and clinical development of extended dosing regimens. Curr Osteoporos Rep 2006; 4: 1420. 6 Reginster J-Y, Felsenberg D, Cooper C et al. A new concept for bisphosphonate therapy: a rationale for the development of monthly oral dosing of ibandronate. Osteoporos Int 2006; 17: 15966. 7 Chesnut CH, Skag A, Christiansen C et al. Effects of oral ibandronate administered daily or intermittently on fracture risk in postmenopausal osteoporosis. J Bone Miner Res 2004; 19: 12419. 8 Miller PD, McClung MR, Macovei L et al. Monthly oral ibandronate therapy in postmenopausal osteoporosis: 1-year results from the MOBILE study. J Bone Miner Res 2005; 20: 131522. 9 Farrugia MC, Summerlin DJ, Krowiak E et al. Osteonecrosis of the mandible or maxilla associated with the use of new generation bisphosphonates. Laryngoscope 2006; 116: 11520. 10 Siris ES, Harris ST, Rosen CJ et al. Adherence to bisphosphonate therapy and fracture rates in osteoporotic women: relationship to vertebral and nonvertebral fractures from 2 US claims databases. Mayo Clin Proc 2006; 81: 101322.

42 Osteoporosis drugs other than bisphosphonates P R O B L E M

227

42 Osteoporosis Drugs Other Than Bisphosphonates

Case History
Mrs JM is a 60-year-old woman who developed a wedge fracture of her L1 vertebra two years ago. She had low bone mineral density (BMD) (T score 2.6 for lumbar spine, 1.6 for hip). At a recent follow-up dual-energy X-ray absorptiometry (DEXA) scan, her BMD had deteriorated further (lumbar spine T score 2.9, hip 2.1). She insists that she has been taking her bisphosphonate, and continues to suffer back pain. Should she persist with bisphosphonate treatment? What other treatments should be considered? Is combination therapy an option?

Background
Bisphosphonate drugs have been the cornerstone of osteoporosis treatment in recent years. However, a small proportion of patients do not appear to respond. While changing to another, sometimes more potent, bisphosphonate may be the answer for some patients, other treatment options are now available.

Raloxifene
Raloxifene is a benzothiophene derivative that acts as a selective oestrogen receptor modulator (SERM). It has protective, oestrogen-like effects on bone and breast. The Multiple Outcome of Raloxifene Evaluation (MORE) trial, published in 1999, included 7705 women and conrmed that raloxifene increased spine and hip BMD. There was decreased risk of vertebral fracture but no denite effect on non-vertebral fracture. A recent meta-analysis1 has analysed seven studies in total, including the MORE trial. Raloxifene consistently decreased risk of vertebral fracture by 40%49% in those with previous vertebral fracture, and by 35% in those without. There is a suggestion of decreased risk of non-vertebral fracture in patients who are at particularly high risk. The issue of non-vertebral fracture is addressed in the Continuing Outcomes Relevant to Evista (CORE) study, in which 4001 women from the MORE trial were followed for up to eight years.2 Those originally taking placebo continued to take placebo. Those originally randomized to 60 mg or 120 mg raloxifene continued with 60 mg. There was no signicant benet of raloxifene in terms of non-vertebral fracture prevention. Atlas Medical Publishing Ltd

228

07 Bone Diseases Tamoxifen is a rst-generation SERM and is widely used in the treatment of breast cancer. It may also decrease the incidence of oestrogen receptor-positive breast cancer by up to 48%. The MORE study also demonstrated that raloxifene prevented the development of aggressive breast cancer. Tamoxifen slightly increases the risk of uterine cancer, while raloxifene may be neutral in this regard. Oestrogen (in hormone replacement therapy [HRT]) has benecial effects on lipid prole. Recent trials, however, have not conrmed that this translates into a decreased risk of cardiovascular disease. One of the hopes for raloxifene, a second-generation SERM, has been that improved lipid prole may lead to better cardiovascular outcomes. The Raloxifene Use for The Heart (RUTH) study enrolled over 10 000 post-menopausal women who had, or were at high risk of, cardiovascular disease. Prior to the results being published, the manufacturers disclosed an increased risk of stroke. A more recent analysis3 of the patients enrolled in MORE and CORE has shown neither benet nor increased risk for cardiovascular disease. Raloxifene is not recommended for cardiovascular protection and should not be administered to women at high risk of stroke. Studies suggest that raloxifene may help protect against cognitive decline with aging, but it remains to be established whether this is clinically relevant. Raloxifene causes menopausal-type vasomotor symptoms in 10%25% of patients and leg cramps in 7%, and increases the risk of venous thromboembolic disease by up to two-fold.

Strontium ranelate
Strontium is an alkaline earth metal with atomic number 38. For pharmacological use, it is administered in a stable complex with an organic moiety (ranelate). Early in vitro studies conrmed that strontium ranelate had benecial effects on bone formation (stimulating production of collagen and non-collagen proteins of the matrix) and on bone absorption (inhibiting both differentiation and activation of osteoclasts). There is now adequate clinical evidence to support the use of the drug in post-menopausal osteoporosis. It is well tolerated, but causes diarrhoea in up to 6% of patients. This usually disappears within three months. Increases in muscle creatine kinase may also occur, but seldom necessitate stopping the drug. The Spinal Osteoporosis Therapeutic Intervention (SOTI) trial4 recruited 1649 postmenopausal women with osteoporosis. Strontium ranelate (2 g/day) was associated with a risk reduction of new vertebral fractures of 49% during the rst year, and 41% over three years. Vertebral BMD increased by 14.4% and hip BMD by 8.3%. The Treatment Of Peripheral Osteoporosis Study (TROPOS)5 recruited 5091 women. Overall, strontium treatment decreased risk of non-vertebral fracture by 16% (relative risk = 0.84), while risk of hip fracture was decreased by 36% in high-risk individuals. A recent Cochrane review has examined evidence from four trials6 and conrms that strontium increases BMD (see Box 42.1) and prevents both vertebral and non-vertebral fracture.

Box 42.1 Strontium ranelate and BMD Strontium has a higher atomic weight than calcium and thus has a disproportionate effect on bone mineral content and BMD, which are therefore overestimated. A 1% increase in strontium can increase BMD by as much as 10%. In patients treated with the drug there is a relatively weak correlation between increases in BMD and fracture protection.

42 Osteoporosis drugs other than bisphosphonates

229

There was a suggestion of increased venous thrombosis and pulmonary embolism, and of nervous system problems including headache, seizures and memory loss.

Teriparatide
Teriparatide (recombinant human parathyroid hormone [PTH] 134) differs from other available agents in that it predominantly stimulates bone formation. Evidence supports its use in both men and women with osteoporosis, in patients with corticosteroidinduced osteoporosis and, particularly, in those at very high risk of fracture.7 Hyperparathyroidism classically causes increased bone turnover with hypercalcaemia. The reasons why intermittent dosing with PTH stimulates net bone formation are incompletely understood. There is evidence that intermittent PTH dosing stimulates expression of a range of genes in the osteoblast including growth factors (transforming growth factor [TGF]-b, epidermal growth factor [EGF], amphiregulin), cell-signalling molecules and MCP-1 (monocyte chemoattractant protein-1). Teriparatide increases osteoblast number and activity, increases the rate of bone remodelling and increases both trabecular thickness and connectivity. The increase in bone turnover is more marked in the rst twelve months of treatment and tails off thereafter. The effect is more marked on trabecular bone than cortical bone. Typically, vertebral BMD increases by about 10% over 1218 months, while femoral neck BMD increases by about 5%. Use of teriparatide decreases vertebral fracture rate by around 65% and non-vertebral fracture rate by 50%. Its effect on fracture reduction is clearly related to increased BMD, although the two are imperfectly correlated. The drug is given subcutaneously at a daily dose of 20 mg and is licensed to be given for up to two years. Antibodies to the molecule develop in about 3% of cases but do not appear to be clinically signicant. Most patients have an increase in plasma calcium, usually within the normal range, but hypercalcaemia has been noted in up to 3% of patients. Plasma calcium, 25-hydroxy-vitamin D, PTH and renal function should be checked at baseline. Calcium and renal function should be checked after a month of treatment. Teriparatide increases uric acid levels and should only be given with caution in patients who are at risk of gout. The most worrying potential side effect is the development of osteosarcoma. This has been described with prolonged usage in animals, and occasional human cases have been recognized. Teriparatide should not, therefore, be given to patients with an unexplained high level of bone alkaline phosphatase, patients who have had previous bone irradiation or to patients with Pagets disease. Recommended practice is to warn patients starting the drug about this potential adverse reaction. There is now reasonable evidence that prior or concurrent exposure to bisphosphonates blunts the effect of teriparatide. Since teriparatide is often given to patients whose osteoporosis continues to progress in spite of treatment with rst-line drugs, many patients will have been exposed to bisphosphonates. It is suggested to stop the latter drugs before treatment with teriparatide. It is justied to restart the bisphosphonate once teriparatide treatment is complete. During treatment, calcium intake (including supplements) should be kept to 1500 mg or less per day and vitamin D intake should be no more than 100 International Units per day. Teriparatide is safe and effective in the elderly and its benets do not depend on baseline BMD.

Combination treatments
Whether combinations of the currently available treatments are useful or desirable is controversial. There is considerable concern that a very marked decrease in bone turnover

230

07 Bone Diseases may lead to increased brittleness of bone. Short-term data (twelve months) show that raloxifene combined with alendronate increases hip and spine BMD and decreases bone turnover markers. It is not known whether these changes translate into decreased fracture risk. It is clear that bisphosphonates should not be combined with teriparatide. Recent data suggest that combination of raloxifene and teriparatide may increase BMD gain at the hip,8 while teriparatide with HRT has also been reported to be synergistic.9 In general, data on combination treatments have proved disappointing, in spite of the fact that combining an anabolic agent with an inhibitor of bone turnover appears attractive. Sequential treatments appear to be a much more viable option. One approach is to use teriparatide relatively early to increase bone formation. The next phase is to use a bisphosphonate to decrease bone turnover, following which the patient is left treatment-free for a period, and then the process is repeated. This approach has been called ADFR (Activate, Decrease osteoclast activity, Free of treatment and Repeat). Osteoporosis is a lifelong condition and we need effective strategies for its long-term management.

Recent Developments
1 Third-generation SERMs are being developed. The ideal drug would protect bone and the cardiovascular system without risk of thromboembolism. Amongst the drugs being developed are lasofoxifene, arzofoxifene and bazedoxifene. Lasofoxifene shows promise as a bone-protective agent, increasing bone density and bone strength in animal studies.10 2 An analysis of patients treated with strontium ranelate in two previous multinational studies has conrmed that the drug was effective in preventing fractures independently of the baseline characteristics of the patient.11 Neither the baseline BMD nor the presence or number of fractures appeared to inuence the subsequent response to strontium. 3 A costbenet model has been used to assess the benet of teriparatide.12 The greatest costbenet ratio was when the drug was used in more recent-onset disease. The cost per quality-adjusted life year (QALY) gained was 20 000 Euros in patients with a recent vertebral fracture, and 64 000 Euros in patients with previous vertebral fracture. This study emphasizes the fact that many treatments for osteoporosis, and other chronic diseases, are often used too late in the course of the disease.

Conclusion
The majority of osteoporotic patients respond well to bisphosphonates, but a signicant proportion shows no or little response. Other options for treating osteoporosis include SERMs, strontium ranelate and teriparatide. These should be considered for patients who do not tolerate or respond to bisphosphonates, and also for patients already treated with bisphosphonate but who require ongoing treatment. The available options are summarized in Figure 42.1. In general, combination treatments are often of limited benet. Bisphosphonates and teriparatide should not be given together. There is some limited evidence favouring a combination of bisphosphonates with oestrogen or SERMs.

42 Osteoporosis drugs other than bisphosphonates

231

Calcium and vitamin D Perimenopausal to prevent OP Elderly (many are vitamin D deficient) No limit to duration Can be used with other agents

Oestrogen Only if menopausal symptoms Early years after menopause Maximum 5 years

Raloxifene Generally 65 years Especially for vertebral OP

Strontium Anabolic and antiresorptive Established general OP (with or without fracture) Use for 3 years (based on available trials)

Bisphosphonate Established OP (especially with fracture) Use up to 5 years All ages

Teriparatide Anabolic ( antiresorptive) Use if high risk of fracture Limit use to 18 months Caution with other agents ? Use before bisphosphonate because of anabolic action

Figure 42.1 Drugs for the treatment of post-menopausal osteoporosis (OP).

Further Reading
1 Seeman E, Crans GG, Diez-Perez A, Pinette KV, Delmas PD. Anti-vertebral fracture efcacy of raloxifene: a meta-analysis. Osteoporos Int 2006; 17: 31316. 2 Siris ES, Harris ST, Eastell R et al. Skeletal effects of raloxifene after 8 years: results from the continuing outcomes relevant to Evista (CORE) study. J Bone Miner Res 2005; 20: 151424.

232

07 Bone Diseases
3 Ensrud K, Genazzani AR, Geiger MJ et al. Effect of raloxifene on cardiovascular adverse events in postmenopausal women with osteoporosis. Am J Cardiol 2006; 97: 5207. 4 Meunier PJ, Roux C, Seeman E et al. The effects of strontium ranelate on the risk of vertebral fracture in women with postmenopausal osteoporosis. New Engl J Med 2004; 350: 45968. 5 Reginster JY, Seeman E, De Vernejoul MC et al. Strontium ranelate reduces the risk of nonvertebral fractures in postmenopausal women with osteoporosis: treatment of peripheral osteoporosis (TROPOS) study. J Clin Endocrinol Metab 2005; 90: 281622. 6 ODonnell S, Cranney A, Wells GA, Adachi JD, Reginster JY. Strontium ranelate for preventing and treating postmenopausal osteoporosis. Cochrane Database Syst Rev 2006; CD005326. 7 Hodsman AB, Bauer DC, Dempster DW et al. Parathyroid hormone and teriparatide for the treatment of osteoporosis: a review of the evidence and suggested clinical guidelines for its use. Endocr Rev 2005; 26: 688703. 8 Deal C, Omizo M, Schwartz EN et al. Combination teriparatide and raloxifene therapy for postmenopausal osteoporosis: results from a 6-month double-blind placebo-controlled trial. J Bone Miner Res 2005; 20: 190511. 9 Ste-Marie LG, Schwartz SL, Hossain A, Desaiah D, Gaich GA. Effect of teriparatide (rhPTH(1-34)) on BMD when given to postmenopausal women receiving hormone replacement therapy. J Bone Miner Res 2006; 21: 28391. 10 Ke HZ, Foley GR, Simmons HA, Shen V, Thompson DD. Long-term treatment of lasofoxifene preserves bone mass and bone strength and does not adversely affect the uterus in ovariectomized rats. Endocrinology 2004; 145: 19962005. 11 Roux C, Reginster JY, Fechtenbaum J et al. Vertebral fracture risk reduction with strontium ranelate in women with postmenopausal osteoporosis is independent of baseline risk factors. J Bone Miner Res 2006; 21: 53642. 12 Lundkvist J, Johnell O, Cooper C, Sykes D. Economic evaluation of parathyroid hormone (PTH) in the treatment of osteoporosis in postmenopausal women. Osteoporos Int 2006; 17: 20111.

43 Male osteoporosis P R O B L E M

233

43 Male Osteoporosis

Case History
A 60-year-old man presents with back pain. X-ray of his spine shows a crush fracture of L2 and a generalized decrease in bone mineral density (BMD). A subsequent dual-energy X-ray absorptiometry (DEXA) scan conrms that he has osteoporosis with T scores of 3.2 for the lumbar spine and 2.6 for the hip. His serum testosterone is 7.2 nmol/l (normal range 930 nmol/l). What are the causative factors in male osteoporosis? What investigations should routinely be carried out? What treatment options are proven to work?

Background
Although osteoporosis is predominantly a disease affecting females, it should be remembered that 20% of fractures, and 30% of hip fractures, occur in males. One in three men over the age of 60 years will suffer a fracture. Osteoporosis in men is more likely to present with a fragility fracture, while many cases in women are diagnosed from screening. Morbidity and mortality following fracture is poorer in men than in women. For example, 80% of men do not return to their pre-fracture physical function after hip fracture and as many as 50% will not return to an independent existence. Underlying causes are more commonly discovered in men, with glucocorticoid excess (mostly iatrogenic) occurring in 20% of cases, heavy alcohol intake in 15%20% and hypogonadism in 15%20%. Other secondary causes such as hyperthyroidism, hyperparathyroidism, malabsorption, antiepileptic treatment and multiple myeloma should be considered as for female osteoporosis (Box 43.1). Biological determinants of bone density in men include activity of the growth hormone/insulin-like growth factor (IGF)-1 axis, decreasing testosterone with age and local oestrogen production and action. With regard to the Box 43.1 Indications for DEXA scanning in men b b b b Low trauma fracture Hypogonadism Glucocorticoid treatment or excess Osteopenia on X-ray b Undernutrition b Hyperparathyroidism b Prolonged immobilization

Atlas Medical Publishing Ltd

234

07 Bone Diseases latter, polymorphisms of both the aromatase gene (responsible for peripheral conversion of androgens to oestrogen) and the oestrogen receptor may be important. Vitamin D status and parathyroid hormone (PTH) activity are also important, and change with age. The data on benets of calcium and vitamin D supplementation are variable, and may relate to the background status of the population being studied.1 Patients who are decient in calcium and vitamin D are more likely to respond to supplementation. The prevalence of vitamin D deciency increases with age, partly because of decreased dietary intake and reduced sunlight exposure. The recommended daily intake of calcium for men under 65 years is 1000 mg, while for men over 65 years it is 1500 mg. Calorie intake generally decreases with age, and the average diet contains less than 400 mg calcium per 1000 kcal. The recommended minimum vitamin D intake is 400 International Units (IU) per day for men under 70 years and 600 IU/day for men over 70. Supplementation is indicated for individuals whose calcium or vitamin D intakes fall below, or are in danger of falling below, these thresholds. This will not necessarily recover lost bone density in patients with established osteoporosis. The mainstay of treatment for patients with established disease is bisphosphonates (BPs). BPs should be used with calcium and vitamin D supplementation where necessary. Currently, three oral BPs are used for male osteoporosis. Alendronate was the rst of the newer generation BPs to be studied. For example, in the study by Gonnelli et al.,2 alendronate treatment of osteoporotic men increased spine BMD by 4.2% at one year, 6.3% at two years and 8.8% at three years. Corresponding increases at the hip were 1.6%, 2.9% and 3.9%, respectively. There was a parallel increase in bone quality as assessed by quantitative ultrasound at the heel. The drug protects against fractures at both vertebral and non-vertebral sites. There is every reason to believe that two other BPs risedronate and ibandronate are equally safe and effective. Alendronate (Fosamax) can be given in daily (10 mg) or weekly (70 mg) dosing schedules and should, like the other bisphosphonate drugs, be taken on an empty stomach with the patient upright, and the tablet should be swallowed with a full glass of water. Alendronate has not only been shown to be clinically effective, but the cost-effectiveness of treatment has also been demonstrated.3 Risedronate (Actonel) may also be given daily (5 mg) or weekly (35 mg) and has effects comparable to alendronate. The newer agent ibandronate (Bonviva) can be given as a single monthly dose of 150 mg. For patients who have gastrointestinal side effects from BPs, pamidronate or ibandronate can be administered parenterally. For patients who either cannot tolerate or do not respond to BPs, treatment with PTH (teriparatide; Forteo) may be indicated. This is given for up to 18 months as a daily subcutaneous injection. Patients should be monitored for hypercalcaemia. The long-term benet and safety of teriparatide have yet to be established. From animal studies there is a theoretical risk of osteosarcoma. Administration of testosterone to hypogonadal men with osteoporosis increases BMD and protects against fracture. Testosterone can be administered orally, transdermally (with either gel or patches), by depot intramuscular injection (every 24 weeks) or using implants that are usually inserted every six months. Testosterone treatment of hypogonadal men also improves muscle bulk and function, improves cardiovascular function and protects against coronary artery disease, as well as generally improving quality of life. Sex-steroid levels tend to decrease with age in normal men. This decrease is more marked in some men than in others. The term partial androgen deciency in aging men

43 Male osteoporosis

235

(PADAM) has been proposed, and it is not clear whether this physiological change with aging merits treatment, particularly in the face of markers of lower androgen status such as decreasing BMD. There is certainly now some evidence that, in the short to medium term, androgen replacement can reverse some of the biological changes that accompany declining androgen status. Risks of androgen therapy include changes in liver function, polycythaemia and predisposition to prostate and breast cancer. In the study by Gennari et al.,4 bone density and sex-steroid status were monitored over four years in 200 aging men. Androgen and oestrogen status both decreased with time, and those with the greatest decrease also had the greatest loss of BMD. The ratio of testosterone to oestradiol, a measure of aromatase activity, varied widely and patients with the lowest estimated levels of aromatase lost bone faster. The role of oestrogen status in regulating bone loss in aging men is controversial; Lormeau et al.5 found no relationship between oestrogen levels and bone loss in aging men. They did, however, report that sex hormone binding globulin (SHBG) levels were related to bone loss, with the highest levels being associated with the fastest bone loss. SHBG tends to increase with age, and by binding free androgen and oestrogen may decrease free, bioavailable levels of sex steroids.

Recent Developments
1 Assessment of vitamin D status is usually by measurement of total circulating 25-hydroxy-vitamin D3 (25[OH]D3). Levels of vitamin D-binding protein are variable and may increase with age. This increase may effectively decrease the amount of free and bioavailable vitamin D. In a recent study,6 25(OH)D3 levels were no different in men with osteoporosis compared with controls. However, levels of vitamin D-binding protein were increased in osteoporotic men and the concentration of 1,25-dihydroxy-vitamin D3 (1,25[OH]D3) was decreased. 2 Androgen deprivation therapy for patients with prostate cancer is now a leading cause of osteoporosis in men.7,8 In recent years, this therapy has become much more effective at decreasing prostatic exposure to androgen, but therefore places men at risk of osteoporosis. BMD loss of up to 8% per year at the spine and 6% per year at the hip is common, and up to 20% of men who survive ve years or more will suffer a fracture. Bone loss can be prevented with BPs but it is not clear whether this translates into decreased fracture risk.

Conclusion
Osteoporosis in males is more likely to have an underlying cause. The most common of these are excessive alcohol intake, use of glucocorticoids and hypogonadism. The diagnostic workup (Figure 43.1) should always include measurement of calcium and vitamin D, serum testosterone, SHBG, luteinizing hormone (LH) and follicle-stimulating hormone (FSH). Even mild calcium and vitamin D deciency should be corrected. At present, testosterone replacement should be reserved for patients with proven hypogonadism and a precise diagnosis should be established in all cases. The mainstay of treatment is bisphosphonate therapy. Alendronate, risedronate and ibandronate are widely used and are probably of comparable efcacy and safety. Teriparatide should be considered for patients with severe or unresponsive disease.

236

07 Bone Diseases

Fragility facture Osteopenia on X-ray High clinical risk

Measure BMD Osteoporosis confirmed Measure calcium, 25(OH)D3, PTH Urinary deoxypyridinolone* Testosterone, SHBG, LH and FSH

Exclude secondary causes Excessive alcohol Glucocorticoids Hypogonadism

Assess calcium and vitamin D status Use supplements if necessary

Bisphosphonate treatment

Good response Follow-up including annual BMD

Poor response Alternative bisphosphonate or teriparatide

Figure 43.1 Diagnosis and management of male osteoporosis. * Measurement of markers of bone turnover may help to predict response to treatment, as they respond quicker than BMD. BMD, bone mineral density; FSH, follicle-stimulating hormone; LH, luteinizing hormone; PTH, parathyroid hormone; SHBG, sex hormone binding globulin.

Further Reading
1 2 3 Kamel HK. Male osteoporosis: new trends in diagnosis and therapy. Drugs Aging 2005; 22: 7418. Gonnelli S, Cepollaro C, Montagnani A et al. Alendronate treatment in men with primary osteoporosis: a three-year longitudinal study. Calcif Tissue Int 2003; 73: 1339. Borgstrm F, Johnell O, Jnsson B, Zethraeus N, Sen SS. Cost effectiveness of alendronate for the treatment of male osteoporosis in Sweden. Bone 2004; 34: 106471.

44 Glucocorticoid-induced osteoporosis
4 5

237

Gennari L, Merlotti D, Martini G et al. Longitudinal association between sex hormone levels, bone loss, and bone turnover in elderly men. J Clin Endocrinol Metab 2003; 88: 532733. Lormeau C, Soudan B, dHerbomez M, Pigny P, Duquesnoy B, Cortet B. Sex hormonebinding globulin, estradiol, and bone turnover markers in male osteoporosis. Bone 2004; 34: 9339. Al-oanzi ZH, Tuck SP, Raj N et al. Assessment of vitamin D status in male osteoporosis. Clin Chem 2006; 52: 24854. Gilbert SM, McKiernan JM. Epidemiology of male osteoporosis and prostate cancer. Curr Opin Urol 2005; 15: 237. Holmes-Walker DJ, Woo H, Gurney H, Do VT, Chipps DR. Maintaining bone health in patients with prostate cancer. Med J Aust 2006; 184: 1769.

6 7 8

P R O B L E M

44 Glucocorticoid-Induced Osteoporosis
Case History
Mrs SP is a 45-year-old woman who has suffered asthma since childhood. The asthma is now stable, but she is concerned about osteoporosis since she has had an average of three courses of steroids each year for the past ten years. She has regular periods but she is approaching the menopause. Her mother developed osteoporosis in later life. A dualenergy X-ray absorptiometry (DEXA) scan conrms that she has low bone mineral density (BMD). What are the causes of steroid-induced osteoporosis? How might the condition be prevented? What treatment options are available?

Background
Overall, 0.5%0.9% of the population requires intermittent or continuous treatment with steroids. Up to 2.5% of the elderly population (age >70 years) take corticosteroids. In addition to the risks imposed by steroids, some of the underlying conditions requiring steroids also predispose to osteoporosis (e.g. rheumatoid arthritis and multiple Atlas Medical Publishing Ltd

238

07 Bone Diseases myeloma). The pathogenesis of glucocorticoid-induced osteoporosis is complex,1,2 occurring in two phases with rapid bone loss of up to 12% of total BMD within the rst year followed by a slower rate of bone loss of typically up to 3% per year. Corticosteroids decrease the formation of osteoblasts and markedly increase (by up to three-fold) apoptosis of osteoblasts. The early phase with increased bone loss is also characterized by increased osteoclastic activity. Decreased osteoprotegerin (OPG) may be partly responsible for this effect. OPG acts as a soluble decoy receptor for the receptor activator of nuclear factor-k-beta ligand (RANKL), and decreased activity of OPG allows increased interaction between RANKL and its native receptor (RANK) leading to activation of the pathway that includes nuclear factor-k-beta (NF-kB). Other mechanisms involved include decreased expression of bone morphogenetic proteins and a diversion of bone marrow precursor cells into the adipocyte lineage. Use of glucocorticoids is the most prevalent cause of secondary osteoporosis. In spite of the widely known detrimental effect of steroids, perhaps only as few as 15% of patients commencing steroids have investigations to exclude osteoporosis or treatment to prevent loss of BMD. Patients with glucocorticoid-induced osteoporosis have a greater risk of fracture than other subjects with comparable BMD. It has, therefore, been suggested that the threshold for intervention should be lower and a T score of 1.5 is widely used. Between 30% and 50% of patients taking long-term steroids will suffer a fragility fracture. The equivalent of prednisolone 10 mg/day for six months or more increases the risk of hip fracture seven-fold and the risk of vertebral fracture 17-fold. Investigation and management is summarized in Figure 44.1. As with all osteoporosis patients, attention should be paid to lifestyle factors, including calcium and vitamin D status, and other potential secondary causes. The patient should be taking 1500 mg of calcium per day and 800 International Units (IU) per day of vitamin D. If it does not seem that these levels of intake are being achieved, supplements should be prescribed. At all times efforts should be made to minimize the exposure to glucocorticoids and to withdraw them completely where possible. There is evidence for recovery of bone density when steroid treatment is withdrawn. Of the available treatments,3,4 the greatest evidence by far is with use of bisphosphonates (BPs). Glucocorticoids impair calcium absorption from the gut and reabsorption in the kidney. Calcium supplementation is therefore indicated in many cases but will not prevent the rapid loss of BMD in the months after steroid therapy is initiated. Vitamin D is more effective and may be particularly indicated in older people. Cholecalciferol at doses between 300 IU/day and 100 000 IU/week helps to minimize the effect of glucocorticoids; vitamin D analogues such as dihydrotachysterol, calcitriol and alfacalcidol are at least as effective. However, no vitamin D analogue or dose is as effective as BPs. Sexsteroid replacement is controversial and relatively short-term use only is recommended for women with menopausal symptoms. Testosterone replacement should be prescribed for men with proven hypogonadism. Current evidence does not favour the use of calcitonin for treatment of glucocorticoid-induced osteoporosis. Alendronate and risedronate are now the most commonly used drugs for prevention and treatment of osteoporosis caused by steroids. Both appear to be effective in at least preserving hip and spine BMD, and there are reasonable data showing that they protect against vertebral and non-vertebral fracture. Intravenous pamidronate may be used where oral drugs are not tolerated or where compliance is poor. Etidronate is now sel-

44 Glucocorticoid-induced osteoporosis

239

Is dose 5 mg prednisolone or equivalent? Is treatment of 3 months or more likely?

Patient at risk

Lifestyle measures Stop smoking Weight-bearing exercise Limit alcohol intake

Minimize steroid Lowest possible dose Topical steroids Other immunosuppressives Alternate day dose regimen

Check calcium and vitamin D status (supplement if necessary)

DEXA scan

T score

1.5

T score

1.5

Alendronate or risedronate

Repeat DEXA in 12 months

Repeat DEXA in 12 months

Figure 44.1 Investigation and management of glucocorticoid-induced osteoporosis. DEXA, dual-energy X-ray absorptiometry.

dom used and is much less potent than second-generation BPs. Teriparatide (parathyroid hormone) increases BMD both at vertebral and other sites, but it is not known whether it is effective in treatment of glucocorticoid-induced osteoporosis.

240

07 Bone Diseases

Recent Developments
1 A number of new, potential therapeutic targets have been identied for primary and glucocorticoid-induced osteoporosis.5 These include sclerostin, a bone morphogenetic protein produced by osteocytes. Deciency of this protein is associated with high BMD. The RANKRANKLOPG pathway is central to osteoclast recruitment. Attempts to manipulate activity of this pathway in animal models include use of a monoclonal antibody to RANKL. CD40 and its ligand belong to the tumour necrosis factor (TNF) superfamily and are involved in apoptosis of bone cells. Inhibition of osteoblast apoptosis may increase bone formation. 2 Agents with improved anti-inammatory and immunosuppressive activity without steroid side effects would offer a major advantage.6,7 The anti-inammatory effects of steroids are mediated through inhibition of the NF-kB and activator protein-1 (AP-1) pathways, while side effects occur through transcriptional modication of a variety of genes. The search for new drugs includes agents with selective glucocorticoid receptor-modulator activity. 3 In a recent study,8 201 patients who were about to start steroids at doses equivalent to prednisolone 7.5 mg/day or greater were randomly assigned to receive alendronate 10 mg/day with an alfacalcidol placebo or alfacalcidol at a dose of 1 mg/day. Patients were followed for 18 months. BMD of the lumbar spine increased by 2.1% in those treated with alendronate, while it decreased by 1.9% in the alfacalcidol-treated group. There was a similar disparity in bone density at the hip.

Conclusion
Use of glucocorticoids is associated with a phase of rapid bone loss due to excessive bone absorption over formation. This is followed by a slower phase of bone loss due mainly to decreased bone formation resulting from decreased recruitment and increased apoptosis of osteoblasts. Prevention should be by minimizing other risk factors for osteoporosis and, where possible, eliminating other secondary causes. Exposure to steroids should be minimized, and combination with other immunosuppressive agents should be considered where prolonged therapy is needed. Adequate calcium and vitamin D status should be ensured. The bisphosphonates alendronate and risedronate are currently the mainstay of treatment but are grossly underused in clinical practice.

Further Reading
1 2 3 4 Weinstein RS. Glucocorticoid-induced osteoporosis. Rev Endocr Metab Disord 2001; 2: 6573. Canalis E, Bilezikian JP, Angeli A, Giustina A. Perspectives on glucocorticoid-induced osteoporosis. Bone 2004; 34: 5938. Orcel P. Prevention and treatment of glucocorticoid-induced osteoporosis in 2005. Joint Bone Spine 2005; 72: 4615. Devogelaer J-P, Goemaere S, Boonen S et al. Evidence-based guidelines for the prevention of glucocorticoid-induced osteoporosis: a consensus document of the Belgian Bone Club. Osteoporos Int 2006; 17: 819.

45 Pagets disease of bone


5 6 7 8 Mazziotti G, Angeli A, Bilezikian JP, Canalis E, Giustina A. Glucocorticoid-induced osteoporosis: an update. Trends Endocrinol Metab 2006; 17: 1449.

241

Rhen T, Cidlowski JA. Antiinammatory action of glucocorticoids new mechanisms for old drugs. New Engl J Med 2005; 353: 171123. Rosen J, Miner JN. The search for safer glucocorticoid receptor ligands. Endocr Rev 2005; 26: 45264. de Nijs RNJ, Jacobs JWG, Lems WF et al. Alendronate or alfacalcidol in glucocorticoidinduced osteporosis. New Engl J Med 2006; 355: 67584.

P R O B L E M

45 Pagets Disease of Bone

Case History
Mr JS is a generally t 74-year-old man. He complains of increasing pain in his back and in his left shin. The latter feels warm but is not deformed. A scintigraphic bone scan shows increased uptake at several areas in the spine and in his left shin. There is no evidence of malignant disease. His plasma calcium and phosphate levels are normal, but alkaline phosphatase is increased at 420 units/l (normal = up to 120 units/l). How should he be investigated and followed up? What is the current thinking on the aetiology of Pagets disease? When should treatment be considered, and what is the best treatment?

Background
First described as osteitis deformans by Sir James Paget in 1877, Pagets disease is a chronic, focal disorder of bone where increased areas of marked bone turnover lead to pain, local bone expansion, deformity and risk of fracture.1,2 It most commonly affects the skull, clavicles, vertebrae, pelvis, femur and tibia. Pagets disease is relatively common and may affect up to 1.5% of the population aged 40 years and over, 5% of those aged 55 years and over and as many as 20% of 85-year-olds. It is particularly common in subjects of Northern European descent and thus relatively common in the United Kingdom, Atlas Medical Publishing Ltd

242

07 Bone Diseases North America, Western Europe and Australia. The prevalence may have decreased by up to 50% in the past 20 years. The vast majority of cases are asymptomatic. The cause of Pagets disease is not known, but it is likely that both genetic and environmental factors are important. Up to 20% of patients have a history of Pagets disease in one or more rst-degree relatives. A high proportion of families with Pagets disease and up to 15% of sporadic cases have mutations in the sequestosome 1/p62 gene, which is involved in the activation of osteoclasts. Infectious agents have been implicated and could account for some of the geographical variation in prevalence. Amongst candidate agents are paramyxoviruses, measles, respiratory syncytial virus and canine distemper virus. The disease typically presents with aching bone pain, which may be worse on weight bearing. Occasionally it presents with deformity or pathological fracture. Many cases are discovered incidentally because of radiological features or increased serum alkaline phosphatase. Complications of Pagets disease are relatively uncommon but important to recognize (Box 45.1). The most common complication is deafness due to involvement of the petrous temporal bone. Box 45.1 Complications of Pagets disease b b b b b b b b Hearing loss involvement of petrous temporal bone Osteogenic sarcoma <1% of cases Pathological fracture Cranial neuropathies Entrapment neuropathies and radiculopathies Spinal stenosis Cardiac failure very rare Hypercalcaemia with prolonged immobility

X-ray of painful or deformed sites should be undertaken, but systematic skeletal survey is not usually indicated. The initial changes are of focal lysis leading to areas with mixed lysis and sclerosis, trabecular expansion, thickening of cortical bone and deformity. Cortical ssure fractures may be identied and may coincide with sites of pain. Metastatic disease may be considered in the differential diagnosis but it is unusual for bone biopsy to be required. Skeletal scintigraphy is much more sensitive than plain radiology but it is also less specic. Widespread disease may be apparent, even in patients who are relatively asymptomatic. The most useful biochemical marker is serum alkaline phosphatase, although it is only increased in 85% of patients. The level of alkaline phosphatase is directly related to activity and to extent of the disease. It is frequently normal in patients with monostotic disease. Measurement of bone-specic alkaline phosphatase may be useful where total alkaline phosphatase is normal or in the presence of liver disease. Other markers of bone turnover may also be useful, particularly in monitoring the early response to therapy. Measurement of urinary deoxypyridinoline or the cross-linked N-telopeptide of type 1 collagen is widely used. Investigation and treatment are summarized in Figure 45.1. Often, supportive measures such as physiotherapy and use of aids such as a walking stick may sufce. For those who require specic treatment, bisphosphonates (BPs) are the mainstay. Indications for treatment include severe pain, extensive disease (particularly if major neurological or

45 Pagets disease of bone

243

vascular structures are at risk), neurological complications, hypercalcaemia resulting from prolonged immobility and where the disease is affecting the site of proposed surgery (such as a hip or knee replacement). Available BPs are summarized in Box 45.2. Etidronate and tiludronate are now seldom used. Alendronate at doses up to 40 mg/day for up to six months or risedronate 30 mg/day for two to six months are equally effective. Intravenous pamidronate can be given for patients who are at risk of gastrointestinal side effects from BPs. A variety of regimens have been used, ranging from a single dose of 3060 mg, to three doses of 30 mg given on consecutive days, or 3060 mg given once per week for up to six weeks. Serum alkaline phosphatase decreases with modern bisphosphonate treatment in the vast majority of cases. Patients are usually followed up at threemonthly intervals and treatment is repeated after six months if there is recurrence of symptoms or if the alkaline phosphatase increases again. BPs are contraindicated when the glomerular ltration rate is less than 35 ml/min/1.73 m2. The newer BPs zoledronic acid and ibandronate may be used. Salmon calcitonin is still useful in some cases, particularly when BPs are either contraindicated or poorly tolerated, or as an add-on therapy when there is severe pain. Calcitonin can be given subcutaneously at doses of 50100 IU/day or on alternate days for up to 18 months. The chemotherapeutic agent plicamycin is now seldom used. Box 45.2 Bisphosphonates for Pagets disease Etidronate Tiludronate Alendronate Risedronate Pamidronate 200400 mg/day 400 mg/day 1040 mg/day 30 mg/day 3060 mg 6 months 6 months 36 months 26 months 16 doses

Recent Developments
1 Osteoclast differentiation is known to be regulated by members of the tumour necrosis factor superfamily, including the receptor activator of nuclear factor-k-beta ligand (RANKL) and osteoprotegerin (OPG). Deactivating mutations to the latter gene (TNFRSF11B) are known to be responsible for most cases of juvenile Pagets disease.3 Mutation of the sequestosome 1 (SQSTM1) gene on the long arm of chromosome 5 is the most consistently identied genetic abnormality in adult Pagets disease. An argument for genetic screening of patients who have or are at risk of Pagets disease has been advanced.4 2 There appears to be very little to choose between the currently available potent BPs in terms of efcacy.5,6 Alendronate, risedronate and pamidronate have very similar effects in inducing remission, although use of a different agent may be justied if there is no response or limited response to the rst agent. 3 Zoledronic acid, a third-generation bisphosphonate, is widely used in the treatment of hypercalcaemia of malignancy. Recent evidence suggests that a single infusion of this agent is superior to current bisphosphonate regimens.7,8 However, as might be expected, side effects may also be more common with more potent agents. These side

244

07 Bone Diseases

Bone pain Deformity Increased AP

X-ray site of symptoms

Measure AP

Provisional diagnosis of Pagets PTH exclude hyperparathyroidism 25(OH)D exclude deficiency

Bone-specific AP Urinary deoxypyridinoline

Bone scintigraphy

Consider need for treatment

Analgesia (NSAIDs) Aids (physiotherapy)

Alendronate or risedronate (IV pamidronate if poorly compliant or GI intolerance)

Review with AP or other bone markers 3 monthly

Repeat treatment if recurrent symptoms or increasing AP

Calcitonin if no response to or does not tolerate bisphosphonate

Figure 45.1 Investigation and management of Pagets disease. 25(OH)D, 25-hydroxy-vitamin D; AP, alkaline phosphatase; GI, gastrointestinal; NSAIDs, non-steroidal anti-inammatory drugs; PTH, parathyroid hormone.

45 Pagets disease of bone

245

effects include hypocalcaemia (often transient) and increases in serum creatinine. Apart from potency in inducing remission, use of potent agents such as zoledronic acid may increase compliance and decrease the cost of administering and monitoring therapy. 4 Patients with Pagets disease have a higher rate of comorbidities than matched controls and the disease is costly, with increased costs particularly relating to physician visits and diagnostic tests.9 Compared with matched controls, patients with Pagets disease had increased rates of pathological fracture (4.9% vs 0.4%), heart murmurs (3.3% vs 0.4%), low back pain (19.7% vs 8.6%) and hearing loss (13.5% vs 5.7%).

Conclusion
The diagnosis of Pagets disease generally relies on the nding of increased serum alkaline phosphatase, along with radiological features and focal areas of increased uptake on isotope bone scan. Other markers of bone turnover can be helpful. Bone biopsy is seldom required. Environmental factors are poorly characterized at present, while genetic markers are recognized for both juvenile and adult forms of Pagets disease. Treatment is generally not required. The most common indication for treatment is bone pain. Bisphosphonates are the mainstay of treatment and alkaline phosphatase is the most useful marker for success of treatment.

Further Reading
1 2 3 4 5 Whyte MP. Pagets disease of bone. New Engl J Med 2006; 355: 593600. Selby PL, Davie MWJ, Ralston SH, Stone MD. Guidelines on the management of Pagets disease of bone. Bone 2002; 31: 36673. Whyte MP. Pagets disease of bone and genetic disorders of RANKL/OPG/RANK/NF-kB signaling. Ann N Y Acad Sci 2006; 1068: 14364. Michou L, Collet C, Laplanche J-L, Orcel P, Cornlis F. Genetics of Pagets disease of bone. Joint Bone Spine 2006; 73: 2438. Walsh JP, Ward LC, Stewart GO et al. A randomized clinical trial comparing oral alendronate and intravenous pamidronate for the treatment of Pagets disease of bone. Bone 2004; 34: 74754. Rendina D, Mossetti G, Viceconti R, Sorrentino M, Nunziata V. Risedronate and pamidronate in the clinical management of patients with severe Pagets disease of bone and acquired resistance to bisphosphonates. Calcif Tissue Int 2004; 75: 18996. Hosking D. Pharmacological therapy of Pagets and other metabolic bone diseases. Bone 2006; 38 (2 Suppl 2): S37. Maricic M. The use of zoledronic acid for Pagets disease of bone. Curr Osteoporos Rep 2006; 4: 404. Briesacher BA, Orwig D, Seton M, Omar M, Kahler KH. Medical care costs of Pagets disease of bone in a privately insured population. Bone 2006; 38: 7317.

7 8 9

246

07 Bone Diseases

P R O B L E M

46 Bone Complications of Renal Disease

Case History
JR is a 64-year-old man with type 2 diabetes. His glycaemic control is reasonable with oral hypoglycaemic agents but blood pressure has been difcult to control. He currently takes an angiotensin-converting enzyme inhibitor, a diuretic and a calcium channel blocker for blood pressure control. Plasma creatinine is increased at 342 mmol/l. His parathyroid hormone (PTH) is marginally high and bone mineral density (BMD), measured using a dual-energy X-ray absorptiometry (DEXA) scan, is low (T score 2.4 for lumbar spine, 1.8 for hip). What other investigations would be useful? Can we be certain that he is developing osteoporosis? Is he a candidate for bisphosphonate therapy?

Background
Bone complications are common in chronic kidney disease (CKD). The bone complications of CKD are underdiagnosed and under-recognized. One of the reasons is that precise diagnosis has been based on bone biopsy ndings. Bone biopsy is not carried out frequently, except in specialist centres. The morphological changes in bone that occur with renal failure have been known as renal osteodystrophy (ROD). The syndrome is important because of the increasing prevalence of CKD, increasing recruitment of patients into dialysis programmes with consequent increase in lifespan for CKD patients, and also because it is accompanied by increased morbidity and premature mortality. The latter relates to vascular changes including ectopic calcication. A broader term CKDmineral and bone disorder (CKD-MBD) has been proposed and encompasses the many facets of this complication of CKD (Box 46.1). ROD can be classied according to whether bone turnover is low, normal or high; whether mineralization is normal or decreased; and whether bone volume is low, normal Box 46.1 Denition of CKD-MBD A constellation of bone disorders present or exacerbated by chronic kidney disease that lead to bone fragility and fractures, abnormal mineral metabolism and extraskeletal manifestations. Atlas Medical Publishing Ltd

46 Bone complications of renal disease

247

or high. Apart from the cellular component, bone comprises mainly organic matrix (osteoid) and inorganic matrix (mineral). Around 15% of bone turns over each year. Osteoid, secreted by the osteoblasts, consists mainly of type 1 collagen with proteoglycans and other, quantitatively, relatively minor proteins including bronectin and osteonectin. The inorganic component is mainly hydroxyapatite crystals. Apart from the amount of mineralized bone (which determines BMD) and intact microarchitecture of the bone, the correct balance of organic and inorganic components is essential for normal mechanical properties (strength, ability to absorb shock and deformability). PTH and levels of active vitamin D (1,25-dihydroxy-vitamin D3 [D3]; calcitriol) are the most important determinants of bone structure and calcium metabolism. PTH maintains plasma calcium by increasing intestinal absorption, increasing renal reabsorption and mobilizing calcium from bone. Excess PTH results in increased bone turnover. This affects both trabecular and cortical bone. More than 98% of active D3 is synthesized in the cells of the proximal renal tubule, and synthesis is decreased in patients with renal impairment. Synthesis is normally increased by PTH, and there is a negative feedback loop whereby D3 decreases PTH secretion. D3 increases calcium absorption from the gut and calcium mobilization from bone. The key measurements made on bone biopsy are bone volume, bone turnover (using double tetracycline labelling), osteoid tissue mineralization and metal deposition (particularly aluminium). This allows for diagnosis of the several forms of renal failure-related bone disease.1,2

1. High turnover bone disease (osteitis brosa cystica)


This is characteristic of PTH excess with increased osteoblast and osteoclast activity. The balance may be tipped towards either excess bone formation or excess breakdown. The former leads to excess osteoid and peritrabecular fibrosis, which may impair bone marrow formation. Excess bone breakdown may lead to the characteristic cystic change. Increased cortical porosity contributes to increased risk of fracture. The disorder is usually asymptomatic, and routine biochemistry is normal apart from increased PTH levels. Increased vitamin D (acting through the vitamin D receptor) and increased calcium (acting through the calcium-sensing receptor) are both powerful stimuli to decrease PTH secretion. Both responses are downregulated in secondary hyperparathyroidism and PTH secretion is inappropriately high. The trigger for this increase in PTH is probably low calcitriol levels. High levels of phosphate lead to calcium binding, with a consequent decrease in ionized calcium and thus increased PTH secretion. In addition, phosphate may directly increase PTH secretion and inhibits renal 1a-hydroxylase.

2. Defective mineralization (osteomalacia)


This is a direct consequence of suboptimal vitamin D status, although it is not usually severe enough to warrant a clinical diagnosis of osteomalacia. In the past, aluminium exposure was a major risk factor, and because this is less prevalent, pure osteomalacia is becoming less common in CKD.

3. Mixed ROD
Features of PTH excess are combined with defective mineralization due to the relative deciency of vitamin D.

248

07 Bone Diseases

4. Adynamic bone disease (ABD; low turnover state)


This is the opposite end of the spectrum from the state that exists with PTH excess, and is particularly common in the elderly and in those with diabetes. It occurs in up to 50% of biopsy specimens. ABD is most common where calcium and vitamin D therapy is excessive, following parathyroidectomy or with aluminium toxicity. There is decreased osteoid and very little evidence of either osteoblast or osteoclast activity, and no tetracycline labelling. ABD is associated with increased fracture risk, bone pain and ectopic calcication. Bone mineralization is normal in ABD. The relative rates of the different forms of bone disease are shown in Table 46.1.

Table 46.1 Distribution of ROD subtypes


Pre-dialysis (%)
High turnover Mixed Low turnover (osteomalacia + ABD) Normal histology 10 20 30 40

Dialysis (%)
0 65 35 0

Figures show the relative distribution of different types of ROD in pre-dialysis patients and in those undergoing dialysis. Normal histology very rarely occurs in the dialysis population. Adapted from Schwarz et al. 2006.1

Evaluation should include measurement of serum PTH, calcium (corrected for albumin concentration or ionized), phosphate, alkaline phosphatase and plasma bicarbonate, as well as imaging for soft tissue calcication. While precise diagnosis of ROD requires bone biopsy, there is probably limited justication for more widespread use of this test as it is invasive, ideally requires prolonged preparation of the patient with double tetracycline labelling and can only be interpreted in specialized centres. PTH is the single most useful biomarker and is increased where there is increased bone turnover. The place of markers of bone turnover in diagnosis is not established, although they may be useful to help follow the effects of treatment. Markers of bone formation include osteoprotegerin, total and bone-specic alkaline phosphatase, osteocalcin and procollagen type 1 carboxyterminal extension peptide. Markers of bone resorption include pyridinoline and deoxypyridinoline, tartrate-resistant acid phosphatase (TRAP) and procollagen type 1 cross-linked carboxy-terminal peptide. BMD measurements do not correlate perfectly with fracture risk. Distal radius is the preferred site for BMD measurement in CKD. The major concern is that low BMD does not necessarily indicate osteoporosis and therefore the need for antiresorptive therapy. Bone biopsy is indicated when biochemical features are difcult to interpret, when fracture or bone pain are prominent and cannot be explained by non-invasive investigations, where aluminium toxicity is suspected and where there is severe and progressive vascular calcication. Bone histomorphometry uses measures of turnover, mineralization and volume to classify ROD the TMV classication. Biopsy is usually undertaken at the iliac crest. Use of two doses of tetracycline given four weeks apart prior to biopsy allows for assessment of bone formation and mineralization rate.

46 Bone complications of renal disease

249

In general, the metabolic abnormalities that lead to altered bone morphology and vascular calcication do not occur until glomerular ltration rate (GFR) has decreased below 60 ml/min/1.73 m2. However, these bone and vascular changes occur in children and young adults with more modest decreases in GFR (to <90 ml/min/1.73 m2). Caution should be exercised in diagnosing pure osteoporosis in a patient who has renal impairment. For routine screening, vascular calcication is best detected on lateral abdominal plain X-rays looking for aortic calcication. Computed tomography (CT) scans give

Low GFR

Measure: Calcium (corrected total and ionized) Phosphate Vitamin D (25[OH]D and 1,25[OH]2D) PTH

Assess vascular risk (BP, lipids) Screen for glucose intolerance/diabetes

Lateral abdominal X-ray (vascular calcification)

Bone biopsy if: Biochemistry does not explain clinical findings Bone pain or fracture Risk of aluminium toxicity

Vitamin D replacement

Decrease phosphate: Diet Phosphate binders

Persistent high PTH: Calcimimetic Parathyroidectomy

Vigorous CV risk reduction

Figure 46.1 Diagnosis and management of bone complications of renal disease. 25(OH)D, 25-hydroxyvitamin D; 1,25(OH)2D, 1,25-dihydroxy-vitamin D; BP, blood pressure; CV, cardiovascular.

250

07 Bone Diseases more precise information and quantitative methods are described for research purposes, but routine CT scanning is not currently justied. High phosphate levels can be improved by effective renal replacement therapy (dialysis or transplant), although response to dialysis is often incomplete. Dietary phosphate should be restricted. For many patients, treatment with phosphate binders is required. Alu-Caps swallowed whole before meals are very effective but, in the long term, there is a risk of bone and brain toxicity. Calcichew, Titralac and Phos-Ex (calcium acetate) are calcium-containing phosphate binders which, again, are taken 15 minutes before meals. Sevelamer has been introduced recently and is a non-aluminium and non-calciumcontaining phosphate binder. Careful replacement of vitamin D should be undertaken. As the problem is with renal generation of active 1,25-dihydroxy-vitamin D3, an active analogue of vitamin D such as alfacalcidol is usually required. Cinacalcet is a calcimimetic drug that acts at the calcium-sensing receptor on parathyroid gland cells and leads to decreased PTH secretion. The drug is relatively expensive but effective. Surgery to remove the parathyroid glands is required for patients with very high PTH levels. Investigation and treatment are summarized in Figure 46.1.

Recent Developments
1 There is emerging evidence that many patients may harbour subtle abnormalities from an early stage of renal decline, and that these abnormalities contribute to prognosis. Levin et al.3 studied 1814 patients with varying degrees of renal function. Suboptimal vitamin D status (<22 pg/ml) was present in 13% of patients with estimated glomerular ltration rate (eGFR) of >80 ml/min and in over 60% of those with eGFR <30 ml/min. High PTH was common in patients with even modest renal impairment (eGFR >80 ml/min). 2 In patients with renal disease, vascular calcication leads to structural changes in the vascular wall and accelerated atherogenesis. However, patients with asymptomatic hyperparathyroidism may have subtle abnormalities in vascular function that contribute to the development of macrovascular disorders. These abnormalities include insulin resistance, endothelial activation and increased arterial stiffness. In asymptomatic patients, the changes in glucose tolerance do not appear to progress rapidly.4 Fallo et al.5 have demonstrated increased soluble E-selectin and von Willebrand factor in patients with hyperparathyroidism. These markers provide evidence of endothelial activation and may help to assess cardiovascular risk prior to and following treatment of hyperparathyroidism. 3 Mineral abnormalities contribute to development of insulin resistance and type 2 diabetes. On the other hand, treatment of diabetes may inuence bone and mineral status. Thiazolidinediones (rosiglitazone and pioglitazone) are widely used in treatment of type 2 diabetes. In bone, these drugs enhance adipogenesis while decreasing osteoblast differentiation. There is concern, therefore, that they may predispose users to osteoporosis. Certainly recent evidence suggests that use of glitazone drugs can decrease BMD.6 It remains to be seen whether this translates into increased fracture risk. 4 Data from the third National Health and Nutrition Evaluation Survey (NHANES III) conrmed that low vitamin D status was a risk factor for type 2 diabetes.7 This is not only conrmed by recent data,8 but it also appears that patients with type 2 diabetes

46 Bone complications of renal disease

251

and low vitamin D status are at greater risk of requiring insulin and are more likely to develop microvascular complications. Consideration should be given to screening for abnormalities of calcium and vitamin D status in patients with insulin-resistant states, and studies are required to evaluate whether vitamin D supplementation improves prognosis for patients with metabolic syndrome and type 2 diabetes.

Conclusion
The term renal osteodystrophy is of limited use in routine clinical practice as it can only be diagnosed and classied with bone biopsy. At present, markers of bone turnover are not able to specically diagnose bone complications of CKD. Vascular calcication is an important risk marker for patients with renal disease and is part of the metabolic disturbance that also leads to bone disease. Biochemical changes in calcium, phosphate, vitamin D and PTH metabolism appear when eGFR falls below 40 ml/min, but subtle changes can occur with mild degrees of renal impairment. The above patient should have assessment of his calcium and vitamin D status. His renal impairment means that bisphosphonate therapy is not indicated without more detailed knowledge of his underlying bone status. Vitamin D supplements may decrease PTH levels and thus bone turnover. Low BMD in this case conrms decreased bone mineral content but may reect decreased vitamin D and increased PTH status and not simply primary osteoporosis.

Further Reading
1 2 3 Schwarz C, Sulzbacher I, Oberbauer R. Diagnosis of renal osteodystrophy. Eur J Clin Invest 2006; 36 (Suppl 2): 1322. Ferreira A. Development of renal bone disease. Eur J Clin Invest 2006; 36 (Suppl 2): 212. Levin A, Bakris GL, Molitch M et al. Prevalence of abnormal serum vitamin D, PTH, calcium, and phosphorus in patients with chronic kidney disease: results of the study to evaluate early kidney disease. Kidney Int 2007; 71: 318. Ayturk S, Gursoy A, Bascil Tutuncu N, Ertugrul DT, Guvener Demirag N. Changes in insulin sensitivity and glucose and bone metabolism over time in patients with asymptomatic primary hyperparathyroidism. J Clin Endocrinol Metab 2006; 91: 42603. Fallo F, Cella G, Casonato A et al. Biochemical markers of endothelial activation in primary hyperparathyroidism. Horm Metab Res 2006; 38: 1259. Schwartz AV, Sellmeyer DE, Vittinghoff E et al. Thiazolidinedione use and bone loss in older diabetic adults. J Clin Endocrinol Metab 2006; 91: 334954. Scragg R, Sowers M, Bell C; Third National Health and Nutrition Examination Survey. Serum 25-hydroxyvitamin D, diabetes, and ethnicity in the Third National Health and Nutrition Examination Survey. Diabetes Care 2004; 27: 281318. Suzuki A, Kotake M, Ono Y et al. Hypovitaminosis D in type 2 diabetes mellitus: Association with microvascular complications and type of treatment. Endocr J 2006; 53: 50310.

5 6 7

S E C T I O N

E I G H T

08

Muscle Diseases
47 48 49 Steroid myopathy Inammatory myopathies Muscle complications of statin therapy

P R O B L E M

47 Steroid Myopathy
Case History
JG is a 62-year-old man with severe obstructive pulmonary disease. He has been taking steroids continuously for ten years generally 510 mg of prednisolone per day but more with exacerbations. He complains of progressive weakness and muscle wasting, and notes particular difculty climbing stairs. He is a moderately heavy drinker. There is no history of diabetes and no family history of muscle disease. How can steroid myopathy be diagnosed? What is his prognosis? What treatment options are available for him?

Background
This chapter is a general background to muscle disease, with specic emphasis on steroid-induced myopathy. While muscle symptoms are common, diseases of muscle are relatively uncommon and many of the specic diagnoses are rare. Table 47.1 is a summary classication of disorders of muscle. Congenital myopathies usually present in infancy with a oppy baby that has poor muscular effort. Plasma creatine kinase (CK) is normal, and electromyography (EMG) shows a myopathic pattern. Central core disease usually presents with mild, nonprogressive weakness in infancy leading to delay in walking and other physical development milestones. It is inherited as an autosomal dominant condition. Nemaline rod Atlas Medical Publishing Ltd

254

08 Muscle Diseases

Table 47.1 Diseases of muscle


A: Congenital
Congenital myopathies Central core disease Nemaline rod myopathy Centronuclear (myotubular)

Muscular dystrophies Channelopathies Inherited metabolic diseases Mitochondial myopathy syndromes

B: Acquired
Inammatory Dermatomyositis Inclusion body myositis Polymyositis Hypothyroidism and hyperthyroidism Vitamin D deciency Cushings syndrome Conns syndrome and Addisons disease Hypokalaemia Critical care myopathy Alcohol Organophosphates Snake venoms Corticosteroids Statins, clobrate Vincristine, cyclosporine Amiodarone Zidovudine Carcinomas Dermatomyositis

Metabolic

Drugs and toxins

Paraneoplastic

myopathy is more serious, presenting with weakness and hypotonia. The consequences are feeding difculties, delay in walking and sometimes respiratory muscle weakness. It is slowly progressive and older children or adults with the condition characteristically have decreased muscle bulk and an abnormally long face with protruding jaw. It can be inherited in either a dominant or a recessive fashion. Centronuclear (myotubular) myopathy again presents in infancy and is relentlessly progressive, usually leading to death at an early age. Involvement of the eye muscles is common. Muscular dystrophies. Onset and severity varies. Specic genetic tests are now available for many of these conditions and all patients/families should receive genetic counselling. Onset is usually in childhood, although some forms typically present later. The most common forms are: b Myotonic dystrophy. This is the most common inherited disease of muscle. Myotonic dystrophy type 1 (DM1) is inherited as an autosomal dominant disorder and is due to an expanded cytosine-thymine-guanine (CTG) trinucleotide repeat in the 3 untranslated region of the myotonic dystrophy protein kinase (DMPK) gene on chromosome 19q13.3. In addition to myopathy, patients suffer cognitive impairment, subcapsular cataracts, cardiac conduction abnormalities, sensorineural

47 Steroid myopathy

255

deafness, frontal balding and hypogonadism. Myotonic dystrophy type 2 (DM2) is also an autosomal dominant condition and is due to expansion of a cytosinecytosine-thymine-guanine (CCTG) repeat in intron 1 of the ZNF9 gene on chromosome 3q. It causes a proximal muscle pattern of myopathy, sometimes with pain and hypertrophy, but no cognitive impairment. Both DM1 and DM2 lead to alternative splicing of the voltage-gated chloride channel (ClC-1) and are therefore considered along with the channelopathies. Duchenne muscular dystrophy. An X-linked condition, which therefore affects boys, that is due to deletion in the dystrophin gene. It usually presents between the ages of two and six years. The patient is usually wheelchair-bound by their early teens, and survival beyond late teens or early adulthood is the exception. Weakness is proximal and limb girdle and there may be pseudohypertrophy of the calves, cardiac conduction abnormalities and scoliosis. Becker muscular dystrophy. This is also X-linked, with a similar distribution of muscle weakness to Duchenne muscular dystrophy. It is usually milder than Duchenne, but is variable in severity. Symptoms may not begin until age ten years or later, and patients can have a long lifespan, albeit with varying degrees of disability. EmeryDreifuss muscular dystrophy. An X-linked disorder that is due to a mutation in the emerin gene. Symptoms start around ve years of age with upper arm and lower leg weakness. Proximal muscle weakness develops later. Contractures and joint problems may develop, and patients are at risk of sudden cardiac death because of conduction abnormalities. Limb girdle muscular dystrophy. There can be dominant (type 1) or recessive (type 2) inheritance. A number of gene abnormalities may cause the syndrome and prognosis is, therefore, variable. Cardiac abnormalities may be present. Boys and girls are equally affected, and symptoms typically begin in late childhood. Facioscapulohumeral muscular dystrophy. This is an autosomal dominant condition affecting males and females. Onset is usually in late childhood or early adulthood. The symptoms can be quite mild, although may become more generalized, affecting the lower limbs later.

Channelopathies are a recently recognized group of conditions where there is a genetic defect in one of the ion channels involved in regulating normal muscle.1 A summary is presented in Table 47.2. Myotonia arises from repetitive bursts of action potential when muscle contraction is voluntarily activated. The result is inability to relax the muscle. Symptoms usually improve during activity. By contrast, paramyotonia worsens with cold and after exercise. Becker myotonia is the most common form while Thomsens disease, although less common, usually presents with milder symptoms. Hyperkalaemic periodic paralysis may be triggered by potassium intake and symptoms are ameliorated by glucose. The reverse applies to hypokalaemic periodic paralysis. Andersen syndrome is an autosomal dominant disorder where episodes of paralysis are provoked by prolonged inactivity (including sleep), calorie deprivation and cold. It is associated with long QT interval on the electrocardiogram, making patients prone to tachycardias. Malignant hyperthermia can be provoked by volatile anaesthetics, depolarizing muscle relaxants or extreme physical activity. A sustained increase in cellular calcium in skeletal muscle leads

256

08 Muscle Diseases

Table 47.2 Channelopathies


Channel
Sodium (SCN4A, Ch 17q23.1)

Disease
Paramyotonia congenita Hypokalaemic periodic paralysis (type 2) Hyperkalaemic periodic paralysis Andersen syndrome Hypokalaemic periodic paralysis (type 1) Malignant hyperthermia Malignant hyperthermia Becker myotonia Thomsens disease Congenital myasthenia

Heritance
Dominant Dominant Dominant Dominant Dominant Dominant Dominant Recessive Dominant Dom/Rec

Potassium (KCNJ2, Ch 17q2324) Calcium (CACNA1S, Ch 1q3132) (RyR1, Ch 19q13.1) * Chloride (CLCN1, Ch 7q32) Acetylcholine receptor

* RyR1, calcium release channel. One of ve genes affected, each encoding different subunits of the nicotinic acetylcholine receptor. Dom/Rec, inheritance may be dominant or recessive.

to intense muscle contraction with hyperthermia, metabolic acidosis, hypoxia and hyperkalaemia. Mitochondrial myopathies are being increasingly recognized in clinical practice, although they remain rare: b MELAS (Myopathy, Encephalopathy, Lactic Acidosis and Stroke) episodic encephalopathy, stroke-like episodes; a progressive neurodegenerative disorder often also causing diabetes b MERRF (Myoclonic Epilepsy with Ragged Red Fibres) optic atrophy, peripheral neuropathy, dementia, myoclonic epilepsy, cerebellar ataxia and sensorineural deafness b KearnsSayre syndrome progressive oculomotor symptoms including ptosis, pigmentary retinal degeneration, sensorineural deafness, proximal myopathy and cardiac conduction defects b CPEO (Chronic Progressive External Ophthalmoplegia) like KearnsSayre syndrome but later onset and not associated with retinal degeneration Congenital metabolic disorders should be considered in the differential diagnosis of muscle disorders, particularly where they occur early in life or where there is a relevant family history. Differential diagnosis includes the glycogen storage diseases (GSDs). Disorders that lead to prominent muscle symptoms in adult patients include: b Pompe disease (Type II GSD) due to deciency of the lysosomal enzyme alpha-1,4glucosidase (acid maltase) leading to unregulated accumulation of glycogen with disruption of muscle structure and function b Cori disease (Type III GSD; limit dextrinosis) due to deciency of the debrancher enzyme leading to accumulation of abnormal glycogen, which cannot be broken down to release glucose

47 Steroid myopathy

257

b McArdle disease (Type V GSD) due to deciency of myophosphorylase which, again, leads to defective glycogen breakdown. Muscle swelling and tenderness occurs, CK levels are generally very high and there may be episodes of rhabdomyolysis b Tarui disease (Type VII GSD) leads to a clinical presentation similar to that of McArdle disease and is due to deciency of muscle phosphofructokinase b Carnitine-palmitoyl transferase deciency causes episodes of muscle pain and weakness, intermittent CK elevation and myoglobinuria In adult practice, except in highly specialized centres, acquired muscle diseases are much more common than congenital disorders. Amongst these, myopathy caused by alcohol or drugs, including corticosteroids, is encountered not uncommonly.2 Acute alcoholic myopathy is relatively rare and leads to muscle necrosis, with variable inammatory inltrate causing muscle weakness and pain. Plasma CK is markedly increased, and there may be myoglobinuria and rhabdomyolysis with accompanying renal impairment. Recovery generally occurs following alcohol withdrawal and supportive measures. Chronic alcoholic myopathy principally affects type II (fast twitch, anaerobic, glycolytic) bres. It typically occurs after ten years of consuming greater than 100 grams (1012 units) of alcohol per day. The aetiology is not precisely known. Factors include ethanol-induced impaired mitochondrial function, leading to defective adenosine triphosphate generation and fatty acid utilization, acetaldehyde accumulation inhibiting protein synthesis, defective protein synthesis because of decreased amino acid availability and decreased activity of the growth hormone/insulin-like growth factor-1 (IGF-1) axis, and free radicals causing cell membrane damage. Steroid myopathy does not always occur with either prolonged exposure or high doses. It is more common with potent uorinated steroids (dexamethasone, betamethasone and triamcinolone). As with alcoholic myopathy, acute and chronic forms are recognized. Acute steroid myopathy usually occurs after acute exposure to high-dose steroid and can take many months to recover. A subacute, necrotizing form of myopathy with steroids is described, and leads to severe symptoms with CK levels more than ten times normal.3 Exposure of myocytes to steroid impairs protein synthesis and leads to loss of the protective effects of IGF-I. Furthermore, increased cellular protease activity increases muscle protein breakdown. Biopsy reveals variation in bre diameters, loss of type II bres and necrotic and basophilic bres throughout the muscle. As with other metabolic myopathies, symptoms characteristically affect the proximal muscles although more general involvement, including respiratory muscles, may occur in severe cases. In patients exposed to steroids long-term, there are usually other clinical features of steroid excess present by the time myopathy develops. Treatment (Figure 47.1) consists of minimizing exposure by decreasing the dose, using topical preparations, using alternate day regimens and avoiding uorinated steroids. Progressive resistance-training exercises are useful in restoring normal function and muscle bulk. Recovery is slow in chronic cases, and complete recovery may not be achieved. Critical illness myopathy (CIM), or acute quadriplegic myopathy, in its full-blown form is a rare entity with acute onset of generalized weakness. Plasma CK levels are normal. It is similar to steroid myopathy indeed many patients have been treated with steroids but is more severe and more generalized. Neuromuscular blocking drugs have also been implicated in the aetiology. EMG shows low or normal action potentials.

258

08 Muscle Diseases

Muscle weakness Muscle wasting

Young age ve Family history Increased CK

Markedly CK ve Inflammatory markers Autoantibodies

? Congenital cause

? Inflammatory cause

Review steroid dose and duration

Alcohol history other drugs

Muscle biopsy

EMG

Steroid myopathy diagnosed

Minimize exposure to steroids Avoid fluorinated steroids Use topical agents Use other immunosuppressive drugs Minimize dose Alternate day dose regimens

Resistance exercises ? Nutritional supplements

Figure 47.1 Diagnosis and management of steroid myopathy. +ve, positive.

Biopsy may show type II bre atrophy or necrosis similar to that seen in steroid myopathy. There is no specic treatment. Recovery is usually complete but can be slow.

Recent Developments
1 CIM is associated with increased hospital stay, increased risk of requiring mechanical ventilation and increased mortality.4,5 Patients are at increased risk of developing CIM if they have sepsis, are hyperglycaemic or require steroid treatment. Amongst

47 Steroid myopathy

259

causative factors are systemic inammation (particularly with sepsis), increased proteolysis and oxidative and metabolic stresses. There are often neuropathic features, and there is impairment of excitationcontraction coupling. Intensive insulin therapy has been considered as a means of protecting patients from the consequences of CIM. 2 Muscle loss with inactivity is exacerbated if it is accompanied by stress and this is thought to be due to hypercortisolaemia. Essential amino acids formulated to replicate the proportion found in muscle are a potent anabolic stimulus in the myopathies of inactivity and steroid exposure.6 Attention should be paid to nutrition in patients exposed to steroids, those who are critically ill and those who are likely to suffer prolonged immobility. 3 Creatine supplementation diminishes the impaired exercise capacity that occurs when experimental animals are administered supraphysiological doses of steroids.7,8 The supplement attenuates loss of muscle mass with steroids. Clinical trials in patients taking steroids or admitted to critical care facilities are required to evaluate this intervention, which may prove a safe prophylactic measure.

Conclusion
Steroid myopathy is diagnosed by excluding other causes of muscle weakness and wasting. Except in the acute necrotizing form of steroid myopathy, there is not usually systemic inammatory activation or increased circulating muscle markers. Steroid dose does not have to be high nor the duration of treatment long for the patient to develop myopathy. Muscle biopsy may be required for denitive diagnosis. Prognosis is variable and relates to severity. Improvement is usual if steroids can be decreased or withdrawn. Other risk factors for muscle loss should be eliminated where possible. These include certain drugs and high intake of alcohol. There is no specic treatment. Resistance exercises to rebuild muscle mass and nutritional supplements have been advocated but there is no evidence to support this from randomized clinical trials.

Further Reading
1 2 3 4 5 Jurkat-Rott K, Lehmann-Horn F. Muscle channelopathies and critical points in functional and genetic studies. J Clin Invest 2005; 115: 20009. Owczarek J, Jasi ska M, Orszulak-Michalak D. Drug-induced myopathies. An overview of the n possible mechanisms. Pharmacol Rep 2005; 57: 2334. Bronner IM, Hoogendijk JE, Wintzen AR et al. Necrotising myopathy, an unusual presentation of a steroid-responsive myopathy. J Neurol 2003; 250: 4805. Deem S. Intensive-care-unit-acquired muscle weakness. Respir Care 2006; 51: 104252; discussion 10523. Friedrich O. Critical illness myopathy: what is happening? Curr Opin Clin Nutr Metab Care 2006; 9: 4039.

260

08 Muscle Diseases
6 7 Paddon-Jones D, Wolfe RR, Ferrando AA. Amino acid supplementation for reversing bed rest and steroid myopathies. J Nutr 2005; 135: 1809S12S. Campos AR, Serani LN, Sobreira C, Menezes LG, Martinez JAB. Creatine intake attenuates corticosteroid-induced impairment of voluntary running in hamsters. Appl Physiol Nutr Metab 2006; 31: 4904. Menezes LG, Sobreira C, Neder L, Rodrigues-Jnior AL, Martinez JAB. Creatine supplementation attenuates corticosteroid-induced muscle wasting and impairment of exercise performance in rats. J Appl Physiol 2007; 102: 698703.

P R O B L E M

48 Inammatory Myopathies
Case History
A 56-year-old woman complains of difculty in rising from a chair and climbing stairs. She has an erythematous rash on the back of her hand. Symptoms have increased over the past four weeks. Her previous health has been very good and she is not taking any medications. What other clinical features should be considered? How should she be investigated? What treatment is available and what is the prognosis?

Background
This group of disorders (Box 48.1) is quite rare but not infrequently needs to be considered in practice. This chapter considers only the primary inammatory myopathies, of which dermatomyositis (DM) and inclusion body myositis (IBM) are the most common. Together, their incidence is 28 per million per year. They present classically with muscle weakness. Pain and stiffness are less prominent symptoms. DM and polymyositis (PM) present with similar distribution of involvement. Proximal muscle weakness predominates over distal weakness. Pelvic girdle involvement usually precedes and is more marked than shoulder girdle involvement. The typical initial symptoms are difculty rising from a chair or climbing stairs. DM is a disorder of humoral immunity where antibody binding/deposition or immune complexes lead to capillary damage, resulting in localized atrophy or infarction. It may present at any age. Atlas Medical Publishing Ltd

48 Inammatory myopathies

261

Box 48.1 Inammatory myopathies 1. Primary Dermatomyositis Inclusion body myositis Polymyositis 2. Connective tissue diseases Mixed connective tissue disease (MCTD) Systemic lupus erythematosus Scleroderma Sjgrens syndrome Rheumatoid arthritis 3. Infections Viral (Coxsackie, HIV etc.) Bacterial Parasitic 4. Miscellaneous Graft-versus-host disease Eosinophilic myositis Macrophagic myofasciitis Sarcoidosis Associated with systemic vasculitis
HIV, human immunodeciency virus. Adapted with permission from Hilton-Jones 2003.1

Twenty per cent of cases in later life occur with an underlying malignancy. There is no association with any particular malignancy. In children, DM does not usually give rise to profound weakness, but is more frequently associated with subcutaneous calcication and facial ushing. However, the characteristic rash is usually absent and the condition is seldom associated with malignancy. Presentation of DM is usually subacute but it may develop very rapidly and present with severe symptoms. The characteristic rash, which occurs in most patients, is a photosensitivity reaction on the cheeks, exposed anterior chest or the knuckles. It can be similar to the rash of systemic lupus erythematosus (SLE). The eyelids may show a purple discolouration and there may be a scaly eruption (Gottrons sign). Interstitial lung disease occurs in up to 30% of cases and in over 60% of those who have antibodies against aminoacyl-tRNA synthetases. Increased mortality in DM relates to lung disease, underlying malignancy and, occasionally, associated myocarditis. The serum of patients with lung disease is usually positive for antiaminoacyl-tRNA synthetase antibodies (particularly anti-Jo). PM is much less common than the other two idiopathic inammatory myopathies. Its onset is usually more insidious than that of DM. It is not associated with skin lesions or with underlying malignancy. Histologically, it is distinct from the other two disorders, but muscle biopsy may not reveal characteristic changes in a patient with apparent PM. PM is predominantly a disorder of cell-mediated immunity where CD8+ cells home to muscle that is aberrantly expressing class I major histocompatibility antigens. IBM classically involves the quadriceps and the long nger exors, leading respectively to unexplained falling or knees giving way and compromised hand grip.2,3 Unlike the

262

08 Muscle Diseases other two conditions, symptoms may be asymmetric with facial involvement or dysphagia from an early stage. Foot drop is common. Microscopic features resemble those of PM, and overlap with the immunocytochemical features of Alzheimers disease in the brain has been suggested. The latter include vacuolar formation with foci of amyloid and phosphorylated tau. The extent to which autoimmunity is involved in the pathogenesis is uncertain. Hereditary forms are well described and, not surprisingly, may present at an early age. IBM typically occurs in the sixth decade or beyond and is now the primary inammatory myopathy most commonly diagnosed in this age group. An association with human immunodeciency virus (HIV) and human T-lymphotropic virus type 1 (HTLV-1) has been described and it is associated with other autoimmune diseases in about one-third of cases. IBM is often relentlessly progressive and response to treatments is disappointing. There is emerging anecdotal evidence to support the use of anti-T-cell therapies and stem-cell therapy has been considered. Diagnosis and management of inammatory myopathies is summarized in Figure 48.1. Measurement of serum creatine kinase (CK) is the simplest screening tool. Mild elevations outside the normal range are common and may be due to exercise, trauma or intramuscular injections. Men have higher values than women and values are higher in patients of Afro-Caribbean origin. Electromyography characteristically shows brillation patterns with sharp waves. Ninety per cent of patients with inammatory myopathy have autoantibodies to nuclear or cytoplasmic antigens. Myositis-specic antibodies (MSA) are directed against cytoplasmic ribonucleoproteins and are present in 30% of cases. Antibodies against aminoacyltRNA synthetases are particularly present in those with rapid onset, skin rash and interstitial lung disease. Antibodies against six of the 20 aminoacyl-tRNA synthetases have been described. The commonest of these, anti-Jo (directed against the histidyl-tRNA synthetase), is present in around 10% of cases. Antibodies to signal recognition particle (SRP; involved in endoplasmic reticulum transport of polypeptides) are found particularly in AfricanAmerican women, and are associated with acute onset, widespread features (including cardiac) and poor prognosis. M1-2 is a nuclear antigen, antibodies to which are found in DM. Muscle biopsy should be carried out where possible given the potential gravity of the conditions and that there may be a need for immunosuppressive treatment. The deltoid or quadriceps muscles are usually used. The best choice is an affected muscle but not one that is severely affected, as the biopsy may then only show severe atrophy. Open biopsy or needle biopsy may be used, and magnetic resonance imaging (MRI) or computed tomography scanning may be used to guide the site of biopsy. MRI studies may be useful diagnostically and in following progress. Ultrasound may also be useful. Exercise is important to ensure that residual muscle capacity is maximized. Also, passive exercises help to minimize the risk of contractures. All patients should have the help of a qualied physiotherapist. Adequate calorie and protein intake should be ensured to minimize muscle catabolism, particularly if the patient has dysphagia. The mainstay of drug treatment is corticosteroids. Initially, intravenous methylprednisolone at a dose of 500 mg for ve days may induce remission. This is followed by prednisolone 1 mg/kg body weight/day, gradually reduced as the plasma CK decreases. Other immunosuppressive drugs are also useful e.g. methotrexate (up to 30 mg/week) or azathioprine (2.5 mg/kg/day). Cyclosporine and cyclophosphamide are also widely used. Intravenous immunoglobulin is effective for DM or PM. IBM is much more refractory to steroids or other immunosuppressive therapies.

48 Inammatory myopathies

263

Muscle weakness stiffness pain

Increased CK ESR and CRP

Possible inflammatory myopathy

Proximal muscle involvement

Quadriceps Hand muscles

? DM or PM

? IBM

? Skin lesions Face, chest, hands ? Lung involvement CXR, pulmonary function tests ? Internal malignancy CXR, CT scan, tumour markers

EMG

Autoantibodies

MRI or U/S

Muscle biopsy

Diagnosis of DM or PM

Corticosteroid

Maintenance dose

Second-line immunosuppressive

Biological agent

Figure 48.1 Diagnosis and management of inammatory myopathies. CK, creatine kinase; CRP, C-reactive protein; CT, computed tomography; CXR, chest X-ray; DM, dermatomyositis; EMG, electromyography; ESR, erythrocyte sedimentation rate; IBM, inclusion body myositis; MRI, magnetic resonance imaging; PM, polymyositis; U/S, ultrasound.

264

08 Muscle Diseases

Recent Developments
1 There is increasing experience with use of biological agents,4,5 although the rarity of the conditions means that controlled trials are not available. Rapamycin or monoclonal antibodies have been used as anti-T-cell agents. There is some experience with tumour necrosis factor (TNF)-a blockers (etanercept and iniximab). The anti-B-cell (CD20) antibody rituximab has been used, as it has in other connective tissue diseases. Eculizumab, a monoclonal antibody that inhibits cleavage of the complement component C5, has also been used. 2 The pathogenesis of IBM is becoming more clearly understood. Although it is frequently associated with immune disturbance, it may primarily be a degenerative disease.2,6 Accumulation of amyloid-beta protein and its precursor, evidence of oxidative stress, abnormal protein folding and disturbed proteasomal degradation of proteins may all contribute to the pathological features. 3 Identication of further antigenic targets and other immune disturbances may improve our understanding of pathogenesis but it is not clear whether these will contribute to streamlining diagnosis and management. Recently, increased interleukin-18 (IL-18) has been described in DM and PM, in keeping with a T helper 1 (Th1)-dominated immune response.7 Two novel nuclear antigens with molecular masses of 140 kDa and 155 kDa have been described in Japanese patients.8 The presence of antibodies to these antigens may be strongly associated with coexisting malignancy.

Conclusion
Although rare, inammatory myopathies are important to diagnose. They carry a signicant mortality but prognosis has been improved by use of modern diagnostic techniques and immunosuppressive therapy.9 Patients with suspected myopathy should be carefully and fully investigated and precise diagnosis relies on muscle biopsy. There should be enquiry and investigation for cutaneous, pulmonary and cardiac involvement and a search for malignancy in DM. The latter may not be apparent at diagnosis but typically manifests itself within one year of diagnosis. Corticosteroids are the rst line of treatment and should be commenced in sufciently high doses to control disease activity quickly. Doses should also be decreased rapidly as the condition becomes less active. If this does not occur, addition or substitution of other immunosuppressive agents should be considered. The outlook for patients with inammatory myopathies has improved in recent years because of more effective diagnosis and availability of a wider range of treatments.

Further Reading
1 2 Hilton-Jones D. Diagnosis and treatment of inammatory muscle diseases. J Neurol Neurosurg Psychiatry 2003; 74 (Suppl 2): ii25ii31. Engel WK, Askanas V. Inclusion-body myositis: clinical, diagnostic, and pathologic aspects. Neurology 2006; 66 (2 Suppl 1): S209.

49 Muscle complications of statin therapy


3 4 5 6

265

Dalakas MC. Inammatory, immune, and viral aspects of inclusion-body myositis. Neurology 2006; 66 (2 Suppl 1): S338. Cordeiro AC, Isenberg DA. Treatment of inammatory myopathies. Postgrad Med J 2006; 82: 41724. Dalakas MC. Therapeutic targets in patients with inammatory myopathies: present approaches and a look to the future. Neuromuscul Disord 2006; 16: 22336. Askanas V, Engel WK. Inclusion-body myositis: a myodegenerative conformational disorder associated with Abeta, protein misfolding, and proteasome inhibition. Neurology 2006; 66 (2 Suppl 1): S3948. Tucci M, Quatraro C, Dammacco F, Silvestris F. Interleukin-18 overexpression as a hallmark of the activity of autoimmune inammatory myopathies. Clin Exp Immunol 2006; 146: 2131. Kaji K, Fujimoto M, Hasegawa M et al. Identication of a novel autoantibody reactive with 155 and 140 kDa nuclear proteins in patients with dermatomyositis: an association with malignancy. Rheumatology 2007; 46: 258. Briani C, Doria A, Sarzi-Puttini P, Dalakas MC. Update on idiopathic inammatory myopathies. Autoimmunity 2006; 39: 16170.

7 8

P R O B L E M

49 Muscle Complications of Statin Therapy

Case History
MO is a 52-year-old man with type 2 diabetes. Glycaemic control is reasonable with diet alone. Six months ago, he was started on a statin because of his cholesterol level. Over the past three months, he has noted muscle aches and pains. These have become much more severe in the past two weeks and his calves are tender bilaterally. What is the risk of muscle complications in patients treated with statins? Does the risk differ with different agents? How should statin-induced muscle problems be prevented, diagnosed and treated?

Atlas Medical Publishing Ltd

266

08 Muscle Diseases

Background
Statins 3-hydroxy-3-methylglutaryl-CoA reductase inhibitors decrease endogenous cholesterol synthesis. Statins are now the most commonly prescribed group of drugs. They lower total and low-density lipoprotein (LDL) cholesterol, as well as slightly lowering triglycerides and increasing high-density lipoprotein (HDL) cholesterol. For secondary prevention of cardiovascular disease, total and LDL cholesterol levels of 4.0 mmol/l and 2.0 mmol/l, respectively, are now recommended. Statins typically decrease LDL cholesterol by 30%40%, while combination treatments or high-dose statins can decrease LDL by up to 55%. Lipid-lowering therapy is also recommended for primary prevention where the estimated risk of a cardiovascular event is greater than 20% in ten years. This includes many patients with type 2 diabetes. While statins are very effective and safe, the number of patients receiving these drugs means that even unusual side effects are encountered routinely in practice. Setting tighter cholesterol targets has led to higher doses being prescribed and increasing use of more potent agents, which also increases the risk of side effects. For similar cholesterol-lowering activity, different doses and drugs are comparable in their fringe benets (levels of triglycerides and HDL cholesterol, anti-inammatory effects) and their risk of side effects. The risk of muscle disorders became widely recognized when cerivastatin (Baycol) was withdrawn ve years ago.1 Muscle problems occurred particularly following high doses and when the drug was combined with the brate gembrozil. Increased hepatic transaminases occurs in 0.5%2.0% of cases, although progression to liver failure is rare. Muscle side effects are now commonly reported but it is not clear to what extent these are greater than in non-treated patients. Myopathy usually implies a condition that leads to muscle weakness. However, the term is used in the statin literature to represent any muscle disorder. Myalgia is dened as muscle symptoms without elevation of serum creatine kinase (CK); myositis signies muscle aches and weakness with CK elevation; rhabdomyolysis is muscle necrosis usually associated with severe symptoms and CK greater than ten times normal. Rhabdomyolysis with renal impairment can be fatal but occurs in less than one per million patients prescribed statins. Available statins appear to have similar likelihood of causing myopathy (0.2%0.5%). Rhabdomyolysis occurs in 0.02%0.04% of patients. Fibrate treatment alone carries a similar risk of muscle side effects. When brates and statins are used together, risk of myopathy increases to around 1%. Patients should be warned to report muscle side effects. There is no role for routine monitoring of CK. The benets of statin therapy are clear. Muscle symptoms are common in the general population. If muscle symptoms occur with normal or less than three times normal CK, it is usually reasonable to follow the symptoms and CK levels at one- to two-weekly intervals without discontinuing the drug. For patients with mild symptoms and modest or no CK elevation, the drug should be discontinued for a few weeks and cautiously reintroduced with careful monitoring. Risk of statin-induced myopathy is increased in the following situations: b b b b Advanced age (>80 years), especially in women Low body weight Strenuous exercise Hypothyroidism

49 Muscle complications of statin therapy b Multisystem disease, particularly renal failure b Perioperative or other catabolic states b With other medications: Nicotinic acid Fibrates Cyclosporine Macrolide antibiotics Antifungal agents Verapamil Amiodarone Protease inhibitors for human immunodeciency virus (HIV) infection Alcohol abuse

267

The prevalence of muscle side effects may have been underestimated in the major statin trials since high-risk individuals were excluded (e.g. the elderly, those with renal impairment). Muscle symptoms are often overlooked in routine clinical practice. The PRIMO (Prediction of Muscular Risk in Observational conditions) survey2 investigated the prevalence of muscular symptoms in patients receiving high-dose statins. Of the 7924 patients surveyed, muscular symptoms developed in 10.5%. Amongst patients who reported symptoms, 38% stated that the exertion required for everyday activities was prevented, while 4% described themselves as being severely limited. Fluvastatin XL had a rate of muscular symptoms (5.1%) that was only half of that with other statins. Drugs that inhibit the cytochrome P450 enzyme CYP3A4 increase risk of statin-induced myopathy since most statins are metabolized by this enzyme. Fluvastatin is metabolized by CYP2C9, and this may explain the lower incidence of muscle symptoms. While the PRIMO study was only observational, it does underline the fact that muscular side effects are common, particularly when the patient is taking higher doses of statins. Rhabdomyolysis is the most severe muscular complication.3 It is important to appreciate that it is more likely to occur with a brate alone than with a statin alone. Adding a brate to statin therapy leads to a twelve-fold increase in risk of muscle complications. These are most common within a month of starting or escalating therapy. Rhabdomyolysis causes myoglobinaemia, myoglobinuria and renal impairment. It may present with back pain or proximal muscle pain, but many cases have a non-specic presentation (e.g. fatigue or u-like symptoms). Risk of rhabdomyolysis does not appear to relate to serum levels of the drugs. However, high serum levels of rosuvastatin have been noted in Asian patients, for whom a lower dose of the drug is recommended. Rhabdomyolysis is diagnosed when muscle symptoms are accompanied by a CK rise to greater than ten times normal. Enquiry should be made about agents that may provoke statin toxicity (including alcohol), and for other causes of CK increase such as exercise, fall and intramuscular injections. If there is doubt as to whether symptoms are related to statin treatment, it is reasonable to stop the statin for two weeks to see whether symptoms improve and CK levels decrease. Plasma levels of thyroid-stimulating hormone (TSH), antinuclear factor (ANF) and anti-Jo antibodies should be checked. One of the postulated mechanisms for muscle toxicity is depletion of coenzyme Q10. Supplementation at doses of up to 1200 mg/day has been advocated. Acute management includes careful monitoring of renal function, rehydration, alkalinizing the urine to prevent myoglobin

268

08 Muscle Diseases

Muscle pain or weakness Fatigue Decreased exercise tolerance

Review drug and alcohol history*

CK, TSH, ANF, Anti-Jo

Toxicity possible

Toxicity probable

Rhabdomyolysis

Drug holiday

Drug holiday

U/E, myoglobin

Lower dose of statin Less toxic statin Pravastatin Rosuvastatin Fluvastatin Ezetimibe

Stop the statin Rehydrate Alkalinize urine Muscle biopsy

Diet resin Ezetimibe

Regular follow-up Muscle symptoms Lipid control Renal function and CK

Figure 49.1 Management of statin-induced muscle disease. * Check for drugs that may precipitate muscle toxicity. Plasma and urine myoglobin should be measured. ANF, antinuclear factor; CK, creatine kinase; TSH, thyroid-stimulating hormone; U/E, urea and electrolytes.

precipitation and discontinuing the statin. The optimal approach to ongoing lipid-lowering therapy is unclear. Changing to a more hydrophilic statin (pravastatin or rosuvastatin) should be considered. Fluvastatin has been associated with lower risk of muscle problems. The risk of recurrence is high if the statin is reinstituted. Ezetimibe may be used alone or with a decreased dose of statin but, in either case, there appears to be some risk of recurrent muscle symptoms. Niacin and brates may cause muscle symptoms in patients who have previously had problems with statins. Resins such as cholestyramine

49 Muscle complications of statin therapy

269

have a low risk of toxicity but patients may not reach current cholesterol targets. Management of statin-induced muscle disease is summarized in Figure 49.1.

Recent Developments
1 Kaufmann et al.4 have demonstrated mitochondrial toxicity whereby exposure to statins dissipated the mitochondrial membrane potential with consequent decreased beta oxidation and disruption in mitochondrial structure. The latter effect may be important in promoting apoptosis of myocytes.5 Effects were seen with the lipophilic statins (cerivastatin, uvastatin, simvastatin and atorvastatin) but not with the hydrophilic pravastatin. Another recent study6 has reported inhibition of the human monocarboxylate transporter 4, which is responsible for lactate efux from the myocyte. Again, inhibition was seen to a major degree only with the lipophilic statins. 2 Potential benets of statins beyond their ability to lower cholesterol include antiinammatory properties and favourable effects on bone remodelling.7 Annual sales of statins are worth over $12.5 billion (United States). Their increasing use includes many elderly patients. It is reassuring that elderly patients do not appear to be particularly susceptible to side effects.8 A meta-analysis of 119 trials including a total of 86 000 patients9 concluded that statins were very safe with an odds ratio for rhabdomyolysis of 1.59 and myositis of 2.56. There was a very low withdrawal rate because of adverse events. It must be borne in mind, however, that high-risk individuals are excluded from clinical trials but frequently require treatment in routine practice. 3 Ezetimibe inhibits intestinal absorption of cholesterol. It is useful in patients who do not achieve target cholesterol values with statins alone, or in patients who are at risk of adverse effects of statins and where it is, therefore, desirable to keep the dose of statin to a minimum. Vytorin is a combination of ezetimibe and simvastatin at respective doses of 10/10, 10/20, 10/40 and 10/80 mg. An analysis of 17 trials with this drug combination10 reports that the incidence of muscle side effects is no greater than with the statin used alone. 4 Rosuvastatin (Crestor) is the rst of a new generation of potent statins. In view of their increased potency, the safety of these newer drugs is obviously a matter of concern. Recent studies are reassuring. Using a database of over two million Dutch patients, more than 45 000 statin users were identied.11 Overall, the incidence of adverse events was less than 1 per 3000 person-years and there was no difference in adverse events comparing rosuvastatin with other available drugs of this class. Shepherd et al.,12 using a large database of nearly 17 000 patients taking statins, again showed a side-effect prole that was very little different to that of patients taking placebo. Rosuvastatin was similar to other statins in the incidence of side effects, including those affecting muscle.

Conclusion
Muscle side effects occur in a minority of patients taking statins but can, at worst, lead to fatality. The risk of clinically apparent muscle side effects has been estimated at around 1 per 3000 patient-years. This may be minimized by avoiding the drugs in very high-risk

270

08 Muscle Diseases patients, by minimizing the dose and being aware of potential drug interactions. The available statins appear to be similar in terms of risk of muscle complications. In vitro studies suggest that the more hydrophilic compounds may be safer, as may uvastatin because its route of metabolism differs from that of other statins. Management of muscle complications depends on the severity. The drug should be withdrawn, but may be cautiously reintroduced after symptoms have abated if the adverse event is mild. Otherwise, management of hydration and renal impairment are usually the most important aspects. For patients whose CK reaches greater than ten times the upper limit of normal, statin should not be reintroduced and management of hyperlipidaemia in this scenario presents a challenge.

Further Reading
1 Pasternak RC, Smith SC, Bairey-Merz CN, Grundy SM, Cleeman JI, Lenfant C. ACC/AHA/NHLBI Clinical Advisory on the use and safety of statins. J Am Coll Cardiol 2002; 40: 56772. 2 Bruckert E, Hayem G, Dejager S, Yau C, Bgaud B. Mild to moderate muscular symptoms with high-dosage statin therapy in hyperlipidemic patients the PRIMO study. Cardiovasc Drugs Ther 2005; 19: 40314. 3 Antons KA, Williams CD, Baker SK, Phillips PS. Clinical perspectives of statin-induced rhabdomyolysis. Am J Med 2006; 119: 4009. 4 Kaufmann P, Trk M, Zahno A, Waldhauser KM, Brecht K, Krhenbhl S. Toxicity of statins on rat skeletal muscle mitochondria. Cell Mol Life Sci 2006; 63: 241525. 5 Dirks AJ, Jones KM. Statin-induced apoptosis and skeletal myopathy. Am J Physiol Cell Physiol 2006; 291: C120812. 6 Kobayashi M, Otsuka Y, Itagaki S, Hirano T, Iseki K. Inhibitory effects of statins on human monocarboxylate transporter 4. Int J Pharm 2006; 317: 1925. 7 Almuti K, Rimawi R, Spevack D, Ostfeld RJ. Effects of stains beyond lipid lowering: potential for clinical benets. Int J Cardiol 2006; 109: 715. 8 Agostini JV, Tinetti ME, Han L, McAvay G, Foody JM, Concato J. Effects of statin use on muscle strength, cognition, and depressive symptoms in older adults. J Am Geriatr Soc 2007; 55: 4205. 9 McClure DL, Valuck RJ, Glanz M, Hokanson JE. Systemic review and meta-analysis of clinically relevant adverse events from HMG CoA reductase inhibitor trials worldwide from 1982 to present. Pharmacoepidemiol Drug Saf 2007; 16: 13243. 10 Davidson MH, Maccubbin D, Stepanavage M, Strony J, Musliner T. Striated muscle safety of ezetimibe/simvastatin (Vytorin). Am J Cardiol 2006; 97: 2238. 11 Goettsch WG, Heintjes EM, Kastelein JJP, Rabelink TJ, Johansson S, Herings RMC. Results of a rosuvastatin historical cohort study in more than 45,000 Dutch statin users, a PHARMO study. Pharmacoepidemiol Drug Saf 2006; 15: 43543. 12 Shepherd J, Vidt DG, Miller E, Harris S, Blasetto J. Safety of rosuvastatin: update on 16,876 rosuvastatin-treated patients in a multinational clinical trial program. Cardiology 2007; 107: 43343.

Index
ACR-70 response, rheumatoid arthritis 84 activator protein-1 (AP-1) pathway 240 acupuncture, value in bromyalgia syndrome 31 acute quadriplegic myopathy 2579 adalimumab use in psoriatic arthritis 182 use in vasculitides 157 AdamantiadesBehets disease see Behets syndrome ADAMTS-5 gene 41 adenine phosphoribosyltransferase deciency 186 adenosine deaminase deciency 186 adenylsuccinate lyase deciency 186 ADFR (Activate, Decrease osteoclast activity, Free of treatment and Repeat) 230 adhesive capsulitis (frozen shoulder) 1416, 212 adiponectin 43 Adsons test 22 adynamic bone disease 248 alcohol consumption as cause of male osteoporosis 233 effect on uric acid synthesis 1856 relationship to RA risk 98 alcoholic myopathy 257 alefacept, use in psoriatic arthritis 182 alemtuzumab 149 alendronate 223 combination with cholecalciferol 220 combination with raloxifene 230 fracture prevention 224 long-term use 224 use during corticosteroid therapy 238, 240 use in male osteoporosis 234 use in Pagets disease 243 alfacalcidol 219, 240, 250 alkaline phosphatase, levels in Pagets disease 242 allantoin 184 allopurinol 106, 187, 188, 1923, 194 alopecia, in SLE 122 alpha2-adrenergic agonists, use in bromyalgia syndrome 30 alpha-adrenergic antagonists, value in Raynauds phenomenon 137 alphaviruses 208 Alu-Caps 250 Alzheimers disease, similarity to inclusion body myositis 262 American College of Rheumatology (ACR), classication criteria for SLE 117 aminoacyl-tRNA synthetase antibodies 261, 262 Andersen syndrome 255 androgen deprivation therapy, osteoporosis risk 235 androgen therapy, male osteoporosis 2345 angiogenesis, abnormalities in psoriasis 182 angiotensin II receptor antagonists, use in Raynauds phenomenon 137 angiotensin-converting enzyme (ACE) inhibitors use in Raynauds phenomenon 137 use in scleroderma renal disease 142, 143 animal bites 201 ANKH gene 198 ankylosing spondylitis 4, 87 BASMI 8890 diagnosis 173 effects on heart 91 mortality risk 91 anterior compartment syndrome 24 anterior cruciate ligament tears 6 antibiotic treatment of osteomyelitis 203 of septic arthritis 201, 204 antibody treatments 149 use in Behets syndrome 166 use in inammatory myopathies 264 use in psoriatic arthritis 1812 use in vasculitides 157, 158 see also anti-TNF therapy anti-CCP antibodies 5, 72, 73, 7980 association with smoking 75 in psoriatic arthritis 182 anti-centromere antibodies 110 scleroderma 141, 142 Sjgrens syndrome 132 anticonvulsants, use in bromyalgia syndrome 30 antidepressants, use in bromyalgia syndrome 30 anti-dsDNA antibodies 11011, 117 in monitoring of SLE 1245 anti-histone antibodies 117, 118 anti-hnRNP antibodies 112 anti-inammatory properties, statins 269 anti-Jo-1 antibodies 112, 261, 262 anti-neutrophil cytoplasmic antibodies (ANCA) 155, 1567, 1578

272

Index
in SLE 122 see also osteoarthritis; psoriatic arthritis; rheumatoid arthritis; septic arthritis; viral arthritis arthritis mutilans 180 arthroscopic release, frozen shoulder 16 arthroscopy, in septic arthritis 201 arylalkanoic acids 61 2-arylpropionic acids (profens) 61 arzofoxifene 230 ASC protein, role in gout 193 aspiration of joints 4 diagnosis of septic arthritis 7, 201 in gout 190 knee 89 in pseudogout 196 aspirin, low dose concurrent non-selective NSAID therapy 61 use in Raynauds phenomenon 137 use in SLE 126 atherosclerosis, therapeutic strategies 158 athletes, risk of osteoarthritis 43 atrophic stage, scleroderma 141 autoantibodies, viral infection as trigger 206 azapropazone 193 azathioprine 148, 151 interaction with allopurinol 192 prediction of response 150 use in Behets syndrome 166 use in GCA and PMR 161 use in inammatory myopathies 262 use during pregnancy 95, 127 use in psoriatic arthritis 181 use in Sjgrens syndrome 132 use in SLE 127 use in vasculitides 157 B19 (parvovirus) 2067 back pain see low back pain bacteraemia, risk factors 204 Barmah Forest Virus 208 basiliximab 149 bazedoxifene 230 Becker muscular dystrophy 255 Becker myotonia 255 Behets syndrome 1656 effect of pregnancy 94 genetic factors 1667 insulin resistance 167 benzbromarone 193 betamethasone, intra-articular injection 51 bicipital tendinitis 13 biological agents use in Behets syndrome 166

anti-nuclear antibodies (ANAs) 789, 110, 11315 anti-dsDNA 11011 anti-hnRNP 112 anti-Jo-1 112, 261, 262 anti-Scl-70 112, 141, 142 anti-Sm and anti-RNP 11112 anti-SSA/Ro and anti-SSB/La 111, 116, 129 in endometriosis 113 environmental factors 112 in scleroderma 141, 142 in Sjgrens syndrome 130 in SLE 11618 antioxidant intake, relationship to RA risk 97, 98 antiphospholipid antibodies 209 antiphospholipid syndrome (APS), risks during pregnancy 94 antiplatelet therapy in giant cell arteritis 162 use in Raynauds phenomenon 137 anti-RA33 antibodies 112 anti-RNA polymerase III antibodies, scleroderma 142 anti-RNP antibodies 11112 anti-Scl-70 antibodies 112, 141, 142 anti-Sm antibodies 11112, 117 anti-SSA/Ro antibodies 111, 116, 129 anti-SSB/La antibodies 111, 129 antisynthetase syndrome 112 anti-TNF therapy 51, 80, 83, 85 in hepatitis C 207 in inammatory myopathies 264 in psoriatic arthritis 182 safety monitoring 79 in vasculitides 157 see also etanercept; iniximab anti-topoisomerase (anti-Scl-70) antibodies 112, 141, 142 antiviral antibodies 206 anxiety disorders, in bromyalgia syndrome 28 aortic calcication 24950 aortic insufciency, AS 91 aplastic anaemia, parvovirus B19 as cause 207 apoB-100 immunization 158 APPROVe (Adenomatous Polyp Prevention on Vioxx) study 61 aromatase activity, relationship to bone loss 235 arthritis acute monoarthritis, differential diagnosis 68 new onset examination 3 history-taking 23 investigations 34 patterns of 4

Index
use in inammatory myopathies 264 use in psoriatic arthritis 1812 use in vasculitides 157, 158 see also anti-TNF therapy biomarkers of osteoarthritis 43 biopsy, in vasculitis 157 see also bone biopsy; muscle biopsy bisphosphonates 218, 21920, 2234, 226, 231 in combination treatment 230 interaction with teriparatide 229 osteonecrosis risk 224 patient compliance 225 structure and function 2223 use during corticosteroid therapy 2389, 240 use in male osteoporosis 234 use in neuropathic arthropathy 213 use in osteoarthritis 51 use in Pagets disease 2423, 245 use in post-menopausal osteoporosis 225 blood cultures, septic arthritis 201 blood lm appearance, rheumatoid arthritis 78 body mass index (BMI), relationship to osteoarthritis risk 45 bone biopsy 247, 248 bone histomorphometry 248 bone mineral density (BMD) 21718 in diabetes 213 effect of strontium ranelate 228 effect of teriparatide 229 relationship to osteophyte formation 43 in renal osteodystrophy 248 bone mineralization, effect of bisphosphonates 223 bone remodelling, effects of statins 269 BONE study 224 bone turnover 247 bone turnover markers, Pagets disease 242 bosentan 144 value in Raynauds phenomenon 138 botulinum toxin injection, bromyalgia syndrome 31 breast cancer risk, effect of SERMS 228 breast-feeding, effect on rheumatoid arthritis risk 74 British Ankylosing Spondylitis Metrology Index (BASMI) 8890 brucellosis 201 Burkholderia pseudomallei 201 buttery rash 122 c-ANCA 156, 157 C-reactive protein (CRP) 4, 161 in giant cell arteritis 162 in rheumatoid arthritis 78, 80

273
in septic arthritis 9, 201 in SLE 1256 caffeine intake, relationship to RA risk 98 Calcichew 250 calcineurin inhibitors 1489 calcinosis, in scleroderma 141 calcitonin fracture prevention 224 use in neuropathic arthropathy 213 use in Pagets disease 243 calcitriol (1, 25-dihydroxy-vitamin D3) 219, 247 calcium channel blockers, value in Raynauds phenomenon 137 calcium levels, in teriparatide therapy 229 calcium pyrophosphate dihydrate (CPPD) deposition disease (pseudogout) 8, 9, 1956 associated conditions 1967 diagnosis 196 hereditary 197 role of ANKH mutations 198 toll-like receptors 198 treatment 1978 calcium supplementation 231 in corticosteroid therapy 238 men 234 cancers association with dermatomyositis 2601 breast cancer risk, effect of SERMS 228 as cause of lower back pain 172 vitamin D levels 219 capsaicin cream, value in osteoarthritis 47 carbon monoxide diffusion capacity, scleroderma 142 CARD15 gene 179 cardiac complications, ankylosing spondylitis 91 cardiovascular disease risk effect of NSAIDs 614 effect of raloxifene 228 in hyperuricaemia 1867 in psoriasis 182 in SLE 1256 carnitine-palmitoyl transferase deciency 257 carpal tunnel, anatomy 22 carpal tunnel release 245 endoscopic 26 carpal tunnel syndrome (CTS) 21 causes 23 clinical signs 23 imaging techniques 256 management 245 cartilage, in osteoarthritis 41 cartilage oligomeric matrix protein 43 caspase-1 role in gout 193 cauda equina syndrome 171

274

Index
cognitive dysfunction, in SLE 127 colchicine 9, 106 use in Behets syndrome 166 use in gout 190, 1923 use in osteoarthritis 51 use in pseudogout 197 use in vasculitides 157 collagen genes 41 combination therapy in osteoporosis 22930 in rheumatoid arthritis 84 compartment syndromes 24 complement deciencies antinuclear antibodies 111 association with SLE 120, 157 compliance, bisphosphonate treatment 225 computer use, dry eyes 130 conduction disturbances, AS 91 congenital heart block 94, 95, 111 congenital myopathies 2534 connective tissue growth factor (CTGF), role in scleroderma 143 continuous-focus osteomyelitis 203 contractures, in scleroderma 141 coracoacromial arch anomalies 1314 CORE (Continuing Outcomes Relevant to Evista) study 227 Cori disease 256 corticosteroid-induced osteoporosis, investigation and management 2389 corticosteroid therapy 1478, 151 as cause of male osteoporosis 233 in carpal tunnel syndrome 24 in frozen shoulder 16 in GCA and PMR 161 in gout 106 in hepatitis C 207 in inammatory myopathies 262 injection 9 in carpal tunnel syndrome 24 epidural 1767 in bromyalgia syndrome 31 in frozen shoulder 16 intravenous, in giant cell arteritis 161, 162 in osteoarthritis 51 in pseudogout 197 in psoriatic arthritis 181 in rotator cuff disease 14 in tennis elbow 19, 20 long-term 162 during pregnancy 95 in rheumatoid arthritis 83, 84, 106 in Sjgrens syndrome 132 in SLE 126, 127 steroid myopathy 257, 258, 259

CCL2 162 CD4* T lymphocytes 93 in Sjgrens syndrome 129 celecoxib, cardiovascular risk 612, 63 CENP antibodies 132 central core disease 253 central nervous system involvement Behets syndrome 166 SLE 127 centronuclear (myotubular) myopathy 254 cerivastatin 266 cervical rib syndrome 22 cervical rotation measurement 89, 90 cervical spinal cord stimulation, Raynauds phenomenon 137 channelopathies 2556 Charcot joints (neuropathic arthropathy) 213 cheiroarthropathy 15, 212 Chikungunya 208 children, investigation of acute monoarthritis 910 chloroquine, value in psoriatic arthritis 181 cholecalciferol 219 combination with alendronate 220 cholestyramine 2689 chondrocalcinosis 195 see also pseudogout chondroitin, value in osteoarthritis 46, 49, 104 chronic kidney disease protein restriction 1878 use of allopurinol 192 chronic kidney disease-mineral and bone disorder (CKD-MBD) 2467, 251 adynamic bone disease 248 diagnosis and management 24850 in early renal disease 250 high turnover bone disease (osteitis brosa cystica) 247 mixed 247 osteomalacia 247 ChurgStrauss syndrome 155 ANCA 157 interferon therapy 158 cilostazol, value in Raynauds phenomenon 138 Cinacalcet 250 citrullination 72 citrulline residues 5 CLASS (Celecoxib Long-term Arthritis Safety Study) 57 clodronate, structure and function 2223 clonidine, use in bromyalgia syndrome 30 coenzyme Q10 depletion 267 cognitive behavioural therapy, chronic low back pain 177 cognitive decline, effect of raloxifene 228

Index
in vasculitides 157 in viral arthritis 210 costoclavicular syndrome 22 COX isoenzymes, role in ulcer healing 57 COX-2 inhibitors 56, 57 cardiovascular risk 614 use in SLE 126 coxibs 61 cardiovascular side effects 613 Coxsackie virus, as trigger for Sjgrens syndrome 132 CPEO (Chronic Progressive External Ophthalmoplegia) 256 creatine kinase (CK) 262 monitoring in statin therapy 266, 270 creatinine supplementation, value during steroid therapy 259 CREST syndrome 110, 140 critical illness myopathy (CIM) 2579 Crohns disease 179 CRTL1 gene 41 CRTM gene 41 cryoglobulinaemia 154 antinuclear antibodies 111 crystal arthritis 78, 9 see also gout; pseudogout crystals, intra-articular 7 culture-negative septic arthritis 203 curcumin, effect on inammation 100 cyclooxygenase (COX) inhibitors 567 COX-2 selective drugs 56, 57 cardiovascular risk 614 use in SLE 126 cyclooxygenase (COX) isoenzymes, role in ulcer healing 57 cyclophosphamide 148, 151 effect on fertility 94 use in inammatory myopathies 262 use in scleroderma 143 use in Sjgrens syndrome 132 use in SLE nephritis 127 use in vasculitides 157 cyclosporin 149 use in Behets syndrome 166 use in inammatory myopathies 262 use in psoriatic arthritis 181 use in vasculitides 157 cytokine patterns in pregnancy 93 daclizumab 149 dactylitis, in psoriatic arthritis 180 dapsone, use in vasculitides 157 dehydroepiandrosterone (DHEA) levels in GCA and PMR 1623

275
use in SLE 1267 dental hygiene, Sjgrens syndrome 131 depression in Sjgrens syndrome 132 in SLE 127 dermatomyositis (DM) 2601 antinuclear antibodies 111, 112, 113 pathogenesis 264 dextropropoxyphene 51 diabetes bone complications 213 cheiroarthropathy 15, 212 joint disorders 213 ligament, tendon and capsular problems 212 musculoskeletal complications 21112, 21415 osteoporosis risk 21314 type 2, risk from low vitamin D status 2501 vitamin D levels 219 diacerein 51 value in osteoarthritis 104 diastolic dysfunction, AS 91 diclofenac cardiovascular risk 64 side effects 57 diet 1002 in inammatory myopathies 262 recommendations in hyperuricaemia and gout 187 dietary risk factors, rheumatoid arthritis 74, 978 dietary supplements 99100 see also vitamin D treatment diffuse cutaneous scleroderma 140 diffuse idiopathic skeletal hyperostosis (DISH) 213 diffuse osteomyelitis 201 disc prolapse 171, 172 discectomy 176 discoid lupus 118 disease-modifying antirheumatic drugs (DMARDs) 834 effect on fertility 934 use during pregnancy 95 disease-modifying drugs, benets in psoriatic arthritis 182 disease-modifying osteoarthritis drugs (DMOADs) 512, 104 dislocation, risk after hip replacement surgery 67 diuretics, use in carpal tunnel syndrome 24 DNA methylation abnormalities 119 docosahexaenoic acid 98 benet in RA 823 doxycycline, value in osteoarthritis 51, 53, 104 DRB1 gene 72 drug abuse, septic arthritis 201

276

Index
in septic arthritis 201 in SLE 1256 etanercept 149 use in Behets syndrome 166 use in inammatory myopathies 264 use in psoriatic arthritis 182 use in vasculitides 157, 158 ethnic differences in GCA and PMR 160 in gout 184 in psoriasis 179 in scleroderma 140 in SLE 1234 in use of statins 267 etidronate 2389 structure and function 2223 use in Pagets disease 243 etoricoxib, risk comparison with diclofenac 64 exercise protection against fractures 21819, 221 value in inammatory myopathies 262 value in osteoarthritis 46 exercise programmes in frozen shoulder 1516 in joint replacement surgery 66 in plantar fasciitis 36 in rotator cuff disease 14 extractable nuclear antigens 110 ezetimibe 268, 269 facet joint hypertrophy 171 facet joint injections 176 facial pain, in bromyalgia syndrome 28 facioscapulohumeral muscular dystrophy 255 family history, in new-onset joint pain 3 fasting, effect on inammation 99 fatigue 91 in bromyalgia syndrome 28 FDG-PET, in diagnosis of infections 204 febuxostat 188, 193 fenobrate, use in gout 191, 193 fertility 934 in Sjgrens syndrome 96 fetus, effects of maternal rheumatic disease 945 fever, signicance in monoarthritis 7 brates, muscle side effects 266, 267, 268 brillin 140 broblast dysfunction, scleroderma 142, 143 broblast growth factors, polymorphisms in psoriasis 182 bromyalgia syndrome (FMS) 278 epidemiology and aetiology 29 investigation 29, 30 outlook and management 2931

drug-induced lupus 11819 drug interactions, with allopurinol 192 dry eyes, Sjgrens syndrome 130 dry mouth, Sjgrens syndrome 1301 dual-energy X-ray absorptiometry (DEXA) 218 indications in men 233 Duchenne muscular dystrophy 255 duloxetine, use in bromyalgia syndrome 30 Dupuytrens disease (DD) 212 matrix metalloproteinases 213 eccentric loading exercises, rotator cuff disease 14 eculizumab, use in inammatory myopathies 264 efalizumab 149 use in psoriatic arthritis 182 eicosapentaenoic acid 98 benet in RA 823 elbow pain golfers elbow 19 tennis elbow 1819, 20 elderly people polyarthritis 103 gout 106 osteoarthritis 1034 rheumatoid arthritis 1046, 150 recurrent haemorrhagic shoulder 14 elemental diets 99 elimination diets 99 EmeryDreifuss muscular dystrophy 255 endometriosis 113 endoscopic carpal tunnel surgery 26 endothelial dysfunction, scleroderma 143 endothelin (ET), role in scleroderma 143, 144 enthesitis, in psoriatic arthritis 180 entrapment neuropathies 214, 256 association with diabetes 211 carpal tunnel syndrome 245 as cause of heel pain 35 eosinophilia myalgia syndrome 112 epidermal growth factor (EGF), polymorphisms in psoriasis 182 epidural steroid injections, value in low back pain 1767 epigenetic modications, SLE 119, 120 EpsteinBarr virus (EBV), as trigger for rheumatoid arthritis 745 erythema infectiosum (slapped cheek) 206 erythrocyte sedimentation rate (ESR) 4, 161 in giant cell arteritis 162 in polymyalgia rheumatica 160 in rheumatoid arthritis 78 in scleroderma 142

Index
ngernails, in psoriasis 3 sh oil, benet in RA 823, 989 avonoids 100 uorinated steroids, risk of myopathy 257 uoxetine, value in Raynauds phenomenon 137 uvastatin, risk of muscle side effects 267, 268 a-fodrin antibodies 132 footwear choice, osteoarthritis 47 fracture prevention raloxifene 227 strontium ranelate 228, 230 teriparatide 229 fractures, osteoporotic 218 in men 2335 risk in glucocorticoid-induced osteoporosis 238 see also osteoporosis frozen shoulder 13, 1416, 17 fructose, effect on uric acid synthesis 185, 187 fruit intake, relationship to RA risk 97, 98 gabapentin, use in bromyalgia syndrome 30 GAIT (Glucosamine/chondroitin Arthritis Intervention Trial) 104 gastric side effects, NSAIDs 55, 56, 59, 61 protection strategies 567 risk evaluation 58 gastro-oesophageal reux, in bromyalgia syndrome 28 gastrointestinal problems in scleroderma 141, 1434 Sjgrens syndrome 131 gembrozil, combination with cerivastatin 266 gender differences in gout 184 in osteoarthritis 103 in osteoporosis risk 218 in Raynauds phenomenon 135 in Sjgrens syndrome 129 in SLE 11819 gene therapy, osteoarthritis 512 genetic factors in Behets syndrome 1667 in osteoarthritis 41, 43 in pseudogout 197, 198 in psoriasis 179 in rheumatoid arthritis 72, 75 in systemic lupus erythematosus 117 genetic screening, Pagets disease 243 giant cell arteritis (GCA) 154, 155, 15960, 1634 antiplatelet therapy 162 biological therapies 158 clinical features 1601

277
DHEA levels 1623 epidemiology 162 genetic basis 162 investigation 161, 163 treatment 1612, 163 ginger extracts, value in osteoarthritis 52 glenohumoral osteoarthritis 12 glitazones, osteoporosis risk 214, 250 glucocorticoid-induced osteoporosis 2378, 240 investigation and management 2389 glucocorticoid therapy see corticosteroid therapy glucosamine, value in osteoarthritis 46, 489, 104 glycogen storage diseases 2567 golfers elbow 19 gonococcal arthritis 7 Gottrons sign 261 gout 78, 9, 10, 106, 189, 1934 acute management 190 association with diabetes 211 dietary recommendations 187 gender and ethnic differences 184 investigations 190 molecular basis for inammation 193 prophylaxis 1903 risk factors 190 serum urate levels 7 see also hyperuricaemia; uric acid graft-versus-host disease 112 similarity to scleroderma 144 Greek Atorvastatin and Coronary Heart Disease Evaluation study 1867 green tea, value in osteoarthritis 52 headaches in bromyalgia syndrome 28 in SLE 127 health status, in psoriatic arthritis 182 heart, effects of ankylosing spondylitis 91 Heberdens nodes, genetic predisposition 41 heel pain common causes 34 plantar fasciitis 337 heel spurs 34 Helicobacter pylori infection, interaction with NSAID therapy 57, 59 HenochSchnlein purpura 154 Hep2000 cell line 110 hepatitis, viral arthritis 206 hepatitis B 208 hepatitis C 2078, 209 cryoglobulinaemia 154 hereditary neuropathy with liability to the pressure palsies (HNPP) 2

278

Index
hypogonadism 233 testosterone treatment 2345 hypokalaemic periodic paralysis 255 hypomethylation, association with SLE 119 hypothyroidism chondrocalcinosis 197 gout 190 hypoxanthine phosphoribosyltransferase (HPRT) deciency 186 ibandronate 223, 224 use in male osteoporosis 234 use in Pagets disease 243 ibuprofen cardiovascular risk 63 interaction with aspirin 61 imaging chronic low back pain 176 rheumatoid arthritis 78 see also magnetic resonance imaging; positron emission tomography (PET); radiography immobility, muscle loss 259 immunoglobulin, use in inammatory myopathies 262 immunosuppressive drugs 147 antibody treatments 149 antiproliferative and antimetabolic drugs 148 see also azathioprine; cyclophosphamide; leunomide; methotrexate; mycophenolate mofetil calcineurin inhibitors 1489 glucocorticoids 1478 see also corticosteroid therapy hydroxychloroquine 149 indications for use 150 sulphasalazine 149 therapeutic monitoring 14950, 151 use in vasculitides 157 impingement theory, rotator cuff disease 1314 incision length, hip replacement 68 inclusion body myositis (IBM) 2612 pathogenesis 264 indurative stage, scleroderma 141 infection, risk after joint replacement surgery 67 infections association with arthritis 3 as cause of lower back pain 172 as cause of Pagets disease 242 as trigger for giant cell arteritis 154, 160 as trigger for rheumatoid arthritis 745 as trigger for Sjgrens syndrome 132 see also septic arthritis inammation assessment, SLE 1256

hexosamine biosynthetic pathway 489 high turnover bone disease (osteitis brosa cystica) 247 hip, septic arthritis 9 hip fracture, morbidity 233 hip fracture prevention, strontium ranelate 228 hip replacement surgery 65, 69 complications 67 exercise programmes 66 indications 66, 67 minimal incision 68 prosthesis survival 678 value of weight reduction 656 histamine H2 receptor antagonists, use with NSAIDs 56 histone modication 119 HIV infection 208 association with inclusion body myositis 262 HLA-B51 166 HLA-Cw0602 allele 179 HLA-DRB1*04 160, 162 homocysteine levels, Behets syndrome 167 homogeneous staining, ANA 110, 117 hormone replacement therapy (HRT) 218, 231 combination with teriparatide 230 in osteoarthritis 43, 51 use in SLE 119 HTLV (human T-lymphotropic virus)-1 infection, association with inclusion body myositis 262 human bites 201 hyaluronan, serum levels 43 hyaluronic acid, intra-articular injection 47, 49, 104 hydralazine, association with DIL 118 hydroxychloroquine 149 use in hepatitis C 207 use in psoriatic arthritis 181 use in Sjgrens syndrome 132 use in SLE 126 hyperbaric oxygen therapy, osteomyelitis 203 hypergammaglobulinaemia, antinuclear antibodies 111 hyperglycaemia, association with osteoarthritis 412 hyperkalaemic periodic paralysis 255 hyperostosis 213 hyperparathyroidism, vascular dysfunction 250 hypertensive disorders of pregnancy 96 hyperuricaemia 188 cardiovascular disease risk 1867 causes 1856 dietary recommendations 187 treatment 187 see also gout

Index
inammatory lower back pain 172, 173 inammatory markers, as outcome measures 88 inammatory myopathies 2602 diagnosis and management 2623 use of biological agents 264 iniximab 149 use in Behets syndrome 166 use in GCA and PMR 161 use in inammatory myopathies 264 use in psoriatic arthritis 182 use in vasculitides 157, 158 insulin resistance in Behets syndrome 167 possible effect of glucosamine supplementation 49 interferon therapy in Behets syndrome 166 in ChurgStrauss syndrome 158 interferon 1, possible role in SLE 11920 topical use in Sjgrens syndrome 131 interleukin-1 inhibitors 83 interleukin-18 association with Behets syndrome 167 levels in DM and PM 164 intermalleolar distance measurement 89, 90 interstitial pulmonary brosis 112 intervertebral disc prolapse 171, 172 discectomy 176 intravenous drug users, septic arthritis 201 intravenous immunoglobulin, use in vasculitides 157 irritable bowel syndrome, in bromyalgia syndrome 28 joint aspiration 4 diagnosis of septic arthritis 7, 201 in gout 190 knee 89 in pseudogout 196 joint count 88, 89 prognostic value in RA 80 joint pain acute monoarthritis, differential diagnosis 68 new onset examination 3 history-taking 23 investigations 34 joint replacement surgery 65, 69 complications 67 exercise programmes 66 indications 66, 67 minimal incision 68 perioperative methotrexate therapy 667 prosthesis survival 678 unicompartmental knee arthroplasty 68 value of weight reduction 656 juvenile Pagets disease 243 Kawasaki syndrome 155 KearnsSayre syndrome 256 KelleySeegmiller syndrome 186 knee joint aspiration 89 magnetic resonance imaging 10 monoarthritis, differential diagnosis 68 osteoarthritis, corticosteroid injection 51 knee replacement surgery 65, 69 complications 67 indications 66, 67 post-operative weight gain 66 prosthesis survival 68 unicompartmental 68

279

laminin-1 113 lansoprazole, gastric protection in NSAID therapy 59 large vessel vasculitis 155 laser therapy, Raynauds phenomenon 137 lasofoxifene 230 late-onset RA (LORA) 1056, 150 lateral cutaneous nerve of the thigh, meralgia paraesthetica 24 lateral epicondylitis (tennis elbow) 1819 RCT of treatments 20 leunomide 148 avoidance during pregnancy 95 safety monitoring 79 use in psoriatic arthritis 181 use in Sjgrens syndrome 132 leptin levels, Behets syndrome 167 LeschNyhan syndrome 186 LIFE (Losartan Intervention for Endpoint Reduction) study 187 lifestyle modication, SLE 126 ligamentum avum hypertrophy 171 limb girdle muscular dystrophy 255 limited cutaneous scleroderma 140 limited joint mobility (LJM), diabetes 212 linoleic acid 46 a-linolenic acid 46 local anaesthetic injection, bromyalgia syndrome 31 losartan, use in gout 191, 193 low back pain acute 16970, 174 ankylosing spondylitis 173 causes 1701

280

Index
MELAS (Myopathy, Encephalopathy, Lactic Acidosis and Stroke) 256 melioidosis 201 meloxicam, cardiovascular risk 62 membranous staining, ANA 110 menopause effect on BMD 218 effect on osteoarthritis risk 43 meralgia paraesthetica 24 mercaptopurine, interaction with allopurinol 192 MERRF (Myoclonic Epilepsy with Ragged Red Fibres) 256 metabolic syndrome, association with hyperuricaemia 186 methotrexate therapy 148, 151 effect on fertility 94 in GCA and PMR 161 in inammatory myopathies 262 joint replacement surgery 667 prediction of response 150 in psoriasis 181 teratogenicity 95 in rheumatoid arthritis 84, 86 safety monitoring 79 in scleroderma 143 in Sjgrens syndrome 132 in vasculitides 157 methylene tetrahydrofolate reductase levels 150 methylprednisolone epidural injection 177 intra-articular injection 51 intravenous, in inammatory myopathies 262 use in GCA 161, 162 microchimaerism hypothesis, scleroderma 144 microscopic polyangiitis 155 ANCA 157 minimal incision, hip replacement surgery 68 minocycline 51 association with DIL 118 misoprostol 56 mitochondrial myopathies 256 mitochondrial toxicity, statins 269 mixed connective tissue disease (MCTD), antinuclear antibodies 111, 112, 113 MMP-3 gene 41 MOBILE study 224 monitoring of immunosuppressive treatment 14950, 151 of systemic lupus erythematosus (SLE) 1235 monoarthritis 4 diagnostic errors 7 differential diagnosis 68 see also gout; septic arthritis monocarboxylate transporter 4 inhibition, statins 269

low back pain contd acute contd management 1723 outcome 173 specic pathology 1712 chronic clinical examination 1756 imaging 176 opioid use 177 treatment 1767 indicators for nerve root problems 171 inammatory 172, 173 red ags 170 risk factors 170 Low-Dye taping, plantar fasciitis 35 lumbar exion measurement 89, 90 LUMINA (lupus in minorities: nature versus nurture) study 124, 126 lupus nephritis 122, 124, 127, 150 Lyme disease 7 lymphoma, risk in Sjgrens syndrome 132 M1-2 antibodies 262 McArdle disease 257 magnetic resonance imaging (MRI) in carpal tunnel syndrome 245 in giant cell arteritis 161 in inammatory myopathies 262 of knee joint 10 in low back pain 176 in osteomyelitis 203 in psoriatic arthritis 180 male osteoporosis 2335 diagnosis and management 236 malignancy association with dermatomyositis 2601 breast cancer risk, effect of SERMS 228 as cause of lower back pain 172 vitamin D levels 219 malignant bone pain 2 malignant hyperthermia 2556 mammalian target of rapamycin (MTOR) 148 manipulation under anaesthesia (MUA), value in frozen shoulder 16 matrix metalloproteinases (MMPs) in Dupuytrens disease 213 as therapeutic targetes 51 medial calcaneal nerve entrapment 35 medial compartment syndrome 24 medial epicondylitis (golfers elbow) 19 median neuritis 23 Mediterranean diet 98 medium vessel vasculitis 1545 medullary osteomyelitis 201

Index
monoclonal antibodies 149 monocyte chemoattractant protein-1 (MCP-1) 162 MORE (Multiple Outcome of Raloxifene Evaluation) trial 227, 228 morphea 140 see also scleroderma movement disorders, in bromyalgia syndrome 28 multidisciplinary management, low back pain 177 multiple sclerosis, vitamin D levels 219 multiplex technologies 113 muscle biopsy 262 muscle complications of statins 21112, 2667, 26970 diagnosis and management 2679 pathogenesis 269 rhabdomyolysis 267 muscular dystrophies 2545 muscular infarction, diabetic 212 muscular pains, Sjgrens syndrome 131 myalgia 266 mycophenolate mofetil (MMF) 148, 150 use in psoriatic arthritis 181 use in scleroderma 143 use in SLE nephritis 127 use in vasculitides 157 Mycoplasma pneumoniae, as trigger for giant cell arteritis 154, 160 myoadenylate deaminase deciency 186 myocardial infarction risk after joint replacement surgery 67 NSAIDs 57, 614 myopathies acquired alcoholic myopathy 257 critical illness myopathy 2579 steroid myopathy 258, 259 channelopathies 2556 classication 254 congenital 2534 in congenital metabolic disorders 2567 inammatory 2602 diagnosis and management 2624 mitochondrial 256 muscular dystrophies 2545 myositis-specic antibodies (MSA) 262 myotonia 255 myotonic dystrophy 2545 myotubular (centronuclear) myopathy 254 n-3 fatty acid supplementation, value in osteoarthritis 467

281
N-arylanthranilic acids 61 nailfold capillaries, Raynauds phenomenon 135, 140, 141 NALP proteins, role in gout 193 naproxen, interaction with aspirin 61 NAT9 gene 179 nemaline rod myopathy 2534 neonatal lupus syndrome 945 Neoral see cyclosporin nerve root problems 171 neural theory of tendinopathy 16 neuropathic arthropathy (Charcot joints) 213 neuropathies, Sjgrens syndrome 1312 niacin, muscle symptoms 268 nifedipine, value in Raynauds phenomenon 137 night pain, shoulder 1112 nitric oxide synthase inhibitors, value in osteoarthritis 51 nitric oxide synthases, role in ulcer healing 57 non-steroidal anti-inammatory drugs (NSAIDs) 54-5 classication 61 gastric side effects 569 mechanism of action 556 safety monitoring 79 use in carpal tunnel syndrome 24 use in elderly people 1056 use in gout 190, 192 use in osteoarthritis 501 use in patients with cardiovascular risk 63 use during pregnancy 95 use in pseudogout 197 use in SLE 126 nuclear factor-k-beta (NF-kB) pathway 53, 240 nucleolar staining, ANA 110 Nurses Health Initiative 218 Nurses Health Study, smoking, relationship to RA risk 74 nutritional supplements, value in osteoarthritis 467, 52 obesity association with osteoarthritis 412, 43, 45 association with plantar fasciitis 34 and joint replacement surgery 656 as risk factor for osteoarthritis 104 occupational risk factors osteoarthritis 43 rheumatoid arthritis 74 oedematous stage, scleroderma 141 oesophageal dysfunction, scleroderma 141, 1434 oestrogen effect on osteoarthritis risk 43

282

Index
prevention in long-term steroid therapy 162 risk factors 21819 risk in insulin-requiring diabetes 21314 risk from thiazolidinediones 250 risk management 220 spinal fracture 172 osteoprotegerin (OPG) 218, 238, 243, 248 osteosarcoma, as side effect of teriparatide 229 outcome measures, RA and AS 8892 oxicams 61 p-ANCA 1567 Pagets disease 2412 avoidance of teriparatide 229 comorbidities 245 complications 242 genetic factors 243 investigation and management 2423, 244 pain 2 pain management in bomyalgia syndrome 30 in low back pain 176 painful arc 13 painful joints acute monoarthritis, differential diagnosis 68 new onset examination 3 history-taking 23 investigations 34 pamidronate 224 structure and function 223 use during corticosteroid therapy 238 use in male osteoporosis 234 use in neuropathic arthropathy 213 paracetamol 61 value in osteoarthritis 501 parainuenza virus 160 paramyotonia 255 parathyroid hormone (PTH) 247 in evaluation of renal osteodystrophy 248 recombinant see teriparatide parathyroidectomy, in renal osteodystrophy 249, 250 parecoxib, cardiovascular side effects 62 partial androgen deciency in aging men (PADAM) 2345 parvovirus B19 160, 2067, 2089 patient education, rheumatoid arthritis 84 peak bone mineral density 218 peginterferon-a therapy, hepatitis C 207 pentazocine 51 peptidylarginine deaminases (PADIs) 72 periodontal disease, effect on rheumatoid arthritis 75

oestrogen contd effect on rheumatoid arthritis risk 74, 75 oestrogen therapy in osteoarthritis 51 in osteoporosis 218, 231 see also hormone replacement therapy (HRT) oligoarthritis 3 olive oil 98 omega-3 friendly foods 101 omega-3 oil 98 benets in rheumatoid arthritis 823, 99 Onyong-nyong 208 opioids, use in chronic low back pain 177 oral contraceptive pills, use in SLE 119 orlistat 45 orthoses value in osteoarthritis 47 value in plantar fasciitis 35 osteitis brosa cystica (high turnover bone disease) 247 osteoarthritis 3940, 1034 aetiology 41 association with diabetes 211 comparison with pseudogout 196 drug treatment 503 non-pharmacological treatment 459 radiological severity scoring 40 risk factors 413 secondary causes 40 osteoblast apoptosis inhibition 240 osteoblasts, effect of corticosteroids 238 osteoclast differentiation 243 osteoclasts, activation at menopause 218 osteoid 247 osteomalacia 247 osteomyelitis 201, 203 plasma procalcitonin 10 spinal 172, 204 osteonecrosis, as side effect of bisphosphonates 224 osteopenia, indications for bisphosphonates 223 osteoporosis 21718, 221 drug treatments 218, 231, 234 bisphosphonates 21920, 2234 combination treatment 22930 raloxifene 2278 sequential treatments 230 strontium ranelate 2289, 230 teriparatide 229, 230 third generation SERMS 230 glucocorticoid-induced 2389 investigations 226 male 1335 diagnosis and management 236 management 225, 226

Index
Phalens sign 23 pharmocogenomics 150 Phos-Ex (calcium acetate) 250 phosphate binders, use in chronic kidney disease 250 phosphoribosylpyrophosphate synthetase gene mutations 186 photosensitivity, SLE 122, 126 pioglitazone, osteoporosis risk 214, 250 piroxicam, side effects 57 plantar fascia stretching 36 plantar fasciitis 335 management 357 plasmapheresis, use in vasculitides 157 pleural disease, SLE 123 Pogosta Disease 208 polyarteritis nodosa 155 polyarthritis 3 in elderly people 103 gout 106 osteoarthritis 1034 rheumatoid arthritis 1046 polymerase chain reaction (PCR) screening, in diagnosis of infection 203 polymyalgia rheumatica (PMR) 105, 155, 15960, 1634 DHEA levels 1623 epidemiology 162 investigation 161, 163 treatment 1612, 163 polymyositis (PM) 260, 261 antinuclear antibodies 111, 112, 113 pathogenesis 264 polyphenols 100 value in osteoarthritis 52 Pompe disease 256 positron emission tomography (PET) in diagnosis of giant cell arteritis 161 in diagnosis of infections 204 posterior compartment syndrome 24 prayer sign 212 prazosin, value in Raynauds phenomenon 137 prednisolone in GCA and PMR 161 in inammatory myopathies 262 intra-articular injection 51 risk of fracture 238 see also corticosteroid therapy pregabalin, use in bromyalgia syndrome 30 pregnancy effect on rheumatoid arthritis risk 74 effect on rheumatoid diseases 93, 94 parvovirus B19 infection 208 use of azathioprine 127 use of hydroxychloroquine 126

283
preventive screening, SLE 124, 1256 primary biliary cirrhosis, antinuclear antibodies 111 primary Raynauds phenomenon 135 PRIMO (Prediction of Muscle Risk in Observational conditions) survey 267 probenecid 193 procainamide, association with DIL 118 procalcitonin (PCT) 10 as marker for infection 203 prolactin 93 prolapsed intervertebral discs 171, 172 discectomy 176 prostacyclin effect of NSAIDs 612 value in Raynauds phenomenon 137 value in scleroderma 143 prostaglandin generation, COX-1 and COX-2 55 prostate cancer, androgen deprivation therapy 235 prosthesis survival, joint replacement 66, 678 protein restriction, chronic kidney disease 1878 proteinuria in rheumatoid arthritis 789 in scleroderma 142 proton pump inhibitors, use with NSAIDs 56, 57, 59 pseudoclaudication 171 pseudogout 8, 9, 1956 associated conditions 1967 diagnosis 196 hereditary 197 role of ANKH mutations 198 toll-like receptors 198 treatment 1978 Pseudomonas aeruginosa joint infections 201 psoriasis 3, 179 cardiovascular disease risk 182 in retrovirus infection 208 psoriatic arthritis 4, 17980, 1823 comparison with rheumatoid arthritis 180 diagnosis 180, 181 management 1812 X-ray changes 78 PSORS loci 179 psychological disorders in Behets syndrome 166 in bromyalgia syndrome 28 in Sjgrens syndrome 132 psychological interventions, value in low back pain 177 psychosis, in SLE 127 PTNP22 gene 72 pulmonary artery hypertension (PAH) bosentan therapy 144 scleroderma 142, 143

284

Index
Sjgrens syndrome 131 in SLE 122, 124, 127, 150 renal osteodystrophy (ROD) 2467, 251 adynamic bone disease 248 diagnosis and management 248 in early stage disease 250 high turnover bone disease (osteitis brosa cystica) 247 mixed 247 osteomalacia 247 renal side effects, NSAIDs 57 rofecoxib 63 resolvins 98 retinal vasculitis, Behets syndrome 166 retroviruses 208 reverse Phalens sign 23 rhabdomyolysis, statin therapy 266, 267 rheumatoid arthritis 712 anti-CCP antibodies 7980 antinuclear antibodies 111, 113 comparison with psoriatic arthritis 180 diagnosis 45, 77 investigations 789 diet, susceptibility factors 978 dietary supplements 99100 disease-modifying antirheumatic drugs 834, 86 effect on fetus 945 effect of pregnancy 93 effect on pregnancy 96 effect of smoking 5, 856 in elderly people 1046, 150 environmental factors 735 fertility 934 genetic factors 72, 75 omega-3 oil supplementation 823 parvovirus B19 seropositivity 2089 patient education 84 primary care and specialist liaison 79 prognostic markers 80 safety monitoring of medications 79 treatment algorithm 85 treatment response 845 treatment response evaluation 878, 902 rheumatoid factor (RF) 4, 72, 73, 77, 78, 156 in hepatitis C 207 in parvovirus B19 infection 207 in psoriatic arthritis 182 in rubella 207 ribavarin therapy, hepatitis C 207 rimonabant 45 risedronate 223 fracture prevention 224 long-term use 224 use during corticosteroid therapy 238

pulmonary embolism, risk after joint replacement surgery 67 pulmonary brosis, scleroderma 1412, 143 pulmonary manifestations dermatomyositis 261 Sjgrens syndrome 131 pulseless disease (Takayasus arteritis) 155 purine metabolism 1845 purine nucleoside phosphorylase deciency 186 purine-rich foods 187 purpura 154, 158 pyrazolidine derivatives 61 quality of life, in psoriatic arthritis 182 quality of life evaluation 901 quercetin 100 radial neuritis 23 radiography in gout 190 in low back pain 176 in osteomyelitis 203 in Pagets disease 242 in pseudogout 196 in rheumatoid arthritis 78 vascular calcication 249 radionuclide scanning, in osteomyelitis 203 raloxifene 2278, 231 in combination treatment 230 fracture prevention 224 RANK-RANKL pathway 218, 238, 240, 243 rapamycin, use in inammatory myopathies 264 RAPTOR gene 179 rashes buttery 122 in dermatomyositis 261 rat-bite fever 201 Raynauds phenomenon 1345 autoantibodies 111, 113 benets of statins 144 pathogenesis 136 in scleroderma 141 treatment 1368 reactive arthritis 4, 7 recurrent haemorrhagic shoulder of the elderly 14 red meat intake, relationship to RA risk 98 referred pain 3 around shoulder 11, 12 remitting seronegative symmetric synovitis with pitting oedema (RS3PE) 105 renal disease in scleroderma 142, 143

Index
use in male osteoporosis 234 use in osteoarthritis 104 use in Pagets disease 243 rituximab 149 use in inammatory myopathies 264 use in SLE 128 use in vasculitides 157 RNA-induced silencing complex (RISC) 209 rofecoxib cardiovascular risk 61, 62, 63 renal toxicity 63 rosiglitazone, osteoporosis risk 214, 250 Ross River Virus 208 rosuvastatin 269 dosage in Asian patients 267 rotator cuff disease 11, 1314, 212 clinical assessment 1113 imaging 16 rubella 206, 207 RUTH (Raloxifene Use for The Heart) study 228 sacroiliitis 4 salicylates 61 see also aspirin, low dose Sandimmune (cyclosporin) 149 SAPHO (Synovitis, Acne, Pustulosis, Hyperostosis and Osteitis) syndrome 180 sarcoidosis 132 Schirmers test 130 sciatica, value of epidural steroid injection 1767 scleroderma 1402, 146 antinuclear antibodies 111, 112 pathophysiology 1423 prognosis 142 similarity to graft-versus-host disease 144 treatment 1434 sclerosant injections, low back pain 176 sclerostin 240 secondary Raynauds phenomenon 135 selective oestrogen receptor modulators (SERMS) 227 third generation 230 see also raloxifene selective serotonin reuptake inhibitors use in bromyalgia syndrome 30 use in Raynauds phenomenon 137 SELENA (Safety of Estrogens in Lupus Erythematosus National Assessment) trials 119 septic arthritis 2001, 204 culture-negative 203 of hip, diagnosis 9 investigation 4, 7

285
investigation and management 201, 202 mortality, relationship to age 204 plasma procalcitonin 10 in retrovirus infection 208 treatment 9 sequential treatments, osteoporosis 230 sequestosome 1 (SQSTM1) gene mutations 243 Sevelamer 250 sex hormone binding globulin (SHBG), relationship to bone loss 235 shoulder pain 11, 212 causes and characteristics 12 clinical assessment 1113 frozen shoulder 1416, 17 imaging 16 rotator cuff disease 1314 sialometry 1301 sibutramine 45 sicca syndrome 130 side effects of allopurinol 192 of androgen therapy 235 of azathioprine 148 of bisphosphonates 224 of colchicine 190 of corticosteroids 106, 127, 148 of cyclophosphamide 148 of cyclosporin 149 of hydroxychloroquine 149 of leunomide 148 of methotrexate 148 of mycophenolate mofetil 148 of NSAIDs 57 cardiac 614 gastrointestinal 55, 569 of raloxifene 228 of statins 21112, 26670 of strontium ranelate 228 of sulphasalazine 149 of teriparatide 229 signal recognition particle (SRP) antibodies 262 sildenal, value in Raynauds phenomenon 1378 simvastatin, combination with ezetimibe 269 Sindbis Virus 208 sirolimus (Rapamune) 148 Sjgrens syndrome 12930 anti-centromere antibodies 132 antiviral antibodies 206 association with hepatitis C 207 autoantibodies 111, 113 dry eyes 130 dry mouth 1301 extraglandular manifestations 1312 fertility 96

286

Index
use in psoriatic arthritis 181 sulphate intake, relationship to osteoarthritis risk 49 sulphinpyrazone 191, 193 supercial osteomyelitis 201 superoxide dismutase gene, Val16 allele 167 suprascapsular neuritis 22 swollen joint count (SJC) 88 synovial uid characteristics 89 systemic lupus erythematosus (SLE) 3, 4, 78, 121, 128 ACR classication criteria 117 aetiology 11718, 11920 antiviral antibodies 206 autoantibodies 110, 111, 113 cardiovascular disease risk 1256 central nervous system involvement 127 clinical features 1223 diagnosis 11617 drug-induced 118 effect on fetus 945 effect of pregnancy 93 effect on pregnancy 94, 96 fertility 94 monitoring 1235 nephritis 127 preventive screening 124, 1256 risk from smoking 74 SELENA trials 119 treatment 1267 systemic sclerosis 140 autoantibodies 111, 112, 113 effect of pregnancy 94 T score, denition of osteoporosis 21718 tacrolimus 149 use in psoriatic arthritis 181 Takayasus arteritis (pulseless disease) 155 tamoxifen 228 taping value in osteoarthritis 47 value in plantar fasciitis 35 Tarui disease 257 temporal artery biopsy 155, 161, 162, 164 tender joint count (TJC) 88 tendinopathy, neural aetiology 16 tennis elbow 1819 RCT of treatments 20 teriparatide 224, 226, 229, 231 in combination treatment 230 cost-benet analysis 230 use during corticosteroid therapy 239 use in male osteoporosis 234 testosterone, use in male osteoporosis 2345

Sjgrens syndrome contd infections as trigger 132 international consensus criteria for diagnosis 130 investigation and management algorithm 133 skin lesions, Sjgrens syndrome 131 slapped cheek disease (erythema infectiosum) 206 SLC9A3R1 gene 179 SLC11A1 gene 167 SLC22A4 gene 72 SLC22A12 (URAT1) 185 sleep abnormalities, in bromyalgia syndrome 28, 29 small vessel vasculitis 1545 smoking 11213 effect on ankylosing spondylitis 91 effect on osteoarthritis 42 effect on Raynauds phenomenon 136 effect on rheumatoid arthritis 5, 74, 75, 856 effect on SLE 126 SOTI (Spinal Osteoporosis Therapeutic Intervention) trial 228 SPARC 140 speckled staining, ANA 110, 117 spinal canal stenosis 171 surgery 176 spinal cord stimulation, Raynauds phenomenon 137 spinal fracture 172 spinal function assessment 8890 spinal fusion surgery 176 spinal infections 172, 203, 204 spondyloarthropathies, effect of pregnancy 94 spondylodiscitis 180 spondylolisthesis 171 statins benets in scleroderma 1445 cholesterol-lowering activity 266 muscle symptoms 212, 2667, 26970 diagnosis and management 2679 pathogenesis 269 rhabdomyolysis 267 steroid myopathy 257, 258, 259 steroids see corticosteroid therapy Streptococcus moniliformis 201 stress reduction, bromyalgia syndrome 31 stroke risk, raloxifene 228 strontium ranelate 2289, 230, 231 fracture prevention 224 subacute cutaneous lupus 118 sulphasalazine 149 effect on fertility 94 safety monitoring 79 use during pregnancy 95

Index
tetracycline labelling, bone biopsy 248 tetracyclines, value in osteoarthritis 51, 53, 104 Th1 and Th2 cell patterns 93 thalidomide, use in Behets syndrome 166 therapeutic monitoring, immunosuppressives 14950, 151 thermal biofeedback (TBF) 1367 thiazolidinediones, osteoporosis risk 214, 250 thiopurine methyltransferase levels 150 Thomsens disease 255 thoracic outlet syndromes 22 three-joint complex, spine 171 thrombosis, risk during pregnancy 94 thromboxane A2, effect of NSAIDs 612 thyroiditis, in Sjgrens syndrome 132 tiludronate structure and function 2223 use in Pagets disease 243 Tinels sign 23 tissue-engineering, osteoarthritis 52 Titralac 250 tizanidine, use in bromyalgia syndrome 30 TNFR2 gene 72 TNFRSF11B mutations 243 toll-like receptors (TLRs) 198 tongue, claudication 155 tophi 189 dissolution 193 topical NSAIDs, value in osteoarthritis 51 topoisomerase 1 140 tragus to wall measurement 89, 90 tramadol 30 use in osteoarthritis 51 transforming growth factor-b (TGF-b), role in scleroderma 143 transplant patients, cyclosporin therapy 149 treatment response evaluation, AS and RA 8792 triamcinolone epidural injection 177 intra-articular injection 51 tricyclic antidepressants, use in bromyalgia syndrome 30 trigger point injections bromyalgia syndrome 31 low back pain 176 trigger points, bromyalgia syndrome 28, 29 triphasic colour changes, Raynauds 135 TROPOS (Treatment of Peripheral Osteoporosis Study) 228 tumour necrosis factor (TNF)-a-1031C allele 166 tumour necrosis factor (TNF)-a antagonists 51, 80, 83, 85, 149, 150 safety monitoring 79

287
tumour necrosis factor (TNF)-a polymorphisms, association with RA 72 turmeric, effect on inammation 100 twin studies osteoarthritis 41 rheumatoid arthritis 72 U3-RNP antibodies, scleroderma 142 ulcer healing, role of COX isoenzymes 57 ulcers in Behcets syndrome 166 in scleroderma 141 ulnar neuritis 223 ultrasound in carpal tunnel syndrome 245 in giant cell arteritis 161 unicompartmental knee arthroplasty 68 URAT1 (SLC22A12) 185 urate levels, in diagnosis of gout 7 uric acid metabolism 1845 see also gout; hyperuricaemia uricase 184 uricase preparations, use in gout prophylaxis 192 uricosuric agents 190, 191, 193 urinary C-terminal peptide of collagen type II 43 urine alkalinization 184, 187 uveitis, Behets syndrome 166 vascular calcication, in renal disease 24950, 251 vascular dysfunction in hyperparathyroidism 250 in scleroderma 143 vascular endothelial growth factor (VEGF) polymorphisms, psoriasis 182 vascular theory, rotator cuff disease 14 vasculitides 1534 ANCA 1578 antibody therapies 158 classication 154 diagnosis 1557 effect of pregnancy 94 large vessel 155 medium vessel 155 small vessel 154 treatment 157, 158 see also Behets syndrome; giant cell arteritis (GCA) VDR gene 41 vegetarian/ vegan diets 99 vertebral fracture prevention 224

288

Index
in corticosteroid therapy 238 men 234 Vytorin 269 walking sticks, use in osteoarthritis 47 warfarin, interaction with allopurinol 192 Wegeners granulomatosis 1545 ANCA 157 weight reduction value before joint replacement surgery 656 value in bromyalgia syndrome 31 women, rheumatoid arthritis risk 74 wrist splints, carpal tunnel syndrome 24 X-rays see radiography xanthine oxidase (XO) 184 xanthine oxidase inhibitors 188 see also allopurinol xerostomia differential diagnosis 131 Sjgrens syndrome 1301 zoledronate dosage 224 structure and function 223 use in Pagets disease 243, 245

vertebral fracture prevention contd raloxifene 227 strontium ranelate 228 teriparatide 229 VIGOR (Vioxx Gastrointestinal Outcomes Research) 57, 61 viral arthritis 2056 alphaviruses 208 causes 206 diagnosis and treatment 20910 hepatitis C 2078, 209 parvovirus B19 2067, 2089 retroviruses 208 rubella 207 viscosupplementation 47, 49 visual symptoms, GCA 161 vitamin C intake, relationship to RA risk 97, 98 vitamin C status, relationship to risk of osteoarthritis 46 vitamin D 219 calcitriol (1, 25-dihydroxy-vitamin D3) 247 vitamin D deciency 221 vitamin D receptor gene 41 vitamin D status in male osteoporosis 235 relationship to rheumatoid arthritis risk 74 relationship to risk of type 2 diabetes 2501 vitamin D treatment 218, 231 in chronic kidney disease 250

You might also like